You are on page 1of 368

UPSC

CSAT
Class
Study Material

Vikrant Bhosale | Aparna Dixit | Sushruth Ravish


Published by
'The Unique Academy'
'Shivtirth', Above Bank of India, Fergusson College Road,
Shivajinagar, Pune 411 005
Ph. 020-25530950 / 9850021524 / 9890192929
Fax - 020-30221950
Email. : admin@theuniqueacademy.com
Website : www.theuniqueacademy.com

UPSC CSAT Class Study Material


By Vikrant Bhosale | Aparna Dixit | Sushruth Ravish

© Unique Academy Publications Pvt. Ltd.

All rights reserved. No part of this publication may be reproduced or


transmitted, in any form or by any means, without permission. Any person
who does any unauthorised act in relation to this publication may be liable
to criminal prosecution and civil claims for damages.

Year 2019

Cover Design : Unique Academy Publications Team


Typesetter : Unique Academy Publications Team
Printed by : M.S. Process, Narayan Peth, Pune
Contents
Sr. No. Topic Page No.
Section – I – Reading Comprehension
1.1. Back to Basics - Building a foundation 10
Practice Passages – 1 12
Chapter 1 - Introduction
1.2 Finding the Main Idea 20
Practice Passages – 2 23
2.1 Active Reading 29
2.2 Speed Reading 32
2.3 Vocabulary 36
Chapter 2 - Elements of
2.3.1 Some Often Used Bases 42
RC Skills
2.3.2 Prefixes 53
2.3.3 Suffixes 56
2.3.4 Strategies for Working with Roots 60
3.1 Author’s Purpose Practice Questions 72
Chapter 3 - Higher Order
3.2 Main Idea Practice Questions 75
Thinking (HoT)
3.3 Worksheet Level – 1 83
4.1 RC section in CSAT 2011 95
Chapter 4 - Analysis of RC
4.2 RC section in CSAT 2012 98
Passages and Questions in
4.3 RC section in CSAT 2013 102
CSAT
4.4 RC section in CSAT 2014 105
5.1 Structure of Passage Approach 109
Chapter 5 - Strategies for
5.2 Storyline Approach 111
Reading Comprehension
5.3 Optimized Reading Approach 114
6.1 CSAT 2011 RC Questions 116
Chapter 6 - RC Questions
6.2 CSAT 2012 RC Questions 126
from CSAT 2011-2014
6.3 CSAT 2013 RC Questions 138
Papers
6.4 CSAT 2014 RC Questions 147
Section II - Logical Reasoning – Syllogisms
Chapter 1 Syllogisms 1 161
Chapter 2 Syllogisms 3 171
Chapter 3 Syllogisms 3 176
Section III - RC Questions from CSAT 2015-2017 Paper
Chapter 1 RC Questions on Inference and Corollary 181
Chapter 2 RC Questions on Assumptions 193
Chapter 3 RC Questions on Main Idea 197
Chapter 4 Specific and Generic Questions 203
Section IV – Logical Reasoning and Analytical Ability
Chapter 1 Simple Exercises 217
Chapter 2 Data Arrangement 225
Chapter 3 Blood Relations + Que. from Previous Years 235
Chapter 4 Data Sufficiency 242
Chapter 5 Questions from Previous Years 247
Chapter 6 Que. from Previous Years 255
Section V – Data Interpretation
Chapter 1 Model Exercises 267
Chapter 2 D.I. Questions in CSAT so far 278
Section VI – Sectional Tests
Chapter 1 Reading Comprehension 299
Chapter 2 Syllogisms 322
Chapter 3 Logical Reasoning & Analytical Ability 328
Chapter 4 LRAA + DI 345
Section - I

Reading
Comprehension
UPSC - CSAT -1

6 USPC - CSAT
UPSC - CSAT -1

Chapter

1 Introduction

Reading Comprehension - What it Means


Reading comprehension or Comprehension (in the context of competitive exams) is a familiar
question type to almost all of us. Back in our school days we had ‘unseen passages’ in the both
English and our regional languages that were more or less on similar lines. As simple as it may sound,
it is the thorn in the neck of many an aspirant! We’ll soon see why.
The word ‘comprehension’ means the ability to understand. So, when we talk about Reading
comprehension, we simply are combining reading and understanding together. Naturally the question
arises, reading and understanding of what? In most cases it is understanding an article or a passage.
Often these articles consist of multiple paragraphs. These paragraphs are in turn made up of sentences
and words. But hidden behind these words would be ideas, views and opinions of the author. Only on
probing further will you come across his/her purpose, objective behind writing the passage. You, as
the recipient are supposed to read these sentences and understand author’s views and reasons.

Why Reading Comprehension?


Reading Comprehension is a question type that is present in almost all exams of repute. From
Indian exams like CSAT (for entry into civil Services), CAT( for entry into management institutes),
CDS( for entry into armed forces) to foreign exams like GRE, GMAT it is an ubiquitous question
type. So why is Reading comprehension such a popular question type? One of the main reasons is
a candidate’s performance in RC gives the examiner a window into the candidate’s thought process
itself. You are presented with a host of opinions and information through the passage. You are supposed
to then sift through them, assimilate the core ideas and internalise them so that you would be able to
apply it to cases that are relevant to the context being addressed. This infact is a microcosm of the
knowledge gaining process itself.

Reading Comprehension : Overview


Reading comprehension, (or RC in popular parlance,) checks a candidate for multiple skills and
abilities. He/ She is supposed to not only understand the passage at both micro and macro levels, but also,
by putting himself / herself in the author’s shoes, be able to offer relevant opinions, whenever asked.
Passages are edited excerpts from larger articles/works written by scholars and eminent personalities.
Such write-ups can be picked from any field, but generally focus on science and technology, social
USPC - CSAT 7
UPSC - CSAT -1

sciences and humanities. Many candidates think that because they know how to ‘read’, RC should
not be a major hurdle. But if we do a reality check, in most cases this complacency is misplaced.
Very few of us would be accustomed to the scholarly standards of the passages generally chosen
in the examinations. Many passages are taken from journals like The Economic and the Political
Weekly, The Scientific American, Nature and the like. A few passages are also taken from research
publications and books by experts and respected academicians. Very few of us would have this in our
regular reading list. Another aspect that makes a difference is, almost all our earlier reading would
have gone unchecked and our understanding would never have been put to test. So when you read an
article in a magazine or on a blog, there is no one to check if you have missed an important idea or
misunderstood the author on a crucial point. Such mistakes, which you might not have been aware of
earlier, can cause lot of harm to your performance in an examination.
A common grouse against the RC section is that it is time consuming. A student with an average
educational background hardly reaches a reading speed of 150 words per minute while tackling
light readings in English, say a children’s story book or a light novel. A vernacular medium student
might be still 25% slower while comprehending a language he/she is not completely familiar with.
Moreover, while dealing with analytical passages, the reading speed is pulled down by the need to
pay attention to arguments and justifications. Further if the language, content or the style used is a
little unfamiliar or even if the student is not particularly interested in the topic of the passage, there
is a drastic drop in the reading speed. This reduction in speed costs most students dearly during their
examinations. So a lot depends on how fast and accurately one reads. Thus, not only comprehension,
but reading quickly also is a critical aspect, if one wants to do well in the Reading Comprehension
(RC) section. We would discuss more about the techniques of speed reading later. For an appreciable
performance in RC questions, one needs to read carefully, accurately and draw reasonable inferences
from the material in the passage. It mandates retention and analysis of ideas, good concentration and
robust reading techniques.
The way, the joy of the pudding is in eating it, the objective of Reading comprehension section is
to answer the questions that are based on it. So let us now focus on the questions we can expect on
passages.
Questions on Reading comprehension would conceptually cover the following aspects:
¾¾ Identifying the main theme or issue of the passage
¾¾ Browsing through the information and obtaining specific details from the passage
¾¾ Identifying the contextual function, or meaning of a specific phrase used in the passage.
¾¾ Identifying the implied meanings and methods an author uses to convey the ideas in the passage
¾¾ Drawing reasonable inferences from the given information
¾¾ Applying the ideas presented in the passage to new situations and contexts

8 USPC - CSAT
UPSC - CSAT -1

Questions on Reading Comprehension in UPSC


CSAT is supposed to test your ability to become a bureaucrat, a person who is committed to social
justice and is a vital cog in the running of a country. So the moral, ethical and social dimensions have
been added to almost every question type. Reading comprehension, which otherwise is anywhere
between 500-800 words long and deals with abstruse topics in philosophy, American history, quantum
mechanics, when it appears in CAT or GRE, comes in CSAT in an entirely different way. Passages
in CSAT are around 250-500 words and cover topics like government policy decisions, culture,
democracy, social awakening, inclusive growth, ecosystems. The vocabulary and the language
used is hardly something worth worrying. This is not to say that they were easy, but the aspirant is
checked on different parameters. The reasoning and the analysis need to be razor- sharp as always
and passages demand every iota of your concentration but the focus is different. Instead of worrying
about vocabulary and time management which are a concern in other competitive exams, one needs
to learn to read between the lines and infer conclusions and occasionally be able to visualize the
application of those inferences to existing social perspectives around them. Let us now do a more
detailed analysis of the reading comprehension passages that have appeared in CSAT so far and get
to know them better! Reading Comprehension has been a regular feature in UPSC GS II. Right from
its inception, at least 40% of the total number of questions have been appearing on it. The following
is the yearwise-break-up of the number of questions
Year 2011 2012 2013 2014 2015 2016 2017
No. of passages 9 11 11 10 23 17 29
No. of Questions 39 40 42 32 30 27 30

Type of Question 2015 2016 2017


Inference 8 10 13
Main Idea 8 2 10
Assumption 5 4 4
Direct and Specific 9 11 3

It needs to be mentioned here that before 2015, when the marks obtained in CSAT was no merely
qualifying but marks used to be counted for calculating cut-off, there was a separate section called
English Language Skills. Around 8-9 questions used to appear under this section. The English
language skills section would generally consist of 2-3 small passages. Since these questions were to
check English language skills, no translation was provided in Hindi. From 2015 onwards that section
has been discontinued. All reading comprehension passages and questions now have both Hindi and
English translations.

USPC - CSAT 9
UPSC - CSAT -1
If you analyse the information provided in the table closely, you will realize a clear shift in
RC. The number of passages have almost doubled since 2015. Passages have got shorter but since the
number of questions asked per passage has gone down, RC section has become more time consuming
than earlier. Hence, those who are depending on RC for qualifying in this paper, need to understand
the basics of question type. The additional trouble with RC is that unlike mathematics or logical
reasoning, where students solve questions and then read the options to pick the correct response, In
RC, the correct response is almost always chosen after reading all the four options. Hence, many
times there is no way to cross-check answers. Hence, attempting RC questions without knowing the
basics can be detrimental in CSAT.

1.1 Back to Basics


Reading Comprehension- Building A Foundation
Critical readers do not let opinions sway them; they want to know what actually happened.
They collect tangible evidence and facts and use this information to draw an informed conclusion.
Separating fact from opinion is essential during a crime scene investigation. It is also a crucial skill
for effective reading.
When you read, look for clues to understand the author’s meaning. What is this passage about?
What is this writer saying? What is his or her message? At times, it may seem like authors are trying
to hide their meanings from you. But no matter how complex a piece of writing may be, the author
always leaves plenty of clues for the careful reader to find. It is your job to find those clues. Open your
eyes and ask the right questions. In other words, read carefully and actively.

Getting the Essential Information


The first step in increasing your reading comprehension is to learn how to get the basic information.
As a reader faced with a text, the first thing you should do is establish the facts. What does this piece
of writing tell you? What happens? To whom? When, where, how, and why? If you can answer
these basic questions, you’re on your way to really comprehending what you read. (You’ll work on
answering the more difficult question—”Why did it happen?” later.)

What Are the Facts?


A fact may be defined as:
¾¾ Something that we know for certain to have happened
¾¾ Something that we know for certain to be true
¾¾ Something that we know for certain to exist
Much of what you read, especially today in this “Information Age,” is designed to provide you

10 USPC - CSAT
UPSC - CSAT -1
with facts. You may read, for example, about a new exam syllabus that you must follow; about how
the new computer system works; about what happened at the international summit. In a standardized
test, you’ll probably have to answer at least a few reading comprehension questions that ask about
the facts in a reading passage. These facts are not always easy to determine, especially if the writing
is dense or complicated. To make it simpler, ask yourself these questions as you read: What facts am
I expected to know? What am I to learn or be aware of? What happened? What is true? What exists?
Jump right into the task of finding facts. The following brief passage is similar to something you
might see in a newspaper. Read the passage carefully, and then answer the questions that follow.
Remember, careful reading is active reading, so mark up the text as you go. Underline key words and
ideas; circle and define any unfamiliar words or phrases; and record your reactions and questions in
the margins.

Passage I

On Friday, April 21, at approximately 8:30 a.m., Amita Jhaweridas, owner of The Jhaweridas
Jewellers, arrived at her shop to find that it had been robbed and vandalized overnight. The front
window of the shop at 28 Bhawanipeth was broken, and chairs and tables were overturned throughout
the area. Additionally, the cash register had been pried open and emptied of money. The thieves
attempted to open the safe as well, but were unsuccessful. Ms. Jhaweridas used her cell phone to
report the crime to the police. She also phoned the proprietor of Papa Jones Pizza, located at 30
Bhawanipeth, as she noticed that the door of that restaurant showed signs of forced entry. The police
department is asking anyone with information to call the nearest police station.
Based on the words underlined alone, answer the following questions:
1. What happened to The Jhaweridas Jewellers shop?
__________________________________________________________________________
2. When was the crime discovered?
__________________________________________________________________________
3. Where did it happen?
__________________________________________________________________________
4. What was stolen?
__________________________________________________________________________
5. Who called the police?
__________________________________________________________________________
6. What other businesses were affected?

__________________________________________________________________________

USPC - CSAT 11
UPSC - CSAT -1
A driver’s license must be renewed every twenty years. A renewal application is sent approximately
five to seven weeks before the expiration date listed on the license. Individuals who fail to renew
within three years of the license expiration date are not eligible for a renewal and must repeat the
initial licensing process. To renew a license, you must visit a Regional Transport Office. You must
present a completed renewal application; your current driver’s license; acceptable proof of age,
identification, and address; income tax return, current pay slip. You must also pay the required fee. If
all the documents and payment are in order, your photo will be taken and a new license will be issued.
Based on the words underlined alone, answer the following questions:
7. What documents does one need to renew a driver’s license?
__________________________________________________________________________
8. How often must one renew a driver’s license?
__________________________________________________________________________
9. How does one obtain the renewal form?
_________________________________________________________________________
Practice Passages - 1

The key to mastering Reading Comprehension is practice, practice and more practice!! Start off
with these short, easy passages and questions. Most ask you to identify only explicit ideas and details,
although a few may require that you make simple inferences. Begin by looking for the basic facts of
the case.

Rehabilitation is a constructive way to reduce crime and generally improve the criminal justice
system in a humane way. The system’s current emphasis on punishment is a failure. Without
rehabilitation before and after their discharge from prison, offenders will usually commit more crimes.
1. The paragraph best supports the statement that
(a) Prisons should be replaced by humane rehabilitation centers
(b) Without rehabilitation, criminals will invariably commit more crimes
(c) If criminals are rehabilitated the crime rate will go down
(d) Most prisons today are too overcrowded for effective rehabilitation

A recent idea in law enforcement is community-oriented policing. This concept is used effectively
in Japan. In every Japanese neighborhood there are Kobans, or guard shacks, where a local police
officer sits. Tokyo has thousands of Kobans. This system has made the Japanese feel safe walking

12 USPC - CSAT
UPSC - CSAT -1
around their cities.
2. According to the passage which of the following statements is true?
(a) Kobans are an inexpensive and efficient way to keep cities safe
(b) In Japan, police officers do not patrol the streets but sit in Kobans instead
(c) Indians would do well to study Japanese law-enforcement methods
(d) Community-oriented policing has made the residents of Tokyo feel secure

Anyone who lives in a large, modern city has heard the familiar sound of electronic security
alarms. Although these mechanical alarms are fairly recent, the idea of a security system is not new.
The oldest alarm system was probably a few strategically placed dogs who would discourage intruders
with a loud warning cry.
3. The paragraph best supports the statement that
(a) Dogs are more reliable than electronic alarms
(b) City dwellers would be wise to use dogs for security
(c) Mechanical alarm systems break down but dogs do not
(d) A dog is an older alarm device than is a mechanical alarm

In cities throughout the country, there is a new direction in local campaign coverage. Frequently
in local elections, journalists are not giving voters enough information to understand the issues and
evaluate the candidates. The local news media devotes too much time to scandal and not enough time
to policy.
4. According to the passage, the local news media
(a) Is not doing an adequate job when it comes to covering local campaigns
(b) Does not understand either campaign issues or politics
(c) Should learn how to cover politics by watching the national news media
(d) Has no interest in covering stories about local political events

The use of desktop computer equipment and software to create high-quality printing such as
newsletters, business cards, letterhead, and brochures is called Desktop Publishing, or DTP. The most
important part of any DTP project is planning. Before you begin, you should know your intended
audience, the message you want to communicate, and what form your message will take.
USPC - CSAT 13
UPSC - CSAT -1
5. Which of the following statements is valid according to the passage?
(a) DTP is one way to become acquainted with a new business audience
(b) Computer software is continually being refined to produce more high-quality printing
(c) The first stage of any proposed DTP project should be organization and design
(d) The planning stage of any DTP project should include talking with the intended audience

Close-up images of Mars by the Mariner 9 probe indicated networks of valleys that looked like
the stream beds on Earth. These images also implied that Mars once had an atmosphere that was thick
enough to trap the sun’s heat. If this is true, something must have happened to Mars billions of years
ago that stripped away the planet’s atmosphere.
6. With reference to the passage, which of the following assumptions are true?
(a) Mars once had a thicker atmosphere than earth does
(b) The Mariner 9 probe took the first pictures of Mars
(c) Mars now has little or no atmosphere
(d) Mars is closer to the sun than Earth is

The criminal justice system needs to change. The system could be more just if it allowed victims
the opportunity to confront the person who has harmed them. Also, mediation between victims and
their offenders would give the offenders a chance to apologize for the harm they have done.
7. With reference to the passage, victims of a crime should
(a) Learn to forgive their offenders
(b) Learn the art of mediation
(c) Insist that their offenders be punished
(d) Have the right to confront their offenders

There are no effective boundaries when it comes to pollutants. Studies have shown that toxic
insecticides that have been banned in many countries are riding the wind from countries where they
remain legal. Compounds such as DDT and toxaphene have been found in remote places like the
Yukon and other Arctic regions.
8. The paragraph best supports the statement that
(a) Toxic insecticides such as DDT have not been banned throughout the world

14 USPC - CSAT
UPSC - CSAT -1
(b) Many countries have ignored their own anti-pollution laws
(c) DDT and toxaphene are the two most toxic insecticides in the world
(d) Even a worldwide ban on toxic insecticides would not stop the spread of DDT pollution

9
During colonial times in India, judges were encouraged to ask questions of the parties in the
courtroom. The judges were, in fact, expected to investigate the facts of the case themselves. If judges
conducted an investigation today, we would throw out the case.
9. The paragraph best supports the statement that
(a) Juries are less important today than they were in colonial times
(b) Courtrooms today are more efficient than they were in colonial times
(c) Jurors in colonial times were more informed than jurors today
(d) The judge system in India has changed since colonial times

10

The City has distributed standardized recycling containers to all households with directions that
read: We would prefer that you use this new container as your primary recycling container as this
will expedite pick-up of recyclables. Additional recycling containers may be purchased from the City.
10. According to the passage, each household
(a) May only use one recycling container
(b) Must use the new recycling container
(c) Should use the new recycling container
(d) Must buy a new recycling container
11. According to the passage, which of the following is true about the new containers?
(a) The new containers are far better than other containers in every way.
(b) The new containers will help increase the efficiency of the recycling program.
(c) The new containers hold more than the old containers did.
(d) The new containers are less expensive than the old.

11

The Competitive Civil Service system is designed to give applicants fair and equal treatment
and to ensure that applicants are hired based on objective criteria. Hiring has to be based solely on
candidates’ Knowledge, Skills, and Abilities (KSA) and not on any external factors such as race,

USPC - CSAT 15
UPSC - CSAT -1
religion, sex, and so on. Whereas employers in the private sector can hire employees for subjective
reasons, government employers must be able to justify their decisions with objective evidence that
the candidate is qualified.
12. Which of the following statements is most likely to be true based on the passage?
(a) Hiring in the private sector is inherently unfair
(b) KSA are not as important as test scores to government employers
(c) Government hiring practices are simpler than those employed by the private sector
(d) The civil service strives to hire on the basis of a candidate’s abilities
13. The government’s practice of hiring on the basis of KSA frequently results in the hiring
of employees
(a) Based on race, religion, sex, and so forth
(b) Who are unqualified for the job
(c) Who are qualified for the job
(d) On the basis of subjective judgment

12

It is well-known that the world urgently needs adequate distribution of food, so that everyone gets
enough. Adequate distribution of medicine is just as urgent. Medical expertise and medical supplies
need to be redistributed throughout the world so that people in emerging nations will have proper
medical care.
14. This paragraph best supports the statement that
(a) the majority of the people in the world have no medical care
(b) medical resources in emerging nations have diminished in the past few years
(c) not enough doctors give time and money to those in need of medical care
(d) many people who live in emerging nations are not receiving proper medical care

13

In the past, suggesting a petrol tax has usually been thought of as political poison. But that doesn’t
seem to be the case today. Several states are pushing bills in their state legislatures that would cut
income or property taxes and make up the revenue with taxes on fossil fuel.
15. The paragraph best supports the statement that
(a) petrol taxes produce more revenue than income taxes
(b) states with low income tax rates are increasing their petrol taxes

16 USPC - CSAT
UPSC - CSAT -1
(c) state legislators no longer fear increasing petrol taxes
(d) taxes on fossil fuels are more popular than property taxes

14

Popular illusions about birds extend further than the use of the word ‘egg-shape’ that would suggest
that all eggs are alike. For instance, there is the popular idea that owls hoot. Actually, only very few
owls hoot and these include the common brown or tawny wood owl. The white barn owl screeches,
the little owl has a wailing cry, the long-eared owl barks, and the short-eared owl snorts! Another
mistaken idea is that all ducks ‘quack’, because the common farmyard duck is a domesticated form of
the common wild duck or mallard that quacks. Actually most wild ducks call with whistles.
16. The main purpose of this passage is
(a) to describe the life of popular birds.
(b) to show our incorrect ideas of bird life
(c) to show our perfect knowledge about birds.
(d) to describe the calls of owls and ducks.
17. The impression created by repeating the terms, ‘popular’ and ‘common’ are
(a) human beings are closely attached to birds.
(b) our ideas about birds are derived from the most common types.
(c) owls and ducks arc our favourite birds.
(d) domesticated birds arc our source of information about the bird world.
18. The common duck is not a separate species but a tamed version of wild variety, because
it
(a) whistles like most wild ducks. (b) grunts like the tufted duck.
(c) has the same call as other ducks. (d) quacks like the mallard.

15

It was Galileo and Newton—notwithstanding that Newton himself was a deeply religious man—
who destroyed the old comfortable picture of a friendly universe governed by spiritual values. And
this was effected, not by Newton’s discovery of the law of gravitation nor by any of Galileo’s brilliant
investigations, but by the general picture of the world which these men and others of their time
made the basis of the science, not only of their own day, but of all succeeding generations down to
the present. That is why the century immediately following Newton, the eighteenth century, was
notoriously an age of religious skepticism. Skepticism did not have to wait for the discoveries of
Darwin and the geologists in the nineteenth century. It flooded the world immediately after the age of

USPC - CSAT 17
UPSC - CSAT -1
the rise of science.
19. ‘The old comfortable picture of a friendly universe’ was:
(a) a universe governed by religious beliefs
(b) a universe with men like Newton who were deeply religious
(c) a universe investigated by Galileo and Newton
(d) the century immediately following Newton
20. Religious skepticism arose because:
(a) Galileo and Newton were not religious being scientists
(b) Newton discovered the law of gravitation
(c) of the discoveries of Darwin and the geologists of the nineteenth century
(d) of the picture of the world that became the basis of science after the seventeenth century

16

Patriotism is easy enough during the war. For one thing, people become actively conscious of their
country when it is threatened. For another, since everyone or nearly everyone feels it, there is a strong
collective emotion flowing through the country. But when the war is over, the country recedes from
the consciousness, and it is rarely even at the back of the mind. The question, “Am I doing any good
to the country ?” doesn’t occur to the mind; and even the question, “Is what I am doing good or bad
for the country?” occurs only rarely. A limited patriotism is better than none but it is not enough.
21. The author says that one reason why patriotism is easy during war-time is that
(a) War is the only factor responsible for arousing patriotism.
(b) When the country is in danger, we become conscious of it.
(c) We become patriotic when we are in a fighting mood.
(d) Patriotism is always in the consciousness.
22. A secondary reason suggested by the author for the emergence of patriotic feeling
during war-time is that
(a) Patriotism is specially required during war-time.
(b) We become deeply conscious of danger to ourselves during war-time.
(c) Patriotism becomes contagious because one finds a lot of other people feeling patriotic.
(d) Everyone feels that he/she should strive to protect the others.
23. When the threat of war is over,

18 USPC - CSAT
UPSC - CSAT -1
(a) There is no need for people to be interested in others’ welfare.
(b) People do not consider it necessary to serve the country.
(c) People tend to become totally unmindful of national interests.
(d) People are generally anxious to serve the country.
24. When the author says that the country “recedes from the consciousness”, he means to
say that
(a) People no longer feel that they belong to the country.
(b) People do not have the country clearly in the forefront of their mind.
(c) People are not capable of understanding what is good and desirable for them.
(d) The moral values which they have so far upheld would disappear from their minds.
25. By a “limited patriotism”, the author refers to
(a) The people’s attitude of indifference to the welfare of their country
(b) The attitude of people who love other countries as much as they love their own country
(c) Those who are self-centered and mainly concerned about their own welfare
(d) The attitude of people who are indifferent to the welfare of their country except during
war-time

Question No. 1 2 3 4 5

Answers

Question No. 6 7 8 9 10

Answers

Question No. 11 12 13 14 15

Answers

Question No. 16 16 18 19 20

Answers

Question No. 21 22 23 24 25

Answers

USPC - CSAT 19
UPSC - CSAT -1

TIP
Hone your reading comprehension skills by answering these six essential
question after reading articles in the daily newspaper:
¾¾ Who?
¾¾ What?
¾¾ When?
¾¾ Where?
¾¾ Why?
¾¾ How

1.2 Finding the Main Idea


In the first part, we talked about establishing the facts— the who, what, when, where, and how—
we omitted one very important question: Why? Now we are ready to tackle that all - important
question. There’s a motive behind every piece of writing. Atleast there is one behind the passages we
can expect in competitive exams.
All writing is communication. A writer writes to convey his or her thoughts to an audience, the
reader: you. Just as you have something to say (a motive) when you pick up the phone to call someone,
writers have something to say (a motive) when they pick up a pen or pencil to write. The reader might
ask, “Why did the author write this? What idea is he or she trying to convey?”. In essence What
you’re really asking is, “What is the writer’s main idea?” Finding the main idea is much like finding
the motive of the crime. It’s the motive of the crime (the why) that usually determines the other factors
(the who, what, when, where, and how). Similarly, in writing, the main idea also determines the who,
what, when, and where the writer will write about, as well as how he or she will write.
Subject vs. Main Idea
There’s a difference between the subject of a piece of writing and its main idea. To see the
difference, let us look at this passage.
Today’s postal service is more efficient and reliable than ever before. Mail that used to take months
to move by horse and foot now moves around the country in days or hours by truck, train, and plane.
First-class mail usually moves from New Delhi to Thiruvananthapuram in three days or less. If your
letter or package is urgent, the Postal Service offers Priority Mail and Speed/Express Mail services.
Priority Mail is guaranteed to go anywhere in India in two to three days or less. Express Mail will get
your package there overnight.
What is the main idea of the passage?
______________________________________________________________________________
The subject of the passage is who or what the passage is about). The main idea must say something

20 USPC - CSAT
UPSC - CSAT -1
about this subject. The main idea of a text is usually an assertion about the subject. An assertion is a
statement that requires evidence (“proof”) to be accepted as true. Once you know what the subject is,
see if you can determine the main idea. Read the passage again and look for the idea that makes an
assertion about the subject and holds together or controls the whole paragraph.
The main idea of a passage is an assertion about its subject, but it is something more: It is the idea
that also holds together or controls the passage. The other sentences and ideas in the passage will all
relate to that main idea and serve as “evidence” that the assertion is true. You might think of the main
idea as a net that is cast over the other sentences. The main idea must be general enough to hold all
of these ideas together.
Thus, the main idea of a passage is
¾¾ An assertion about the subject.
¾¾ The general idea that controls or holds together the paragraph or passage.
Finding the Main Idea
If you are having trouble identifying the main ideas in a story, try asking yourself these questions:
¾¾ What unifying concept is the author striving to communicate?
¾¾ Is there a moral or lesson that the author is trying to teach?
¾¾ Are there any reoccurring symbols or imagery that the author is using to communicate a deeper
meaning?

Topic Sentences
You’ll notice that in the paragraph about the postal service, the main idea is expressed clearly in
the first sentence: “Today’s postal service is more efficient and reliable than ever before.” A sentence,
such as this one, that clearly expresses the main idea of a paragraph or passage is often called a topic
sentence. In many cases, as in the postal service paragraph, the topic sentence is at the beginning
of the paragraph. You will also frequently find it at the end. Less often, but on occasion, the topic
sentence may be in the middle of the passage. Whatever the case, the topic sentence— like “Today’s
postal service is more efficient and reliable than ever before”— is an assertion, and it needs “proof.”
The proof is found in the facts and ideas that make up the rest of the passage. (Not all passages
provide such a clear topic sentence that states the main idea. Less obvious passages will come up in
later examples.)

Practice in Identifying Topic Sentences


Remember that a topic sentence is a clear statement of the main idea of a passage; it must be
general enough to encompass all the ideas in that passage, and it usually makes an assertion about
the subject of that passage. Knowing all that, you can answer the following question even without
reading a passage.

USPC - CSAT 21
UPSC - CSAT -1
Exercise 1

Which of the following sentences is general enough to be a topic sentence?


(a) The new health club has a great Yoga class.
(b) Many different classes are offered by the health club.
(c) Aerobics is a popular class at the health club.
(d) The yoga class is offered on Saturday mornings.
Exercise 2

Now look at the following paragraph. Underline the sentence that expresses the main idea, and
notice how the other sentences work to support that main idea.
(a) Eshwar always played cops and robbers when he was a boy; now, he’s a police officer.
(b) Suzanne always played school as a little girl; today, she is a high-school math teacher.
(c) Kashmira always played store; today, she owns a chain of retail clothing shops.
(d) Long before they are faced with the question, “What do you want to be when you grow up?”
some lucky people know exactly what they want to do with their lives.
Exercise 3

Among the following eight sentences are two topic sentences. The other sentences are supporting
sentences. Circle the two topic sentences. Then write the numbers of the supporting sentences that go
with each topic sentence.
1. Finally, there is a security guard on duty 24 hours a day.
2. Some duties, like writing reports, have no risk at all.
3. For example, there is a pool on the top floor.
4. Not all police duties are dangerous.
5. Others, like traffic duty, put police officers at very little risk.
6. Tenants of the luxury apartment building enjoy many amenities.
7. Still other duties, like investigating accidents, leave officers free of danger.
8. In addition, the lobby has a dry cleaner, an ATM, and a coffee shop.
You might have noticed that the supporting sentences in the first paragraph about police duties
begin with the following words: some, others, and still other. These words are often used to introduce
examples. The second paragraph uses different words, but they have the same function: for example,
in addition, and finally. If a sentence begins with such a word or phrase, that is a good indication it is
not a topic sentence—because it is providing a specific example.
Here are some words and phrases often used to introduce specific examples:
¾¾ For example

22 USPC - CSAT
UPSC - CSAT -1
¾¾ For instance
¾¾ In addition
¾¾ Furthermore
¾¾ In particular
¾¾ Some
¾¾ Others
If you’re having trouble finding the main idea of a paragraph, you might try eliminating the
sentences that you know contain supporting evidence
Practice Passages - 2

Try your hand at identifying the explicit (or stated) main idea of each of these brief passages.
Remember that there is a difference between the subject of a passage and its main idea. When looking
for the main idea, look for one that best encompasses or sums up the passage as a whole.

Recent history has been about ideologies: communism versus capitalism, fascism versus
democracy. But the end of the cold war has resulted in many subtle challenges throughout the world.
Today, global politics is being reconfigured along cultural lines. Political boundaries are increasingly
redrawn along ethnic and religious lines.
1. The main idea of the passage is that, since the cold war,
(a) In most countries religion and ethnicity have become more important than communism
or capitalism
(b) In countries throughout the world, religion and ethnicity have become more important
than political ideology
(c) In countries throughout the world, political boundaries should be redrawn
(d) In most countries, fascism and communism no longer exist

Lawyer-bashing is on the increase in India. Lawyers are accused of lacking principles, clogging
the justice system, and increasing the cost of liability insurance. Lawyers have received undeserved
criticism. A lawyer is more likely than not to try to dissuade a client from litigation by offering to
arbitrate and settle conflict.
2. The main idea of the paragraph is best expressed in which of the following statements
from the passage?
(a) Lawyer-bashing is on the increase in India.

USPC - CSAT 23
UPSC - CSAT -1
(b) Lawyers have received undeserved criticism.
(c) Lawyers are accused of lacking principles.
(d) A lawyer is more likely than not to try to dissuade a client from litigation by offering to
arbitrate and settle conflict.

Whether or not you can accomplish a specific goal or meet a specific deadline depends first on how
much time you need to get the job done. What should you do when the demands of the job exceed
the time you have available? The best approach is to divide the project into smaller pieces. Different
goals will have to be divided in different ways, but one seemingly unrealistic goal can often be
accomplished by working on several smaller, more reasonable goals.
3. The main idea of the passage is that
(a) Jobs often remain only partially completed because of lack of time
(b) The best way to complete projects is to make sure your goals are achievable
(c) The best way to tackle a large project is to separate it into smaller parts
(d) The best approach to a demanding job is to delegate responsibility

For most judges, sentencing a person who has been convicted of a crime is a difficult decision.
In the majority of jurisdictions throughout the country, judges have few sentencing options from
which to choose. Generally, their options are confined to a fine, probation, or incarceration. Crimes,
however, cover a wide spectrum of criminal behavior and motivation, and a wide variety of sanctions
should be available.
4. The main idea of the paragraph is that
(a) There should be laws that dictate which sentence a judge should hand down
(b) Someone other than a judge should be allowed to sentence a criminal
(c) Judges should be given more sentencing options from which to choose
(d) More money should be spent on the criminal justice system

5
Managing job and family is not simple. Both commitments make strong demands on people and
are sometimes in direct opposition to each other. Saying yes to one means saying no to the other, and
stress can often result. Being realistic and creating a balance in life can help set priorities.

24 USPC - CSAT
UPSC - CSAT -1
5. The main idea of the paragraph is that
(a) Most family responsibilities cause stress at home and at work
(b) Because it pays the bills, a job must take priority over other commitments
(c) It is important to have a balance between job and family responsibilities
(d) Because they are so important, family duties must take priority over the job
6
Generation Xers are those people born roughly between 1965 and 1981. As employees, Generation
Xers tend to be more challenged when they can carry out tasks independently. This makes Generation
Xers the most entrepreneurial generation in history.
6. This paragraph best supports the statement that Generation Xers
(a) Work harder than people from other generations
(b) Have a tendency to be self-directed workers
(c) Tend to work in jobs that require risk-taking behavior
(d) like to challenge their bosses’ work attitudes

7
Electronic mail (E-mail) has been in widespread use for more than a decade. E-mail simplifies
the flow of ideas, connects people from distant offices, eliminates the need for meetings, and often
boosts productivity. But E-mail should be carefully managed to avoid unclear and inappropriate
communication. E-mail messages should be concise and limited to one topic. When complex issues
need to be addressed, phone calls are still best.
7. The main idea of the paragraph is that E-mail
(a) Is not always the easiest way to connect people from distant offices
(b) Has changed considerably since it first began a decade ago
(c) Causes people to be unproductive when it is used incorrectly
(d) Is effective for certain kinds of messages but only if managed wisely
8. Which of the following would be the most appropriate title for the passage?
(a) Appropriate Use of E-Mail
(b) E-Mail’s Popularity
(c) E-Mail: The Ideal Form of Communication
(d) Why Phone Calls Are Better than E-Mail

USPC - CSAT 25
UPSC - CSAT -1
8
Children start out in a world where fantasy and imagination are not substantially different from
experience. But as they get older, they are shocked to discover that the world in which people reliably
exist is the physical world. Computer games and virtual reality are two ways in which children can
come to terms with this dilemma.
9. The main idea of the paragraph is that computer games and virtual reality
(a) can be important tools in children’s lives
(b) keep children from experiencing reality
(c) help children to uncover shocking truths about the world
(d) should take the place of children’s fantasy worlds
9
Native Indian art often incorporates a language of abstract visual symbols. The artist gives a poetic
message to the viewer, communicating the beauty of an idea, either by using religious symbols or a
design from nature such as rain on leaves or sunshine on water. The idea communicated may even be
purely whimsical, in which case the artist might start out with symbols developed from a bird’s tracks
or a child’s toy.
10. The main idea of the passage is that Native Indian art
(a) is purely poetic and dream-like
(b) Is usually abstract, although it can also be poetic and beautiful
(c) Communicates the beauty of ideas through the use of symbols
(d) Is sometimes purely whimsical

10
In criminal cases, the availability of readable fingerprints is often critical in establishing evidence
of a major crime. It is necessary, therefore, to follow proper procedures when taking fingerprints. For
elimination purposes, major case prints should be obtained from all persons who may have touched
areas associated with a crime scene.
11. The main idea of the paragraph is that
(a) Because fingerprints are so important in many cases, it is important to follow the correct
course in taking them.
(b) All fingerprints found at a crime scene should be taken and thoroughly investigated.
(c) If the incorrect procedure is followed in gathering fingerprints, the ones taken may be
useless.
(d) The first step in investigating fingerprints is to eliminate those of non-suspects.

26 USPC - CSAT
UPSC - CSAT -1
12. The paragraph best supports the statement that
(a) no crimes can be solved without readable fingerprints
(b) all persons who have touched an area in a crime scene are suspects
(c) all fingerprints found at a crime scene are used in court as evidence
(d) all persons who have touched a crime-scene area should be fingerprinted

11
Police officers who routinely investigate violent crimes can’t help but become somewhat jaded.
Paradoxically, the victims and witnesses with whom they work closely are often in a highly vulnerable
and emotional state. The emotional fallout from an assault, for example, can be complex and long
lasting. Police officers must be trained to handle people in emotional distress and must be sensitive to
the fact that for the victim the crime is not routine. At the same time, Police officers must recognize
the limits of their role and resist the temptation to act as therapists or social workers, instead referring
victims to the proper agencies.
13. What is the main idea of the passage?
(a) The best Police officers do not become emotionally hardened by their jobs.
(b) Victims of violent crime should be referred to therapists and social workers.
(c) Police officers should be sensitive to the emotional state of victims of violent crime.
(d) Police officers should be particularly careful in dealing with victims of an assault.
14. According to the passage, what is paradoxical about the detective’s relationship to the
victim?
(a) Police officers know less about the experience of violent crime than do victims.
(b) What for the Police officers is routine is a unique and profound experience for the
victim.
(c) Police officers must be sensitive to victims’ needs but can’t be social workers or
psychologists.
(d) Not only must Police officers solve crimes, but they must also handle the victims with
care.
15. Which of the following is NOT advocated by the passage for Police officers who
investigate violent crimes?
(a) They should refer victims to appropriate support services.
(b) They should be aware of the psychological consequences of being victimized.
(c) They should not become jaded.
(d) They should not become too personally involved with victims’ problems.

USPC - CSAT 27
UPSC - CSAT -1
Answers- Practice Passages - 2

Question No. 1 2 3 4 5

Answers

Question No. 6 7 8 9 10

Answers

Question No. 11 12 13 14 15

Answers

TIP
To identify the main idea in a story it can be helpful to create a story map or graphic
organizer. In separate boxes in your graphic organizer, you should include -
¾¾ The names of major and minor characters.
¾¾ Major and minor settings.
¾¾ Conflicts occurring between characters.
¾¾ Major resolutions.
¾¾ Author’s purpose. (Is the author’s goal to entertain, teach inform, or persuade
readers to embrace a particular philosophical view-point or concept?)

Summary
Now you can answer the last question— the why. What is the writer’s motive? What’s the main
idea he or she wants to convey? By finding the sentence that makes an assertion about the subject
of the paragraph and that encompasses the other sentences in the paragraph, you can uncover the
author’s motive.


28 USPC - CSAT
UPSC - CSAT -1

Chapter

2 Elements of RC Skills

WHAT CAN BE DONE TO IMPROVE RC


Most test takers are likely to experience more than one of the following problems as they tackle
Reading Comprehension, at least to some degree:
¾¾ Lack of concentration, perhaps because of being unfamiliar with or un- interested in the topic
¾¾ Poor reading speed
¾¾ The need to go through the passage several times to find the information needed.
¾¾ Difficulty in understanding meanings of words in the context of the passage
¾¾ Difficulty in narrowing down the answer choices to one that’s clearly the most appropriate.
Three main areas of concern emerge for a candidate to work on his/her reading Comprehension:
¾¾ Active reading Skills
¾¾ Speed Reading Skills
¾¾ Vocabulary

2.1 Active Reading


One of the reasons for these problems stem from the habit of what we like to call casual reading.
This means that you simply read the passage from start to finish, casually reading every sentence,
without thinking as to what particular information might be important in answering the questions. The
likely result of this habit, however, is that all you’ll remember are some scattered facts and ideas that
help you respond correctly to only few questions. The casual reading habit won’t help you answer
questions that measure your ability to understand or apply the ideas in the passage. Much of our
reading is neither accurate nor thoughtful but the reading of the Comprehension passages must be
painstakingly careful and must involve attention, retention and awareness. Comprehension passages
in tests need to be effectively read:
¾¾ To gain and understand accurate information and ideas
¾¾ To recognize author’s organization of the content and style of writing.
¾¾ To interpret author’s point of view
¾¾ To analyze and evaluate the textual matter
¾¾ To find out where answers to specific questions lie

USPC - CSAT 29
UPSC - CSAT -1
¾¾ To get the gist of each paragraph
¾¾ To distinguish opinions or interpretations from factual assertions; this is an important skill in
reading
¾¾ To obtain the meaning of words/phrases from context
¾¾ Finally get the author’s purpose of writing the text, to help you answer inferential questions

Further steps for Becoming an Active Reader


Critical reading and thinking skills require active reading. Being an active reader means you have
to engage with the text, both mentally and physically. Many of the guidelines here are applicable to
your general reading as well. Becoming an active reader is being able to able to read all texts better.
Once you develop these good habits, implementing them specifically to Reading Comprehension
passages should not be a difficult task.
Following tasks can be carried out to become an active reader.
¾¾ Skim ahead and Jump back.
¾¾ Mark up the text.
¾¾ Make specific observations about the text.

Skimming Ahead and Jumping Back


Skimming ahead enables you to see what’s coming up in your reading. Go through the text you’re
about to read. Notice how the text is broken down, what the main topics are, and the order in which
they are covered.
Skimming through the text beforehand will prepare you for what you are about to read. It’s a lot
like checking out pitch and the length of the boundaries in a stadium where you are to play a cricket
match. If you know what’s ahead, you know how to pace yourself, so you’re prepared to handle
what’s to come.
When you finish your reading, jump back. Notice both what the author highlighted ( through
inverted commas, use of italics etc.) and what you highlighted while reading. By jumping back, you
help solidify in your mind the ideas and information you just read. You’re reminded of how each idea
fits into the whole, how ideas and information are connected. When you make connections, you’re
much more likely to remember them.

Marking Up the Text


Marking up the text creates a direct physical link between you and the words you’re reading. It forces
you to pay closer attention to the words you read and takes you to a higher level of comprehension.
Use these three strategies to mark up text:
1. Highlight or underline key words and ideas.

30 USPC - CSAT
UPSC - CSAT -1
2. Circle and define any unfamiliar words or phrases ( During preliminary reading only)
3. Record your reactions and questions in the margins.
1. Highlighting or Underlining Key Ideas
When you highlight or underline key words and ideas, you are identifying the most important parts
of the text. There’s an important skill at work here: You can’t highlight or underline everything, so
you have to distinguish between the facts and ideas that are most important (major ideas) and those
facts and ideas that are helpful but not so important (minor or supporting ideas).
Highlight only the major ideas, so you don’t end up with a text that’s completely highlighted. An
effectively highlighted text will make for an easy and fruitful review. When you jump back, you’ll be
quickly reminded of the ideas that are most important to remember. Highlighting or underlining major
points as you read also allows you to retain more information from the text.
2. Circling Unfamiliar Words (During preliminary reading only)
One of the most important habits to develop is that of circling and looking up unfamiliar words and
phrases. If possible, don’t sit down to read without a dictionary by your side. It is not uncommon for
the meaning of an entire sentence to hinge on the meaning of a single word or phrase, and if you don’t
know what that word or phrase means, you won’t understand the sentence. Besides, this habit enables
you to quickly and steadily expand your vocabulary, so you’ll be a more confident reader and speaker.
If you don’t have a dictionary readily available, try to determine the meaning of the word as best you
can from its context—that is, the words and ideas around it (more about this later). Then, make sure
you look up the word as soon as possible so you’re sure of its meaning.
3. Making Notes in the margins
Recording your questions and reactions in the margins turns you from a passive receiver of
information into an active participant in a dialogue. (If you’re reading a book, where writing directly
may not be possible, write your reactions in a notebook.) You will get much more out of the ideas and
information you read about if you create a ‘conversation’ with the writer.

Making Specific Observations


Good readers know that writers use many different strategies to express their ideas. Even if you
know very little about those strategies, you can make useful observations about what you read to
better understand and remember the author’s ideas. You can notice, for example, the author’s choice
of words; the structure of the sentences and paragraphs; any repetition of words or ideas; important
details about people, places, and things; and so on. This step—making observations—is essential
because your observations (what you notice) lead you to logical inferences about what you read.
Inferences are conclusions based on reason, fact, or evidence. You are constantly making inferences
based on your observations, even when you’re not reading. For example, if you notice that the sky is
full of dark, heavy clouds, you might infer that it is going to rain; if you notice that your friend has a

USPC - CSAT 31
UPSC - CSAT -1
lot of books on cricket on her/his desk, you might infer that he likes cricket.
If you misunderstand what you read, it is often because you haven’t looked closely enough at the
text.
As a result, you base your inferences on your own ideas and experiences, not on what’s actually
written in the text. You end up forcing your own ideas on the author (rather than listening to what
the author has to say) and then forming your own ideas about it. It’s critical, then, that you begin to
really pay attention to what writers say and how they say it. If any of this sounds confusing now, don’t
worry. Each of these ideas will be thoroughly explained in the lessons that follow. In the meantime,
start practicing active reading as best you can.

2.2 Speed Reading


Reading effectively is reading with both comprehension and speed. An efficient reader reads
through units, not word by word. Your aim should be to reduce the number of fixations, and lengthen
the eye span. Your reading rate will increase as you learn to do this efficiently and so will your
comprehension. A skilful reader does not work with isolated units but with context – what precedes
and follows the particular material being read. A good reader rarely loses time by having to refer to
the beginning of a sentence or paragraph. Rather, the thought will have been carried through in one
series of lengthened glances.
The best way of learning to read with speed as well as comprehension is to ‘read with your mind
assisted by your eyes’. Doing this will enhance comprehension by reducing the number of fixations
and increasing concentration. Learn to find key words and phrases that summate the main ideas of the
passage. These steps will greatly increase reading speed, although you must not forget that different
kinds of material require different reading speeds.
Six rules for faster comprehension
One of the important goals of your training is to transform your potential speed into a normal,
habitual speed. To achieve this goal, you should constantly and repeatedly follow—these important
rules for improving your reading:

1. Read More
You will have to read much, much more than you are now in the habit of reading. If you’re a slow
reader, for a major part of your life, it is very likely that you have done little more than go through the
daily papers and a few magazines. You have read only when you happen to have a few spare minutes,
you read merely to pass time. Or perhaps you hardly ever read at all unless you absolutely have to.
From now on, you must make time for reading. Speed can be developed into a permanent habit
only if you do what naturally fast and skillful readers have always done, from childhood : read a lot.
Unless you develop the habit of reading for two hours or more at a stretch, several stretches every
week, do not expect ever to become an efficient or a rapid reader.

32 USPC - CSAT
UPSC - CSAT -1
2. Learn To Read For Main Ideas
Stop wasting time and effort on details. When you read an article, push through efficiently for a
quick recognition of the main idea that the details support and illustrate; be more interested in the
writer’s basic thinking than in his minor points.
When you read a volume of nonfiction, be intent on getting the theme, the broad ideas, the
framework on which the author has built the book. Don’t let an occasionally perplexing paragraph,
page, or chapter slow you up. Keep speeding through. As the complete picture is filled in by rapid
overall reading, the few puzzling details will either turn out to have been inconsequential or will be
cleared up as you move along.

3. Challenge Your Comprehension


Fast readers are good readers. They’re fast because they have learned to understand print quickly,
and they understand quickly because they give themselves constant practice in understanding. To
this end, they read challenging material; and you must do the same. Does a book of nonfiction seem
difficult? Does an article in a magazine look as if it will require more thinking than you feel prepared
to do? Then that’s the type of reading that will give you the most valuable training.
You will never become a better reader by limiting yourself to easy reading—you cannot grow
intellectually by pampering yourself. Ask yourself: “Do I know more about myself and the rest of the
world, as a result of my reading, than I did five years ago?” If your honest answer is no, then you’d
better get started, today, on a more challenging type of reading than you’ve been accustomed to in the
past. This perhaps goes without saying that unless you develop studying at lengths any competitive
examination will prove a tough nut to crack.

4. Budget Your Time


Say to yourself: “I have five chapters in sociology, history, geography (or whatever) to read by
next week.”
And then give yourself a limited, specific time in which to complete the assignment; Set the exact
time when you will start and be determined to finish by a fixed time.
You may be amazed to discover that setting a specific time and a definite (but reasonable) time
limit will mobilize your concentration and will be one small, but important, step in improving your
comprehension.
You’ll speed up, if you force yourself to. You’ll develop tricks of getting ahead, of skimming parts
that are less essential, of looking for main ideas, of reading at your top potential rate.
Good readers always have a feeling of going fast, for they have developed fast habits. Indeed,
adults and college students who have trained themselves to read rapidly would find their original slow
pace uncomfortable and unpleasant.

USPC - CSAT 33
UPSC - CSAT -1
While you are training, give yourself a time limit on whatever you read—and live up to that time
limit. In this way you will mobilize yourself for reading as an intellectual pursuit, and only in this way
will you train yourself to understand at your highest potential rate

5. Pace Yourself
When you start a new book, magazine read for quick understanding for 15 minutes. Count the
number of pages you’ve finished in that time, multiply by 4, and you have your potential speed for
that book/ magazine in pages per hour. (Of course, some books are slower reading than others—it
takes more time to cover 50 pages in an academic text than in a light novel. The more solidly packed
the ideas are on a page, the more time it will take to cover that page. But throughout a given book, all
the material will likely be on the same level.)
Keep to the rate you‘ve set for yourself in pages per hour. In this way, you will learn to devise
personal tricks that will speed you up and that will, at the same time, sharpen your comprehension
skill. But you must practice every day, or nearly every day, if you wish to make high speed natural
and automatic, if you wish to become efficient in rapid comprehension.

6. Develop Habits Of Immediate Concentration


Nothing makes concentration so easy, so immediate, as the technique of sweeping through material
purposefully looking for main ideas and broad concepts.
All people of normal intelligence can concentrate when they read, but slow readers put themselves
at a disadvantage by getting distracted. If, through laziness, you read at a slower rate than the rate at
which you are able to comprehend, there is great temptation for your mind to wander.
Even the brightest students become bored if the work is too easy for them. They think of more
interesting things, they day­dream, they stop paying attention. This analogy explains why a slow reader
picks up a book or magazine, goes through a few pages, and finding that attention is wandering, puts
it down and turns to something else.
By reading always at your top comprehension speed, you constantly challenge your understanding,
you stimulate your mind, you get involved in the author’s thoughts. As an added dividend, you soon
find that the increased concentration you get from fast, aggressive reading sharpens your understanding
and enjoyment, for every distracting thought is pushed out of your mind.
But reading about the principles of efficient and rapid comprehension is not going to make you a
faster or better reader. Only putting those principles into practice, over a period of time, can do that
for you.
How long will it take? That depends on what sort of person you are and how assiduously you
apply yourself under prime conditions, habits of speed and aggressive comprehension can become
automatic after a few months of daily, or almost daily, practice. When, as a result of your training, you
find yourself not only doing much more reading than ever before, but also getting much more out of

34 USPC - CSAT
UPSC - CSAT -1
your reading, you will agree that it was time and effort well spent.
The important thing is that you now know, from actual self-testing, that you have the ability to read
faster than you generally do.
Training can help you to capitalize on this ability, will help you make habitual and normal the rapid
rate, the quick and self-assured grasp of main ideas, that characterize the efficient reader. Exercise
by exercise, drill by drill, selection after selection, you can learn to eliminate the faulty habits and
inefficient techniques that interfere with total concentration, that slow up your comprehension, that
keep your rate of reading down to a much lower level than you are potentially capable of achieving.
Inefficient readers do not quite trust the adequacy or their comprehension. They read meticulously,
digesting and re-digesting every sentence, every paragraph. Paradoxically enough, not only their
speed suffers as a result of such extreme care, but their comprehension also, for they get so involved
in details and relatively unimportant minor points that they often miss the main theme of the writing.
They are an excellent example of not seeing the forest because of the thickness of the trees. If they
are reading a book, they may struggle mightily to master every page, down to the last comma and
semicolon, and may nevertheless fail to understand the overall ideas and implications of a chapter. In
short, inefficient readers strive too hard to be perfect in their grasp of every word, every phrase, every
detail, instead of pushing through swiftly to follow the basic concepts.
Therefore, they often regress. Having no confidence in their comprehension, they go back to check
on figures, minor points, statistics, bits of description whose only purpose is to lend atmosphere—and
the regressions cut their train of thought, make them over-conscious of words, ruin their concentration,
break the smoothness of their absorption of ideas, and, of course, radically reduce their speed. They
have never trained themselves to plow straight ahead as fast as their understanding makes possible.
They have simply never learned to develop the habit of moving along rapidly, of reading with “a
sense of urgency.”
There are additional factors that are relevant to reading efficiency:
Efficient readers have so large a vocabulary that the words they meet are quick conveyors of
thought. The vocabulary of inefficient readers, on the other hand, is so limited that many of the
words they encounter represent a mystery to be puzzled out before ideas can be fully grasped and
appreciated.
Efficient readers have already read so much that they can constantly compare and contrast their
present reading with their previous reading experiences; they have a background on which to build.
Inefficient readers too often have to approach every little bit of reading as a new and unrelated
experience.
Efficient readers have developed strong intellectual curiosity; and all the reading they do helps
in some measure to satisfy that curiosity. The inefficient reader’s intellectual curiosity has gradually
grown weaker because reading has never been a sufficiently comfortable or rapid process to make the
satisfaction of their curiosity worth the effort.

USPC - CSAT 35
UPSC - CSAT -1
If you suspect that you are not normally as efficient, as rapid, or as responsive a reader as you would
like to be, let me tell you this—without qualification. The good habits needed for fast and skillful
reading can be developed in a comparatively short time. You can train the speed and accuracy of your
visual perception. You can learn to attack material with the kind of aggressiveness that will sharpen
your concentration and increase your rate of comprehension. You can learn to eliminate regressions,
to by-pass your vocal apparatus to decrease an over-dependency on “hearing” the words you read.
You can learn to stop paying unnecessary attention to minor details. You can, with the proper practice
and guidance, learn to plow ahead reading always with “a sense of urgency,” speedily absorbing the
main ideas, getting the overall picture. You can start building your vocabulary and stimulating your
intellectual curiosity. And as a result, you will, in all likelihood, make tremendous gains in speed.
Permanent, habitual gains that will come from radically improved habits and techniques of reading.
This is not as hard as it may at first sound—it is not a bit hard if you will make an honest attempt
to alter some of the comfortable and probably inefficient reading patterns you have grown too used
to, that you have become too fond of, that it may be unpleasant, even painful, to give up.
Perhaps you have got into the habit of sauntering leisurely, too leisurely, when it’s just as easy to
run. Perhaps you get bogged down in details instead of concentrating purposefully on finding and
understanding the main idea. Perhaps you think that every word has to be chewed and digested before
you can go on to the next one, that every sentence has to be mulled over, that every thought has to be
studied before you can really understand it; believe me, this is not so. All the words, all the sentences,
all the thoughts in any selection add up to a final point, a final effect, a dominant and central idea.
Most of the times, your job is done if we understand the central idea. If you have any success in
putting these instructions into practice, both your overall understanding and your increase.

2.3 Vocabulary
ROOTS - THE BASIC BUILDING BLOCKS OF WORDS
Vocabulary is knowledge of word meanings. The simplicity of this definition does not quite convey
what it means to “know” a word. For example, Merriam-Webster’s Collegiate Dictionary lists 18
definitions (several of them with sub-definitions) for the word place. Although we rarely stop to think
about it, the issue of knowing words is complex. For example, each of us has an active vocabulary
and a passive vocabulary. An active vocabulary includes words we can quickly generate for speaking
or writing because we know them well. We can recognize words in our passive vocabulary when we
encounter them, but we don’t regularly use them. For instance in Pandit Nehru’s famous ‘Tryst with
destiny’ speech, chances are we understood what he meant by “tryst” because we have seen it in
other contexts. Yet, when is the last time you used this word in a sentence? Any goal of vocabulary
improvement must be to increase both active and passive vocabularies.
Fortunately, well over half of English words—nearly 75 percent according to some estimates—are

36 USPC - CSAT
UPSC - CSAT -1
derived from Greek or Latin. So a focus on these word parts makes sense as part of a vocabulary
improvement. Latin and Greek prefixes, bases, and suffixes are fairly consistent in their meanings
and spelling patterns. Consequently, you can figure out the pronunciation and meaning of many new
words by looking at their roots. You can understand the logic in the spelling pattern. A student who
knows that the root spec means “look,” for example, has a head start in figuring out what speculate,
spectacular, and spectacle mean when encountering them in a text. The student can then use context
to determine whether the spectacle in question is a “big event” or, when used in the plural form, a
“pair of glasses.” This clear link among pronunciation, meaning, and spelling is especially useful
when encountering new and challenging words.
Using roots to unlock word meanings will do more than expand your vocabularies. Each word
built from roots has taken a unique path into our language. Did you know, for example, that the
words vocabulary and vowel come from the Latin root voc, which means “voice”? In ancient Rome,
students were required to recite lists of new words orally, or using their “voices.” And, of course, we
need our “voices” to say “a, e, i, o, u.” Studying word roots can potentially give you a glimpse of word
histories. It will help you grasp an essential linguistic principle: English words have a discernible
logic because their meanings are historically grounded. This knowledge, used in conjunction with
word analysis skills, can empower you as learners.

What Is a Root?
Words, like stories, have a structure. We all know that most good stories have a beginning, middle,
and end, and that these parts of stories connect with one another. Each part has a purpose and advances
the overall story line. Likewise, many English words are made up of parts. Like the parts of a story,
the parts of a word also carry meaning. And this is precisely what a ‘root’ is: a part of a word that
carries meaning. A word is composed, of course, of letters. But letters, by themselves, carry only
sound, not meaning. The letter r, for example, has no meaning by itself. It’s a sound, nothing more
and nothing less. Letters, then, are word parts, but they are not roots because they have no meaning.
When a root appears inside a word, it lends its meaning to the word and thus helps create the word’s
meaning. Moreover, words that contain the same root also share meaning. We call these related words
cognates (from Latin cognatus, which means “born together, related in origin”). A root conveys not
only sound but also meaning.
One of the most commonly encountered roots in English words, for example, is mot. The root mot
is not a word in itself, but it means something; This particular root means “move.” Nearly any time it
appears in a word, that word will have a meaning associated with movement. Take a minute to think
of mot words that have to do with movement
¾¾ A motor makes things move.
¾¾ Some outdoor lights are triggered by motion detectors.
¾¾ Some classrooms have a lot of commotion.
¾¾ We all were promoted last year.

USPC - CSAT 37
UPSC - CSAT -1
¾¾ Who lost the remote control to the television?
¾¾ A motif in a novel is a theme that “moves” around the story

The Three Kinds of Roots


Many English words can also be divided into three parts:
¾¾ the prefix
¾¾ the base
¾¾ the suffix
Prefixes, bases, and suffixes are the three kinds of roots, found in many words. Root is the generic
term for any part of a word that holds meaning Prefixes, bases, and suffixes are kinds of roots. In
terms of a word’s structure, the prefix appears at the beginning of a word, the base in the middle, and
the suffix at the end. The base holds up the entire word, providing its essential or “basic” meaning.
What Does a Base Do?
The root mot is a base that means “move.” Here are a few more examples of bases:
¾¾ tract = “pull, draw, drag”  duc, duct = “lead”
 port = “carry”  ven, vent = “come”
¾¾ dic, dict = “speak”  voc, vok = “call”
¾¾ viv, vit = “life”  sol, helio = “sun”
¾¾ dom = “house”  terra = “land”
¾¾ aqua, hydro = “water”
These bases generate entire words that carry their basic meanings:
¾¾ A motor makes things “move.”
¾¾ A tractor “pulls” farm equipment.
¾¾ We need good traction to “pull” a car through sand or snow.
¾¾ When we trace something, we make a “drawing.”
¾¾ A duct “leads” heated air from a Air conditioner to the rooms of the house.
¾¾ A portable television can be “carried” from one room to another.
Notice that each of the italicized words in the above list has a meaning directly associated with the
meaning of the base itself. In fact, the base duct can even function as a whole word all by itself! And
as we noted earlier, these bases carry fundamental, easily understood meanings: move, pull or drag,
lead, and carry.
You may have noticed that none of the words in this particular list begins with a prefix. The first
unit in these words is the base itself.

What Does a Prefix Do?


When a prefix is attached to the base of a word, the prefix does one of three things: it gives a word

38 USPC - CSAT
UPSC - CSAT -1
direction, negates a word by meaning “not,” or intensifies the meaning of a word by adding the notion
of “very.”
Most of the prefixes found in English words—about 25 in all—are derived from Latin. These few
prefixes are used in a large number of words. For example, every theatre/ auditorium has exit signs
that point to the way “out of” the building. You can use this everyday word to teach that the prefix
ex- sometimes means the direction “out, out of.” Ex- can have an intensifying meaning as well. For
example, an exhausted person is “very” drained (haust = “drain”). Another example of an intensifying
prefix is the per- in perfection, “made well” (fect = “make, do”). Many students will have seen a band
or orchestra conductor wave a baton, leading musicians “together” as they play their instruments. The
prefix con- is a directional prefix meaning “with, together.” When students redo an assignment, they
must go “back” and do it “again.” Thus re- is a directional prefix meaning “back, again.” A student
who is unable to come to a birthday party is “not” able to do so: un- is a negative prefix. Likewise, in-,
im-, and il- are negating prefixes. Consider words like invisible, impossible, and illegal. Most of the
prefixes you encounter in school texts are directional in nature. This means that they indicate a path
of some kind: “with, together,” “under,” “in,” “out,” “back, again,” “away from,” “out of.” Here are
a few examples of the most common directional prefixes:
 at-, ad- = “to, toward, add to”  de- = “down, off”
 dis- = “apart, in different directions”  con- = “with, together”
 ex- = “out”  pro- = “forward, ahead”
 sub- = “under, below”  in- = “in”
Once you have learned the directional meanings of these prefixes, you can generate a large number
of words from even a single base. Here are some tract (“pull, draw, drag”) words with directional
prefixes:
¾¾ Previews of coming attractions “draw” us “to” the theater.
¾¾ The ugly building detracted (“pulled down”) from the beauty of the neighborhood.
¾¾ Noises in a school hallway are distracting because they “draw” our attention “away” from the
lesson.
¾¾ Muscles contract when they “pull together” and tighten.
¾¾ A dentist extracts a bad tooth by “pulling” it “out.”
¾¾ A protracted war is one that is “drawn forward.”
¾¾ When we subtract, we “draw” the number “below” its amount by taking away from it.
All of the italicized words above are cognates derived from a single base, tract. Each of these
words, furthermore, begins with a directional prefix that indicates the direction of the “pulling,
drawing, or dragging.”

What Does a Suffix Do?


If a word has a suffix, it comes at the end of a word. Unlike prefixes and bases, which have fixed

USPC - CSAT 39
UPSC - CSAT -1
and stable meanings, suffixes have meanings that are fluid. The essential function of a suffix is to
indicate the part of speech of a particular word. By the time you have reflected on the prefix and the
base, many times you usually understand the word well enough to figure it out. For these reasons,
only a few suffixes merit intensive scrutiny. Words of Greek origin, for example, tend to be long and
often carry technical meanings. Thus, it can be useful to learn that the suffix -ology means “study of.”
Then you will be able to think about the base meaning in such words as:
 Geology is the “study of” the earth.  Theology is the “study of” God.
 Hematology is the “study of” blood.  Anthropology is the “study of” human cultures.
Here are two more Latin bases and their meanings:
 audi, audit = “hear, listen”  vid, vis = “see”
The meaning of many words built on such bases is often immediately clear. An audible sound,
for example, is one that “can” be “heard.” A visible image “can” be “seen.” An inaudible voice “can
not” be “heard,” just as an invisible force “can not” be “seen.” An auditorium is a large room for
“listening” to speakers or performers. This is why auditoriums are designed for acoustics (acous is the
Greek base for “hear”). The audio portion of a TV program is the part we “hear” (as opposed to the
video portion, which we “see”). We only “listen” to an audio cd, while we can “see” the images on a
video. When we audition for a school play, we must speak a part, sing a song, or play an instrument
for the judges to “hear” or “listen” to. A vista offers a panoramic “view” and enables us to “see” large
expanses of scenery. We wear a sun visor to shade our eyes so that we can “see” things in the glaring
sunlight. In words such as these, the straightforward meaning of the base leads directly to the meaning
of the entire word. Students will quickly find the basic idea of “hearing” or “seeing” as readily evident
in these words (e- = intensifying prefix; things that are evident are “very” easy to “see”).

Figurative Meaning
Let us consider the word supervisor and talk about it in terms of seeing (prefix super- = “over”). A
supervisor is someone who “oversees” someone else’s work. We may speak of our supervisor at work
as hovering over us and watching our every move. The basic meaning of “seeing” forms the core of
this word. A supervisor takes a close “look” at our work and inspects it for accuracy. We can find the
same basic idea of “seeing” in the words provide and provisions. We use these words in such contexts
as “providing for our children,” “providing for a rainy day,” “buying weekly provisions at the grocery
store,” and so on. When we shop for our provisions, we are “seeing ahead” (pro- = “forward, ahead”)
to what we will need for the coming week. When we provide for our children, we are “seeing ahead”
to their future needs. We are envisioning the future, trying to form a mental picture of what they will
need. The basic sense of “seeing” in all these words is not literal, but figurative. If you know that the
base of the word revolution is volut, meaning “roll, turn,” you can work with the base meaning and
arrive at a correct understanding of the word in each context whether we are speaking of the American
Revolution or the revolution of the moon around Earth.
Let’s think about the literal and figurative meanings of bases with another example. The Latin

40 USPC - CSAT
UPSC - CSAT -1
bases cur(r), curs, and cour(s) mean “run.” In many words, this base refers to physical “running,” or
something close to it: a courier service “runs” letters and packages with runners who either run on
foot, ride bikes, or drive delivery trucks. The idea of “running, speed, rapid movement” is evident
in this word. We may also think of a racecourse on which athletes, cars, or horses physically “run.”
What about writing in cursive script? There is no physical running in writing cursive. In this word,
the “running” occurs on a figurative level. When we write in cursive, our letters “run” together with
ligatures, and we can write more quickly than when we merely print in block letters. As we think
about figurative running, we observe that the cursor on our computer screen “runs” across the monitor
screen as we move our mouse. The current of a river has “running water.” Reflect on this phrase for a
moment. If we thought in merely literal terms, the phrase “running water” would strike us as absurd.
Water does not run—it flows; it spills; it drips. But it does not literally run. It figuratively runs. The
figurative meaning also pertains when we speak of current events, of films currently showing at the
theater. These events and films are now “running,” as we say. We employ figurative language in our
daily speech, and this can lead us to a roots-level understanding of many words.
Here are a few more “running” words that employ figurative language. We speak of cash as currency
because we look on money as flowing, fleeting, running—similar to the current of water. Compare
phrases like “cash flow,” “liquidated assets,” and “frozen assets.” We incur debts and “run up” our
bills. When we concur with others on an issue, we agree with them because we go or “run together
with” them. When we take an excursion, we are usually riding “out,” not “running out.” Nevertheless,
the basic figurative use of “running” applies. Likewise, one country may conduct a military incursion
of another, an aggressive “running into” someone else’s territory. The invading soldiers may arrive
in tanks or by parachute without actually running on foot. In a relatively new application of the word
incursion, we hear of runway incursions at airports with heavy traffic. Airplanes have near misses and
almost “run” “into” one another as they land and take off.
The roots approach to vocabulary learning and teaching is important because it mirrors the very
process through which we, as users of language, arrive at an understanding of new ideas and concepts.

Practicing with Roots


You can adhere to the following procedure for working with roots.
Step 1: Divide the word into what to you seems the best combination of a base/prefix and suffix
and understand their individual meanings.
Step 2: Combine the different parts now to form a comprehensible, coherent meaning.
Step 3: Verify the meaning whenever possible through context or by referring to a good dictionary.
The list below contains useful bases, prefixes and suffixes. Along with these are given their
meanings and some helping words. Going through this list will familiarize you with some important
roots and will help you to realize that many of them are actually familiar or are forms of other roots
that are familiar.

USPC - CSAT 41
UPSC - CSAT -1

Some Often Used Bases


A misanthrope (one who hates humans)
act, ag ig - to act aper, over - open
e.g transact (carry through, conduct business ) e.g. aperture (an opening) overt (not secret or
intransigent (stubbon, i,e refusing to do) hidden)
agr - land field apt, ept - fit
e.g. agrarian (relating to farms or farming. e.g. adaptation (a change which makes an
agrarian animal
economy ), agronomy (science of soil etc. fit for an environment ); maladaptation (
management and crop production) makes one unfit), inept (unskillful or lacking in
alt - high judgment ; be unfit for the task)
e.g. altar (a raised platform for offering aqu - water
sacrifices), altitide (height), exalt (raise to a e.g. aquarium (a fish tank), aqueous (of or
higher position , praise) relating to water e.g. an aqueous solution is
alter - other one made using water as the solvent), aquifer
e.g. alter (to make something ‘other’ or different (a layer of rock bearing water)

from what it was ) alter ego (other self, a close arch - first, chief, rule, govern
friend or inseparable companion) alternate (a e.g. arch-enemy, architect; monarch,
pattern having first one and then the other) aster - star
am - friend , love e.g.asterisk (a star - shaped mark ), disaster
e.g. amicable (friendly) , amity (friendship) , (from the idea that the disaster was caused by
anim - life soul, mind ‘bad’ stars), astronaut (literally a star- sailor)

e.g. animal (a thing which has life ), animate aud - hear


(living; opposed to inanimate), animosity/ e.g. audience (‘the king granted me an audience’
animus (dislike) i. e. a hearing), audit (this comes from the idea
ann, enn - year e.g. that accounts were originally heard, not read as
they are today)
annals (historical records of the main events of
every year), biannual (twice a year ), biennial B
(once in two years), perennial (permanent or bas - low
through the year ) e.g. abase (to lower or disgrace oneself),
anthrop - man baseness (the quality of being low in birth, status,
e.g. anthropic (relating to humans), or qualities of mind; cowardly, contemptible;
anthropomorphic (having or representing a the opposite of nobility)
human shape - morph relates to form of shape),

42 USPC - CSAT
UPSC - CSAT -1
bell - war capit - head
e.g. belligerent (ready to fight, warlike), rebel e.g. captain (the head of a ship), capitulate
(one who makes war against an established (surrender- the idea is connected with the
authority) bowed head of the loser)
bell - beautiful carn - flesh
e.g. belle (a beautiful woman), embellish (to e.g. carnival (originally a festival that was an
decorate or make more beautiful) indulgence in fleshly desires, which involved a
ben - good, well 40
e.g. benefit (something that is good because day fast), incarnation (a god in human form i.e.
it provides an advantage), benevolent (kindly, one who is ‘in the flesh,)
wishing to do good) ced, cess - yield, go
bibl - book e.g. cede (to surrender territory especially after
e.g. bible (the bible is actually a collection of losing a battle), intercede (literally, this means
books - 66 in all), bibliophile (a book lover) to go in between; the actual meaning is to plead
brev - short; on behalf of someone), recess (a niche, or a
hollow space in a wall - the wall goes back at
e.g. brevity (briefness), abbreviation
that point) recession (a slump in the market-it is
(shortform).
when the economy goes down)
C
chron - time;
cad, cas, cid - fall;
e.g. anachronism (something exists or is
e.g. cadaver (a dead body; the dead are referred
mentioned as existing out of its appropriate
to as ‘the fallen’), cadence (the rises and falls
historical time frame e.g. a bullock cart in the
in the tone of speech), decay (rotting or going
age of space travel seems like an anachronism),
into a lower or degraded condition), incident
chronicle (a detailed and continuous record
(something that happens or /falls’ e.g. Sankrant
of events over time), crony (a close friend or
falls on the 14th of January this year)
associate - someone who you spend time with)
cand - white, shining;
cid, cis - cut, kill;
e.g candid (frank- a white background helps to
e.g. incise (to make a cut), precise (the idea
see things as they are), candidate (in ancient
is that in order to be precise something is cut
Rome candidates used to appear dressed in
short); pesticide, homicide (murder), genocide
white robes),
(the killing of a race of people)
incandescent (glowing or shining with heat -
clin - lean, bend
light
e.g. decline (to sink to a lower status or
bulbs are incandescent)
position, ‘declining standards’ or ‘the patient is
cap, capt - take, hold declining” - originally the idea was of bending
e.g. capacity (how much a thing can hold), down); recline (lie - or bend - back)
capture (to catch hold of) clos, clois, clud, clus - close, shut

USPC - CSAT 43
UPSC - CSAT -1
e.g. cloister (a covered walk around the computer screen), precursor (a forerunner),
courtyard of a vent, monastery etc.; a tranquil incursion (an invasion - lots of people running
secluded place), occlude (block - when a in)
passage is closed it gets blocked), seclude D
(separate or cut off from the outside) dem - people
cogn - know e.g. demagogue (a political agitator, rabble-
e.g. cognition (the ability or act of knowing a rouser - literally a leader of the people; agogos
sensation, perception or idea), cognizable (an means leader), pandemic (a plague which has
offence that can be tried by a court an offence spread across the continents, so called because
the court can take cognizance (notice) of), it affects all peoples - ‘pan’ means ‘all’)
recognize die, dict - speak, say
cord - heart e.g. edict (a command, or decree - which you
e.g. concord (agreement- the hearts are have to ‘say out’- ‘e’ means ‘out’), predict (say
together), discord (disagreement - the hearts before hand - ‘pre’ means ‘before’), benediction
are apart) (a blessing - literally it means ‘saying out
corp - body good’), malediction (a curse -literally it means
e.g. corporation (e.g. the Municipal Corporation ‘saying out evil’)
is a body that governs the working of the city), domin - lord, master;
corpuscle (a small body like a red blood cell), e.g.domineering (bossy -behaving like a master
incorporate (literally, to make part of the body). to a slave), dominion (kingdom or the land
cosm - world of which someone is master), indomitable
e.g. cosmos (the universe - the larger world we (unconquerable - literally cannot be mastered)
live in), microcosm (a miniature representation due, duct - lead
(of the world) ‘a marathon is a microcosm of e.g. abduct (kidnap - originally it meant to
life’ or ‘his street was his microcosm, a universe lead away, which is still part of the meaning),
from which he never strayed’) conductor (that which leads things like
cred - believe electricity along or with itself), aqueduct (a
e.g. credit (given only if the shopkeeper channel for water - water is lead along it),
believes in you), credibility (how believable duct (a passage throught which something can
people think you are), incredulous (astonished, be lead), produce (this literally means to lead
unable to believe), creed credo (beliefs - the forth),
Indian Constitution guarantees we will not be E
discriminated against on grounds of race, cast equ - equal
and creed) e.g. equidistant (at equal distance from),
curr, curs - run equivocal (speaking ambiguously, with the
e.g. currency (money - it runs or circulates in intention of misleading - it literally means
society), cursor (the thing that runs across your ‘saying two things equally’, the idea being that

44 USPC - CSAT
UPSC - CSAT -1
that creates confusion), inequity (unfairness - forte (a strong point, something one is good at)
from the idea that things are unequal) fract, frag, frang, fring - break
F e.g. fracture(The result of breaking, esp. of a
fac, fect, fic - make, do bone),
e.g. facile (do-able or easy to do), facility fragile (something that can break easily),
(something which helps you to do), manufacture frangible (able to be broken), infringe (violate
(make), effective (capable of doing), fictive or break esp. an oath or a pledge)
(made up), prolific (fertile) fug, fugi, fugal, fuge - flee
fid - faith, trust e.g. fugitive (one who flees from justice or
e.g. confident (having faith in oneself), diffident danger),
(unconfident, not having faith in oneself), refugee (a person who flees from his homeland
fidelity (faithfulness), infidel (an unbeliever, and seeks shelter esp. in a foreign country),
one not from the faith) centrifugal (moving or directed away from
fin - end an axis or center), subterfuge (a statement or
e.g. affinity (liking or similarity - the Latin word action resorted to in order to avoid blame or
‘finis’ means border and the idea of this word is defeat; an evasion)
of two things which are close together or linked fus, found, fuse - pour
though they are separated by a border), define e.g. fusion (union of different things as a
(from the idea of determining) the border or whole;pouring together, confound (confuse;
boundary of, infinitude (the quality of being mix up, throw/pour into disorder), diffuse
infinite), infinitesimal (very small in amount, (pour or spread out; disseminate)
nearly zero) G
flet, flex - bend gen - to produce; kind, race, birth
e.g. deflect (bend or turn aside e.g. deflect e.g. generate (bring into existence), generic
a blow), flex (the muscles), genuflect (bend (not specific, applicable to a genus or a class),
the knee(s) down to the ground in an act of generation (production; the offspring of the
worship), inflection (a change in the tone or same parent or parents regarded as a single
pitch of a voice), inflexible (stubborn), reflect step or stage in descent)
form - shape gen - spirit
e.g.non-conformist (one who rebels against e.g. genial (cordial and kindly, of cheerful
conventional rules and regulations -literally spirits),
one who is not the same shape as others),
genius (special intellectual and creative spirit)
reform (to get rid of faults or flaws)
gest - carry, bring, give birth to
fort - strong
e.g. congestion (overfill, bring together),
e.g. comfort (originally this meant to come
gestation (conception and development of a
alongside and give strength to someone), effort
plan an idea; pregnancy)
(it requires you to bring out your strength) fort,

USPC - CSAT 45
UPSC - CSAT -1
grad, gress, grade - step, go to another turn one language into another)
e.g. graduate (one who has taken a degree; to leg, lig, lect - choose, gather, read
arrange in steps or degrees), progress (move/ e.g. legible (able to be read), dilligent (marked
step forward), retrograde (moving back) by painstaking effort), elect (to pick out by
graph, graphy - write voting)
e.g. autograph (written with one’s own hand), lib, libert - free
graphic (relating to writing, vividly described) e.g. liberal (not limited to traditional attitudes),
grat, grac - favor; pleasing, favorable liberation, (freedom from some kind of
e.g. gratitude(thankfulness) , gratuity (gift of oppression)
money in return for services, a tip), congratulate loc, lieu - place
(felicitate), e.g. dislocate (put or move out of place), local
grace (charm, refinement of movement,action (pertaining to a particular place), in lieu of (in
or expression; favor, privilege) place of), milieu (environment, place or setting)
I logue, logy - word, study, proportion or
integr - entire, whole reasoning
eg. integrate (to make whole by bringing parts e.g. logic (study of the principles of reasoning),
together), integrity (completeness; complete prologue (preface or introduction to a literary
adherence to a certain moral code) or musical work), psychology (study of the
J nature and functioning of the mind)

jac, ject, jet - throw loqu, loquy, locut - speak, talk

e.g. ejaculate (throw out; to exclaim aloud), e.g. loquacious (talkative), soliloquy(talking
inject (drive something into; force fluid into), to oneself), circumlocution (evasive language,
reject (refuse; throw back), jettison (throw goods using more words than is necessary)
overboard esp. from a ship or an aeroplane) luc, lux, lum, lumin - light
jug, junct, join - join e.g. lucent (emitting light, clear) translucent
e.g. conjugate (joined or paird together; in (allowing light to partially pass through),
Maths illuminate (throw light on; enlighten
intellectually)
or Physics complex numbers joined in a
reciprocal relation are complex conjugates) M
conjunction (action of joining; part of speech magn - great
joining two or more clauses), juncture (joint; e.g. magnitude (a quantity, great size or
junction) importance),
L magnificent (splendid, grand), magnanimous
late, lation - carry (generous; of great courage, of a noble mind -
e.g.dilate (enlarge, expand), relate (link in animus to the spirit or mind.
association), translation (change from one form man - hand

46 USPC - CSAT
UPSC - CSAT -1
e.g. manufacture (make into a product using another)
hands), manicure (care of the hands) mon - advise, remind
mar - sea e.g. admonish (warn; reprimand), monument (a
e.g. marine (pertaining to/inhabiting the sea), structure or edifice intended to commemorate a
maritime (intended for service at sea) great person or event)
matr, mater - mother mort - death
e.g. maternal (motherly), matrix (a place or e.g. immortal (living forever, imperishable),
medium in which something originates or mortuary (a place where dead bodies are
develops), matriarch (female who rules the kept before burial or cremation), post mortem
family, tribe or group) (analysis of a body done after death to ascertain
ment - mind the cause of death)
e.g. mentality (character or disposition of the mot, mov, mob- move
mind), e.g. motor (a thing that causes motion), motility
remind (remember), mention (refer to, bring to (ability to move), demote (reduce/move down
mind) in rank), automobile (self propelled vehicle),
merg - plunge, sink immobile

e.g. submerge (sink in water), merger (unable to move)


(combination of two or more organizations) mult - many
meter - measure e.g. multitude (a great number), multifarious
e.g. chronometer (an instrument that measures (diverse, of many kinds), multiply (increase in
time), symmetry (correspondence of form or number), multiplex (having many elements, a
measure on opposite sides of an axis) building with

migr - wander many units)

e.g. migrate (move from one place to another), mut - change


immigration (process of settling as a permanent e.g. mutation (an alteration; a genetic change),
resident in another country) transmute (change from one form to another),
mir - look immutable (unalterable)

e.g. admire (look/regard with respect), mirror N


(a looking glass) nasc, nat, nate, nant, - birth, born
mit, miss, mise, mission - send e.g. renascence (rebirth, revival), natal
e.g. admit (allow to approach; concede; (pertaining to birth, dating from birth), innate
confess), mission (a body of persons sent on a (inborn, inherent), pregnant (carrying a child
special task), before birth)

demise (death, termination), submission (the naut, naus, nav - ship


action of yielding to authority or another e.g. nautical (relating to ships or navigation),
person), transmit (send from one place to nausea (stomach sickness with urge to vomit,

USPC - CSAT 47
UPSC - CSAT -1
characteristic of sea sickness), navy (the whole onym, onymy- word, name
body of a nation’s ships of war or trade), e.g. anonymous (having an unacknowledged
navigate (sail, direct the course of an aeroplane name),
or a ship) synonym (a word having similar meaning),
neg - deny pseudonym (a false or fictitious name esp.
e.g. negate(make invalid), renege (go back on assumed by an author)
a deal/oath), abnegate (renounce, practice self P
denial) pater, patri - father
nomen - name e.g. paternal (characteristic of a father), patriot
e.g. nominee(a person named in connection (one who loves and supports one’s country land
with a grant , honor or award), nomenclature of fathers), expatriate (send into exile, send
(manner in which names are assigned), away from one’s native place)
cognomen (a surname;nickname), misnomer path, pathy, pathos - emotion, suffering,
(wrong name) feeling e.g.pathology (study of the essential
nov - new nature of diseases), sympathy (feeling of pity),
e.g. novelty (something new), novice (beginner, pathetic (able to arouse feelings of compassion
someone who is new to a field), innovation orcontempt)
(creativity, new ideas) ped, pod - foot
O e.g. impede (hinder, obstruct), biped (having
ocul - eye two feet), expedite (to speed up/ facilitate
e.g. oculist (a specialist in the treatment of eye progress, free [the feet] from entanglement)
disorders), binocular, inoculate (introduce a ped - child, education
weakened infecting agent into the body so as to e.g. pediatrics (medical care of infants), pedant
immunize; graft a bud into a plant of a different (a bookish person), pedagogue (a teacher esp.
type) a strict or dogmatic one)
oper, oeuvre - work pel, puls - drive
e.g. cooperation (work together for the same e.g. compel (to force someone to act), expulsion
purpose), operate (to perform a function), (act of driving out), repel (to push/drive back),
oeuvre (a whole body of work produced by an appellate (have the power to hear appeals
artist,composer), esp. in law), pulsate (expand and contract
omn, omni - all, everywhere rhythmically)
e.g. omnibus (motor vehicle for passengers, pend, pens, pond, poise - hang, weigh carefully
vehicle for all; a publication containing all/a e.g. suspend (hang something; cease
variety of items), omnipotent (all powerful esp. temporarily; debar from a position), pension
God), omnivorous (eating both animal and (a regular payment made to a retired
vegetable person), ponder (think deeply), compensate
food) (counterbalance; offset; reimburse), propensity

48 USPC - CSAT
UPSC - CSAT -1
(inherent tendency, inclination) impotent (powerless; infertile)
pet - seek, beg, attack prehend, prehens- take grasp
e.g. impetus (moving force) petition (a formal e.g. apprehend (understand; arrest, grasp),
request), impetuous (impulsive and passionate) comprehension (understand, take in the
petr – stone, rock meaning of), prehensile (adapted for grasping
e.g. petrify (turn to stone; terrify), petroleum or holding esp. for a limb-some monkeys have
(oil from the rocks) prehensile tails)

phil - loving proto – first in time, order or rank

e.g. philosophy (love of wisdom), philanderer (a e.g. protagonist (the main character in a drama
male flirt/- andros in Greek Fmeans man) or a movie), prototype (original form serving
as a model)
phob - fear
psych - mind
e.g. hydrophobia (an excessive and irrational
fear of water) e.g. psychology (study of the nature, functioning
and development of the mind), psychic ( a
phon - sound
person having paranormal powers), psychopath
e.g. telephone (instrument used for transmitting
( a person who is mentally and emotionally
and receiving speech or sound), phonic
unstable and/or aggressive and anti­social)
(pertaining to sound), phonetics (branch of
Q
study that concerns itself with the sounds of
speech), euphonious (pleasant sounding) quer, quir quis ques - ask, seek

plic - fold, bend e.g. query (question), inquiry (investigation,


examination), inquisition (relentless
e.g. complicate (make difficult or confusing),
questioning), quest
implicate (intertwine; involve)
R
pon, pos, post, pose - set aside, place, put
reg,rect-rule, govern, correct
e.g. component (a constituent part), compose
(to put together as a whole), position (place), e.g. regent, (one who rules in the absence of
depose (remove from power) the monarch), rectitude (moral righteousness,
correctness), incorrigible (incurably bad,
port - carry, bring
beyond correction), interregnum (time between
e.g. porter (one who carries luggage), import
the end of one reign and the start of another)
(bring in from an external source), deport
rid,ris - laugh
(expel from a country)
e.g. ridiculous (absurd, deserving or inviting
pot -drink
laughter), risible (capable of exciting laughter)
e.g. potion (kind of drink or medicine), potable
rupt - break
(fit for drinking)
e.g. rupture (process of breaking or bursting
poten - powerful
open),
e.g. omnipotent (all powerful), potentate
erupt(to burst out), interrupt (break into a
(monarch; powerful state, city or body),

USPC - CSAT 49
UPSC - CSAT -1
conversation or an action), corrupt (debase, e.g. sedate (calm, serenely composed), reside
make rotten; break entirely) (live permanently, settle down), assiduous
S (hardworking)
sacr - holy sent, sens - feel, think, sense
e.g. sacred (holy), sacrifice (holy offering), e.g. sentiment (a thought or view colored by
sacrilege (violation of something holy), emotion), sensible (showing good sense or
consecrate (make sacred), execrate (to declare practical wisdom), sentient (conscious, having
evil; curse) sense perception), sentence (grammatically
sanct - holy complete thought), presentiment (a mental
e.g. sanction (official permission; a law or a impression or feeling about a future event, a
decree foreboding)

esp. ecclesiastical), sanctify (make holy) sequ, secut - follow

sci, scio - know e.g. sequel (ensuing part that follows a narrative

e.g. science (knowledge, systematic study), or a discourse e.g. a play or a movie),


conscious (aware; knowing or Perceiving consecutive (successive, one after the other),
within obsequious (slavishly attentive, excessively
eager to follow orders or directions), execute
oneself), omniscient (all knowing), prescience
(to carry out fully), persecute (persistently
(foresight; foreknowledge of actions)
annoy; oppress with ill treatment)
scop, skep - watch, examine
serv – serve, save
e.g. microscope (instrument used for magnifying
e.g. servant (one employed to serve), servile
details), skeptic (one who habitually doubts
(cravenly submissive)
and questions normally accepted conclusions;
serv - keep safe or well
one who examines critically
e.g. reserve (save or keep for future use;
scrib, script - write
avoidance of plain speaking or openness),
e.g. describe (write down, give a detailed
conservation (keep from harm, decay or loss)
account),
stat, stit, stant, sist - place, stand
prescription (laying down rules or directions;
e.g. static (stationary, standing still), constitute
establishment of a claim), manuscript (written
(make up of), assist (give help to), resist
by hand), proscribe (condemn as dangerous,
(withstand, fend off), distant (far removed,
prohibit)
standing apart), destitute (extremely poor,
sec, sect, seg - cut
utterly lacking material
e.g. section (a cut portion, a piece), bisect (cut/
possessions), superstition (irrational belief
split into two equal parts), insect (a creature
that contradicts natural laws, standing beyond
with a segmented body), secant (line intersecting
nature)
a curve at two or more points)
solv, solu - loosen, release
sed, sid, sess - sit, seat, settle

50 USPC - CSAT
UPSC - CSAT -1
e.g. dissolve (to pass into a solution; vanish), e.g. assume (take as being true; take it upon
resolute (firm, determined), absolution oneself), presumption ( an action of taking for
(forgiveness of sins generally granted by an granted)
ecclesiastical authority). T
somn - sleep tact, tang, tig, ting - touch;
e.g. somnambulist (one who walks in his sleep- e.g. contact (coming together or touching of
to ambulate is to walk), insomnia (chronic things), tangent (touching at a single point;
sleeplessness), somniloquist (one who talks in irrelevant), contiguous (adjacent, sharing a
his sleep- loq, loc pertain to the idea of talking) boundary, contingency ( a thing dependent on
soph - wisdom an uncertain event; a chance occurrence; an
e.g. sophisticated (experienced, worldly, incidental expense)
cultured, philosophy (love of wisdom-philo teg, techn, tect, text - cover; art, skill
pertains to love), sophistry (misleading and e.g. technical (relating to a practical scientific
false argumentation) subject, involving skill), detect (discover the
spec, spect, spic - look, appear existence of), context (surrounding part of a
e.g. specimen (representative term or sample statement around a particular word), protégé
to be looked at), inspect (examine, look into), (one who is trained or protected), architect
conspicuous (striking, noticeable), specious (master builder- archi is the idea of rule or first
(believable but fallacious), perspicacious (keen, in rank)
having a clear understanding) tele - distance
spir, spire, spirat - breathe, spirit e.g. telescope (tubular optical magnifying
e.g. expire (to come to an end; breathe one’s object to bring distant objects into closer view),
last), telepathy (communication of thought or feeling
conspire (to plan together secretly), spirit (vital by extrasensory means- pathos means feeling
principle; force animating life), respirator or emotion)
(filtering device covering the mouth; an temper, temper - mixture, balance; time
apparatus for the artificial maintenance of e.g. temperance (moderation or restraint),
respiration) temperature (degree of hot or cold) temporary
strain, string, strict - to draw tight, bind (lasting for a limited time), extemporaneous
e.g. stringent (strict, with rigorous standards), (impromptu, unprepared), contemplate
stricture (a restraint, limitation), constrain (to (meditate, ponder, regard thoughtfully)
restrict, confine) ten, tain - hold, reach
stru, struct - build; arranged pattern e.g. abstain (hold back from; refrain) , tenant
e.g. construe (explain the meaning of, interpret) (rent paying dweller), entertain (to hold
instruct (provide with knowledge), instrument attention) tenacious (persistently holding on) ,
(agency, tool) term - end
sum, sumpt - take e.g. interminable (tediously long, having no

USPC - CSAT 51
UPSC - CSAT -1
prospect of an end), terminal ( final, ultimate, U
occurring at the end; in the last stage of a umbra - shade
fatal disease), terminate (to end; dismiss from
e.g. umbrella (device that gives shade or
employment)
protection from the rain), umbrage (displeasure,
ter, terr - dry land, earth annoyance)
e.g. inter (bury a body in a tomb or earth), penumbra (shaded outer region of a shadow)
terrace (level platform of earth), territory
urb - city
(specified area of land, sphere of influence),
e.g. suburb (the outskirts of a city), urban (of or
Mediterranean (in the middle of the land; the
relating to a city)
almost land-locked sea)
V
therm - heat
vac, van, vast, void- empty, desolated, ravaged
e.g. thermometer (instrument for measuring
e.g.
temperature - metry pertains to measurement
theroms (vacuum flask for keeping things hot vacate (to cease to occupy), evacuation (to
), diathermy (generation of heat in tissue by empty or withdraw), evanescent (vanishing
passing quickly), devastate (to lay waste), devoid
(destitute, totally lacking)
an electrical current through it) thermodynamic
(the study of the relation between heat and vad, vas - go
motion- dynam pertains to force or power) e.g. evade (get away; contrive to avoid),
tort, tors - twist , wind val, vail -be strong to do well
e.g. contort (to twist into an abnormal shape), e.g. value (worth, power), valid (sound, to the
torsion (act of twisting or turning, extract point), prevail (to be superior in strength, to
(extract by force) succeed), convalescence (period of regaining
tract, treat - draw , pull strength following an illness).

e.g. attract (pull toward, appeal to ), extract (pull ven, venu, vent - come
out ), retract (draw back, take back), entreat (to e.g. convene (to come together officially),
plead or beg) avenue (way of approach), event (an
trit - rub occurrence), covenant (formal contract esp.
with God
e.g. trite (no longer novel or fresh, hackneyed),
attrition (regret for sin through fear of ver - true
punishment; e.g. veracity (truthfulness, accuracy), aver (to
rubbing away by friction) testify to the truth of), verdict (decision of a
jury)
trud , trus - thrust
verb - word
e.g. intrude (thrust in, enter forcibly without
consent), abstruse (difficult to comprehend) e.g. verbose (wordy), verbatim (using exactly

52 USPC - CSAT
UPSC - CSAT -1
the same words, word for word) viv, vit - live
vert, vers - turn e.g. vivacious (full of animation or spirit),
e.g. convert (to turn about or from), reverse vitalize (infuse with life), survive (remain alive;
(opposite, contrary; turn back), controversy outlast)
(disputation on a matter of opinion) voc, vok - call, summon
vid, vis, view - see e.g. vocal (relating to the voice), vocation ( a
e.g. evident (obvious, clear the eye or mind), regular occupation, a calling), revoke (to call
visible (able to be seen), provide - (to furnish, back, withdraw; to annul by recalling)
supply ahead of time), review (look over again) volv, volut - roll, turn
vine, vict - conquer e.g. convolute (twist or coil around), involve (to
e.g. invincible (unable to be vanquished), evict contain as a part, include), revolution (orbital
(to force out legally, to expel), convict (to find movement or turning around a point, evolve
guilty, condemn) (to gradually change and adapt and turn into
something else)

Prefixes
A prefix comes before the basic element of a word
A arch-enemy (chief enemy; the Devil)
a, ab, abs - from, away auto - self
e.g. avert (turn away; prevent, ward off) abjure e.g. autobiography (a story of one’s life written
(renounce on oath), absent (away, not present) by oneself), automatic (self-acting)
ambi, amphi - around, both B
e.g. ambidextrous (capable of writing with both ben - good, well
hands), amphibious (living on both land and in e.g. benediction (a blessing), benefactor (a
water) person who gives friendly aid)
ante, anti - before bi - two
e.g. antedate (affix an earlier than true date, e.g. bilateral (of two sides, involving two parties)
precede in time), anticipate (take into before bisect (divide into two equal parts)
due time) C
anti - against circum - around
e.g. antidote (medicine given to counteract e.g. circumnavigate (sail around), circumvent
poison), (find a way around, evade)
antislavery (against slavery) com, con, col, cor, co - together;
arch - first, chief e.g. commit (entrust or consign for treatment),
e.g. arch-angel (an angel of the highest rank),
USPC - CSAT 53
UPSC - CSAT -1
concord (agreement or harmonious relations), e.g. hypercritical (excessively critical, fault
collect (assemble together at a place), correct finding), hypersensitive (sensitive to emotions
(free from error), co-worker (a fellow worker) or artistic impressions to an abnormal degree)
contra, contro, counter - against; hypo - too little, under;
e.g. contradict (speak against), controvert e.g. hypochondriac (a person with depression
(argue about, dispute), counteract (act against) or low spirits, a person who is persistently
D anxious about his heath, the hypochondria is
de - down, away from, about; the part of the abdomen lying under the ribs.
It was regarded as the seat of melancholy),
e.g. descend (climb down), depart (do away),
hypodermic (situated below the skin)
describe (portray in words, write down)
I
demi - half;
in, il, im - into, in, on
e.g. demigod (half-god)
e.g. invade (attack, enter into something by
dia - across, through;
force),
e.g. diameter (line passing through the center
illustrate (make clear, elucidate; shed light
of a circle and ending at the circumference),
upon.
diastole (the phase of the heartbeat where the
heart relaxes and fills with blood) lustrare means to shed light on), immerse (dip
or plunge in a liquid)
dis, di, dif - apart, not
in, il, im, ir - not
e.g. dissension (disagreement of opinion),
division (splitting into parts and branches), e.g. indistinct (not clear or distinct), illegal (not
diffident (lacking self-confidence) legal, forbidden by law), impossible (unable to
be done or exist), irresponsible (lacking a sense
E
of responsibility)
equi - equal
inter, intro - between, among
e.g. equilateral (having all sides of equal length),
e.g. interpose (come between, interfere;
equinox (an occasion in the year when the day
interrupt
and night are of equal duration), equivalent
(equal in value or significance) someone), introduce (bring, put or lead into;
insert)
e, ef, ex - out of, from
M
e.g. extract (pull out), eject (throw out), efface
(rub out from a surface) mal - bad

extra - out of, beyond e.g. maltreat (treat badly, handle roughly or
rudely), malevolent (wishing ill of others),
e.g. extraordinary (out of the usual or regular
malfunction (function improperly, break down)
course, special), extracurricular (outside the
normal curriculum or routine) mono - one, single;

H e.g. monotone (an utterance in a single tone


without change of pitch), monorail (pertaining
hyper - too much

54 USPC - CSAT
UPSC - CSAT -1
to a railway in which the track is a single rail), trench or a submarine to look all around)
monogamy (practice of marrying only once). poly - many
N e.g. polygon (many sided figure), polygamy
neo - new (practice of marriage with several spouses)
e.g. neoplasm (new and abnormal growth, a post - after
tumor), neophyte (a beginner, a novice) e.g. postpone (put off to a future time),
non - not postmortem (examination of a body after death
e.g. non-entity (a non-existent thing; a person to ascertain the cause of death)
of no consequence or importance), non- pro - forward, before
conformist (a rebel, one who does not conform e.g. proceed (go or travel forward), provide
to traditional beliefs) (supply or make available; make adequate
0 preparation)
ob, of, op - against R
e.g. obviate (encounter and dispose of), offend re - back, again
(attack or assault; hurt or wound feelings. e.g. recur (happen again or periodically),
fendere is to strike), oppose (fight or argue recede (go or move back)
against) retro - backward ;
omni - all e.g. retrogress (move back, deteriorate)
e.g. omnibus (a publication containing a variety retrospect
of items), omnivorous (eating both plants and (look back in time, survey the past
meat)
S
ortho - straight;
se - apart, away
e.g. orthodox (having rigid adherence to
e.g. seduce (lead a person astray in condition
traditional beliefs), orthopedic (pertaining to
or belief). sedition (a concerted movement to
disorders of the bones or joints)
overthrow an established government)
P
semi - half
pan - all
e.g. semicircular (half circle) semiconscious
e.g. pantheism (worship that tolerates all gods), (partially conscious)
Pan-American (involving all the countries of
sub - under
north and South America)
e.g. submarine (under water), subversive
peri - around
(tending to disturb or overthrow)
e.g. perimeter (a defended boundary; a
super - above, beyond
continuous
e.g. superpose above or on something else),
line forming a boundary of a geometrical
supernatural (beyond the natural or ordinary)
figure), periscope (tubular device containing
syn, sym - with, at the same time
prisms or mirrors that enables a person in a

USPC - CSAT 55
UPSC - CSAT -1
e.g. synonymous, sympathetic uninformed (not having knowledge or
T information)
trans - across uni - one
e.g. transcontinental (that which crosses a e.g. unanimous (of one mind or opinion, general
continent), transmit (send across an intervening agreement), uniform (of one unchanging form,
space, convey) having similar characteristics)
U V
ultra - beyond vice - instead of
e.g. ultrastable (stable against all subsequent e.g. vice-chancellor (the acting representative
disturbances), ultramodern (extremely of the chancellor of the university), viceroy (a
progressive, sophisticated) ruler of a colony acting on behalf of a sovereign)
un - not
e.g. unaware (not aware or conscious),

Suffixes
A suffix follows the basic element of a word
able - capable of being or doing or providing by e.g. formal, marital (relating to marriage),
e.g. drinkable, educable, malleable (capable optimal (relating to the optimal), thermal
of being moulded), tenable (literally ‘holdable, (relating to heat);
the actual meaning is logical, well-founded, functional (capable of functioning), hierarchical
reasonable - it is used to describe a point of (having a quality related to a hierarchy, a
view, opinion etc. that you can reasonably hold), system of organization with a series of ranks
durable, suitable, comfortable, favourable one below the other), martial (having a quality
amicable (friendly), equitable (fair), equable connected to the military), patriarchal (having
(uniform, free from fluctuation), the quality of a patriarch, a male head or ruler
acious, cious - having the quality of of a family or a tribe)
e.g. audacious (bold, daring) capricious (guided ance, ence - state or quality of
by whim, unpredictable), mendacious (lying, e.g. abeyance (a state of suspension or
untruthful) inactivity),
age - act, condition dominance (a state of having the greater power
e.g. coinage (the act of forming a new word or authority), indifference, obsolescence (the
or phrase, the new word or phrase formed), state of falling into disuse, or of becoming
carnage (the slaughter of a great number), outdated), senescence (the condition of growing
verbiage (wordiness) old, related
al - relating to or having the quality of to the word senile), somnolence (sleepiness),
virulence (the quality of being poisonous or

56 USPC - CSAT
UPSC - CSAT -1
bitter in feeling) e.g. the proprietor/name of a medicine like
ate - characterized by paracetamol);
e.g. commensurate (adj. in proportion), gustatory (relating to taste),
graduate (adj. relating to graduates e.g. minatory (threatening),
graduate courses; also noun) prepatory (for the purpose of preparation)
neonate (n. new born), potentate (n. a ruler or cy - state, position of, condition of
powerful person), precipitate (adj. sudden e.g. e.g. emergency, efficiency, expediency
precipitate action ), aggravate (make worse), (convenience) excellency, insufficiency,
eradicate (destroy completely), exacerbate immediacy, malignancy, urgency,
(make worse), initiate (start), irritate, sedate
dom - state of
(put to sleep)
e.g. boredom, kingdom, martyrdom (condition
vindicate (to prove to be right or to clear or
of being a martyr i.e someone who dies for a
blame)
cause),
ary, eer, er, or, ory - one who or that which
fiefdom (one’s sphere of activity or control),
does
stardom, wisdom
e.g. commentary, commissionary (a deputy or
escent - becoming;
delegate), corollary (a deduction that follows
e.g. acquiescent (agreeing), convalescent
logically from something else), functionary (an
(recovering, getting better),
official), mercenary (a soldier for hire), mortuary
(a morgue or place where dead bodies are kept), deliquescent (absorbing water to the point of
obituary (a notice of death); seer (a prophet, turning into a solution),
literally one who sees), pioneer, volunteer; evanscent (shortlived),
employer, maker, producer, watchmaker; actor flourescent, incandescent (glowing with heat),
contractor, director, emancipator (liberator), irridescent (glowing with many colours like
proctor (an invigilator), lavatory (washroom mother
from the French ‘lavere’ meaning to wash’),
of pearl),
signatory (one who signs a treaty etc.), laudatory
luminescent (glowing, giving out light),
(having the function of praising)
nascent (just coming into being),
ary, ory - having the quality of or related to
obsolescent (the state of becoming outdated),
e.g. cautionary (having the function of
cautioning senescent (the condition of growing old)

ex- cautionary reminders), rotary or rotatory, fy - make e.g. crucify (to nail on a cross),
stationary, visonary (one who is foresighted deify (make into a god), edify (make wise,
or a dreamer, literally one who has visions i.e. instruct),
dreams), fructify (to produce fruit or results), fortify (to
voluntary; parliamentary, proprietary strengthen),
(signifying ownership or proprietorship - horrify, mortify (kill; punish the body, deprive

USPC - CSAT 57
UPSC - CSAT -1
it of static,
pleasure), terrific; acid, arid (dry, boring),
nullify (to make null and void i.e. with no legal candid (frank),
effect e.g. a marriage or a law), intrepid (fearless),
qualify, rectify (to set right) putrid (rotten), rancid (rotten), torrid (steamy,
hood - state of passionate), turgid (swollen), sordid (cheap,
e.g. brotherhood, falsehood (a lie), godhood dirty),
(the state of being god), livelihood (means of solid, valid (fitting the specifications, having
living), likelihood (probability), neighborhood, legal
parenthood, personhood, priesthood, force or effect, allowable);
womanhood
android (a humanlike robot),
ic, id, oid - of, like, related to, characterized by,
anthropoid (human in appearance),
causing
cuboid (cube like), humanoid (humanlike),
e.g. antipyretic (a medicine for fever),
ovoid
analgesic (medicine for pain),
(like an egg),
autistic (a mental condition characterized by
geoid (shaped like the earth)
abnormal withdrawal from reality), arable,
il, ile - capable of being
dogmatic (inflexible in opinions or beliefs,
e.g. (in/un)civil (polite), evil, tranquil (calm);
unwilling to listen to reason), docile
carcinogenic (causing cancer), (easily managed, submissive),
eclectic (gathered from many different sources), facile (easily done), febrile (feverish), fertile,
eccentric (odd in behaviour etc.), (im)mobile,
ecstatic (extremely delighted), elastic, prehensile (capable of grasping or gripping),
erratic (irregular, random), puerile (childish), senile (old and losing
esoteric (known to only a few, hard to memory, mental sharpness, and control over
understand), bodily functions),
fanatic, fantastic, frantic, frenetic (=frantic), tactile (capable of being touched or sensed
gigantic, through touch),
hectic (stirring, exciting, disturbing), versatile (capable of being put to many uses),
heretic (one who has unorthodox beliefs), volatile (capable of evaporating quickly; quick
idealistic, kinetic, lactic (relating to milk), tempered),
lunatic, malefic virile (manly, powerful)
(intending evil), mystic, optic, optimistic, pacific ion - act of
(peaceful), pessimistic, e.g. benediction (blessing), division,
pyretic (causing fever), scientific, specific, enervation (weakening),

58 USPC - CSAT
UPSC - CSAT -1
eradication (wiping something out), fission, ish - like
inanition (exhaustion), e.g. clownish, boorish (ill-mannered),
induction (take in or cause to begin), interaction, brackish (salty),
interdiction (prohibition), malediction (curse), devilish, doltish (like a dolt or an idiot,
production, irradiation (expose to radiation), nightmarish, peevish (bad-tempered),
sanction (v. grant; n. permission or embargo) prudish (moralistic), sluggish (slow),
ious, ous, ose - full of; characterized by squeamish (easily disgusted or offended)
e.g. audacious, edacious (greedy), ism - belief in or practice of;
fallacious (false), gracious, e.g. alcholism, altruism (unselfishness, the
perspicacious (keen intelligence), principle of putting others first),

pertinacious (stubborn, persistent), agnosticism (the belief that one cannot know

sanctimonious (pretending to be holy or whether god exists or not)


righteous), anachronism (placing things in the wrong
veracious (truthful), historical time),

vexatious (troublesome, frustrating); atavism (resemblance to remote ancestors),

assiduous (hardworking, persevering), atheism (belief that there is no god),

boisterous (rough and aggressive or full of high deism (belief that God created the universe but
does not intervene in it),
spirits; difficult to control),
exorcism (casting out demons from people or
cadaverous (like a corpse),
places that are possessed),
cantankerous (quarrelsome, bad-tempered),
recidivism (falling back into crime),
dolorous (sorrowful),
theism (belief in god)
erroneous (incorrect),
ist - one who practices or is devoted to
ferrous (containing iron), e.g. activist, alchemist (one who sought to
perspicuous (clear e.g. a perspicuous change other metals into gold and to make a
explanation), drink that would give Immortality),
sedulous (hardworking, endeavouring to artist, marxist, nihilist (one who rejects all
please), religious principles and believes that life is
specious (showy, having a false appearance of meaningless), physicist, scientist
truth), ity - conditon
vacuous (empty) e.g. acclivity (an ascending slope; a descending
adipose (fatty), slope is a declivity),

bellicose (aggressive, warlike), acerbity (bitterness and sharpness), ambiguity,

comatose (in a coma) acuity (mental sharpness),


dexterity (skill),
grandiose (grand), morose (gloomy),

USPC - CSAT 59
UPSC - CSAT -1
imbecility (stupidity) made), factory,
ive - relating to purgatory (in Roman Catholicism a place where
e.g. assertive (confident, strong-willed), the souls of the dead go to be purified by
expansive (tending to spread out), suffering
furtive (done secretly), before they go to heaven)
lucrative (profitable), ory - having the function or quality of
pre-emptive (acting or striking before an e.g. accusatory, adulatory (praising),
opponent cursory (quick, fleeting),
can, so as to prevent him striking) dilatory (tending to cause delay)
ly - in the manner of ship - state of, skill
e.g. doubly, dastardly (cowardly), e.g. courtship, hardship, ownership, partnership,
niggardly (miserly), partisanship (the state of being biased)
grimly (sternly or cruelly), unearthly some - having the characteristic of
(supernatural, e.g. cumbersome (bulky, inconvenient, clumsy),
mysteriously) quarrelsome,
mony - state of frolicsome (playful),
e.g. hegemony (dominance or undue influence gruesome (inspiring fear or horror)
of a country), tude - state of
parsimony (stinginess) e.g. altitude, attitude, certitude (certainty),
ness - quality or condition of fortitude (strength),
e.g. daintiness (fineness, elegance), latitude, magnitude (size),
earthiness (coarseness), plenitude (the condition of being full),
imperiousness (the quality of being verisimilitude (appearance of truth or reality)
commanding, y - full of
or overmastering), e.g. bleary (of the eyes dim, dull, full of sleep),
scurviness (the condition of being contemptible) chary (cautious), cheery,
ory - a place for leery (cautious),
e.g. armory (a place where arms and armour wary (cautious), weary
are

Strategies for Working with Roots


¾¾ Once you identify the prefix and base of a word, how can you form a definition?
¾¾ Sometimes definitions don’t really make much sense. Other times, you seem to get stuck at the
prefix.
¾¾ Some roots have multiple forms. How to keep them simple? For example, the base “to move”

60 USPC - CSAT
UPSC - CSAT -1
has three forms: mov, mot, and mobil. The base “to put” has the forms pon, pos, posit, and even
pound.
¾¾ Some prefixes have multiple forms. How can we keep them straight? For example, the prefix
“with, together” is sometimes cited in three forms: con-, com-, col-. How can you divide and
conquer words like these at the right spots?
Answers to these questions suggest some strategies or routines for you to use. All of these strategies
emphasize the importance of inculcating a sense of flexibility in our approach towards words and
their meanings. This sense of flexibility is important when we work with words. Although language
is systematic, it is not rigid. It is a living entity produced by human beings in a variety of contexts.
This is why we speak of “language arts” and not “language science.”
Word Comprehension: Start with the Base, Not with the Prefix
Students usually learn to pronounce a word from the front and work through it to the end. This
approach, though mechanical, works well when students are reading words they already know. But
the roots approach aims to expand vocabulary and introduce students to words they do not already
know.
Very often, a word’s base is preceded by a prefix. This is especially so in academic vocabulary. If
students try to understand a new word according to the system they learned as beginning readers—
attacking it from the front and working through to the end—they will usually produce explanations
that make no sense. Let’s assume that the root of the week is tract = “pull, draw, drag” and that
students are working with the word contract. Most students will quickly divide the word at the right
spot: con/tract. Moreover, students may know that the prefix con- means “with, together” and the
base tract means “pull, draw, drag.” But if they combine these word parts in order of appearance,
the resulting definition may come out as “with pull” or “together drag,” neither of which makes
sense. You can solve this problem by “slashing off” the prefix and identify the base first and then
use the meaning of the base as the first word of a definition. Using contract as an example, begin
with con/tract and then determine that the base is tract, which means “pull.” Having established
this core meaning, students can then add the meaning of the prefix. In other words, understand that
although we read the word contract from left to right, we comprehend it as the base tract, to which
the prefix con- has been appended. The revised definition of contract = “pull or draw together” makes
sense. When our muscles contract, they “draw together”; when business partners sign a contract, they
“draw” it up “together.”
Prefixes are “attachments” placed “before” the base. Prefixes exist only as attachments to the
bases, and the bases provide words with their essential meanings. Like postage stamps, prefixes are
not used in isolation. So it makes sense that the meaning of a word depends primarily not on its prefix
but on its base. A word may often begin with a prefix, but its meaning always begins with its base.
¾¾ A dentist who extracts a tooth “pulls” it “out.” (extract = “pull out,” not “out pull”)
¾¾ Vanilla extract is the oily liquid “drawn out” of a vanilla bean.
¾¾ We feel “drawn to” people who are attractive. (attract = “draw to, draw toward,” not “toward

USPC - CSAT 61
UPSC - CSAT -1
draw”)
¾¾ When we subtract numbers in a column, we “draw” one number “from under” another. (subtract
= “draw from under,” not “under draw”)
¾¾ A journalist retracts a false statement and “withdraws” it by taking or “pulling it back.” (retract
= “pull back,” not “back pull”)
¾¾ We can also list words based on tract that have no prefix:
¾¾ A tractor is a machine that “pulls” farm equipment.
¾¾ When we trace a picture, we “draw” or “drag” our pencil across the paper as we follow the lines
of the original picture.
When we go “back” over lines we have just “drawn,” we retrace them.
In all of these examples, we see the importance of flexibility as we use and understand the base
meaning of “pull, draw, drag” in a variety of applications.
Multiple Forms of Bases: Look for Core Meaning
Words in Latin and Greek change significantly in form as they express differences in tense (past/
present/future), voice (active/passive), and part of speech (noun/adjective/verb). But even as they
vary, the different forms of Latin roots remain recognizable. Remaining flexible is key.
For example, the Latin verb (in its four principal parts) moveo, movere, movi, motus means “move.”
The participle, with its forms motus, mota, motum, means “having been moved.” The adjective mobilis,
mobile means “able to be moved, mobile.” English words dealing with “movement”: mov, mot, mobil.
These three forms, all sharing the foundation of mo, are easily recognized as being related. Such
words as movement, remove, promotion, commotion, and mobility are all cognates. All you need to do
is associate the concept of “movement” with all of them.
pon, pos, posit,(Latin) pound(French) all deal with the idea of “put, place”: an opponent is someone
who “puts, places” himself or herself against a challenger; we pose for a picture by “placing” ourselves
in a flattering position; we produce compounds when we “put” two or more words “together.” The
Latin bases pung, punct, to cite another example, mean to “pierce” and give us such words as pungent
(smells and tastes that “pierce” our senses); puncture (to “pierce” a hole); punctuation (the periods,
commas, and apostrophes that we form by “piercing” the paper with dot-like strokes); and punctual
(“on the dot,” i.e., the pierce mark). This same base has a French variant in poign (note the diphthong
oi) in place of the single Latin u. Thus, the word poignant is cognate with this family: poignant
feelings are emotionally “piercing,” and we feel them keenly.
In words derived from Greek, it is common to find a connecting o between combined roots. This
is why we find so many o’s in medical and technical terminology (e.g., electr-o-cardi-o-gram, pyr-o-
phobia, dem-o-cracy).
Knowing this and being comfortable with flexible thinking can lead students to successful
comprehension of bases that have multiple spellings. Helping students keep their focus on meaning
is the key.

62 USPC - CSAT
UPSC - CSAT -1
Double Consonants Within a Word: Teaching Assimilation
Some prefixes have multiple forms. For example, we find the prefix con- in various forms in the
words connect, combine, and collect; we find variations of the prefix ad- in advertise, attract, allusion,
and affect. These slight changes follow a pattern that is recognizable and often even predictable:
this is the phenomenon known as “assimilation.” Assimilation simply means that some consonants
change and become like (“similar to” = assimilate) the consonants that follow next within words.
Assimilation is a common feature of many Latin prefixes. Latin prefixes, like variant forms of
Latin bases, undergo spelling changes with assimilation, but the meaning does not change. These
spelling changes are simply to make words easier to pronounce, which makes the language sound
better. This is the principle of euphony (phon = “sound”; eu- = “good or well”) or “sounding good.”
Although many consonants coexist in English words, some combinations are avoided. For example,
English does not like the sound of a word like conlect, so it changes the n- of the prefix to match the
first letter of the attached base. Thus, conlect becomes collect. Not only is it easier to say, but it also
sounds better.
Latin prefixes that end in consonants (e.g., con- and in-) may change when they are attached
to bases that begin with consonants. The final consonant of the prefix often changes into another
consonant, facilitating pronunciation and enhancing euphony. In general, this change occurs only
when the resulting consonant cluster would otherwise be difficult to pronounce or strike the ear as
unpleasant.
When full assimilation occurs, the final n- of the Latin prefix often changes into the same consonant
as the first letter of the base. Here are some examples:
¾¾ con + lect = collect
¾¾ con + motion = commotion
¾¾ con + mit = commit
¾¾ con + rect = correct
¾¾ in + legal = illegal
¾¾ in + legible = illegible
¾¾ in + migrant = immigrant
Prefixes ending in the consonant n- may also assimilate the final n- into an m- if the base begins
with b or p. For example:
¾¾ con + bine = combine
¾¾ con + pose = compose
¾¾ in + bibe = imbibe
¾¾ in + possible = impossible
This change is called “partial assimilation.” The final n- of a prefix does not double into the next
consonant base but only becomes an m- for the purposes of euphony.
when you encounter a word beginning with a- followed by a double consonant, you have found
an assimilated ad- and should look for the meaning of “to, toward, add to” in the assimilated prefix.
USPC - CSAT 63
UPSC - CSAT -1
For example:
¾¾ ad + celerate = accelerate
¾¾ ad + fect = affect
¾¾ ad + gravate = aggravate
¾¾ ad + legiance = allegiance
¾¾ ad + pendix = appendix
¾¾ ad + rogant = arrogant
¾¾ ad + similate = assimilate
¾¾ ad + tract = attract

How to Divide and Conquer Words with Two Prefixes


One final issue related to “flexing” has to do with words that have multiple prefixes. Here are some
examples:
¾¾ incorruptible (in- and assimilated con-)
¾¾ reconstruction (re- and con-)
¾¾ reproductive (re- and pro-)
¾¾ misconstrue (mis- and con-)
The overall procedure for dividing and conquering these words is similar to the procedure for
words that have single prefixes. Words with two prefixes are generally longer, of course, so it may
take a bit more time to zero in on the base or root word. In general, we suggest the following tips:
¾¾ identify and remove the first prefix.
¾¾ See if the remainder of the word is recognizable.
¾¾ Finally, apply the meaning of the first prefix (i.e., the one they removed in the first step above)
to the rest of the word: re- + construction = reconstruction: rebuilding; the process of
constructing something again.

Summary
¾¾ Find the base. Begin defining a word by defining its base.
¾¾ Realize that some bases and some prefixes have multiple spellings.
¾¾ Be flexible!
Having seen how useful, roots are to understanding the meaning of the words, it is however
necessary to provide some caveat regarding this approach. Roots can at times be confusing or owing
to the change in meanings of words with time, might no longer be relevant today. There can be
difficulties in dividing the words into bases and prefixes too. Many words cannot be broken up inspite
of seeming so and would have completely different meanings.


64 USPC - CSAT
UPSC - CSAT -1

Chapter

3 Higher Order Thinking (HoT)

There are three things we need to deal with while tackling reading comprehension questions:
[A] The Question stem
[B] The Relevant part of the passage
[C] The Options.
We’ll look at each in greater detail:

[A] The Question Stem


Usually in any Reading Comprehension question, you would have a question followed by options.
Just the question, without the options is also called as the question stem.
Reading Comprehension questions come in a variety of forms, but they can essentially be placed
into 2 major categories:
I. GENERIC questions
II. SPECIFIC questions

I. Generic Questions

Generic questions deal with the main idea, purpose, organization, and structure of a passage.
Typical generic questions are phrased as follows:
¾¾ What is the primary purpose of the passage?
¾¾ What is the main topic of the passage?
¾¾ What is the author’s primary objective in the passage?
¾¾ The passage as a whole can best be characterized as which of the following?
¾¾ What is the central theme of the passage?

II. Specific Questions

Specific questions deal with details, inferences, assumptions, and arguments. Typical specific
questions are phrased as follows:
¾¾ According to the passage, which of the following statements is true?
¾¾ Which of the following is most likely to be the reason for…..?
¾¾ With reference to the passage, which of the following assumptions are valid?

USPC - CSAT 65
UPSC - CSAT -1
¾¾ Which of the following assumptions is not supported by the passage?
Although you can answer generic questions without having read the details contained in the
passage, you will need to use the details in the passage to answer the specific questions. However,
you should use a passage sketch to identify the paragraph of the passage that contains the details that
are relevant to the question.
If you have browsed through reading comprehension based question in any competitive examina-
tion, you would have observed that both Generic and Specific question stems are generally framed in
fixed patterns. A handful of these patterns/types constitute the bulk of questions in this section. The
ability to recognize these question types quickly and understand the aim of the question and the com-
mon traits of correct and incorrect answers is extremely important. The commonly asked question
types are:
1. Main Idea /Title ( Generic Question)
2. Author’s Purpose ( Generic Question)
3. Supporting evidence ( Specific Question )
4. Inference ( Generic/Specific )
5. Tone & Style ( Generic Question)
6. Passage Structure ( Generic Question)
7. Contextual vocabulary ( Specific Question )
8. Application ( Generic/Specific )

1. Main Idea / Title


Main idea questions ask you to identify the ‘Theme’ or ‘Main Point’ of the passage. In order to
answer these questions correctly, you must be able to identify the agenda/topic of the passage and
those ideas that support this agenda or explain the topic.
¾¾ Common Question Stems
yy Which of the following most accurately describes the main idea of the passage?
yy What is the central theme of the passage?
yy The passage is primarily concerned with which of the following?
yy The author of this passage is primarily concerned with….
yy The main point made by the passage is that……
¾¾ How to Identify the Correct Answer
Main Idea questions ask you to identify the crux of the author’s point. A one line answer to
the question what is the author talking about in the passage? generally will lead you closer to
the answer. The correct answer to generic questions should relate to the passage as closely as
possible. A mental sketch of how the passage is organised should help in answering generic
questions. You should be able to answer generic questions without having to read each line of

66 USPC - CSAT
UPSC - CSAT -1
the passage. In fact, reading the entire passage can be distracting. At least one of the incorrect
answer choices will usually pertain to a key detail contained in only one of the body paragraphs.
If you have not read these isolated details, you will not be tempted to select these incorrect an-
swer choices. You must identify which ideas in the passage play a supporting role and which
idea is primarily being supported. The correct answer to a main point question is often a para-
phrasing of the conclusion or a summary statement of the passage. Common incorrect answer
choices are those that feature only supporting ideas.

2. Author’s Purpose
Both Main idea and Author’s purpose are very closely related and getting the correct answer to one
is most likely to lead to the correct answer to the other too.
¾¾ Common Question Stems
yy What is the purpose of the author?
yy Why has the author used the example of……in the passage?
¾¾ How to Identify the Correct Answer
If the question tag ‘what is the author talking about?’ gave you the main idea, the question
‘Why is the author writing the passage?’ gives you the answer here. It also helps to think what
the general structure of the passage is. Is the author describing various things? Is he criticizing
a particular action? Is he explaining a problem? And so on… This would also help you in un-
derstanding the tone of the passage.

3. Supporting evidence
Supporting evidence questions are often prefaced by “according to the passage” or “the passage
states that”. Most of the questions that fit into this category could be called as detail-based as they rely
on your ability to find a specific piece of information, often contained in two or three sentences. These
questions tend to be more difficult than main idea questions because they require a more detailed
recollection of the text. If necessary, you can return to the text and quickly re-read a few sentences.
¾¾ Common Question Stems
yy According to the passage, which of the following assumptions is not valid?
yy The passage states that x occurs because...
yy According to the passage, which of the following is true of x?
yy The passage mentions each of the following except….
yy According to the passage, if x occurs then…
¾¾ How to Identify the Correct Answer
In trying to identify the correct answer, it is extremely important that you stick to the text.
The words “according to the passage” should be taken seriously. Answers that seem logical but
that are not directly supported by the text should be avoided.

USPC - CSAT 67
UPSC - CSAT -1
4. Inference
Inference questions are often prefaced by the phrase “the passage implies” or “the author implies”,
In some ways, inference and supporting idea questions are similar. They both require you to stick
closely to the text and rely on specific facts. However, inference questions tend to go further and ask
you to make a very small logical conclusion that is strongly implied, based upon information in the
passage. Answer choices that require multiple assumptions or inferences will often not be correct. In
inference questions, the answer lies directly in the text and requires a very small logical step (e.g., if
the text says that “all the men in Kerala are literate”, an inference would be that “there are no illiterate
men in Kerala”). In other words inference questions require you to draw a conclusion, albeit a very
small one, based upon what the passage states explicitly. However, the inference question type asks
for an answer that is often a near paraphrase of a fact in the passage or a fact that the information in
the passage rules out (e.g., if a species of an animal has existed for 1 million years, you can infer that
the animal is not new to the earth). On the contrary, the application question type (to be seen later in
detail) asks you to use the information in the passage as premises and draw a conclusion that is not
directly addressed in the passage. In other words, the answer to inference questions is a conclusion
made in the passage while the answer to application questions is a conclusion that is applied outside
of the passage to an idea or action.
¾¾ Common Question Stems
yy The passage implies that which of the following was true of ‘x’
yy It can be inferred from the passage that...
yy The passage suggests which of the following about ‘x’
yy The author implies that x occurred because...
yy The author implies that all of the following statements about ‘x’ are true EXCEPT
¾¾ How to Identify the Correct Answer
The correct answer to these questions is usually an obvious logical conclusion of a sentence
in the text. The logical conclusion will be extremely clear. The difficulty, often in these ques-
tions, is finding the specific sentence in the passage that provides the basis for the conclusion in
the correct answer. Stay away from answer choices that do not directly and closely follow from
a statement in the passage, even if this statement seems plausible based upon the general idea
of the passage or commonly held belief.

5. Tone & Style


Tone questions ask you to identify the attitude or mood of a specific part of the passage or of the
entire passage. A common characteristic of this question type is answer choices that are marked by
phrases containing adjectives. Tone questions test your ability to recognize an attitude or disposition

68 USPC - CSAT
UPSC - CSAT -1
of the author, which is signaled by the use of a handful of key words. Your guesses about the author’s
tone should be based on substantial evidence and not just a single word found in the passage. It might
not be enough to define the tone of the entire passage.
¾¾ Common Question Stems
yy The attitude of the author of the passage towards ‘x’ is best described as one of…
yy The tone of the author is best described as…
yy Which of the following best describes the attitude of the author towards…?
yy Considering the style of the author, his most likely profession is…
¾¾ How to Identify the Correct Answer
Whenever the question of tone arises, try to identify words that indicate opinions/ emotions.
Any description that indicates the author’s stand would help in identifying the tone/attitude.

6. Passage Structure
Though it is rarely asked, Passage structure questions need you to determine the relationship be-
tween different parts of a passage. The key to this question type is, understanding the relationship
between each idea and paragraph. You must be able to separate ideas that support a topic/agenda,
from the topic/agenda itself.
¾¾ Common Question Stems
yy One function of the third paragraph of this passage is to…
yy Which of the following best describes the relation of the first paragraph to the passage as a
whole?
yy The author refers to ‘x’ in the passage primarily to…
yy In the context of the passage, the word… most closely corresponds to which of the following
phrases?
¾¾ How to Identify the Correct Answer
Forming a mental structure of the entire passage helps in getting hold of the structure. The
entire passage can be likened to a series of arguments leading us to some conclusion. Sum-
marising each paragraph in a sentence and then seeing how it fits into the broad scheme of the
passage would get you answers to such question types.

7. Contextual Vocabulary
A common question type, it would simply ask you to identify the meaning of a specified word or
phrase, in the given context. It helps to have a good vocabulary and know the word beforehand, but
the meaning can always be gleaned from the context. Also many times a shade of meaning, you are
not familiar with, might have been used. So it is important to combine both, your knowledge of the

USPC - CSAT 69
UPSC - CSAT -1
word and its usage in the given context to arrive at the correct answer
¾¾ Common Question Stems
yy The meaning of the word ‘x’ in the last paragraph is closest in meaning to….
yy The author uses the word ‘x’ to indicate which of the following?
yy What does the author mean by the term ‘x’?
¾¾ How to Identify the Correct Answer
Read the line in which the word/phrase appears and try to imagine what other word could
have been used there without any change in the overall meaning of the sentence. This word has
to be a synonym (even if it’s a distant one). Sometimes reading the line before and after the
concerned sentence also helps.

8. Application
Application questions ask you to take information and conclusions in the passage and extrapolate
them to similar situations or ideas. The key to this question type is the ability to identify the crux of an
argument and see how it relates to a similar situation. These might include working with hypothetical
situations recognizing scope of the text outside its context; evaluating analogous situations; the ideas
the author would agree / disagree with
¾¾ Common Question Stems
yy T he author of the passage would be most likely to agree with which of the following?
yy Which of the following statements would provide the most logical continuation of the final
paragraph?
yy An idea or action described in the passage is most similar to which of the following?
yy Which of the following, if true, would most weaken the explanation of x provided in the
passage
yy According to the passage which of the following is not
¾¾ How to Identify the Correct Answer
Application questions can be extremely tricky as no direct reference can be found in the
passage. However a statement that seems truest to the author’s intention and is in sync with his
line of argument needs to be marked as the answer. Only in this question type we go explicitly
beyond what is given in the passage.

[B] The Relevant Part of the Passage


If you ask yourself ‘Where in the passage, am I to go for the answer?’ the part of the passage
you are led to, is the relevant part of the passage for that particular question. More often than not
even after a preliminary reading, we find ourselves reading the passage again to either confirm some
detail or clarify a complex point. It would not be very wise to try to completely avoid this step. It

70 USPC - CSAT
UPSC - CSAT -1
is impractical to expect yourself to remember the entire passage with all its details while working
under such pressure situations like examinations. What in fact can be done is, minimize the overall
time. This can be done by spending as little time as possible during the first reading and ensuring that
you have a clear mental picture of the structure of the passage and an overview of the location of the
arguments. Asking the following questions might help:
¾¾ Which is the relevant part of the passage; how does it relate to the other parts of the passage?
¾¾ What does it say – what specific words and phrases have been used? And what exactly do these
mean?
¾¾ How can it be summarized (i.e. what is its main point)?
¾¾ What does it assume?
¾¾ What does it imply
¾¾ What can be inferred from it?

[C] The Options


The options are as important as the question stem. Close attention needs to be paid to specific
words and phrases that are used in the wording of the options. Whenever you read the options ask if
what is said in the option statement matches exactly with what can be understood/inferred (as need
be) from the passage? Whenever you think you have zeroed in on the right answer check if you are
able to defend the said answer choice by a sentence or two taken from the passage. Very rarely would
you need to go beyond 2-3 sentences to justify the answer choice. Further, it’ll help to keep the fol-
lowing points in mind
¾¾ Every Phrase in the chosen option should be substantiated.
In the correct answer choice, every word should be completely true and within the scope of
the passage. If you cannot justify every word in the answer choice, it can be eliminated. In
a case where more than one option falls under this category the answer option closest to the
information given in the passage, should be selected.
¾¾ Avoid ‘Too broad’ and ‘Too narrow’ options
Reading Comprehension answer choices that use extreme words, such as all, never, impossible
etc.—unnecessarily broaden/ narrow down the scope of an answer choice. Moderate language
and ideas are generally preferred.
¾¾ Stick to what is given in the passage
Many Reading Comprehension questions ask you to infer something from the passage.
An inference is an informed deduction based on the information in the passage. Reading
Comprehension inferences don’t go far beyond what is intended in the passage. In general, you
should infer so little that the inference seems obvious.

USPC - CSAT 71
UPSC - CSAT -1
Author’s Purpose Practice Questions
Read the passages below and answer the questions.

1
About 17 million children and adults in the country suffer from asthma, a condition that makes it
hard to breathe. Today it is a problem that is treatable with modern medicine. In days gone by, there
were many different superstitions about how to cure asthma. Some people thought that eating crickets
with a little wine would help. Eating raw cat’s meat might be the cure. Another idea was to gather
some spiders’ webs, roll them into a ball, and then swallow them. People also thought that if you ate
a diet of only boiled carrots for two weeks, your asthma might go away. This carrot diet may actually
have done some good for asthma patients, since vitamin A in carrots is good for the lungs.
1. The main purpose of the passage is to:
(a) Describe herbal remedies
(b) Explain some of the measures for treating asthma from long ago
(c) Define superstitions
(d) Extol the virtues of modern medicine

2
National Parks support a wide diversity of animal species, reflecting the range in elevation, cli-
mate, and habitat variety there. Over 260 native vertebrate species are in the parks; numerous addi-
tional species may be present but have not been confirmed. Of the native vertebrates, five species are
extirpated (here meaning extinct), and over 150 are rare or uncommon. There have been some studies
of invertebrates in the area, but there is not enough information to know how many species occur
specifically in the parks. Many of the parks’ caves contain invertebrates, some of which exist only in
one cave and are known nowhere else in the world. Plant life in the foothills, where summers are hot
and dry and winters are mild, is largely chaparral on the lower slopes, with blue oak and buckeye in
the valleys and on higher slopes. A number of animals live in this area year-round; some breed here,
while others winter here. Local species include the gray fox, bobcat, striped and spotted skunks, black
bear, wood rat, pocket gopher, white-footed mouse, quail, scrub jay, lesser goldfinch, wrentit, acorn
woodpecker, gopher snake, king snake, striped racer, western whiptail lizard, and the newt.
2. What was the author’s purpose in writing this passage?
(a) To inform the reader about local species.
(b) To persuade the reader to visit a national park

72 USPC - CSAT
UPSC - CSAT -1
(c) To inform the reader about National park and the range of animals it houses.
(d) To convince readers to conduct studies on invertebrates.

3
Grapes are one of the oldest cultivated fruits. Hieroglyphics show that Egyptians were involved
in grape and wine production. Also, the early Romans were known to have developed many grape
varieties.
Grapes have been grown in the country for more than 200 years. The tradition of viticulture (grow-
ing grapes) began in 1769 when Spanish friars established missions throughout California. Then the
boom in grapes planted for eating arose in the early 1800s. William Wolfskill, founder of citrus indus-
try, planted the first table grape vineyard in 1839.
Today table grapes, and raisins are all important agricultural commodities, with approximately
700,000 acres planted in vineyards. About 85% of table grape production is in the southern region,
with the Valley region accounting for most of the remaining production.
3. The author most likely wrote this passage to:
(a) To give an introduction about grape cultivation
(b) To educate the reader about Spanish friars
(c) Persuade the reader to consume grapes.
(d) To inform the reader about William Wolfskill

4
It is tempting to think that your eyes are simply mirrors that reflect whatever is in front of them.
Researchers, however, have shown that your brain is constantly working to create the impression of
a continuous, uninterrupted world.
For instance, in the last 10 minutes, you have blinked your eyes around 200 times. You have prob-
ably not been aware of any of these interruptions in your visual world. Something you probably have
not seen in a long time without the aid of a mirror is your nose. It is always right there, down in the
bottom corner of your vision, but your brain filters it out so that you are not aware of your nose unless
you purposefully look at it.
Nor are you aware of the artery that runs right down the middle of your retina. It creates a large
blind spot in your visual field, but you never notice the hole it leaves. Your brain works hard to make
the world look continuous!
4. What is the main purpose of this passage?
(a) To persuade the reader to pay close attention to blind spots

USPC - CSAT 73
UPSC - CSAT -1
(b) To explain the way visual perception works
(c) To persuade the reader to consult an optometrist if you are not able to see your nose
(d) To prove that vision is a passive process

5
On a bad day, have you ever been irritable? Have you ever used a harsh tone or even been verbally
disrespectful to your parents or teachers? Everyone has a short temper from time to time, but current
statistics indicate that between 16% and 20% of a school’s population suffer from a psychological
condition known as oppositional defiant disorder, or ODD.
ODD symptoms include difficulty complying with adult requests, excessive arguments with adults,
temper tantrums, difficulty accepting responsibility for actions, low frustration tolerance, and behav-
iors intended to annoy or upset adults. Parents of children with ODD often feel as though their whole
relationship is based on conflict after conflict.
Unfortunately, ODD can be caused by a number of factors. Some students affected by ODD suffer
abuse, neglect, and severe or unpredictable discipline at home. Others have parents with mood disor-
ders or have experienced family violence. Various types of therapy are helpful in treating ODD, and
some drugs can treat particular symptoms. However, no single cure exists.
The best advice from professionals is directed toward parents. Therapists encourage parents to
avoid situations that usually end in power struggles, to try not to feed into oppositional behavior by
reacting emotionally, to praise positive behaviors, and to discourage negative behaviors with timeouts
instead of harsh discipline
5. The author’s purpose in writing this passage is to:
(a) Express frustration about ODD
(b) Prove that parents are the cause of ODD
(c) Inform the reader about this complex condition
(d) Persuade the reader to keep students with ODD out of public school

6
The United States has always been a pluralistic society, meaning it has embraced many points of
view and many groups with different identities from its beginning. That is not to say that these groups
have always seen eye to eye. The first political parties developed in the United States as a result of
conflicting visions of the American identity. Many politicians believed that wealthy merchants and
lawyers represented the country’s true identity, but many others saw it in the farmers and workers who
formed the country’s economic base.

74 USPC - CSAT
UPSC - CSAT -1
The event that brought this disagreement to the surface was the creation of the Bank of the United
States in 1791. The bank set out to rid the country of the debts it had accumulated during the Ameri-
can Revolution. Until then, each state was responsible for its own debts. The Bank of the United
States, however, wanted to assume these debts and pay them off itself. While many people considered
this offer to be a good financial deal for the states, many states were uncomfortable with the arrange-
ment because they saw it as a power play by the federal government. If a central bank had control over
the finances of individual states, the people who owned the bank would profit from the states in the
future. This concern was the basis of the disagreement: Who should have more power: the individual
states or the central government?
The Democratic-Republican Party developed to protest the bank, but it came to represent a vision
of America with power spread among states. The Federalist Party was established in defense of the
bank, but its ultimate vision was of a strong central government that could help steer the United States
toward a more competitive position in the world economy. These different points of view-central
government versus separate states-would not be resolved easily. These same disagreements fueled the
tension that erupted into the Civil War over half a century later.
6. What is the author’s purpose in writing this passage?
(a) To persuade the reader to accept the Federalist Party’s point of view
(b) To explain the disagreements between early American political parties
(c) To explain the importance of a strong central government
(d) To criticize the founders of the Bank of the United States

Main Idea Practice Questions


7
Americans have always been interested in their Presidents’ wives. Many First Ladies have been
remembered because of the ways they have influenced their husbands. Other First Ladies have made
the history books on their own.
At least two First Ladies, Bess Truman and Lady Bird Johnson, made it their business to send
signals during their husbands’ speeches. When Lady Bird Johnson thought her husband was talking
too long, she wrote a note and sent it up to the platform. It read, “It’s time to stop!” And he did. Once
Bess Truman didn’t like what her husband was saying on television, so she phoned him and said,” If
you can’t talk more politely than that in public, you come right home.”
Abigail Fillmore and Eliza Johnson actually taught their husbands, Millard Fillmore and Andrew
Johnson, the thirteenth and seventeenth Presidents. A schoolteacher, Abigail eventually married her
pupil, Millard. When Eliza Johnson married Andrew, he could not read or write, so she taught him

USPC - CSAT 75
UPSC - CSAT -1
herself.
It was First Lady Helen Taft’s idea to plant the famous cherry trees in Washington, DC Each spring
these blossoming trees attract thousands of visitors to the nation’s capital. Mrs. Taft also influenced
the male members of her family and the White House staff in a strange way: she convinced them to
shave off their beards!
Shortly after President Wilson suffered a stroke, Edith Wilson unofficially took over most of the
duties of the Presidency until the end of her husband’s term. Earlier, during World War I, Mrs. Wilson
had had sheep brought onto the White House lawn to eat the grass. The sheep not only kept the lawn
mowed but provided wool for an auction sponsored by the First Lady. Almost $100,000 was raised
for the Red Cross.
Dolly Madison saw to it that a magnificent painting of George Washington was not destroyed dur-
ing the War of 1812. As the British marched toward Washington, DC, she remained behind to rescue
the painting, even after the guards had left. The painting is the only object from the original White
House that was not burned.
One of the most famous First Ladies was Eleanor Roosevelt, the wife of President Franklin D
Roosevelt. She was active in political and social causes throughout her husband’s tenure in office.
After his death, she became famous for her humanitarian work in the United Nations. She made life
better for thousands of needy people around the world.
7. What is the main idea of this passage?
(a) The Humanitarian work of the First Ladies is critical in American government.
(b) Eleanor Roosevelt transformed the First Lady image.
(c) The First Ladies are important in American culture.
(d) The First Ladies are key supporters of the Presidents.

8
Of the many kinds of vegetables grown all over the world, which remains the favorite of young and
old alike? Why, the potato, of course.
Perhaps you know them as “taters,” “spuds,” or “Kennebees,” or as “chips,” “Idahoes,” or even
“shoestrings.” No matter, a potato by any other name is still a potato- the world’s most widely grown
vegetable. As a matter of fact, if you are an average potato eater, you will put away at least a hundred
pounds of them each year.
That’s only a tiny portion of the amount grown every year, however. Worldwide, the annual potato
harvest is over six billion bags- each bag containing a hundred pounds of spuds, some of them as large
as four pounds each. Here in this country, farmers fill about four hundred million bags a year. That

76 USPC - CSAT
UPSC - CSAT -1
may seem like a lot of “taters,” but it leaves us a distant third among world potato growers. Polish
farmers dig up just over 800 million bags a year, while the Russians lead the world with nearly 1.5
billion bags.
The first potatoes were grown by the Incas of South America, more than four hundred years ago.
Their descendants in Ecuador and Chile continue to grow the vegetable as high as fourteen thousand
feet up in the Andes Mountains. ( That’s higher than any other food will grow.) Early Spanish and
English explorers shipped potatoes to Europe, and they found their way to North America in the early
1600s.
People eat potatoes in many ways-baked, mashed, and roasted, to name just three. However, in the
United States most potatoes are devoured in the form of French fries. One fast-food chain alone sells
more than $1 billion worth of fries each year. No wonder, then, that the company pays particular at-
tention to the way its fries are prepared. Before any fry makes it to the people who eat at these popular
restaurants, it must pass many separate tests. Fail any one and the spud is rejected. To start with, only
russet Burbank potatoes are used. These Idaho potatoes have less water content than other kinds,
which can have as much as eighty percent water. Once cut into “shoestrings” shapes, the potatoes are
partly fried in a secret blend of oils, sprayed with liquid sugar to brown them, steam dried at high heat,
then flash frozen for shipment to individual restaurants.
Before shipping, though, every shoestring is measured. Forty percent of a batch must be between
two and three inches long. Another forty percent has to be over three inches. What about the twenty
percent that are left in the batch? Well, a few short fries in a bag are okay, it seems.
So, now that you realize the enormous size and value of the potato crop, you can understand why
most people agree that this part of the food industry is no “small potatoes.”
8. What is the main idea of this passage?
(a) Potatoes from Ireland started the Potato Revolution.
(b) The average American eats 50 lbs of potatoes a year.
(c) Potatoes are a key vegetable of the world.
(d) The various terms for potatoes have a long history.

9
What does the word patent mean to you? Does it strike you as being something rather remote from
your interests? If it does, stop and think a moment about some of the commonplace things that you
use every day, objects that you take for granted as part of the world around you. The telephone, radio,
television, the automobile, and the thousand and one other things (even the humble safety pin) that
enrich our lives today once existed only as ideas in the minds of men. If it had not been possible to
patent their ideas and thus protect them against copying by others, these inventions might never have

USPC - CSAT 77
UPSC - CSAT -1
been fully developed to serve mankind.
If there were no patent protection there would be little incentive to invent and innovate, for once
the details of an invention became known, hordes of imitators who did not share the inventor’s risks
and expenses might well flood the market with their copies of his product and reap much of the benefit
of his efforts. The technological progress that has made America great would wither rapidly under
conditions such as these.
The fundamental principles in the U. S. patent structure came from England. During the glorious
reign of Queen Elizabeth I in England, the expanding technology was furthered by the granting of
exclusive manufacturing and selling privileges to citizens who had invented new processes or tools- a
step that did much to encourage creativity. Later, when critics argued that giving monopoly rights to
one person infringed on the rights of others, an important principle was added to the patent structure:
The Lord Chief Justice of England stated that society had everything to gain and nothing to lose by
granting exclusive privileges to an inventor, because a patent for an invention was granted for some-
thing new that society never had before.
Another basic principle was brought into law because certain influential people in England had
managed to obtain monopoly control over such age-old products as salt, and had begun charging as
much as the traffic would bear. The public outcry became so great that the government was forced
to decree that monopoly rights could be awarded only to those who created or introduced something
really unique. These principles are the mainstays of our modern patent system in the United States.
In colonial times patent law was left up to the separate states. The inconsistency, confusion, and
unfairness that resulted clearly indicated the need for a uniform patent law, and the men who drew up
the Constitution incorporated one. George Washington signed the first patent law on April 10,1790,
and less than four months later the first patent was issued to a man named Samuel Hopkins for a
chemical process, an improved method of making potash for use in soapmaking.
In 1936 the Patent Office was established as a separate bureau. From the staff of eight that it main-
tained during its first year of operation it has grown into an organization of over 2500 people handling
more than 1600 patent applications and granting over 1000 every week.
The Patent Office in Washington, DC, is the world’s largest library of scientific and technical data,
and this treasure trove of information is open for public inspection. In addition to more than 3 million
U. S. patents, it houses more than 7 million foreign patents and thousands of volumes of technical
literature. Abraham Lincoln patented a device to lift steam vessels over river shoals, Mark Twain
developed a self-pasting scrapbook, and millionaire Cornelius Vanderbilt invented a shoe-shine kit.
A patent may be granted for any new and useful process, machine, article of manufacture, or com-
position of matter ( a chemical compound or combinations of chemical compounds), or any distinct
and new variety; of plant, including certain mutants and hybrids.

78 USPC - CSAT
UPSC - CSAT -1
The patent system has also helped to boost the wages of the American worker to an unprecedented
level; he can produce more and earn more with the computer, adding machines, drill press or lathe.
Patented inventions also help keep prices down by increasing manufacturing efficiency and by stimu-
lating the competition that is the foundation of our free enterprise system.
The decades of history have disclosed little need for modification of the patent structure. Our pat-
ent laws, like the Constitution from which they grew, have stood the test of time well. They encour-
aged the creative processes, brought untold benefits to society as a whole, and enabled American
technology to outstrip that of the rest of the civilized world.
9. What is the main idea of this passage?
(a) The patent system encourages free enterprise.
(b) The patent system in England has been influential in American patent development.
(c) Patents are important tools for inventors.
(d) Patented inventions protect the inventor, free enterprise, and the creative process.

10
In 1976, Sichan Siv was crawling through the jungle, trying to escape from Cambodia. By 1989,
however, Siv was working in the White House, in Washington DC, as an advisor to the President of
the United States. How did this strange journey come about?
Like millions of Cambodians, Siv was a victim of a bloody civil war. One of the sides in this war
was the Cambodian government. The other was a group called the Khmer Rouge. When the Khmer
Rouge won the war, the situation in Cambodia got worse. Many people were killed, while others were
forced into hard labor. Sometimes entire families were wiped out.
Siv came from a large family that lived in the capital of Cambodia. After finishing high school, Siv
worked for a while with a Cambodian airline company. Later, he taught English. After that, he took a
job with CARE, an American group that was helping victims of the war.
Siv had hope to leave Cambodia before the Khmer Rouge took over the country. Unfortunately, he
was delayed. As a result, he and his family were taken from their homes and forced to labor in rice
fields. After a while, Siv managed to escape. He rode an old bicycle for miles, trying to reach Thailand
where he would be free and safe. For three weeks he slept on the ground and tried to hide from the
soldiers who were looking for him. Caught at last, he was afraid he would be killed. Instead, he was
put into a labor camp, where he worked eighteen hours each day without rest. After several months,
he escaped again; this time he made it. The journey, however, was a terrifying one. After three days
of staggering on foot through mile after mile of thick bamboo, Siv finally made his way to Thailand.
Because he had worked for an American charity group, Siv quickly found work in a refugee camp.
Soon he was on his way to the states. He arrived in June of 1976 and got a job-first picking apples

USPC - CSAT 79
UPSC - CSAT -1
and then cooking in a fast-food restaurant. Siv, however, wanted more than this; he wanted to work
with people who, like himself, had suffered the hardship of leaving their own countries behind. Siv
decided that the best way to prepare for this kind of work was to go to college. He wrote letters to
many colleges and universities. They were impressed with his school records from Cambodia, and
they were impressed with his bravery. Finally, in 1980, he was able to study at Columbia University
in New York City. After finishing his studies at Columbia, Siv took a job with the United Nations. He
married an American woman and became a citizen. After several more years, he felt that he was very
much a part of his new country.
In 1988, Siv was offered a job in the White House working for President Reagan’s closest advisors.
It was a difficult job, and he often had to work long hours. However the long hard work was worth it,
because Siv got the opportunity to help refugees in his work.
10. What is the main idea of this passage?
(a) Persistence and courage are global ideas.
(b) Siv persevered to become an American citizen
(c) Siv overcame numerous challenges, came to America and helped a lot of people.
(d) American charity groups helped Siv.

11
What if someone told you about a kind of grass that grows as tall as the tallest trees? A grass that
can be made as strong as steel? A grass from which houses, furniture, boats, and hundreds of other
useful things can be made? A grass that you would even enjoy eating? Would you believe that person?
You should, for that grass is bamboo, the “wood” of 1,001 uses.
Bamboo may look like wood, but it is part of the family of plants that includes wheat, oats, and
barley. It is a kind of grass. This grass is not just a material for making useful products. Young bam-
boo is eaten, often mixed with other vegetables, in many Asian foods.
Bamboo grows in many parts of the world. In the United States it grows in an area from Virginia
west to Indiana and south to Florida, Louisiana, and Texas. Most bamboo, however, is found in warm,
wet climates, especially in Asia and on the islands of the South Pacific Ocean.
In most Asian countries, bamboo is nearly as important as rice. Many Asians live in bamboo
houses. They sit on bamboo chairs and sleep on bamboo mats. They fence their land with bamboo and
use the wood for cages for chickens and pigs.
Bamboo is used to build large buildings as well as homes. When it is glued in layers, it becomes
as strong as steel. On some islands in the South Pacific, bamboo is even used for water pipes. This
extraordinary material has many other uses. It is used to make musical instruments, such as flutes and
recorders. Paper made from bamboo has been highly prized by artists for thousands of years.

80 USPC - CSAT
UPSC - CSAT -1
Bamboo is light and strong, and it bends without breaking. It is cheap, floats on water, almost never
wears out, and is easy to grow. Nothing else on earth grows quite so fast as bamboo. At times you can
even see it grow! Botanists have recorded growths of more than three feet in just twenty-four hours!
Bamboo is hollow and has a strong root system that almost never stops growing and spreading. In
fact, only after it flowers, an event that may happen only once every thirty years, will bamboo die.
There are more than a thousand kinds of bamboo. The smallest is only three inches tall and one-
tenth of an inch across. The largest reaches more than two hundred feet in height and seven inches in
diameter. No wonder, then, that the lives of nearly half the people on earth would change enormously
if there were no longer any bamboo. No wonder, too, that to many people bamboo is a symbol of hap-
piness and good fortune.
11. What is the main idea of this passage?
(a) Bamboo has at least 2,000 uses.
(b) Bamboo grows at an amazing rate and is found primarily in Asia.
(c) Bamboo is an amazing grass that can be used in multiple ways.
(d) Bamboo could be considered a flower in some cases.

12
Every year since 1986, some of the world’s most daring runners have gathered in the desert of
Morocco. They are there to take part in one of the most difficult races in the world. The Marathon
of the Sands, as it is called, covers over 125 miles of desert and mountain wilderness. The runners
complete the course in fewer than seven days, and they run with their food, clothing, and sleeping
bags on their backs.
The Marathon of the Sands was founded in 1986 by Patrick Bauer. His idea was to give the run-
ners, who come from all over the world, a special kind of adventure. Most of the runners in this race
have found that they form deep friendships with the other runners during their days and nights in the
desert. Facing terrible heat and complete exhaustion, they learn much about themselves and each
other.
For most of the runners, though, the challenge of the race is the main reason for coming. On the
first day, for example, they run fifteen miles across a desert of sand, rocks, and thorny bushes. Few
runners finish the day without blistered and raw feet. They also suffer from a lack of water. (They are
allowed less than nine quarts of water during each day of the race.) Most of all, they are exhausted
when they arrive at the campsite for the night.
The second day, the runners are up at 6:00 A.M. Within a few hours, it is 1000F, but the runners do
not hesitate. They must cover eighteen miles that day. That night, they rest. They must be ready for
the next day’s run.

USPC - CSAT 81
UPSC - CSAT -1
On the third day, the runners must climb giant sand dunes- the first they have faced. Dust and sand
mix with the runners’ sweat. Soon their faces are caked with mud. After fifteen miles of these condi-
tions, the runners finally reach their next camp.
The race continues like this for four more days. The fourth and fifth days are the worst. On the
fourth day, the runners pass through a level stretch and a beautiful, tree-filled oasis, but then, on this
and on the next day, they cross more than twenty-one miles of rocks and sand dunes. The temperature
soars to 1250 and many runners cannot make it. Helicopters rush fallen runners to medical help. Run-
ners who make it to the end of the fifth day know that the worst is over.
On the sixth day, heat and rocks punish the racers terribly. In the Valley of Dra, the wind picks up
and, as the desert heat is thrust against them with great force, they grow more and more exhausted.
The seventh day is the last, with only twelve miles to be covered. The dusty, tired, blistered run-
ners set out at daybreak. Near the finish line, children race along with the runners, for everybody has
caught the excitement. The ones who have run the whole marathon know they have accomplished
what most people could not even dream of. “During the hard moments,” says one contestant who has
raced here twice, “I’d think, ‘Why am I here?’ Then I’d realize I was there to find my limits.”
12. What is the main idea of this passage?
(a) The Marathon of the Sands race tests the limits of human endurance.
(b) The runners run at their own pace.
(c) The race causes the strong to stumble and the weak to not finish.
(d) Every runner runs the race to find their human limits.

13
High in the Andes Mountains in Peru stands the ancient city of Machu Picchu. No one knows why
this great city was built, nor is it likely that we will ever know. Nevertheless, the deserted city of Ma-
chu Picchu is important for what it reveals about the ancient Inca people of South America.
The Incas once ruled a great empire that covered a large part of the South American continent. The
empire was more than five hundred years old when the first Spanish explorers, looking for gold, went
to that continent in the sixteenth century.
The Incas were an advanced people. They were skillful engineers who paved their roads and built
sturdy bridges. They plowed the land in such a way that rains would not wash away valuable soil.
They dug ditches to carry water into dry areas for farming.
Even though they did not know about the wheel, the Incas were able to move huge stone blocks-
some as heavy as ten tons- up the sides of mountains to build walls. The blocks were fitted so tightly,
without cement of any kind, that it would be impossible to slip a knife blade between them! The walls

82 USPC - CSAT
UPSC - CSAT -1
have stood firm through great storms and earthquakes that have destroyed many modern buildings.
The Incas were great artists, too. Today, Incan dishes and other kinds of pottery are prized for their
wonderful designs. Since both gold and silver were in great supply, the Incas created splendid objects
from these precious metals.
While it is true that the Incas had no written language, they kept their accounts by using a system
of knotted strings of various lengths and colors. The sizes of the knots and the distances between them
represented numbers.
At its height, the Incan empire included as many as thirty million people. The emperor ruled them
with an iron hand. He told his subjects where to live, what to plant, how long they should work-even
whom they could marry. Since he owned everything, the emperor gave what he wished when he
wished- and in the amount he wished -to his people.
In 1533 Spanish explorers led by Francisco Pizarro murdered the emperor of the Incas. Earlier, the
heir to the Incan empire had also been killed. The Incas, who had always been entirely dependent on
their emperor, now had no recognized leader. The Spaniards easily conquered the empire and plun-
dered its riches.
Have the Incas disappeared from South America? Not at all. In Peru alone, once the center of
that great empire, eighty percent of the twenty million people are descendants of the Inca people.
Evidence of the Incan empire can be found in many other places in South America as well. You can
even visit Machu Picchu. The remains of this ancient city still stand high in the mountains of Peru, an
awesome tribute to this once powerful empire.
13. What is the main idea of this passage?
(a) The Incas once inhabited the ancient city of Machu Picchu.
(b) Peru was the primary country of the Incas.
(c) The Incas, once a great empire were plundered by the spanish.
(d) Inspite of the plundering by the Spanish, Incas remain a presence.

Worksheet Level - 1
The world economy, according to most forecasters, is expected to grow in 2010 by nearly five per
cent. This surprisingly good performance, however, hides the fact that the recovery continues to be
uneven among the three broad categories — the United States, the Euro zone, and the developing
countries. Equally significantly, the recovery is taking place despite a marked lack of coordination
among the major economic powers in solving their common problems. Dire predictions of a year ago
have not materialized to the extent feared. Financial sector concerns shared by many countries have
dissipated in recent months, although, as the world’s leading central banks never fail to warn, the

USPC - CSAT 83
UPSC - CSAT -1
possibility of another global financial crisis cannot be ruled out.
1. Which of the following statements is correct about the World Economy?
(a) The growth of the world economy in 2010 will be 5%
(b) The economy of the US, Euro zone and the developing countries has been performing
badly
(c) Had the major economic power cooperated, the economy would have performed better
(d) The world economy had faced a crisis and is now in the process of recovering
2. Which of the following statements is not correct?
(a) The world economy is recovering in 2010
(b) There has been uneven recovery in US, Euro zone and the developing countries
(c) The dire predictions of the financial sector have been proved completely incorrect.
(d) The possibility of another global financial crisis is imminent
3. The expected growth of world economy in 2010 by 5% signifies which of the following
(a) It is a good recovery especially after a crisis
(b) It is important to understand the discrepancies in the growth amongst the three broad
categories — the United States, the Euro zone, and the developing countries.
(c) There should be coordination among the major economic powers in solving their com-
mon problems
(d) All of the above 
4. Which of the following statements is correct about the role of the world’s leading cen-
tral banks?
(a) Their predictions have failed
(b) The credibility of the leading banks’ is doubtful
(c) The bank’s job is to warn of the eventuality of a financial crisis
(d) The banks are waiting for another financial crisis
5. The main theme of the paragraph is
(a) highlighting the growth rate of 5% in 2010
(b) highlighting the recovery of the world economy after a financial crisis
(c) highlighting the uneven recovery of the global economy
(d) cautioning about another financial crisis

84 USPC - CSAT
UPSC - CSAT -1
Passage 2

A major consequence of fast-paced motorization and expansion of roads and highways in India
is the mounting rate of fatalities and injuries from traffic accidents. More than 110,000 people are
killed on the roads each year, with the death toll rising by eight per cent annually; the estimate for
serious injuries is 1.6 million. India’s roads are now rated the worst in the world. Viewed against
this background, the road safety initiative launched by the central government and the World Bank
to cover 3,000 km of high-risk national and State highways in Assam, Gujarat, and Karnataka is an
incremental and appreciable step to improve the situation. Under the plan, affordable improvements
based on the latest technologies will be put in place to reduce crashes and fatalities. The project will
draw upon the invaluable experience of the International Road Assessment Programme supported by
the World Bank in several countries. The investments can improve the safety record of some roads
to begin with. What is important, however, is for the government to demonstrate the political will to
move beyond limited schemes in a few States. The continuing carnage demands a policy of zero toler-
ance to crashes covering the entire network of 65,000-plus km of national highways and the quarter
million km of urban roads. Almost three years ago, the Sundar Committee recommended a national
road safety policy but precious little has been done by way of implementation.
6. In the beginning of the paragraph the author wants to convey a message that
(a) India is experiencing a fast-paced motorization and expansion of roads and highways
(b) India is witnessing the death of 110,000 people on the roads each year
(c) The estimated number of serious injuries is 1.6 million in India
(d) The quality of roads is really poor in India. 
7. What does the author intend to convey when he says that “India’s roads are now rated
the worst in the world”
(a) The roads in India are actually the worst in the world
(b) India is one of the worst countries in the world
(c) There is an urgent need to improve the conditions of road in India
(d) So many deaths and injury on roads justify India’s reputation as the worst country
8. The road safety initiative has been launched by the central government to
(a) incrementally improve the conditions of the roads.
(b) please world bank
(c) obtain funds from world bank.
(d) completely prevent crashes and fatalities on roads.

USPC - CSAT 85
UPSC - CSAT -1
9. What is special about the road safety initiative?
(a) It is being assisted by the World Bank
(b) It will cover 3,000 km of high-risk national and State highways in Assam, Gujarat, and
Karnataka
(c) It will reduce crashes and fatalities on some roads
(d) It will draw upon the experience of the International Road Assessment Programme sup-
ported by the World Bank in several countries

Passage 3

Sachin Tendulkar said it best when asked what his 50th Test century meant to him. He struggled at
first to articulate himself but eventually conveyed the sense that it was just  another number — a sat-
isfying number no doubt, but just another. As he has said time and again, records matter little to him;
what has driven him to play for 21 years is an unaffected love for the game, which hasn’t dimmed
since he knew it first as a little boy. Yet it’s through his numbers that we can better value his genius.
It’s these records — his ODI double-hundred earlier this year, for instance — that help nuance the
understanding of greatness. So while a 50th hundred is no more significant than a 51st or a 49th, the
roundness of the number presents the opportunity to stand back and appreciate what the achievement
involves. Longevity is the litmus test of greatness — over a long career, even more so in the case of
Tendulkar which has spanned cricket eras, no facet remains untested. Consider that he started against
Imran Khan, Wasim Akram, and Waqar Younis — three of the greatest fast-bowlers of all time — and
made his 50th hundred against Dale Steyn, who will soon have a similar standing, and the scale of
the feat becomes apparent.
10. Which of the following is the closest to what Sachin said when asked, what his 50th Test
century meant to him?
(a) A significant achievement (b) A new land mark in his cricket career
(b) Just another normal achievement (d) Another satisfying achievement
11. Which of the following is most important for Sachin?
(a) 50th Test century (b) ODI double-hundred
(c) Both (a) and (b) (d) Love for cricket
12. Why is the author emphasizing on Sachin’s 50th Test century?
(a) It is a great record
(b) Records do not matter for Sachin
(c) It is a number by which the author wants to appreciate Sachin’s achievement
(d) 50 is the standard number for judging any record
86 USPC - CSAT
UPSC - CSAT -1
13. According to the author which of the following is special about Sachin?
(a) His 21 year cricket career
(b) Ability to face fast bowlers like Imran Khan, Wasim Akram, and Waqar Younis
(c) His 50th Test century against Dale Steyn
(d) Successfully facing all challenges in a long cricket career
14. What can be deduced about Dale Steyn?
(a) He is a great bowler
(b) He is a fast bowler
(c) He could not prevent Sachin from making a century
(d) Nothing can be said about Dale Steyn

Passage 4

Some people seem to think that sports and games are unimportant things that people do, at times
when they are working, instead of going to the cinema, listening to the radio or sleeping. But in actual
fact, sports can be of great value, especially to people who work with their brains most of the day,
and should not be treated only as amusements. Sports and games make our bodies strong, prevent us
from getting fat and keep us healthy. But these are not their only uses. They give us valuable practice
in making eyes, brain and muscles work together. In tennis , our eyes see the ball coming, judge its
speed and direction and pass this information on to the brain. The brain has to decide what to do and
so on, so that the ball is met and hit back where it ought to go. All this must happen with very great
speed and only those who have had a lot of practice at tennis can carry out this complicated chain
of events successfully. For those who work with brains most of the day the practice of such skills is
specially useful.
15. The author thinks that
(a) Sports and games are important (b) Sports and games are unimportant
(c) Going to the cinema is important (d) Sleeping is important
16. Sports and games are important especially to people who work with their
(a) Brains (b) Hands
(c) Heart (d) Body
17. Sports make
(a) Hand and legs work together
(b) The members of the team work together

USPC - CSAT 87
UPSC - CSAT -1
(c) The players and spectators come together
(d) Eyes, brains and muscles work together
18. The game cited by the authors is
(a) Football (b) Tennis
(c) Cricket (d) Hockey
19. In tennis, what is to be done, is decided by
(a) The umpires’ action (b) The audience’s reaction
(c) The players’ brain (d) The player’s heart

Passage 5

There was once a fisherman who was also an excellent flute player. One day he took his flute and a
fish basket, jumped upon a rock and began to play his flute. He was convinced that the fish would be
enchanted by his music and led into his basket willingly. Unfortunately, he did not catch a single fish,
so he went home, brought his net and threw it into the water. He caught so may fish that his basket
overflowed. “Stupid creatures” he exclaimed as he watched the fish twist and jump into his net “ You
did not dance when, played my flute, but when I am not playing , you can’t stop dancing”
20. With what could the fisherman succeed in catching the fish?
(a) a flute (b) a fish basket
(c) a fishing net (d) None of the above
21. Why did he play his flute?
(a) the fish would be charmed by the music
(b) the fish would dance on a rock
(c) the fish loved the fish basket
(d) None of the above
22. Why could he not catch a single fish ?
(a) The fish did not like his music (b) It is not possible to catch fish by playing flute
(c) His flute did not play well (d) None of the above
23. Why did he call the fish ‘stupid creatures’
(a) The fish are really stupid (b) The fish did not like his music
(c) The fish were playing in the net (d) None of the above
24. Were the fish dancing in the net?
(a) No, they were writhing (b) Yes, they were happy
(c) They liked to be out of water (d) None of the above age

88 USPC - CSAT
UPSC - CSAT -1
Passage 6

Education should not be taken for mere book learning. It means the bringing out of the best in us.
It is often another name for culture or cultivation of our various faculties. An all round development
or growth of our whole being in all its aspects is true education. One sided development, say of the
body at the expense of the mind and spirit or of mind at the expense of body and soul, may help man
little but not much in life. Education is the power to make the best use of the forces not only of nature
but also of man- intellectual moral and physical.
25. Education means
(a) Mere book learning (b) Mere book collecting
(c) Bringing out the best in students (d) Teaching of civilization
26. True education is
(a) An all round growth of the whole being
(b) Growth of body
(c) Growth of mind
(d) Growth of spirit
27. Education gives us power
(a) To impress other
(b) To use force on others
(c) To use force of both man and nature
(d) None of the above
28. Body must grow at the expense
(a) Of mind (b) Of spirit
(c) Not of mind (d) Not of mind and spirit

Passage 7

How often one hears children wishing they were grown up and old people wishing they were
young again. Each age has its pleasures and pains and the happiest person is the one who enjoys what
each age gives him without wasting time in useless regrets. In childhood there are few responsibili-
ties. A child is fed, looked after and loved by his parents. He gets so much without giving anything in
return. But a child has its pain. He is not free to do what he wants. He is constantly told to do or not
to do. When he earns, he becomes free from the discipline of school and parents but is loaded with
responsibilities. Now others will not work for him. Old age has been thought of as the worst age to
be, but it is not necessary for the old to be unhappy. With old age should come wisdom and ability to

USPC - CSAT 89
UPSC - CSAT -1
help others with advice wisely given. They can see their children and grand children making progress.
They can feel the happiness of having come through the battle of life safely and of having reached a
time when they can lie and rest.
29. Who is the happiest person?
(a) One who wants a change of age (b) One who is young
(c) One who is a child (d) One who enjoys each age
30. What things does a child not like?
(a) The pain in his life (b) His desire to do things
(c) He can not have his own way (d) Being told not to do this or that
31. What things can an adult do which a child can not do?
(a) Be irresponsible (b) Ability to earn
(c) Enjoy full freedom (d) None of the above
32. What are the advantage of being old?
(a) Help others with wise counsel (b) See grand children growing
(c) Take rest (d) All of these
33. Childhood is a period of
(a) Irresponsibility (b) Carelessness
(c) Care-freeness (d) Neglect

Passage 8

We suffer from an illusion that India is a poor country in need of capital infusion from aboard. As
a matter of fact, India is one of the capital rich counties. We import more gold (officially, $ 8 billion
worth a year, how much more is smuggled is anybody’s guess) than any other country, the US in-
cluded. Reportedly, Indians are the greatest patrons of Swiss Bank accounts. There is little doubt, we
have a lot of money but we do not invest it in technology, not even in the Indian economy, because of
our “Socialist” culture, we officially detest anyone who demonstrates a capacity to make money – ex-
cept in the case of corrupt politicians whom the electorate endows with virtues beyond imagination.
In any case, there is no money available in India for investment in new venture technology. These are
few venture capitalists in India who are prepared to fund ideas. New ideas may fail, but at times, they
produce fantastic profits. India does not allow anyone to make unusual profits. So no one can dare
take big risks.
34. What does India import more than even the US?
(a) Silver (b) Technology
(c) Gold (d) Coffee

90 USPC - CSAT
UPSC - CSAT -1
35. Indians do not fund
(a) Agriculture (b) Buildings
(c) New ideas (d) Politics
36. Indians are greatest patrons of
(a) Corruption (b) Beauty products
(c) Corrupt politicians (d) Swiss Bank Accounts
37. Venture capitalist is one who
(a) Invests in new ideas (b) Goes on adventures
(c) Detests high profits (d) Has a ‘Swiss Bank Account’
38. Innovative Means
(a) Commercial (b) Official
(c) Experimental (d) Social

Passage 9

Gandhiji recognized that while all men should have equal opportunity, all did not have the same
capacity. Some had the ability to earn more than others. But he believed that those who had talent
would be performing the work of society, if they used their talent wisely and well. Gandhiji said he
would allow a man of intellect to earn more and not suppress his talent. But it was his view that the
bulk of his larger earnings should go to the common fund. Those with talent and opportunity would
find their fulfillment as trustee. Gandhiji extended this concept of trusteeship to cover all fields of life.
39. The title of the passages should be
(a) Gandhiji’s philosophy (b) Gandhiji’s services
(c) Gandhiji’s views (d) Gandhiji’s character
40. According to Gandhiji, one can serve the society
(a) If he works honestly (b) If he earns well
(c) If he is talented (d) If he uses his talent wisely
41. Gandhiji never believed in
(a) Political equality (b) Social equality
(c) Opportunity of equality (d) Complete equality in all respects
42. The meaning of ‘Trustee’ is a
(a) Person having confidence (b) Person who has charge of property in trust
(c) A number of trusts (d) State official who executes wills and trusts

USPC - CSAT 91
UPSC - CSAT -1
Passage 10

Do not study for too long. So long as the mind works with ease, it may be allowed to continue
working but if we find that it moves slowly and extra trouble is needed to fix our attention on a par-
ticular thing, it is far better to break off and make a walk or have some other recreation, than to go on
plodding until one feels wholly exhausted. To continue to force the mind to work is likely to lead to
injurious result and May end in a nervous breakdown from which recovery is slow and troublesome.
43. While making the observation ‘Do not study for too long’ , the author suggests that
(a) Excess of everything is bad.
(b) Reading continuously for long hours is tiring.
(c) Mind is delicate and it should not be taxed too much.
(d) Doing mental work beyond a certain limit may cause serious injury to man.
44. The understanding tone of the passage is that
(a) Man should work within limits
(b) Man is delicate and should be handled carefully
(c) An injury to the mind is more difficult to cure than an injury to any other part of the
body.
(d) Mind controls the entire physical system and should be kept in a perfect state.
45. A man feels that he is exhausted when
(a) He finds his mind working slowly.
(b) He has a headache.
(c) His thought process is clouded.
(d) He finds it difficult to concentrate.
46. The writer suggests that the main cause of nervous breakdown is
(a) Life full of anxiety and worry
(b) Too much work and little diversion
(c) Excess mental work leading to exhaustion
(d) A strenuous life

Passages 11

Speech is a great blessing but it can also be a great curse, for while it helps us to make our inten-
sions and desires known to our fellow beings, it can also, if we use it carelessly, make our attitude

92 USPC - CSAT
UPSC - CSAT -1
completely misunderstood. A slip of the tongue, the use of an unusual word, or of an ambiguous word,
may create an enemy where we had hoped to win a friend. Again, different classes of people use dif-
ferent vocabularies, and the ordinary speech of an educated man may strike an uneducated listener
as being pompous. Unknowingly, we may use a word which bears a different meaning to our listener
from what it does, to men of our own class. Thus, speech is not a gift to be used lightly without
thought, but one which demands careful handling. Only a fool will express himself alike to all kinds
of men in all conditions.
47. Speech can be a curse, because it can
(a) Reveal our intentions (b) Lead to carelessness
(c) Hurt others (d) Create misunderstanding
48. A ‘slip of the tongue‘ means something said
(a) Unintentionally (b) Wrongly by chance
(c) Without giving proper thought (d) To hurt another person
49. The best way to win a friend is to avoid ……. in speech.
(a) Ambiguity (b) Verbosity
(c) Pomposity (d) Irony
50. While talking to an uneducated person, we should use
(a) Polite language (b) Ordinary speech
(c) His vocabulary (d) Simple words

Practice Passages - 3

Question No. 1 2 3 4 5

Answers

Question No. 6 7 8 9 10

Answers

Question No. 11 12 13

Answers

USPC - CSAT 93
UPSC - CSAT -1
Worksheet Level - 1

Question No. 1 2 3 4 5

Answers

Question No. 6 7 8 9 10

Answers

Question No. 11 12 13 14 15

Answers

Question No. 16 17 18 19 20

Answers

Question No. 21 22 23 24 25

Answers

Question No. 26 27 28 29 30

Answers

Question No. 31 32 33 34 35

Answers

Question No. 36 37 38 39 40

Answers

Question No. 41 42 43 44 45

Answers

Question No. 46 47 48 49 50

Answers



94 USPC - CSAT
UPSC - CSAT -1

Chapter

4 Analysis of RC Passages and


Questions in CSAT
4.1 Reading Comprehension section in CSAT 2011
CSAT 2011 was a balanced question paper which provided a level playing to all. A close analysis
of the same would stand us in good stead. There were 37 questions based on reading comprehension
out of total 80 questions in CSAT 2011. This is almost 50% of the total questions asked and nothing
else needs to be said about the importance of this question type. There were totally 10 passages with
each passage having 3-4 questions based on it. The last three passages were supposed to test your
english language skills and did not have hindi translations. Here’s more comprehensive analysis of
the Reading comprehension passages in the question paper.

Passage 1

A country under foreign domination seeks escape from the present in dreams of a vanished age,
and finds consolation in visions of past greatness.....

No. of words in the passage 200


No. of words Total 400
No. of questions 4
Generic/ Detail based distribution 1 generic/3 detail
Level of difficulty Moderate – difficult
Attemptability 3 Easy, 1 tricky
Discovery of India (Chapter: Acceptance and Negation of
Source
life; Page 81)

Passage 2

For achieving inclusive growth there is a critical need to rethink the role of the State. The early
debate among economists about the size of the Government can be misleading. …….

No. of words in the passage 280


No. of words Total 400
No. of questions 5
Generic/ Detail based distribution 1 Generic/ 4 detail
Level of difficulty Easy – Moderate

USPC - CSAT 95
UPSC - CSAT -1
Attemptability 2 Easy; 3 Medium
2009-10 Economic Survey ‘Micro-foundations of
Source
Inclusive growth’

Passage 3

The concept of ‘creative society’ refers to a phase of development of a society in which a large
number of potential contradictions become articulate and active……

No. of words in the passage 120


No. of words Total 250
No. of questions 3
Generic/ Detail based distribution 1 Generic/ 2detail
Level of difficulty Moderate
Attemptability 2 Medium; 1 Tricky
Source ‘Class, Caste and Gender’ by Manoranjan Mohanti

Passage 4

Now India’s children have a right-to receive at least eight years of education, the gnawing question
is whether it will remain on paper or become a reality……

No. of words in the passage 250


No. of words Total 610
No. of questions 5
Generic/ Detail based distribution 1 Generic/ 4 detail
Level of difficulty Moderate – Difficult
Attemptability 4 Medium/ 1 tricky
Source Hindu OP-Ed 02. 04.10 by Krishna Kumar

Passage 5

Ecosystems provide people with a variety of goods and services; food, clean water, clean air,
flood control, soil stabilization, pollination, climate regulation, spiritual fulfillment and aesthetic
enjoyment, to name just a few......

No. of words in the passage 170


No. of words Total 400
No. of questions 3
Generic/ Detail based distribution 3 detail

96 USPC - CSAT
UPSC - CSAT -1
Level of difficulty Easy – Moderate
Attemptability 3 Medium
Source W H Freeman Publications, Life and ecology

Passage 6

A moral act must be our own act; must spring from our own will. If we act mechanically, there is
no moral content in our act. Such action would be moral, if we think it proper to act like a machine
and do so.….

No. of words in the passage 170


No. of words Total 400
No. of questions 3
Generic/ Detail based distribution 1 generic/ 2 detail
Level of difficulty Moderate – Difficult
Attemptability 1 Medium, 2 tricky
Ethical Religion (Chapter-Moral Act)by Mahatma
Source
Gandhi

Passage 7

A species that exerts an influence out of proportion to its abundance in an ecosystem is called a
keystone species. The keystone species may influence both the species richness of communities and
the flow of energy and materials through ecosystems.…

No. of words in the passage 240


No. of words Total 500
No. of questions 4
Generic/ Detail based distribution 2 Generic/ 2 detail
Level of difficulty Easy – Moderate

Passage 8

I opened the bag and packed the boots in; and then, just as I was going to close it, a horrible idea
occurred to me. Had I packed my toothbrush ? I don’t know how it is, but I never do know whether
I’ve packed my toothbrush.……

No. of words in the passage 150


No. of words Total 220
No. of questions 3

USPC - CSAT 97
UPSC - CSAT -1
Generic/ Detail based distribution 3 Detail
Level of difficulty Easy
Attemptability Easy
Source Three men in a boat by Jerome K Jerome

Passage 9

In spring, polar bear mothers emerge from dens with three month old cubs.

No. of words in the passage 110


No. of words Total 200
No. of questions 3
Generic/ Detail based distribution 3 Detail
Level of difficulty Easy – Moderate
Attemptability Easy
Source NGC Magzine

4.2 Reading Comprehension section in CSAT 2012


CSAT 2012 question paper provided a level playing to all. A close analysis of the same would
stand us in good stead. There were 40 questions based on reading comprehension out of total 80
questions in CSAT 2012. This is 50% of the total questions asked underscoring the importance of
this question type. There were totally 10 passages with each passage having 3-4 questions based on
it. The last three passages were supposed to test your english language skills and did not have hindi
translations. Let us look at a more comprehensive analysis of the Reading comprehension questions
in the question paper.

Passage 1

The poor especially in market economies need the strength that collectivities offer for creating
more economic…

Approx. no. of words ( Passage) 170


Approx. no. of words (Total) 320
No. of questions 3
Generic/ Detail based distribution 1 generic/2 detail based
Level of difficulty Moderate – difficult
Attemptability 1 Easy, 2 tricky

98 USPC - CSAT
UPSC - CSAT -1
Rethinking Agricultural Production
Collectivities:The case for a group approach to
Source energize agriculture and empower poor farmers;
IEG Working Paper / EPW 27.2.2010 - Bina
Agarwal

Passage 2

In a typical Western liberal context, deepening of democracy invariably leads to consolidation of


‘liberal values’…..
Approx. no. of words ( Passage) 200
Approx. no. of words (Total) 370
No. of questions 3
Generic/ Detail based distribution 2 Generic/ 1 detail based
Level of difficulty Easy – Moderate
Attemptability 1 tricky, 2 easy
Source Indian Politics and Society Since
Independence: Events, Processes and Ideol
ogy – Bid Chakrabarty

Passage 3

Education, without a doubt, has an important functional, instrumental and utilitarian dimension….

Approx. no. of words ( Passage) 200


Approx. no. of words (Total) 330
No. of questions 3
Generic/ Detail based distribution 1 Generic/ 2 detail
Level of difficulty Moderate- Difficult
Attemptability 1 tricky/ 2 Easy

Passage 4

Chemical pesticides lose their role in sustainable agriculture if the pests evolve resistance….
Approx. no. of words ( Passage) 400
Approx. no. of words (Total) 850
No. of questions 6
Generic/ Detail based distribution 4 Generic/ 2 detail

USPC - CSAT 99
UPSC - CSAT -1
Level of difficulty Moderate – Difficult
Attemptability 4 moderate/ 2 tricky
Source Trinity Green website article on Species
Richness

Passage 5

Today’s developing economies use much less energy per capital than developed countries such as
the United States did…

Approx. no. of words ( Passage) 300


Approx. no. of words (Total) 750
No. of questions 6
Generic/ Detail based distribution 4 Generic/ 2 Detail
Level of difficulty Moderate- Difficult
Attemptability 2 Moderate/ 4 tricky
World Development Report 2010:
Source Development and Climate Change – World
Bank

Passage 6

Invasions of exotic species into new geographic areas sometimes occur naturally and without
human agency….

Approx. no. of words ( Passage) 360


Approx. no. of words (Total) 670
No. of questions 5
Generic/ Detail based distribution 3 generic/ 2 detail
Level of difficulty Moderate – Difficult
Attemptability 3 moderate, 2 tricky
Essentials of Ecology - Townsend, C.R., Harper,
Source
J.L. & Begon, M. (2000)

Passage 7

Most champions of democracy have been rather reticent in suggesting that democracy would itself
promote development and enhancement of social welfare…

Approx. no. of words ( Passage) 350


Approx. no. of words (Total) 530

100 USPC - CSAT


UPSC - CSAT -1
No. of questions 3
Generic/ Detail based distribution 3 Detail
Level of difficulty Easy – Moderate
Attemptability 2 Easy, 1 Tricky
Source The Idea of Justice- Amartya Sen

Passage 8

The need for Competition Law becomes more evident when foreign direct investment (FDI) is
liberalized..

Approx. no. of words ( Passage) 230


Approx. no. of words (Total) 460
No. of questions 3
Generic/ Detail based distribution 2 Generic / 1 Detail
Level of difficulty Easy- Moderate
Attemptability 3 Moderate
Economic Times ; Competition law &
Source
inclusive growth- Madhav Mehra 4.11.2009

Passage 9

For fourteen and a half months I lived in my little cell or room in the Dehradun jail, and I began
to….

Approx. no. of words ( Passage) 120


Approx. no. of words (Total) 270
No. of questions 3
Generic/ Detail based distribution 2 Generic / 1 Detail
Level of difficulty Easy- Moderate
Attemptability 1 Easy; 1 moderate ; 1 Tricky
Source Animals in Prison- Jawaharlal Nehru

Passage 10

We started pitching the highest camp that has ever been made….
Approx. no. of words ( Passage) 100
Approx. no. of words (Total) 250

USPC - CSAT 101


UPSC - CSAT -1
No. of questions 3
Generic/ Detail based distribution 3 Detail
Level of difficulty Easy – Moderate
Attemptability 1 Moderate/ 2 easy

Passage 11

A local man, staying on the top floor of an old wooden house, was….

Approx. no. of words ( Passage) 80


Approx. no. of words (Total) 160
No. of questions 2
Generic/ Detail based distribution 2 Detail
Level of difficulty Easy
Attemptability 2 easy

4.3 Reading Comprehension Section in CSAT 2013


CSAT 2013 was an easy question paper which again provided everybody equal opportunity to do
well. There were 31 questions based on reading comprehension out of total 80 questions in CSAT
2013. Though the number of questions per passage decreased from that of 2012, the number of pas-
sages increased. Reading Comprehension had 12 passages with each passage having 2-3 questions
based on it. Most passages were attemptable. In our opinion this was the easiest Reading comprehen-
sion section so far in CSAT. The last three passages were supposed to test your english language skills
and did not have hindi translations. Let us look at a more comprehensive analysis of the Reading
comprehension questions in the question paper.

Passage 1

The Subject of democracy has become severely muddled because of the way…

Approx. no. of words ( Passage) 280


Approx. no. of words (Total) 470
No. of questions 2
Generic/ Detail based distribution 2 detail based
Level of difficulty Easy – Moderate
Attemptability 2 Medium
Source The Idea of Justice- Amartya Sen

102 USPC - CSAT


UPSC - CSAT -1
Passage 2

Corporate governance is based on principles such conducting the business with all integrity…

Approx. no. of words ( Passage) 220


Approx. no. of words (Total) 400
No. of questions 2
Generic/ Detail based distribution 2 detail based
Level of difficulty Easy – Moderate
Attemptability 1 Medium, 1 easy

Passage 3

A number of empirical studies find that farmers are risk- averse, though only moderately …

Approx. no. of words ( Passage) 260


Approx. no. of words (Total) 370
No. of questions 2
Generic/ Detail based distribution 1 Detail/ 1 Generic
Level of difficulty Easy- Medium
Attemptability 2 Easy/ 1 Moderate

Passage 4

Financial markets in India have acquired greater depth and liquidity over the years…

Approx. no. of words ( Passage) 250


Approx. no. of words (Total) 500
No. of questions 4
Generic/ Detail based distribution 4 Detail
Level of difficulty Easy- Medium
Attemptability 2 Easy / 2 Moderate
Source 2011-12; Economic Survey

Passage 5

Crude mineral oil comes out of the earth as a thick brown or black liquid with a strong smell…

Approx. no. of words ( Passage) 170


Approx. no. of words (Total) 280
No. of questions 2

USPC - CSAT 103


UPSC - CSAT -1
Generic/ Detail based distribution 2 detail
Level of difficulty Easy- Medium
Attemptability 1 Easy, 1 moderate

Passage 6

The law in many parts of the world increasingly restricts the discharge of agricultural …

Approx. no. of words ( Passage) 350


Approx. no. of words (Total) 680
No. of questions 5
Generic/ Detail based distribution 1 Generic / 4 Detail
Level of difficulty Moderate- Difficult
Attemptability 2 Easy, 2 Tricky; 1 Medium
Source The Essentials of Ecology; Blackwell Publishers

Passage 7

The miseries of the world cannot be cured by physical help only…


Approx. no. of words ( Passage) 120
Approx. no. of words (Total) 250
No. of questions 2
Generic/ Detail based distribution 1 Generic / 1 Detail
Level of difficulty Easy- Moderate
Attemptability 1Easy / 1 Moderate
Source The secret of Work- Swami Vivekananda

Passage 8

Ecological research over the last quarter of the century has established the deleterious…

Approx. no. of words ( Passage) 110


Approx. no. of words (Total) 200
No. of questions 2
Generic/ Detail based distribution 1 Generic / 1 Detail
Level of difficulty Easy- Moderate
Attemptability 2 Easy
Source Deccan Herald ‘07

104 USPC - CSAT


UPSC - CSAT -1
Passage 9

Seven-year-old Jim came home from the park without his new bicycle…
Approx. no. of words ( Passage) 110
Approx. no. of words (Total) 200
No. of questions 2
Generic/ Detail based distribution 2 Detail distribution
Level of difficulty Easy – Moderate
Attemptability 1 Moderate/ 1 Easy

Passage 10

It was already late when we set out for the next town, which according to the...

Approx. no. of words ( Passage) 150


Approx. no. of words (Total) 280
No. of questions 3
Generic/ Detail based distribution 3 Detail
Level of difficulty Easy
Attemptability 3 easy

Passage 11

A stout old lady was walking with her basket down the middle of a street in Petrograd…

Approx. no. of words ( Passage) 130


Approx. no. of words (Total) 240
No. of questions 3
Generic/ Detail based distribution 3 Detail
Level of difficulty Easy- Moderate
Attemptability 2 easy 1 Moderate

4.4 Reading Comprehension Section in CSAT 2014

Passage 1

The Himalayan ecosystem is highly vulnerable to damage, both due to geological reasons. …….

Approx. no. of words ( Passage) 260


Approx. no. of words (Total) 520

USPC - CSAT 105


UPSC - CSAT -1
No. of questions 4
Generic/ Detail based distribution 1 generic/3 detail
Level of difficulty Moderate – difficult
Attemptability 3 medium / 1 tricky
Nature at work, Ongoing saga of evolution ; VP
Source
Sharma Additional comments

Passage 2

It is often forgotten that globalization is not only about policies on international economic relation-
ships and transactions …….

Approx. no. of words ( Passage) 250


Approx. no. of words (Total) 460
No. of questions 4
Generic/ Detail based distribution 1 Generic/ 3 detail
Level of difficulty Difficult
Attemptability 1 moderate, 3 tricky
Additional comments

Passage 3

In recent times, India has grown fast not only compared to its own past but also in comparison with
other nations ……………

Approx. no. of words ( Passage) 450


Approx. no. of words (Total) 610
No. of questions 3
Generic/ Detail based distribution 1 Generic/ 2detail
Level of difficulty medium
Attemptability 2 easy 1 tricky
Micro-foundations of Inclusive Growth ;
Source
Economic survey 2009-10
Additional comments

Passage 4

It is easy for the government to control State-owned companies through nods and winks. So what
really needs to be done as a first step is to put petrol pricing ……………

106 USPC - CSAT


UPSC - CSAT -1
Approx. no. of words ( Passage) 190
Approx. no. of words (Total) 340
No. of questions 2
Generic/ Detail based distribution Generic/ 4 detail
Level of difficulty Moderate – Difficult
Attemptability 1 moderate/ 1 tricky
Micro-foundations of Macroeconomic Policy ;
Source
Economic Survey 2011-12
Additional comments

Passage 5

Many nations now place their faith in capitalism and governments choose it as the strategy to cre-
ate wealth for their people.. …………….

Approx. no. of words ( Passage) 200


Approx. no. of words (Total) 350
No. of questions 3
Generic/ Detail based distribution 3 detail
Level of difficulty Easy – Moderate
Attemptability Moderate
Creating wealth without justice; The Hindu
Source
27.4.10
Additional comments

Passage 6

Net profits are only 2.2% of their total assets for central public sector undertakings, lower than for
the private corporate sector. …….

Approx. no. of words ( Passage) 370


Approx. no. of words (Total) 600
No. of questions 4
Generic/ Detail based distribution 4 detail
Level of difficulty Moderate- Difficult
Attemptability 2 moderate, 2 tricky
From disinvestment to privatisationBy Vijay
Source
Kelkar, economictimes 2.2.10
Additional comments

USPC - CSAT 107


UPSC - CSAT -1
Passage 7

Climate change poses potentially devastating effects on India’s agriculture.......


Approx. no. of words ( Passage) 220
Approx. no. of words (Total) 350
No. of questions 2
Generic/ Detail based distribution 1 Generic/ 1 detail
Level of difficulty Easy – Moderate
Attemptability 1 Easy, 1 difficult
India 2039: An Affluent Society in One Generation
Source
edited by Harinder S Kohli, Anil Sood ;p127
Additional comments

Passage 8

It is essential that we mitigate the emissions of greenhouse gases and thus avoid some of the worst
impacts of climate change that would take place in coming years and decades. ……………………….

Approx. no. of words ( Passage) 400


Approx. no. of words (Total) 680
No. of questions 4
Generic/ Detail based distribution 1 Generic/ 3 Detail
Level of difficulty Difficult
Attemptability 2 Moderate, 2 Tricky
Source The Economic Times; 5.6.09 Dr RK Pachauri
Additional comments



108 USPC - CSAT


UPSC - CSAT -1

Chapter

5 Strategies for Reading


Comprehension
5.1 Structure of Passage Approach
Think of any Reading comprehension passage as a structure of ideas. Each passage is designed
to convey a number of ideas that are connected in some way. If you understand these ideas and the
connections between them, then you truly understand the passage as a whole. Focusing on structure
helps you in several ways:
¾¾ It makes it easy to see the “big picture.”
It is important to see what the passage is about as a whole.
¾¾ It tells you the purpose of the supporting details.
Understanding the function of supporting details will help you become a close reader.
¾¾ The logical structure organizes all the information in the passage.
This makes it easy to locate any detail to which a particular question might refer.
¾¾ The structure explains how the author’s main points are related to one another.
Structure is the backbone of the author’s passage. The interrelation of the author’s ideas
becomes clear the moment you understand the structure of the passage.
Focus on the Passage’s Logical Structure
Although passages don’t always have clear-cut, logical structures, you’ll almost always detect a structure
of some kind. Here’s a list of the most common types of logical structures found in passages. Either alone
or in combination, these structures underlie most of the passages you’ll encounter on the exam:
¾¾ Statement and example: The author sets forth a theory or idea, which he or she illustrates
with two or more examples or supports with two or more arguments.
¾¾ Argument and counterargument: The passage presents two or more alternative theories,
each of which seeks to explain a certain phenomenon (the passage might also argue for one
theory over another).
¾¾ Theory and criticism: A commonly held theory, notion, or belief is presented; then the
author points out its flaws.
¾¾ Advantages vs. Disadvantages: The passage presents arguments for both sides of a single
issue or presents the benefits and drawbacks of a certain policy or course of action.
¾¾ Compare and contrast: The passage points out similarities and differences between two or
more events, ideas, phenomena, or people.
¾¾ Historical cause and effect: The passage is a cause-and-effect sequence showing how one

USPC - CSAT 109


UPSC - CSAT -1
event led to another (presented either in chronological order or with later events described
before earlier ones).
¾¾ Classification: The passage identifies and distinguishes between two or more basic types,
categories, or classes of a phenomenon, and then branches out to subclasses. (This structure
is most common in passages involving the natural sciences.)

Step 1 : Read the First Question and Answer Choices Before Reading the Passage

Try to anticipate what the passage is about and the sort of information you should be looking for
to answer the first question.

Step 2 : Read the Passage with a Possible summary in Mind

Begin reading the passage, actively thinking about a possible thesis (central idea) and how the
author attempts to support that thesis. Keep an eye out for information that will be useful in answering
the first question.

Step 3 : Choose a Tentative Answer

When you think you’ve learned enough to take a stab at the first question, choose a tentative
answer. You probably won’t have to read very far to take at least a reasoned guess at the first question.
But don’t confirm your selection yet.

Step 4 : Read the Remainder of the Passage, Formulating an Outline as You Go

As you read, try to accomplish the following :


¾¾ Separate main ideas from supporting ideas and examples.
¾¾ Determine the basic structure of the passage (e.g., chronology of events; classification of ideas
or things; comparison between two or more ideas, events, or things);
¾¾ Determine the author’s opinion or position on the subject.
¾¾ Make notes on the margins as needed to see the “flow” of the passage and to keep the passage’s
details straight in your mind.

Step 5 : Sum Up the Passage and Formulate a Brief Main Idea Statement

Take a few seconds to review your outline, then express the author’s main point in your own
words, keeping it to one sentence. If possible jot down the sentence on a rough paper.
Your statement should reflect the author’s opinion or position (e.g., critical, supportive, neutral)
toward the ideas presented in the passage wherever necessary.

Step 6 : Confirm Your Selection for the First Question

Eliminate any answer choice that is inconsistent with your thesis statement, that doesn’t respond
to the question, or that doesn’t make sense to you.

110 USPC - CSAT


UPSC - CSAT -1
Step 7 : Move On to the Remaining Questions

Make sure you consider all four answer choices for each question.

Reading Comprehension – Different Approaches


Before we learn different approaches for improving accuracy and managing our time better, it
is necessary to understand some fundamental concepts. In order to improve our understanding or
‘comprehension’ of given text/passage it is necessary to be able to locate key points given in the
paragraph. Key points can be understood by asking key questions.
Key Questions
These are the questions asked in order to gather information and facts from the passage as opposed
to opinions and attitude of the author. These questions are –
What?
How?
Why?
When?
Where?
Irrespective of the topic of the passage, there will some information in the passage which can be
gathered by answering the key questions.

5.2 Storyline Approach


Storyline Approaches are very useful in Reading Comprehension. This approach is particularly
useful when one finds it difficult to concentrate through long passages. This is a common grievance
of many students who find it difficult to remember the previous points as they read ahead. Storyline
approach helps you in:
¾¾ Concentrating thoroughly on passage
¾¾ Reaching Main point of the passage easily
¾¾ Consolidating answers for “how?”, “why?”, and “what?”
¾¾ Increasing your accuracy for Generic Questions

How does Storyline Approach work?


Storyline approach is called as storyline approach because the reader tries to link all important
points of the passage into a form, very close to a story. While reading any passage, there are shifts
in the focus of thought that are triggered by asking appropriate questions at appropriate places. Any
particular concept/thought may extend several sentences and asking apt questions is very useful since
they take you to the most important point in a passage or from one important point in a passage to
another. This approach involves following steps:
¾¾ Ask and note key points of the passage.

USPC - CSAT 111


UPSC - CSAT -1
(Key points are the points which are obtained by asking “how?”, “why?”, and “what?”)
¾¾ After obtaining a certain key point, sometimes it still wouldn’t make any sense. At this point
ask ‘significance question’
(Significance Question is the question which says, “I have understood the point, but what is the
significance of this key point?”) Here no other questions like “how?”, “why?”, and “what?”
will help, because it is not the actual point you are looking for, but significance of the point.
Ø Once you have obtained answer for significance question, it will help you in linking all the
key points.
However it is very important to remember that one should not jump to significance question,
unless you have obtained key points. Answers to all significance questions would lead you to form a
story like structure, which is easy to remember.
This approach helps you in retaining even the very first point you read, till you finish reading
the very last point.
Let’s try and apply it to an actual RC passage. Mark and number the appropriate places where you
think it is necessary to know the significance of the key point.
Literature and history are twin sisters, inseparable. In the days of our own grandfathers, and
for many generations before them, the basis of education was the Greek and Roman classics for the
educated, and the Bible for all. In the classical authors and in the Bible, history and literature were
closely intervolved, and it is that circumstance which made the old form of education so stimulating
to the thought and imagination of our ancestors. To read the classical authors and to read the Bible
was to read at once the history and the literature of the three greatest races of the ancient world. No
doubt the classics and the Bible were read in a manner we now consider uncritical but they were read
according to the best tenets of the time and formed a great humanistic education. Today the study
both of the classics and of the Bible has dwindled to small proportions. What has taken their place?
To some extent the vacuum has been filled by a more correct knowledge of history and a wider range
of literature. But I fear that the greater part of it has been filled up with rubbish.

Note down Significant Questions –

————————————————————————————————————————
————————————————————————————————————————
————————————————————————————————————————
————————————————————————————————————————
————————————————————————————————————————
Let’s now analyze flow of thoughts and find out the places where understanding significance of
given information becomes extremely important.
Literature and history are twin sisters, inseparable. In the days of our own grandfathers, and
for many generations before them, the basis of education was the Greek and Roman classics for the
educated, and the Bible for all.1 In the classical authors and in the Bible, history and literature were
closely intervolved,2 and it is that circumstance which made the old form of education so stimulating
112 USPC - CSAT
UPSC - CSAT -1
to the thought and imagination of our ancestors. To read the classical authors and to read the Bible
was to read at once the history and the literature of the three greatest races of the ancient world. No
doubt the classics and the Bible were read in a manner we now consider uncritical but they were read
according to the best tenets of the time and formed a great humanistic education. Today the study
both of the classics and of the Bible has dwindled to small proportions.3What has taken their place?
To some extent the vacuum has been filled by a more correct knowledge of history and a wider range
of literature. But I fear that the greater part of it has been filled up with rubbish.
Here if you look at the passage carefully, you will notice that author varies the focus of what he/
she wants to say from first sentence to last sentence to a great extent. However, there is a definite
link between the points, which when assimilated lead to the main point, the author wants to convey.
Therefore it becomes extremely important to understand significance and arrangement of thoughts
through which author reaches the main point. Let us apply this to the passage we earlier saw.
First Significance Question – to be asked is what is significance of the information, who read
what during author’s grandfather’s time? We get the answer in the following sentence. However,
it raises Second Significance Question, - what if literature and history were intervolved? Next 2
sentences provide answer for Second Significance Question, and raises Third and last Significance
Question – What is the significance of the fact that ‘study of both bible and classics has dwindled?’
Sentences that follow give us the answer to this question and highlight the main point author is
trying to make. In this way, by linking one point to another one can remember maximum details. This
approach improves the concentration and engages the reader with the text.
Such reading habits make good foundation for Active Reading.
There are several other situations where the ‘Storyline Approach’ leads to the important point
rather than the main point. These are as follows:
1. Proof- conclusion (this is the proof, what’s the significance? – this is the conclusion)
2. Comparison/contrast-conclusion (in this case we see ‘x’; in that case we see ‘y’; what’s the
significance? – this is the conclusion we can arrive at).
3. Cause-effect (this is what has happened; what’s the significance? – this will be the effect). –
this is why it has been happening)
4. Problem-solution (this is the problem; what’s the significance? – this is the solution). Note
however, solution – problem is a key-sentence flow (this is what has been suggested; why? –
because of this problem)
5. Thing-importance/role/function (this is the thing, situation etc; what’s the significance? –
this is its importance/role/function)
6. Thing-definition (this is what we are talking about; what’s the significance? – this is its
definition)
7. Point-explanation/example. (This is the point; so what’s the significance? – this is its
explanation/this example will help you understand it)

USPC - CSAT 113


UPSC - CSAT -1
5.3 Optimized Reading Approach
In this approach, we try to minimize the reading and maximize the number of questions which can
be solved based on that reading. This approach is very helpful when a passage is long and comprises
of, say more than 3-4 paragraphs. If such passages are carefully analyzed it can be seen that generally
they follow a very good structure, where ideas follow a logical sequence. One can guess this Logical
Sequence by reading only certain key sentences. Which are these key sentences? These sentences
are generally first and last sentences of the paragraph. Whenever author thinks that one key point has
been completely described, while moving on to next key point, he/she would start a new paragraph. It
can be observed that new paragraph starts with the last key point of earlier paragraph.
How does Optimized Reading Approach work?
For all paragraphs except the last paragraph
¾¾ Reading the first and last Sentences
¾¾ Understand the first (introductory) and last (concluding) sentences
¾¾ Guess, on this basis, what the author will talk of (or has) and then
¾¾ Confirm your guesses (or modify them if necessary)
For last Paragraph
¾¾ Complete reading (storyline approach can be used for last paragraph alone as it helps in
establishing main point/ conclusion/ inference etc)
In this approach all the paragraphs except last paragraph is supposed to be read only with the help
of first and last sentences. As you try to understand meaning of these sentences, you should also try
to take an educated guess, deducing as to what must have been given in that particular passage. Make
a small note in the margins and move on to next paragraph.
As you start reading next paragraph you can confirm whether your guess was correct or not. If
next paragraph’s first sentence does not show any relevance to the guess you had taken previously,
you can modify the guess. Sometimes you can think of an alternative possibility, of what might have
been given in earlier paragraph. Repeat this procedure till you reach last paragraph.
Last paragraph should be read completely. This will give you clues if you have missed any major
point that passage is trying to convey. In addition it will help you reconcile all your previous guesses.
With some practice this can be done in very short span of time. As you practice more, your accuracy
of guessing what paragraph is trying to convey increases and helps you gain the overall understanding
about passage.
Let’s try this by using actual RC passage. You will try to note down your guess about each paragraph.
Chemical pesticides lose their role in sustainable agriculture if the pests evolve resistance. The
evolution of pesticide resistance is simply natural selection in action. It is almost certain to occur
when vast numbers of a genetically variable population are killed. One or a few individuals may
be unusually resistant (perhaps because they possess an enzyme that can detoxify the pesticide).
If the pesticide is applied repeatedly, each successive generation of the pest will contain a larger

114 USPC - CSAT


UPSC - CSAT -1
proportion of resistant individuals. Pests typically have a high intrinsic rate of reproduction, and so a
few individuals in one generation may give rise to hundreds or thousands in the next, and resistance
spreads very rapidly in a population.
This problem was often ignored in the past, even though the first case of DDT
(dichlorodiphenyltrichloroethane) resistance was reported as early as 1946. There is exponential
increase in the numbers of invertebrates that have evolved resistance and in the number of pesticides
against which resistance has evolved. Resistance has been recorded in every family of arthropod
pests (including dipterans such as mosquitoes and house flies, as well as beetles, moths, wasps, fleas,
lice and mites) as well as in weeds and plant pathogens. Take the Alabama leafworm, a moth pest of
cotton, as an example. It has developed resistance in one or more regions of the world to aldrin, DDT,
dieldrin, endrin, lindane and toxaphene.
If chemical pesticides brought nothing but problems, — if their use was intrinsically and acutely
unsustainable — then they would already have fallen out of widespread use. This has not happened.
Instead, their rate of production has increased rapidly. The ratio of cost to benefit for the individual
agricultural producer has remained in favour of pesticide use. In the USA, insecticides have been
estimated to benefit the agricultural products to the tune of around $5 for every $1 spent.
Moreover, in many poorer countries, the prospect of imminent mass starvation, or of an epidemic
disease, are so frightening that the social and health costs of using pesticides have to be ignored.
In general the use of pesticides is justified by objective measures such as ‘lives saved’, ‘economic
efficiency of food production’ and ‘total food produced’. In these very fundamental senses, their
use may be described as sustainable. In practice, sustainability depends on continually developing
new pesticides that keep at least one step ahead of the pests — pesticides that are less persistent,
biodegradable and more accurately targeted at the pests.
————————————————————————————————————————
————————————————————————————————————————
————————————————————————————————————————
————————————————————————————————————————
————————————————————————————————————————

Major Benefits of Optimized Reading Approach

This approach is very helpful in solving Specific Questions. If you have noted, what points are
elaborated in which paragraphs, whenever you come across any specific questions – you can locate
the paragraph is which you would find the answer for that specific detail. This saves the time of
reading repetitive points which might have been given in passage and additionally does not distract
our attention to points with lesser significance.



USPC - CSAT 115


UPSC - CSAT -1

Chapter

6 RC Questions from CSAT


2011-2014 Papers
6.1 CSAT 2011 RC Questions
Passage 1

A country under foreign domination seeks escape from the present in dreams of a vanished age, and
finds consolation in visions of past greatness. That is a foolish and dangerous pastime in which many
of us indulge. An equally questionable practice for us in India is to imagine that we are still spiritually
great though we have come down in the world in other respects. Spiritual or any other greatness
cannot be founded on lack of freedom and opportunity, or on starvation and misery. Many western
writers have encouraged that notion that Indians are other-worldly. I suppose the poor and unfortunate
in every country become to some extent other-worldly, unless they become revolutionaries, for this
world is evidently not meant for them. So also subject peoples. As a man grows to maturity he is
not entirely engrossed in, or satisfied with, the external objective world. He seeks also some inner
meaning, some psychological and physical satisfactions. So also with peoples and civilizations as
they mature and grow adult. Every civilization and every people exhibit these parallel streams of an
external life and an internal life. Where they meet or keep close to each other, there is an equilibrium
and stability. When they diverge conflict arises and the crises that torture the mind and spirit.
1. The passage mentions that “this world is evidently not meant for them”. It refers to
people who
1. seek freedom from foreign domination.
2. live in starvation and misery.
3. become revolutionaries.
Which of the statements given above is/are correct ?
(a) 1 and 2 (b) 2 only
(c) 2 and 3 (d) 3 only
2. Consider the following assumptions :
1. A country under foreign domination cannot indulge in spiritual pursuit.
2. Poverty is an impediment in the spiritual pursuit.
3. Subject peoples may become other-worldly.

116 USPC - CSAT


UPSC - CSAT -1
With reference to the passage, which of the above assumptions is/are valid ?
(a) 1 and 2 (b) 2 only
(c) 2 and 3 (d) 3 only
3. The passage thematically centres on
(a) the state of mind of oppressed people
(b) starvation and misery
(c) the growth of civilization
(d) body, mind and spirit of people in general
4. According to the passage, the torture of the mind and spirit is caused
(a) by the impact of foreign domination.
(b) by the desire to escape from foreign domination and find consolation in visions of past
greatness.
(c) due to lack of equilibrium between an external life and an internal life.
(d) due to one’s inability to be either revolutionary or other wordly.

Passage 2

For achieving inclusive growth there is a critical need to rethink the role of the State. The early
debate among economists about the size of the Government can be misleading. The need of the
hour is to have an enabling Government. India is too large and complex a nation for the State to be
able to deliver all that is needed. Asking the Government to produce all the essential goods, create
all the necessary jobs, and keep a curb on the prices of all goods is to lead to a large cumbersome
bureaucracy and widespread corruption. The aim must be to stay with the objective of inclusive
growth that was laid down by the founding fathers of the nation and also to take a more modern view
of what the State can realistically deliver. This is what leads to the idea of an enabling State, that is,
a Government that does not try to directly deliver to the citizens everything that they need. Instead,
it (1) creates an enabling ethos for the market so that individual enterprise can flourish and citizens
can, for the most part, provide for the needs of one another, and (2) steps in to help those who do not
manage to do well for themselves, for there will always be individuals, no matter what the system,
‘who need support and help. Hence we need a Government that, when it comes to the market, sets
effective, incentive-compatible rules and remains on the sidelines with minimal interference, and,
at the same time, plays an important role in directly helping the poor by ensuring that they get basic
education and health services and receive adequate nutrition and food.
5. According to the passage :
1. The objective of inclusive growth was laid down by the founding fathers of the nation.

USPC - CSAT 117


UPSC - CSAT -1
2. Need of the hour is to have an enabling Government.
3. The Government should engage in maximum interference in market processes.
4. There is a need to change the size of the Government.
Which of the statements given above are correct ?
(a) 1 and 2 only (b) 2 and 3 only
(c) 1 and 4 only (d) 1, 2, 3 and 4
6. According to the passage, the strategy of inclusive growth can be effected by focusing
on
(a) Meeting all the needs of every citizen in the country.
(b) Increasing the regulations over the manufacturing sector.
(c) Controlling the distribution of manufactured goods.
(d) Delivery of the basic services to the deprived sections of the society.
7. What constitutes an enabling Government?
1. A large bureaucracy.
2. Implementation of welfare programmes through representatives.
3. Creating an ethos that helps individual enterprise.
4. Providing resources to those who are underprivileged.
5. Offering direct help to the poor regarding basic services.
Select the correct answer from the codes given below :
(a) 1, 2 and 3 only (b) 4 and 5 only
(c) 3, 4 and 5 only (d) 1, 2, 3, 4 and 5
8. Why is the State unable to deliver “all that is needed?
1. It does not have sufficient bureaucracy.
2. It does not promote inclusive growth.
Select the correct answer from the codes given below :
(a) 1 only (b) 2 only
(c) Both 1 and 2 (d) Neither 1 nor 2
9. What is the essential message being conveyed by the author of the passage ?
(a) The objectives of inclusive growth laid down by the founding fathers of the nation
should be remembered.

118 USPC - CSAT


UPSC - CSAT -1
(b) The Government needs to make available more schools and health services.
(c) The Government needs to establish markets and industries to meet the needs of the poor
strata of the society.
(d) There is a need to rethink the role of the State in achieving inclusive growth.

Passage 3

The concept of ‘creative society’ refers to a phase of development of a society in which a large
number of potential contradictions become articulate and active. This is most evident when oppressed
social groups get politically mobilised and demand their rights. The upsurge of the peasants and
tribals, the movements for regional autonomy and self-determination, the environmental movements,
and the women’s movements in the developing –countries are signs of emergence of creative society
in contemporary times. The forms of social movements and their intensity may vary from country
to country and place to place within a country. But the very presence of movements for social
transformation in various spheres of a society indicates the emergence of a creative society in a
country.
10. What does the author imply by “creative society” ?
1. A society where diverse art forms and literary writings seek incentive.
2. A society where social inequalities are accepted as the norm.
3. A society where a large number of contradictions are recognised.
4. A society where the exploited and the oppressed groups grow conscious of their human
rights and upliftment.
Select the correct answer using the codes given below :
(a) 1, 2 and 3 (b) 4 only
(c) 3 and 4 (d) 2 and 4
11. What according to the passage are the manifestations of social movements ?
1. Aggressiveness and being incendiary.
2. Instigation by external forces.
3. Quest for social equality and individual freedom.
4. Urge for granting privileges and self-respect to disparaged sections of the society.
Select the correct answer using the codes given below :
(a) 1, 3 only (b) 2 , 4 only

USPC - CSAT 119


UPSC - CSAT -1
(c) 3, 4 only (d) All of the above
12. With reference to the passage, consider the following statements :
1. To be a creative society, it is essential to have a variety of social movements.
2. To be a creative society, it is imperative to have potential contradictions and conflicts.
Which of the statements given above is/are correct ?
(a) 1 only (b) 2 only
(c) Both 1 and 2 (d) Neither 1 nor 2

Passage 4

Now India’s children have a right-to receive at least eight years of education, the gnawing question
is whether it will remain on paper or become a reality. One hardly needs a reminder that this right
is different from the others enshrined in the Constitution, that the beneficiary – a six year old child
cannot demand it, nor can she or he fight a legal battle when the right is denied or violated. In all
cases, it is the adult society which must act on behalf of the child. In another peculiarity, where a
child’s right to education is denied, no compensation offered later can be adequate or relevant. This
is so because childhood does not last. If a legal battle fought on behalf of a child is eventually won,
it may be of little use to the boy or girl because the opportunity missed at school during childhood
cannot serve the same purpose later in life. This may be painfully true for girls because our society
permits them only a short childhood, if at all. The Right to Education (RTE) has become law at a
point in India’s history when the ghastly practice of female infanticide has resurfaced in the form of
foeticide. This is “symptomatic of a deeper turmoil” in society which is compounding the traditional
obstacles to the education of girls. Tenacious prejudice against the intellectual potential of girls runs
across our cultural diversity and the system of education has not been able to address it.
13. With reference to the passage, consider the following statements :
1. When children are denied education, adult society does not act on behalf of them.
2. Right to Education as a law cannot be enforced in the country.
Which of the statements given above is/are correct ?
(a) 1 only (b) 2 only
(c) Both 1 and 2 (d) Neither 1 nor 2
14. According to the passage, what could be the traditional obstacles to the education of
girls ?
1. Inability of parents to fight a legal battle when the Right to Education is denied to their
children.

120 USPC - CSAT


UPSC - CSAT -1
2. The traditional way of thinking about girls’ role in society.
3. The prejudice against the intellectual potential of girls.
4. Improper system of education.
Select the correct answer from the codes given below :
(a) 1 and 2 only (b) 2, 3 and 4 only
(c) 1, 3 and 4 only (d) 1, 2, 3 and 4
15. On the basis of the passage, consider the following statements :
1. Right to Education is a legal right and not a fundamental right.
2. For realising the goal of universal education, the education system in the country must
be made identical to that of developed countries.
Which of the statements given above is/are correct ?
(a) 1 only (b) 2 only
(c) Both 1 and 2 (d) Neither 1 nor 2
16. Which one of the following statements conveys the key message of the passage ?
(a) India has declared that education is compulsory for its children.
(b) Adult society is not keen on implementing the Right to Education.
(c) The Right to Education, particularly of a girl child, needs to be safeguarded.
(d) The system of education should address the issue of Right to Education.
17. Which one of the following statements conveys the inference of the passage ?
(a) The society has a tenacious prejudice against the intellectual potential of girls.
(b) Adults cannot be relied upon to fight on behalf of children for their Right to Education.
(c) The legal fight to get education for children is often protracted and prohibitive.
(d) There is no sufficient substitute for education received in childhood.

Passage 5

Ecosystems provide people with a variety of goods and services; food, clean water, clean air, flood
control, soil stabilization, pollination, climate regulation, spiritual fulfilment and aesthetic enjoyment,
to name just a few. Most of these benefits either are irreplaceable or the technology necessary to replace
them is prohibitively expensive. For example, potable fresh water can be provided by desalinating
sea-water, but only at great cost. The rapidly expanding human population has greatly modified the
Earth’s ecosystems to meet their increased requirements of some of the goods and services, particularly

USPC - CSAT 121


UPSC - CSAT -1
food, fresh water, timber, fibre and fuel. These modifications have contributed substantially to human
well being and economic development. The benefits have not been equally distributed. Some people
have actually been harmed by these changes. Moreover, short-term increases in some ecosystem
goods and services have come at the cost of the long-term degradation of others. For example, efforts
to increase the production of food and fibre have decreased the ability of some ecosystems to provide
clean water, regulate flooding and support biodiversity.
18. With reference to the passage, consider the following statements. Expanding human
population has an adverse effect on :
1. Spiritual fulfillment
2. Aesthetic enjoyment
3. Potable fresh water
4. Production of food and fibre
5. Biodiversity
Which of the statements given above are correct ?
(a) 1, 2, 3 only (b) 2, 4, 5 only
(c) 3, 5 only (d) All
19. The passage mentions that “some people have actually been harmed by these changes”.
What does it imply ?
1. The rapid expansion of population has adversely affected some people:
2. Sufficient efforts have not been made to increase the production of food and fibre.
3. In the short term some people may be harmed, but in the long term everyone will benefit
from modifications in the Earth’s ecosystems.
Which of the statements given above is/are correct ?
(a) 1 only (b) 2
(c) 1 and 3 (d) None of the above
20. With reference to the passage, consider the following statements:
1. It is imperative to modify the Earth’s ecosystems for the well being of mankind.
2. Technology can never replace all the goods and services provided by ecosystems.
Which of the statements given above is/are correct?
(a) 1 only (b) 2 only

122 USPC - CSAT


UPSC - CSAT -1
(c) Both 1 and 2 (d) Neither 1 nor 2

Passage 6

A moral act must be our own act; must spring from our own will. If we act mechanically, there is
no moral content in our act. Such action would be moral, if we think it proper to act like a machine
and do so. For, in doing so, we use our discrimination. We should bear in mind the distinction between
acting mechanically and acting intentionally. It may be moral of a king to pardon a culprit. But the
messenger carrying out the order of pardon plays only a mechanical part in the king’s moral act. But
if the messenger were to carry out the king’s order considering it to be his duty, his action would be
a moral one. How can a man understand morality who does not use his own intelligence and power
of thought, but lets himself be swept along like a log of wood by a current ? Sometimes a man defies
convention and acts on his own with a view to absolute good.
21. Which of the following statements best describe/describes the thought of the writer ?
1. A moral act calls for using our discretion.
2. Man should react to a situation immediately.
3. Man must do his duty.
4. Man should be able to defy convention in order to be moral.
Select the correct answer from the codes given below :
(a) 1 only (b) 1 and 3
(c) 2 and 3 (d) 1 and 4
22. Which of the following statements is the nearest definition of moral action, according to
the writer ?
(a) It is a mechanical action based on official orders from superiors.
(b) It is an action based on our sense of discretion.
(c) It is a clever action based on the clarity of purpose.
(d) It is a religious action based on understanding.
23. The passage contains a statement “lets himself be swept along like a log of wood by a
current.” Among the following statements, which is/are nearest in meaning to this ?
1. A person does not use his own reason.
2. He is susceptible to influence/pressure.
3. He cannot withstand difficulties/ challenges.
4. He is like a log of wood.
Select the correct answer using the codes given below :
(a) 1 only (b) 1 and 2
USPC - CSAT 123
UPSC - CSAT -1
(c) 2 and 3 (d) 1 and 4

Passage 7

A species that exerts an influence out of proportion to its abundance in an ecosystem is called a
keystone species. The keystone species may influence both the species richness of communities and
the flow of energy and materials through ecosystems. The sea star Pisaster ochraceus, which lives in
rocky intertidal ecosystems on the Pacific coast of North America, is also an example of a keystone
species. Its preferred prey is the mussel Mytilus californianus. In the absence of sea stars, these
mussels crowd out other competitors in a broad belt of the intertidal zone. By consuming mussels,
sea star creates bare spaces that are taken over by a variety of other species. A study at the University
of Washington demonstrated the influence of Pisaster on species richness by removing sea stars from
selected parts of the intertidal zone repeatedly over a period of five years. Two major changes occurred
in the areas from which sea stars were removed. First, the lower edge of the mussel bed extended
farther down into the intertidal zone, showing that sea stars are able to eliminate mussels completely
where they are covered with water most of the time. Second, and more dramatically, 28 species of
animals and algae disappeared from the sea star removal zone. Eventually only Mytilus, the dominant
competitor, occupied the entire substratum. Through its effect on competitive relationships, predation
by Pisaster largely determines which species live in these rocky intertidal ecosystems.
24. What is the crux of the passage ?
(a) Sea star has a preferred prey.
(b) A preferred prey determines the survival of a keystone species.
(c) Keystone species ensures species diversity.
(d) Sea star is the only keystone species on the Pacific coast of North America.
25. With reference to the passage, consider the following statements :
1. Mussels are generally the dominant species in intertidal ecosystems.
2. The survival of sea stars is generally determined by the abundance of mussels.
Which of the statements given above is/are correct ?
(a) 1 only (b) 2 only
(c) Both 1 and 2 (d) Neither 1 nor 2
26. Which of the following is/are implied by the passage?
1. Mussels are always hard competitors for sea stars.
2. Sea stars of the Pacific coast have reached the climax of their evolution.
3. Sea stars constitute an important component in the energy flow in intertidal ecosystem.

124 USPC - CSAT


UPSC - CSAT -1
Which of the statements given above is/are correct?
(a) 1 and 2 (b) 2 only
(c) 1 and 3 (d) 3 only
27. Consider the following assumptions:
1. The food chains/food web in an ecosystem are influenced by keystone species.
2. The presence of keystone species is a specific characteristic of aquatic ecosystems.
3. If the keystone species is completely removed from an ecosystem, it will lead to the
collapse of the ecosystem.
Which of the above assumptions is/are valid ?
(a) 1 only (b) 2 and 3 only
(c) 1 and 3 only (d) All

Question No. 1 2 3 4 5

Answers

Question No. 6 7 8 9 10

Answers

Question No. 11 12 13 14 15

Answers

Question No. 16 17 18 19 20

Answers

Question No. 21 22 23 24 25

Answers

Question No. 26 27

Answers

USPC - CSAT 125


UPSC - CSAT -1
6.2 CSAT 2012 RC Questions
Passage 1

The poor especially in market economies, need the strength that collectivities offer for creating
more economic, social and political space for themselves, for enhancing their socio-economic well-
being and voice, and as a protection against free market individualism. It has been argued that a group
approach to farming, especially in the form of bottom up agricultural production collectivities, offers
substantial scope for poverty alleviation and empowering the poor as well as enhancing agricultural
productivity. To realize this potential, however, the groups would need to be voluntary in nature,
small in size, participative in decision making and equitable in work sharing and benefit distribution.
There are many notable examples of such collectivities to be found in varied contexts, such as in the
transition economies. All of these bear witness to the possibility of successful cooperation under given
conditions. And although the gender impact of the family cooperatives in the transition economies
are uncertain, the Indian examples of women-only groups farming offer considerable potential for
benefiting women.
1. Agricultural collectivities such as group based farming can provide the rural poor.
1. Empowerment
2. Increased agricultural productivity.
3. Safeguard against exploitative markets.
4. Surplus production of agricultural commodities.
Select the correct answer using the codes given below:
(a) 1, 2, 3 and 4 (b) 1, 2 and 3 only
(c) 2 and 4 only (d) 1, 3 and 4 only
2. What does the author imply by “gender impact”?
(a) Women are doubtful participants in cooperatives.
(b) Family cooperatives may not include women.
(c) Women benefiting from group farming.
(d) Women’s role in transition economies is highly restrictive.
3. Consider the following assumptions:
1. It is imperative for transition economies to have agricultural collectivities.
2. Agricultural productivity can be increased by group approach to farming.
With reference to the above passage, which of these assumptions is/are valid?
(a) 1 only (b) 2 only
(c) Both 1 and 2 (d) Neither 1 nor 2

126 USPC - CSAT


UPSC - CSAT -1
Passage 2

In a typical Western liberal context, deepening of democracy invariably leads to consolidation


of ‘liberal values’. In the Indian context, democratization is translated into greater involvement
of people not as ‘individuals’ which is a staple to liberal discourse, but as communities or groups.
Individuals are getting involved in the public sphere not as ‘optimized’ individuals but as members
of primordial communities drawn on religious or caste identity. Community-identity seems to be the
governing force. It is not therefore surprising that the so-called peripheral groups continue to maintain
their identities with reference to the social groups (caste, religion or sect) to which they belong while
getting involved in the political processes despite the fact that their political goals remain more or
less identical. By helping to articulate the political voice of the marginalized, democracy in India has
led to ‘a loosening of social structures’ and empowered the peripherals to be confident of their ability
to improve the socio-economic conditions in which they are placed. This is a significant political
process that had led to a silent revolution through a meaningful transfer of power from the upper caste
elites to various subaltern groups within the democratic framework of public governance.
4. According to the passage, what does “deepening of democracy” mean in the Western
context?
(a) Consolidation of group and class identities.
(b) Democratization translated as greater involvement of people.
(c) Democratization as greater involvement of ‘atomized’ individuals in the public sphere.
(d) None of the statements (a), (b) and (c) given above is correct in this context.
5. Greater democratization in India has not necessarily led to
(a) The dilution of caste and communal identities in the public sphere.
(b) Irrelevance of community identity as a governing force in Indian politics.
(c) Marginalization of elite groups in society.
(d) Relative unimportance of hereditary identities over class identities.
6. What is the “silent revolution” that has occurred in the Indian democratic process?
(a) Irrelevance of caste and class hierarchies in political processes.
(b) Loosening of social strictures in voting behavior and patterns.
(c) Social change through transfer of power from upper caste elites to subaltern groups.
(d) All the statements (a), (b) and (c) given above are correct in this context.

Passage 3

Education, without a doubt, has an important functional, instrumental and utilitarian dimension.
This is revealed when one asks questions such as ‘what is the purpose of education?’ The answers,

USPC - CSAT 127


UPSC - CSAT -1
too often, are ‘to acquire qualifications for employment/upward mobility’, ‘wider/higher (in terms of
income) opportunities’, and ‘to meet the needs for trained human power in diverse fields for national
development’. But in its deepest sense education is not instrumentalist. That is to say, it is not to
be justified outside of itself because it leads to the acquisition of formal skills or of certain desired
psychological – social attributes. It must be respected in itself. Education is thus not a commodity to
be acquired or possessed and then used, but a process of inestimable importance to individuals and
society, although it can and does have enormous use value. Education then, is a process of expansion
and conversion, not in the sense of converting or turning students into doctors or engineers, but the
widening and turning out of the mind — the creation, sustenance and development of self-critical
awareness and independence of thought. It is an inner process of moral – intellectual development.
7. What do you understand by the ‘instrumentalist’ view of education?
(a) Education is functional and utilitarian in its purposes.
(b) Education is meant to fulfil human needs.
(c) The purpose of education is to train the human intellect.
(d) Education is meant to achieve moral development.
8. According to the passage, education must be respected in itself because:
(a) It helps to acquire qualifications for employment.
(b) It helps in upward mobility and acquiring social status.
(c) It is an inner process of moral and intellectual development.
(d) All the (a), (b) and (c) given above are correct in this context.
9. Education is a process in which:
(a) Students are converted into trained professionals.
(b) Opportunities for higher income are generated.
(c) Individuals develop self-critical awareness and independence of thought.
(d) Qualifications for upward mobility are acquired.

Passage 4

Chemical pesticides lose their role in sustainable agriculture if the pests evolve resistance. The
evolution of pesticide resistance is simply natural selection in action. It is almost certain to occur
when vast numbers of a genetically variable population are killed. One or a few individuals may
be unusually resistant (perhaps because they possess an enzyme that can detoxify the pesticide).
If the pesticide is applied repeatedly, each successive generation of the pest will contain a larger
proportion of resistant individuals. Pests typically have a high intrinsic rate of reproduction, and so a
few individuals in one generation may give rise to hundreds or thousands in the next, and resistance
spreads very rapidly in a population.

128 USPC - CSAT


UPSC - CSAT -1
This problem was often ignored in the past, even though the first case of DDT
(dichlorodiphenyltrichloroethane) resistance was reported as early as 1946. There is exponential
increase in the numbers of invertebrates that have evolved resistance and in the number of pesticides
against which resistance has evolved. Resistance has been recorded in every family of arthropod
pests (including dipterans such as mosquitoes and house flies, as well as beetles, moths, wasps, fleas,
lice and mites) as well as in weeds and plant pathogens. Take the Alabama leafworm, a moth pest of
cotton, as an example. It has developed resistance in one or more regions of the world to aldrin, DDT,
dieldrin, endrin, lindane and toxaphene.
If chemical pesticides brought nothing but problems, — if their use was intrinsically and acutely
unsustainable — then they would already have fallen out of widespread use. This has not happened.
Instead, their rate of production has increased rapidly. The ratio of cost to benefit for the individual
agricultural producer has remained in favour of pesticide use. In the USA, insecticides have been
estimated to benefit the agricultural products to the tune of around $5 for every $1 spent.
Moreover, in many poorer countries, the prospect of imminent mass starvation, or of an epidemic
disease, are so frightening that the social and health costs of using pesticides have to be ignored.
In general the use of pesticides is justified by objective measures such as ‘lives saved’, ‘economic
efficiency of food production’ and ‘total food produced’. In these very fundamental senses, their
use may be described as sustainable. In practice, sustainability depends on continually developing
new pesticides that keep at least one step ahead of the pests — pesticides that are less persistent,
biodegradable and more accurately targeted at the pests.
10. “The evolution of pesticide resistance is natural selection in action.” What does it
actually imply?
(a) It is very natural for many organisms to have pesticide resistance.
(b) Pesticide resistance among organisms is a universal phenomenon.
(c) Some individuals in any given population show resistance after the application of
pesticides.
(d) None of the statements (a), (b) and (c) given above is correct.
11. With reference to the passage, consider the following statements:
1. Use of chemical pesticides has become imperative in all the poor countries of the world.
2. Chemical pesticides should not have any role in sustainable agriculture.
3. One pest can develop resistance to many pesticides.
Which of the statements given above is/are correct?
(a) 1 and 2 only (b) 3 only
(c) 1 and 3 only (d) 1, 2 and 3

USPC - CSAT 129


UPSC - CSAT -1
12. Though the problems associated with the use of chemical pesticides is known for a long
time, their widespread use has not waned. Why?
(a) Alternatives to chemical pesticides do not exist at all.
(b) New pesticides are not invented at all.
(c) Pesticides are biodegradable.
(d) None of the statements (a), (b) and (c) given above is correct.
13. How do pesticides act as agents for the selection of resistant individuals is any pest
population?
1. It is possible that in a pest population the individuals will behave differently due to their
genetic makeup.
2. Pests do possess the ability to detoxify the pesticides.
3. Evolution of pesticide resistance is equally distributed in pest population.
Which of the statements given above is/are correct?
(a) 1 only (b) 1 and 2 only
(c) 3 only (d) 1, 2 and 3
14. Why is the use of chemical pesticides generally justified by giving the examples of poor
and developing countries?
1. Developed countries can afford to do away with use of pesticides by adapting to
organic farming, but it is imperative for poor and developing countries to use chemical
pesticides.
2. In poor and developing countries, the pesticide addresses the problem of epidemic
diseases of crops and eases the food problem.
3. The social and health costs of pesticide use are generally ignored in poor and developing
countries.
Which of the statements given above is/are correct?
(a) 1 only (b) 1 and 2 only
(c) 2 only (d) 1, 2 and 3
15. What does the passage imply?
(a) Alternative options to chemical pesticides should be promoted.
(b) Too much use of chemicals is not good for the ecosystem.
(c) There is no scope for the improvement of pesticides and making their use sustainable.
(d) Both the statements (a) and (b) above are correct.

130 USPC - CSAT


UPSC - CSAT -1
Passage 5

Today’s developing economies use much less energy per capital than developed countries such as
the United States did at similar incomes, showing the potential for lower-carbon growth. Adaptation
and mitigation need to be integrated into a climate-smart development strategy that increases resilience,
reduces the threat of further global warming, and improves development outcomes. Adaptation and
mitigation measures can advance development, and prosperity can raise incomes and foster better
institutions. A healthier population living in better-built houses and with access to bank loans and
social security is better equipped to deal with a changing climate and its consequences. Advancing
robust, resilient development policies that promote adaptation is needed today because changes in the
climate, already begun, will increase even in the short term.
The spread of economic prosperity has always been intertwined with adaptation to changing
ecological conditions. But as growth has altered the environment and as environmental change
has accelerated, sustaining growth and adaptability demands greater capacity to understand our
environment, generate new adaptive technologies and practices, and diffuse them widely. As economic
historians have explained, much of humankind’s creative potential has been directed at adapting to the
changing world. But adaptation cannot cope with all the impacts related to climate change, especially
as larger changes unfold in the long term.
Countries cannot grow out of harm’s way fast enough to match the changing climate. And some
growth strategies, whether driven by the government or the market, can also add to vulnerability —
particularly if they overexploit natural resources. Under the Soviet development plan, irrigated cotton
cultivation expanded in water-stressed Central Asia and led to the near disappearance of the Aral Sea,
threatening the livelihoods of fishermen, herders and farmers. And clearing mangroves – the natural
coastal buffers against storm surges – to make way for intensive farming or housing development,
increases the physical vulnerability of coastal settlements, whether in Guinea or in Louisiana.
16. Which of the following conditions of growth can add to vulnerability?
1. When the growth occurs due to excessive exploitation of mineral resources and forests.
2. When the growth brings about a change in humankind’s creative potential.
3. When the growth is envisaged only for providing houses and social security to the
people.
4. When the growth occurs due to emphasis on farming only.
Select the correct answer using the codes given below:
(a) 1 only (b) 2, 3 and 4 only
(c) 1 and 4 only (d) 1, 2, 3 and 4
17. What does low-carbon growth imply in the present context?
1. More emphasis on the use of renewable sources of energy.

USPC - CSAT 131


UPSC - CSAT -1
2. Less emphasis on manufacturing sector and more emphasis on agriculture sector.
3. Switching over from monoculture practices to mixed farming.
4. Less demand for goods and services.
Select the correct answer using the codes given below:
(a) 1 only (b) 2, 3 and 4 only
(c) 1 and 4 only (d) None of the above implies low-carbon growth
18. Which of the following conditions is/are necessary for sustainable economic growth?
1. Spreading of economic prosperity more.
2. Popularizing/spreading of adaptive technologies widely.
3. Investing on research in adaptation and mitigation technologies.
Select the correct answer using the codes given below:
(a) 1 only (b) 2 and 3 only
(c) 1 and 3 only (d) 1, 2 and 3
19. Which of the following inferences can be made from the passage?
1. Rainfed crops should not be cultivated in irrigated areas.
2. Farming under water-deficient areas should not be a part of development strategy.
Select the correct answer using the codes given below:
(a) 1 only (b) 2 only
(c) Both 1 and 2 (d) Neither 1 nor 2
20. Consider the following assumptions:
1. Sustainable economic growth demands the use of creative potential of man.
2. Intensive agriculture can lead to ecological backlash.
3. Spread of economic prosperity can adversely affect the ecology and environment.
With reference to the passage, which of the above assumptions is/are valid?
(a) 1 only (b) 2 and 3 only
(c) 1 and 3 only (d) 1, 2 and 3
21. Which one of the following statements constitutes the central theme of this passage?
(a) Countries with greater economic prosperity are better equipped to deal with the
consequences of climate change.
(b) Adaptation and mitigation should be integrated with development strategies.
(c) Rapid economic growth should not be pursued by both developed and developing

132 USPC - CSAT


UPSC - CSAT -1
economies.
(d) Some countries resort to overexploitation of natural resources for the sake of rapid
development.

Passage 6

Invasions of exotic species into new geographic areas sometimes occur naturally and without human
agency. However, human actions have increased this trickle to a flood. Human-caused introductions
may occur either accidentally as a consequence of human transport, or intentionally but illegally to
serve some private purpose or legitimately to procure some hoped-for public benefit by bringing a
pest under control, producing new agricultural products or providing novel recreational opportunities.
Many introduced species are assimilated into communities without much obvious effect. However,
some have been responsible for dramatic changes to native species and natural communities. For
example, the accidental introduction of the brown tree snake Boiga irregularis into Guam, an island
in the Pacific, has through, nest predation reduced 10 endemic forest bird species to the point of
extinction.
One of the major reasons for the world’s great biodiversity is the occurrence of centers of endemism
so that similar habitats in different parts of the world are occupied by different groups of species that
happen to have evolved there. If every species naturally had access to everywhere on the globe, we
might expect a relatively small number of successful species to become dominant in each biome.
The extent to which this homogenization can happen naturally is restricted by the limited powers of
dispersal of most species in the face of the physical barriers that exist to dispersal. By virtue of the
transport opportunities offered by humans, these barriers have been breached by an ever-increasing
number of exotic species. The effects of introductions have been to convert a hugely diverse range of
local community compositions into something much more homogeneous.
It would be wrong, however, to conclude that introducing species to a region will inevitably cause
a decline in species richness there. For example, there are numerous species of plants, invertebrates
and vertebrates found in continental Europe but absent from the British Isles (many because they have
so far failed to recolonize after the last glaciations). Their introduction would be likely to augment
British biodiversity. The significant detrimental effect noted above arises where aggressive species
provide a novel challenge to endemic biotas ill-equipped to deal with them.
22. With reference to the passage, which of the following statements is correct?
(a) Introduction of exotic species into new geographical areas always leads to reduced
biodiversity.
(b) Exotic species introduced by man into new areas have always greatly altered the native
ecosystems.
(c) Man is the only reason to convert a hugely diverse range of local community compositions
into more homogeneous ones.

USPC - CSAT 133


UPSC - CSAT -1
(d) None of the statements (a), (b) and (c) is correct in this context.
23. Why does man introduce exotic species into new geographical areas?
1. To breed exotic species with local varieties.
2. To increase agricultural productivity.
3. For beautification and landscaping.
With of the above statements is/are correct?
(a) 1 only (b) 2 and 3 only
(c) 1 and 3 only (d) 1, 2 and 2
24. How is homogenization prevented under natural conditions?
(a) Evolution of groups of species specific of local habitats.
(b) Presence of oceans and mountain ranges.
(c) Strong adaptation of groups of species to local physical and climatic conditions.
(d) All the statement (a), (b) and (c) given above are correct in this context.
25. How have the human beings influenced the biodiversity?
1. By smuggling live organisms.
2. By building highways.
3. By making ecosystems sensitive so that new species are not allowed.
4. By ensuring that new species do not have major impact on local species.
Which of the statements given above are correct?
(a) 1 and 2 (b) 2 and 3
(c) 1 and 3 (d) 2 and 4
26. What can be the impact of invasion of exotic species on an ecosystem?
1. Erosion of endemic species.
2. Change in the species composition of the community of the ecosystem.
Select the correct answer using the codes given below:
(a) 1 only (b) 2 only
(c) Both 1 and 2 (d) Neither 1 nor 2

Passage 7

Most champions of democracy have been rather reticent in suggesting that democracy would itself
promote development and enhancement of social welfare — they have tended to see them as good

134 USPC - CSAT


UPSC - CSAT -1
but distinctly separate and largely independent goals. The detractors of democracy, on the other hand,
seemed to have been quite willing to express their diagnosis of what they see as serious tensions
between democracy and development. The theorists of the practical split — “Make up your mind
: do you want democracy, or instead, do you want development ?” — often come, at least to start
with, from East Asian countries, and their voice grew in influence as several of these countries were
immensely successful — through the 1970s and 1980s and even later — in promoting economic
growth without pursuing democracy.
To deal with these issues we have to pay particular attention to both the content of what can be
called development and to the interpretation of democracy (in particular to the respective roles of
voting and of public reasoning). The assessment of development cannot be divorced from the lives
that people can lead and the real freedom that they enjoy. Development can scarcely be seen merely in
terms of enhancement of inanimate objects of convenience, such as a rise in the GNP (or in personal
incomes), or industrialization — important as they may be as means to the real ends. Their value must
depend on what they do to the lives and freedom of the people involved, which must be central to the
idea of development.
If development is understood in a broader way, with a focus on human lives, then it comes
immediately clear that the relation between development and democracy has to be seen partly in
terms of their constitutive connection, rather than only through their external links. Even though
the question has often been asked whether political freedom is “conductive to development”, we
must not miss the crucial recognition that political liberties and democratic rights are among the
“constituent components” of development. Their relevance for development does not have to be
established indirectly through their contribution to the growth of GNP.
27. According to the passage, which is a serious tension perceived between democracy and
development by the detractors of democracy?
(a) Democracy and development are distinct and separate goals.
(b) Economic growth can be promoted successfully without pursuing a democratic system
of governance.
(c) Non-democratic regimes deliver economic growth faster and far more successfully than
democratic ones.
(d) All the statements (a), (b) and (c) given above are correct in this context.
28. According to the passage, what should be the ultimate assessment/aim/view of
development?
(a) Rise in the per capita income and industrial growth rates.
(b) Improvement in the Human Development Index and GNP.
(c) Rise in the savings and consumption trends.
(d) Extent of real freedom that citizens enjoy.

USPC - CSAT 135


UPSC - CSAT -1
29. What does a “constitutive” connection between democracy and development imply?
(a) The relation between them has to be seem through external links.
(b) Political and civil rights only can lead to economic development.
(c) Political liberties and democratic rights are essential elements of development.
(d) None of the statements (a), (B) and (c) given above is correct in this context.

Passage – 8

The need for Competition Law becomes more evident when foreign direct investment (FDI)
is liberalized. The impact of FDI is not always pro-competitive. Very often FDI takes the form of
a foreign corporation acquiring a domestic enterprise or establishing a joint venture with one. By
making such an acquisition the foreign investor may substantially lessen competition and gain a
dominant position in the relevant market, thus charging higher prices. Another scenario is where the
affiliates of two separate multinational companies (MNCs) have been established in competition with
one another in a particular developing economy, following the liberalization of FDI. Subsequently, the
parent companies overseas merge. With the affiliates no longer remaining independent, competition
in the host country may be virtually eliminated and the prices of the products may be artificially
inflated. Most of these adverse consequences of mergers and acquisitions by MNCs can be avoided
if an effective competition law is in place. Also, an economy that has implemented an effective
competition law is in a better position to attract FDI than one that has not. This is not just because
most MNCs are expected to be accustomed to the operation of such a law in their home countries and
know how to deal with such concerns but also that MNCs expect competition authorities to ensure a
level playing field between domestic and foreign firms.
30 With reference to the passage, consider the following statements:
1. It is desirable that the impact of Foreign Direct Investment should be pro-competitive.
2. The entry of foreign investors invariably leads to the inflated prices in domestic markets.
Which of the statements given above is/are correct.
(a) 1 only (b) 2 only
(c) Both 1 and 2 (d) Neither 1 nor 2
31 According to the passage, how does a foreign investor dominate the relevant domestic
market?
1. Multinational companies get accustomed to domestic laws.
2. Foreign companies establish joint ventures with domestic companies.
3. Affiliates in a particular market/sector lose their independence as their parent companies
overseas merge.
4. Foreign companies lower the cost of their products as compared to that of products of

136 USPC - CSAT


UPSC - CSAT -1
domestic companies.
Which of the statements given above are correct?
(a) 1 and 2 only (b) 2 and 3 only
(c) 1, 2 and 3 only (d) 1, 2, 3 and 4
32. What is inference from this passage?
(a) Foreign investors and multinational companies always dominate the domestic market.
(b) It is not in the best interests of the domestic economy to allow mergers of companies.
(c) With competition law, it is easy to ensure a level playing field between domestic and
foreign firms.
(d) For countries with open economy, Foreign Direct Investment is essential for growth.

Question No. 1 2 3 4 5

Answers

Question No. 6 7 8 9 10

Answers

Question No. 11 12 13 14 15

Answers

Question No. 16 17 18 19 20

Answers

Question No. 21 22 23 24 25

Answers

Question No. 26 27 28 29 30

Answers

Question No. 26 27 28 29 30

Answers

USPC - CSAT 137


UPSC - CSAT -1
6.3 CSAT 2013 RC Questions
Passage 1

The subject of democracy has become severely muddled because of the way the rhetoric surrounding
it has been used in recent years. There is, increasingly, an oddly confused dichotomy between those
who want to ‘impose’democracy on countries in the non- Western world (in these countries’ ‘own
interest’, of course) and those who are opposed to such imposition (because of the respect for the
countries’ ‘own ways’) But the entire language of imposition used by both sides is extraordinarily
inappropriate since it makes the implicit assumption that democracy belongs exclusively to the west,
taking it to be a quintessentially ‘Western’ idea which has originated and flourished only in the west.
But the thesis and the pessimism it generates about the possibility of democratic practice in
the world would be extremely hard to justify. There were several experiments in local democracy
in ancient India. Indeed, in understanding the roots of democracy in the world, we have to take an
interest in the history of people participation and public reasoning in different parts of the world. We
have to look beyond thinking of democracy only in terms of Europenan and American evolution. We
would fail to understand the pervasive demands for participatory living, on which Aristotle spoke
with far- reaching insight, if we take democracy to be a kind of a specialized cultural product of the
West.
It cannot, of course, be doubted that the institutional structure of the contemporary practice of
democracy is largely the product of European and American experience over the last few centuries.
This is extremely important to recognize since these developments in institutional formats were
immensely innovative and ultimately effective. There can be little doubt that there is a major Western
achievement here.
1 Which of the following is closest to the view of democracy as mentioned in the above
passage?
(a) The subject of democracy is a muddle due to a desire to portray it as a western concept,
‘alien’ to non- Western countries.
(b) The language of imposition of democracy is inappropriate. There is, however, a need to
consider this concept in the backdrop of culture of own ways of non- western society
(c) While democracy is not essentially a western idea belonging exclusively to the west, the
institutional structure of current democratic practices has been their contribution
(d) None of the statements (a), (b) and (c) given above is correct.
2 With reference to the passage, the following assumptions have been made:
(i) Many of the non - Western countries are unable to have democracy because they take
democracy to be a specialized cultural product of the west.

138 USPC - CSAT


UPSC - CSAT -1
(ii) Western countries are alway trying to impose democracy on non- western countries.
Which of the above is/ are valid assumption/assumptions?
(a) 1 only (b) 2 only
(c) Both 1 and 2 (d) Neither 1 nor 2

Passage 2

Corporate governance is based on principles such as conducting the business with all integrity
and fairness, being transparent with regard to all transactions, making all the necessary disclosures
and decisions, complying with all the laws of the land, accountability and responsibility towards the
stakeholders and commitment to conducting business in an ethical manner. Another point which is
highlighted on corporate governance is the need for those in control to be able to distinguish between
what are personal and corporate funds while managing a company.
Fundamentally, there is a level of confidence that is associated with a company that is known to
have good corporate governance. The presence of an active group of independent directors on the
board contributes a great deal towards ensuring confidence in the market. Corporate governance is
known to be one of the criteria that foreign institutional investors are increasingly depending on when
deciding on which companies to invest in. It is also known to have a positive influence on the share
price of the company. Having a clean image on the corporate governance front could also make it
easier for corporate governess front could also make it easier for companies to source capital at more
reasonable costs. Unfortunately, corporate governance often become the centre of discussion only
after the exposure of a large scam.
3 According to the passage, which of the following should be the practice / practices in
good corporate governance?
(i) Companies should always comply with labour and tax laws of the land.
(ii) Every company in the country should have a government representative as one one of
the independent directors on the board to ensure transparency.
(iii) The manager of a company should never invest his personal funds in the company.
Select the correct answer using the codes given below
(a) (i) only (b) (ii) and (iii) only
(c) (i) and (iii) only (d) (i), (ii) and (iii)
4 According to the passage, which of the following is/ are the major benefit/ benefits of
good corporate governance?
(i) Good corporate governance leads to increase in share price of the company
(ii) A company with good corporate governance always increases its business turnover
rapidly

USPC - CSAT 139


UPSC - CSAT -1
(iii) Good corporate governance is the main criterion for foreign institutional investors when
they decide to buy a company
Answer options :
(a) (i) only (b) (ii) and (iii) only
(c) (i) and (iii) (d) (i), (ii) and (iii)

Passage 3

Malnutrition most commonly occurs between the ages of six months and two years. This happens
despite the child’s food requirements being less than that of an older child. Malnutrition is often
attributed to poverty, but it has been found that even in households where adults eat adequate quantities
of food, more than 50 per cent of children-under-five do not consume enough food. The child’s
dependence on someone else to feed him/her is primarily responsible for the malnutrition. Very often
the mother is working and the responsibility of feeding the young child is left to an older sibling. It
is therefore crucial to increase awareness regarding the child’s food needs and how to satisfy them.
5 According to the passage, malnutrition in children can be reduced
(a) if the children have regular intake of food
(b) after they cross the age of five
(c) if the food needs of younger children are known
(d) if the responsibility of feeding younger children is given to adults.
6 According to the author, poverty is not the main cause of malnutrition, but the fact that
(i) taking care of younger ones is not a priority for working mothers.
(ii) awareness of nutritional needs is not propagated by the Public Health authorities.
Select the correct answer using the given below :
(a) (i) only (b) (ii) only
(c) Both (i) and (ii) (d) Neither (i) nor (ii)

Passage 4

A number of empirical studies find that farmers are risk- averse, though only moderately in many
cases. There is also evidence to show that farmers’ risk aversion results in cropping patterns and input
use designed to reduce risk rather than to maximize income. Farmers adopt a number of strategies to
manage and cope with agricultural risks. These include practices like crop and field diversification,
non-farm employment storage of stocks and strategic migration of family members. There are also
institutions ranging from share tenancy to kinship, extended family and informal credit agencies. One

140 USPC - CSAT


UPSC - CSAT -1
major obstacle to risk sharing by farmers is that the same type of risks can affect a large number of
farmers in the region. Empirical studies show that the traditional methods are not adequate. Hence
there is a need for policy interventions, especially measures that cut across geographical regions.
Policies may aim at tackling agricultural risks directly or indirectly. Examples of risk-specific
policies are crop insurance, price stabilization and the development of varieties resistant to pests and
diseases. Policies which affect risk indirectly are irrigation, subsidized credit and access to information.
No single risk-specific policy is sufficient to reduce risk and is without side-effects, whereas policies
not specific to risk influence the general situation and affect risks only indirectly. Crop insurance,
as a policy measure to tackle agricultural risk directly, deserves careful consideration in the Indian
context and in many other developing countries - because the majority of farmers depend on rain-fed
agriculture and in many areas yield variability is the predominant cause of their income instability.
7 The need for policy intervention to mitigate risks in agriculture is because?
(a) farmers are extremely risk-averse.
(b) farmers do not know how to mitigate risks.
(c) the methods adopted by farmers and existing risk sharing institutions are not adequate.
(d) majority of farmers depend on rain-fed agriculture.
8 Which of the following observations emerges from the above passage ?
(a) One can identify a single policy that can reduce risk without any side-effect.
(b) No single risk-specific policy is sufficient to reduce agricultural risk.
(c) Policies which affect risk indirectly can eliminate it.
(d) Government’s policy intervention can mitigate agricultural risk completely.

Passage 5

Financial markets in India have acquired greater depth and liquidity over the years. Steady reforms
since 1991 have led to growing linkages and integration of the Indian economy and its financial
system with the global economy. Weak global economic prospects and continuing uncertainties in the
international financial markets therefore, have had their impact on the emerging market economies.
Sovereign risk concerns, particularly in the Euro area, affected financial markets for the greater part
of the year, with the contagion of greece’s sovereign debt problem spreading to India and other
economies by way of higher - than - normal levels of volatility.
The funding constraints in international financial markets could impact both the availability and
cost of foreign funding for banks and corporate. Since the Indian financial system is bank dominated,
Bank’s ability to withstand stress is critical to overall financial stability. Indian banks, however,
remain robust, notwithstanding a decline in capital to risk -weighted assets ratio and a rise in non

USPC - CSAT 141


UPSC - CSAT -1
- Performing asset levels in the recent past. Capital adequacy levels remain above the regulatory
requirements. The financial market infrastructure continues to function without any major disruption.
With further globalization, consolidation, deregulation, and diversification of the financial system,
the banking business may become more complex and riskier Issues like risk and liquidity management
and enhancing skill therefore assume greater significance.
9. According to the passage, the financial markets in the emerging market economies
including India had the adverse impact in recent years due to
(i) weak global economic prospects.
(ii) uncertainties in the international financial markets.
(iii) sovereign risk concerns in the Euro area.
(iv) bad monsoons and the resultant crop loss.
Select the correct answer using the code given below :
(a) i and ii only (b) i, ii and iii
(c) ii and iii only (d) ii, iii and iv
10 The Indian financial markets are affected by global changes mainly due to the.... ?
(a) Increased inflow of remittances from abroad.
(b) Enormous increase in the foreign exchange reserves.
(c) Growing global linkages and integration of the Indian financial markets.
(d) Contagion of greece’s sovereign debt problem.
11 According to the passage, in the Indian financial system, bank’s ability to withstand
stress is critical to ensure overall financial stability because Indian Financial system is :
(a) controlled by the government of India
(b) less integrated with banks
(c) controlled by Reserve Bank of India
(d) dominated by banks.
12 Risk and liquidity management assumes more importance in the Indian banking
system in future due to
(i) further globalization
(ii) more consolidation and deregulation of the financial system
(iii) further diversification of the financial system
(iv) more financial inclusion in the economy

142 USPC - CSAT


UPSC - CSAT -1
Select the correct answer using the code given below :
(a) i, ii and iii (b) ii, iii and iv
(c) i and ii only (d) iii and iv only

Passage 6

Crude mineral oil comes out of the earth as a thick brown or black liquid with a strong smell. It is
a complex mixture of many different substances, each with its own individual qualities. Most of them
are combinations of hydrogen and carbon in varying proportions. Such hydrocarbons are also found
in other forms such as bitumen, asphalt and natural gas. Mineral oil originates from the carcasses of
tiny animals and from plants that live in the sea. Over millions of years, these dead creatures form
large deposits under the sea-bed; and ocean currents cover them with a blanket of sand and silt. As this
mineral hardens, it becomes sedimentary rock and effectively shuts out the oxygen, so preventing the
complete decomposition of the marine deposits underneath. The layers of sedimentary rock become
thicker and heavier. Their pressure produces heat, which transforms the tiny carcasses into crude oil
in a process that is still going on today.
13 Mineral oil deposits under the sea do not get completely decomposed because they
(a) are constantly washed by the ocean currents.
(b) become rock and prevent oxygen from entering them.
(c) contain a mixture of hydrogen and carbon.
(d) are carcasses of organisms lying in saline conditions.
14 Sedimentary rock leads to the formation of oil deposit because :
(a) there are no saline conditions below it.
(b) it allows some dissolved oxygen to enter the dead organic matter below it.
(c) weight of overlying sediment layers causes the production of heat
(d) it contains the substances that catalyze the chemical reactions required to change dead
organisms into oil.

Passage 7

The law in many parts of the world increasingly restricts the discharge of agricultural slurry into
watercourses. The simplest and often the most economically sound practice returns the material to
the land as semisolid manure or as sprayed slurry. This dilutes its concentration in the environment
to what might have occurred in a more primitive and sustainable type of agriculture and converts
pollutant into fertiliser. Soil microorganisms decompose the organic components of sewage and slurry
and most of the mineral nutrients become available to be absorbed again by the vegetation.

USPC - CSAT 143


UPSC - CSAT -1
The excess input of nutrients, both nitrogen and phosphorus - based, from agricultural runoff (and
human sewage) has caused many ‘healthy’ oligotrophic lakes (low nutrient concentrations, low plant
productivity with abundant water weeds, and clear water) to change to eutrophic condition where high
nutrient input lead to high phytoplankton productivity (sometimes dominated by bloom - forming
toxic species). This makes the water turbid, eliminates large plants and, in the worst situations, leads
to anoxia and fish kills; so called cultural eutrophication. Thus, important ecosystem services are
lost, including the provisioning service of wild-caught fish and the cultural services associated with
recreation.
The process of cultural eutrophication of lakes has been understood for some time. But only
recently did scientists notice huge ‘dead zones’ in the oceans near river outlets, particularly those
draining large catchment areas such as the Mississippi in North America and the Yangtze in China.
The nutrient-enriched water flows through streams, rivers and lakes, and eventually to the estuary and
ocean where the ecological impact may be huge, killing virtually all invertebrates and fish in areas up
to 70,000 km2 in extent. More than 150 sea areas worldwide are now regularly starved of oxygen as
a result of decomposition of algal blooms, fuelled particularly by nitrogen from agricultural runoff of
fertilizers and sewage from large cities. Oceanic dead zones are typically associated with industrialized
nations and usually lie off countries that subsidize their agriculture, encouraging farmers to increase
productivity and use more fertilizer.
15. According to the passage, why should the discharge of agricultural slurry into
watercourses be restricted?
(i) Losing nutrients in this way is not a good practice economically.
(ii) Watercourses do not contain the microorganisms that can decompose organic components
of agricultural slurry.
(iii) The discharge, may lead to the eutrophication of water bodies.
Select the correct answer using the codes given below :
(a) i only (b) ii and iii only
(c) i and iii only (d) i, ii and iii
16. The passage refers to the conversion of ‘pollutant to fertilizer’. What is pollutant and
what is fertilizer in this context?
(a) Decomposed organic component of slurry is pollutant and microorganisms in soil
constitute fertilizer.
(b) Discharged agricultural slurry is pollutant and decomposed slurry in soil is fertilizer.
(c) Sprayed slurry is pollutant and watercourse is fertilizer.
(d) None of the above expressions is correct in this context.

144 USPC - CSAT


UPSC - CSAT -1
17. According to the passage, What are the effects of indiscriminate use of fertilizers?
(i) Addition of pollutants to the soil and water.
(ii) Destruction of decomposer microorganisms in the soil.
(iii) Nutrient enrichment of water bodies.
(iv) Creation of algal blooms.
Select the correct answer using the codes given below :
(a) i ,ii and iii only (b) i, iii and iv only
(c) ii and iv only (d) i, ii, iii and iv
18. What is/are the characteristics of a water body with cultural eutrophication?
(i) Loss of ecosystem servies
(ii) Loss of flora and fauna
(iii) Loss of mineral nutrients
Select the correct answer using the codes given below :
(a) i only (b) i and ii only
(c) ii and iii only (d) i, ii and iii
19. What is the central theme of this passage?
(a) Appropriate legislation is essential to protect the envionment.
(b) Modern agriculture is responsible for the destruction of environment.
(c) Improper waste disposal from agriculture can destroy the aquatic ecosystems.
(d) Use of chemical fertilizers is undersirable in agriculture.

Passage 8

The miseries of the world cannot be cured by physical help only. Until man’s nature changes, his
Physical needs will always arise, and miseries will always be felt, and no amount of physical help
will remove them completely. The only solution of the problem is to make mankind pure. Ignorance
is the mother of evil and of all the misery we see. Let men have light, let them be pure and spiritually
strong and educated; then alone will misery cease in the world. We may convert every house in the
country into a charitable asylum, we may fill the land with hospitals, but human misery will continue
until man’s character changes.
20. According to the passage, which of the following statements is most likely to be true as
the reason for man’s miseries?
(a) The poor economic and social conditions prevailing in society.

USPC - CSAT 145


UPSC - CSAT -1
(b) The refusal on the part of man to change his character.
(c) The absence of physical and material help from his society.
(d) Ever increasing physical needs due to changing social structure.
21. With reference to the passage, the following assumptions have been made:
(i) The author gives primary importance to physical and material help in eradicating human
misery.
(ii) Charitable homes, hospitals, etc. can remove human misery to a great extent.
Which of the assumptions is/ are vaild?
(a) i only (b) ii only
(c) Both i and ii (d) Neither i nor ii

Passage 9

Ecological research over the last quarter of the century has established the deleterious effects of
habitat fragmentation due to mining, highways and such other intrusions on forests. When a large
block of forests gets fragmented into smaller bits, the edges of all these bits come into contact with
human activities resulting in the degradation of the entire forests. Continuity of forested landscapes and
corridors gets disrupted affecting several extinction-prone species of wild life. Habitat fragmentation
is therefore considered as the most serious threat to biodiversity conservation. Ad hoc grants of forest
lands to mining companies coupled with rampant illegal mining is aggravating this threat.
22. What is the central focus of this passage?
(a) Illegal mining in forests (b) Extinction of wildlife
(c) Conservation of nature (d) Disruption of habitat
23. What is the purpose of maintaining the continuity of forested landscapes and corridors?
(i) Preservation of biodiversity
(ii) Management of mineral resources.
(iii) Grant of forest lands for human activities
Select the correct answer using the codes given below :
(a) i only (b) i and ii only
(c) ii and iii only (d) i, ii and iii

146 USPC - CSAT


UPSC - CSAT -1

Question No. 1 2 3 4 5

Answers

Question No. 6 7 8 9 10

Answers

Question No. 11 12 13 14 15

Answers

Question No. 16 17 18 19 20

Answers

Question No. 21 22 23

Answers

6.4 CSAT 2014 RC Questions


Passage - 1

The Himalayan ecosystem is highly vulnerable to damage, both due to geological reasons and on
account of the stress caused by increased pressure of population, exploitation of natural resources
and other related challenges. These aspects may be exacerbated due to the impact of climate change.
It is possible that climate change may adversely impact the Himalayan ecosystem through increased
temperature, altered precipitation patterns, episodes of drought and biotic influences. This would
not only impact the very sustenance of the indigenous communities in uplands but also the life of
downstream dwellers across the country and beyond. Therefore, there is an urgent need for giving
special attention to sustain the Himalayan ecosystem. This would require conscious efforts for
conserving all the representative systems.
Further, it needs to be emphasized that the endemics with restricted distribution, and most often
with specialized habitat requirements, are among the most vulnerable elements. In this respect the
Himalayan biodiversity hotspot, with rich endemic diversity, is vulnerable to climate change. The
threats include possible loss of genetic resources and species, habitats and concomitantly a decrease
in ecosystem services, Therefore, conservation of endemic elements in representative ecosystems/
habitats assumes a great significance while drawing conservation plans for the region. Towards
achieving the above, we will have to shift toward contemporary conservation approaches, which
include a paradigm of landscape level interconnectivity between protected area systems. The concept
advocates a shift from the species-habitat focus to an inclusive focus on expanding the bio-geographic
range so that natural adjustments to climate change can proceed without being restrictive.

USPC - CSAT 147


UPSC - CSAT -1
1. Consider the following statements:
According to the passage, the adverse impact of climate change on an ecosystem can be a
(1) Permanent disappearance of some of its flora and fauna.
(2) Permanent disappearance of ecosystem itself.
Which of the statements given above is/are correct?
(a) 1 only (b) 2 only
(c) Both 1 and 2 (d) neither 1 nor 2
2. Which one of the following statements best implies the need to shift toward contemporary
conservation approach?
(a) Exploitation of natural resources causes a stress on the Himalayan ecosystem.
(b) Climate change alters precipitation patterns, causes episodes of drought and biotic
interference.
(c) The rich biodiversity, including endemic diversity, makes the Himalayan region a
biodiversity hotspot.
(d) The Himalayan bio-geographic region should be enabled to adapt to climate change
smoothly.
3. What is the most important message conveyed by the passage?
(a) Endemism is a characteristic feature of Himalayan region.
(b) Conservation efforts should emphasize on bio-geographic ranges rather than on some
species or habitats.
(c) Climate change has adverse impact on the Himalayan ecosystem.
(d) Without Himalayan ecosystem, the life of the communities of uplands and downstream
will have no sustenance.
4 With reference to the passage, the following assumptions have been made.
(1) To maintain natural ecosystems, exploitation of natural resources should be completely
avoided.
(2) Not only anthropogenic but also natural reasons can adversely affect ecosystems.
(3) Loss of endemic diversity leads to the extinction of ecosystems.
Which of the above assumptions is/are correct?
(a) 1 and 2 only (b) 2 only
(c) 2 and 3 only (d) 3 only

148 USPC - CSAT


UPSC - CSAT -1
Passage - 2

It is often forgotten that globalization is not only about policies on international economic
relationships and transactions, but has equally to do with domestic policies of a nation. Policy
changes necessitated by meeting the internationally set conditions (by WTO etc.) of free trade
and investment flows obviously affect domestic producers and investors. But the basic philosophy
underlying globalization emphasizes absolute freedom to markets to determine prices and production
and distribution patterns, and view government interventions as processes that create distortions and
bring in inefficiency. Thus, public enterprises have to be privatized through disinvestments and sales;
sectors and activities hitherto reserved for the public sector have to be opened to the private sector.
This logic extends to the social services like education and health. Any restrictions on the adjustments
in workforce by way of retrenchment of workers should also be removed and exit should be made
easier by removing any restrictions on closures. Employment and wages should be governed by free
play of market forces, as any measure to regulate them can discourage investment and also create
inefficiency in production. Above all, in line with the overall philosophy of reduction in the role of the
State, fiscal reforms should be undertaken to have generally low levels of taxation and government
expenditure should be kept to the minimum to abide by the principle of fiscal prudence. All these are
policy actions on the domestic front and are not directly related to the core items of the globalization
agenda, namely free international flow of goods and finance.
5. According to the passage, under the globalization, government interventions are viewed
as processes leading to
(a) Distortions and inefficiency in the economy.
(b) Optimum use of resources.
(c) More profitability to industries.
(d) Free play of market forces with regard to industries.
6. According to the passage, the basic philosophy of globalization is to
(a) Give absolute freedom to producers to determine prices and production.
(b) Give freedom to producers to evolve distribution patterns.
(c) Give absolute freedom to markets to determine prices, production and employment.
(d) Give freedom to producers to import and export.
7. According to the passage, which of the following is/are necessary for ensuring
globalization?
(1) Privatization of public enterprises
(2) Expansionary policy of public expenditure

USPC - CSAT 149


UPSC - CSAT -1
(3) Free play of market forces to determine wages and employment.
(4) Privatization of social services like education and health.
Select the correct answer using the codes given below.
(a) 1 only (b) 2 & 3 only
(c) 1, 3 & 4 (d) 2, 3 & 4
8. According to the passage, in the process of globalization the State should have
(a) Expanding role. (b) Reducing role.
(c) Statutory role. (d) None of the above roles.

Passage - 3

In recent times, India has grown fast not only compared to its own past but also in comparison
with other nations. But there cannot be any room for complacency because it is possible for the Indian
economy to develop even faster and also to spread the benefits of this growth more widely than has
been done thus far. Before going into details of the kinds of micro-structural changes that we need
to conceptualize and then proceed to implement, it is worthwhile elaborating on the idea of inclusive
growth that constitutes the defining concept behind this Government’s various economic policies
and decisions. A nation interested in inclusive growth views the same growth differently depending
on whether the gains of the growth are heaped primarily on a small segment or shared widely by the
population. The latter is cause for celebration but not the former. In other words, growth must not
be treated as an end in itself but as an instrument for spreading prosperity to all. India’s own past
experience and the experience of other nations suggests that growth is necessary for eradicating
poverty but it is not a sufficient condition. In other words, policies for promoting growth need to be
complemented with policies to ensure that more and more people join in the growth process and,
further, that there are mechanisms in place to redistribute some of the gains to those who are unable
to partake in the market process and, hence, get left behind.
A simple way of giving this idea of inclusive growth a sharper form is to measure a nation’s
progress in terms of the progress of its poorest segment, for instance the bottom 20 per cent of the
population. One could measure the per capita income of the bottom quintile of the population and also
calculate the growth rate of income; and evaluate our economic success in terms of these measures
that pertain to the poorest segment. This approach is attractive because it does not ignore growth
like some of the older heterodox criteria did. It simply looks at the growth of income of the poorest
sections of the population. It also ensures that those who are outside of the bottom quintile do not
get ignored. If that were done, then those people would in all likelihood drop down into the bottom
quintile and so would automatically become a direct target of our policies. Hence the criterion being
suggested here is a statistical summing up of the idea of inclusive growth, which, in turn, leads to two

150 USPC - CSAT


UPSC - CSAT -1
corollaries : to wish that India must strive to achieve high growth and that we must work to ensure
that the weakest segments benefit from the growth.
9. The author’s central focus is on
(a) Applauding India’s economic growth not only against its own past performance, but
against other nations.
(b) Emphasizing the need for economic growth which is the sole determinant of a country’s
prosperity.
(c) Emphasizing inclusive growth where gains of growth are shared widely by the
population.
(d) Emphasizing high growth.
10. The author supports policies which will help
(a) Develop economic growth.
(b) Better distribution of incomes irrespective of rate of growth.
(c) Develop economic growth and redistribute economic gains to those getting left
behind.
(d) Put an emphasis on the development of the poorest segments of society.
11. Consider the following statements.
According to the author, India’s economy has grown but there is no room for complacency as
(1) Growth eradicates poverty.
(2) Growth has resulted in prosperity for all.
Which of the statements given above is/are correct?
(a) 1 only (b) 2 only
(c) Both 1 & 2 (d) neither 1 nor 2

Passage - 4

It is easy for the government to control State-owned companies through nods and winks. So what
really needs to be done as a first step is to put petrol pricing on a transparent formula - if the price
of crude is x and the exchange rate y, then every month or fortnight, the government announces
a maximum price of petrol, which anybody can work out from the x and the y. The rule has to be
worked out to make sure that the oil-marketing companies can, in general, cover their costs. This will
mean that if one company can innovate and cut costs, it will make greater profits. Hence, firms will be
more prone to innovate and be efficient under this system. Once the rule is announced, there should
be no interference by the government. If this is done for a while, private companies will re-enter this

USPC - CSAT 151


UPSC - CSAT -1
market. And once a sufficient number of them are in the fray, we can remove the rule-based pricing
and leave it truly to the market (subject to, of course, the usual regulations of anti-trust and other
competition laws).
12. Consider the following statements:
According to the passage, an oil company can make greater profits, if a transparent formula
for petrol pricing is announced every fortnight or month, by
(1) Promoting its sales. (2) Undertaking innovation.
(3) Cutting costs. (4) Selling its equity shares at higher prices.
Which of the statements given above is/are correct?
(a) 1 only (b) 2 and 3 only
(c) 3 and 4 only (d) 1, 2, and 4
13. Consider the following statements:
According to the passage, private oil companies re-enter the oil producing market if
(1) A transparent rule-based petrol pricing exists.
(2) There is no government interference in the oil producing market.
(3) Subsidies are given by the government.
(4) Regulations of anti-trust are removed.
Which of the statements given above are correct?
(a) 1 and 2 only (b) 2 and 3 only
(c) 3 and 4 only (d) 2 and 4 only

Passage - 5

Many nations now place their faith in capitalism and governments choose it as the strategy to
create wealth for their people. The spectacular economic growth seen in Brazil, China and India
after the liberalization of their economies is proof of its enormous potential and success. However,
the global banking crisis and the economic recession have left many bewildered. The debates tend to
focus on free market operations and forces, their efficiency and their ability for self correction. Issues
of justice, integrity and honesty are rarely elaborated to highlight the failure of the global banking
system. The apologists of the system continue to justify the success of capitalism and argue that the
recent crisis was a blip.
Their arguments betray an ideological bias with the assumptions that an unregulated market is fair and
competent, and that the exercise of private greed will be in the larger public interest.

152 USPC - CSAT


UPSC - CSAT -1
Few recognize the bidirectional relationship between capitalism and greed; that each reinforces the
other. Surely, a more honest conceptualization of the conflicts of interest among the rich and powerful
players who have benefited from the system, their biases and ideology is needed; the focus on the
wealth creation should also highlight the resultant gross inequity.
14. The apologists of the “Free Market System”, according to the passage, believe in
(a) Market without control by government authorities.
(b) Market without protection by the government.
(c) Ability of market to self correct.
(d) Market for free goods and services.
15. With reference to “ideological bias”, the passage implies that
(a) Free market is fair but not competent.
(b) Free market is not fair but competent.
(c) Free market is fair and competent.
(d) Free market is neither fair nor biased.
16. “The exercise of private greed will be in the larger public interest” from the passage
(1) Refers to the false ideology of capitalism.
(2) Underlies the righteous claims of the free market?
(3) Shows the benevolent face of capitalism.
(4) Ignores resultant gross inequity.
Which of the statements given above is/are correct?
(a) 1 only (b) 2 and 3
(c) 1 and 4 (d) 4 only

Passage - 6

Net profits are only 2.2% of their total assets for central public sector undertakings, lower than
for the private corporate sector. While the public sector or the State-led entrepreneurship played an
important role in triggering India’s industrialization, our evolving development needs, comparatively
less-than-satisfactory performance of the public sector, enterprises, the maturing of our private sector,
a much larger social base now available for expanding entrepreneurship and the growing institutional
capabilities to enforce competition policies would suggest that the time has come to review the role
of public sector.
What should the portfolio composition of the government be? It should not remain static all times.

USPC - CSAT 153


UPSC - CSAT -1
The airline industry works well as a purely private affair. At the opposite end, rural roads, whose
sparse traffic makes tolling unviable, have to be on the balance-sheet of the State. If the government
did not own rural roads, they would not exist. Similarly public health capital in our towns and cities
will need to come from the public sector. Equally, preservation and improvement of forest cover will
have to be a new priority for the public sector assets.
Take the example of steel. With near-zero tariffs, India is a globally competitive market for
the metal. Indian firms export steel into the global market, which demonstrates there is no gap in
technology. Indian companies are buying up global steel companies, which shows there is no gap in
capital availability. Under these conditions, private ownership works best.
Private ownership is clearly desirable in regulated industries, ranging from finance to infrastructure,
where a government agency performs the function of regulation and multiple competing firms are
located in the private sector. Here, the simple and clean solution-government as the umpire and the
private sector as the players is what works best. In many of these industries, we have a legacy of
government ownership, where productivity tends to be lower, fear of bankruptcy is absent, and the risk
of asking for money from the tax payer is ever present. There is also the conflict of interest between
government as an owner and as the regulator. The formulation and implementation of competition
policy will be more vigorous and fair if government companies are out of action.
17. According to the passage, what is/are the reason/reasons for saying that the time has
come to review the role of public sector?
(1) Now public sector has lost its relevance in the industrialization process.
(2) Public sector does not perform satisfactorily.
(3) Entrepreneurship in private sector is expanding.
(4) Effective competition policies are available now.
Which of the statements given above is/are correct in the given context?
(a) 1 and 3 only (b) 2 only
(c) 2, 3 and 4 only (d) 1, 2, 3 and 4
18. According to the passage, rural roads should be in the domain of public sector only.
Why?
(a) Rural development work is the domain of government only.
(b) Private sector cannot have monetary gains in this.
(c) Government takes money from tax payers and hence it is the responsibility of government
only.
(d) Private sector need not have any social responsibility.

154 USPC - CSAT


UPSC - CSAT -1
19. The portfolio composition of the government refers to
(a) Public sector assets quality.
(b) Investment in liquid assets.
(c) Mix of government investment in different industrial sectors.
(d) Buying Return on Investment yielding capital assets.
20. The author prefers government as the umpire and private sector as players because
(a) Government prescribes norms for a fair play by the private sector.
(b) Government is the ultimate in policy formulation.
(c) Government has no control over private sector players.
(d) None of the above statements is correct in this context.

Passage - 7

Climate change poses potentially devastating effects on India’s agriculture. While the overall
parameters of climate change are increasingly accepted- a 100C average temperature increase over the
next 30 years, sea level rise of less than 10 cm in the same period, and regional monsoon variations
and corresponding droughts- the impacts in India are likely to be quite site and crop specific. Some
crops may respond favourable to the changing conditions, others may not. This emphasizes the need
to promote agricultural research and create maximum flexibility in the system to permit adaptations.
The key ingredient for “drought proofing” is the managed recharge of aquifers. To ensure continued
yields of important staple crop (e.g. wheat); it may also be necessary to shift the locations where these
crops are grown, in response to temperature changes as well as to water availability. The latter will be
a key factor in making long term investment decisions.
For example water runoff from the Himalayas is predicted to increase over the next 30 years as
glaciers melt, but then decline substantially thereafter. It will be critical to provide incentives to plan
for these large-scale shifts in agro-ecological conditions.
Indian needs to make long term investment in research and development in agriculture. India is
likely to experience changed weather patterns in future.
21. Consider the following statements:
Climate change may force the shifting of locations of the existing crops due to
(1) Melting of glaciers.
(2) Water availability and temperature suitability at other locations.
(3) Poor productivity of crops.
(4) Wider adaptability of crop plants.
USPC - CSAT 155
UPSC - CSAT -1
Which of the statements given above are correct?
(a) 1, 2 and 3 (b) 2 and 3 only
(c) 1 and 4 only (d) 1, 2, 3 and 4
22. According to the passage, why is it important to promote agricultural research in India?
(a) To predict variations in monsoon patterns and to manage water resources
(b) To make long term investment decisions for economic growth
(c) To facilitate wider adaptability of crops
(d) To predict drought conditions and to recharge aquifers

Passage - 8

It is essential that we mitigate the emissions of greenhouse gases and thus avoid some of the worst
impacts of climate change that would take place in coming years and decades. Mitigation would
require a major shift in the way we produce and consume energy. A shift away from overwhelming
dependence on fossil fuels is now long overdue, but unfortunately, technological development has
been slow and inadequate largely because government policies have not promoted investments in
research and development myopically as a result of relatively low prices of oil. It is now, therefore,
imperative for a country like India treating the opportunity of harnessing renewable energy on a large
scale as a national imperative. This country is extremely well endowed with solar, wind and biomass
sources of energy. Where we have lagged, unfortunately, is in our ability to develop and to create
technological solutions for harnessing these resources.
One particular trajectory for carrying out stringent mitigation of greenhouse gas emissions assessed
by the Intergovernmental Panel on Climate Change (IPCC) clearly shows the need for ensuring that
global emissions of greenhouse gases peak no later than 2015 and reduce rapidly thereafter. The cost
associated with such a trajectory is truly modest and would amount, in the estimation of IPCC, to not
more than 3 percent of the global GDP in 2030. In other words, the level of prosperity that the world
would have reached without mitigation would at worst be postponed by a few months or a year at the
most. This is clearly not a very high price to pay for protecting hundreds of millions of people from
the worst risks associated with climate change. Any such effort, however, would require lifestyles
to change appropriately also. Mitigation of greenhouse gas emissions is not a mere technological
fix, and clearly requires changes in lifestyles and transformation of a country’s economic structure,
whereby effective reduction in emissions is brought about, such as through the consumption of much
lower quantities of animal protein. The Food and Agriculture Organization (FAO) has determined that
the emissions from the livestock sector amount to 18 percent of the total. The reduction of emissions
from this source is entirely in the hands of human beings, who have never questioned the impacts
that their dietary habits of consuming more and more animal protein are bringing about. Mitigation
overall has huge co-benefits, such as lower air pollution and health benefits, higher energy security

156 USPC - CSAT


UPSC - CSAT -1
and greater employment.
23. According to the passage, which of the following would help in the mitigation of
greenhouse gases?
(1) Reducing the consumption of meat
(2) Rapid economic liberalization
(3) Reducing the consumerism
(4) Modern management practices of livestock
Select the correct answer using the code given below:
(a) 1, 2 and 3 (b) 2, 3 and 4
(c) 1 and 3 only (d) 2 and 4 only
24. Why do we continue to depend on the fossil fuels heavily?
(1) Inadequate technological development
(2) Inadequate funds for research and development
(3) Inadequate availability of alternative sources of energy
Select the correct answer using the code given below:
(a) 1 only (b) 2 and 3 only
(c) 1 and 3 only (d) 1, 2 and 3
25. According to the passage, how does the mitigation of greenhouse gases help as?
(1) Reduces dependence on public health
(2) Reduces dependence on livestock
(3) Reduces energy requirements
(4) Reduces rate of global climate change
Select the correct answer using the code given below:
(a) 1, 2 and 3 (b) 1, 3 and 4
(c) 2, 3 and 4 (d) 1 and 4 only
26. What is the essential message of the passage?
(a) We continue to depend on fossil fuels heavily
(b) Mitigation of the greenhouse gases is imperative
(c) We must invest in research and development
(d) People must change their lifestyle

USPC - CSAT 157


UPSC - CSAT -1

Question No. 1 2 3 4 5

Answers

Question No. 6 7 8 9 10

Answers

Question No. 11 12 13 14 15

Answers

Question No. 16 17 18 19 20

Answers

Question No. 21 22 23 24 25

Answers

Question No. 26

Answers



158 USPC - CSAT


UPSC - CSAT -1

Section - II

Logical Reasoning
Syllogisms

USPC - CSAT 159


UPSC - CSAT -1

160 USPC - CSAT


UPSC - CSAT -1

Chapter

1 Syllogisms - 1

Broadly, you need to understand two types of arguments. We have already come to see what
arguments look like. Now it is time to comprehend some ‘categories’ of arguments.
Analyse this, (P1= Premise 1, P2= Premise 2, C= Conclusion)
¾¾ P1 – All men are fools.
¾¾ P2 – Ravi is a man.
¾¾ C – Ravi is a fool.
¾¾ Now, the second type,
¾¾ P1 – Ravi is an engineer.
¾¾ P2 – Ravi is a fool.
¾¾ C – All engineers are fools.
This second form of argumentation is known as ‘Inductive logic’. Here, the conclusion arrived at
is not an LN but a logical ‘possibility’ (LP). The conclusion that we derived here ‘may or may not’ be
true. And hence we call it an LP.
When you are attempting a question, you must always try to look for a ‘logical necessity’ as an
answer, not a ‘logical possibility’. We mark LP as an answer only if the answer choices do not HAVE
a necessity answer choice in the first place.
One more thing before we move on to the next topic. DO NOT include anything external to the
premises in the conclusion.

¾¾ In syllogism questions, we are looking for LNs, not LPs.


¾¾ Do not take the variables of the questions to heart, treat them as Xs and Ys.
¾¾ Do not add anything external to the premises in the conclusion.

There are four basic premises to understand in syllogisms.


(a) All X is/are Y (b) No X is/areY
(c) Some X is/areY (d) Some X is/are not Y
Let us deal with each in detail.

USPC - CSAT 161


UPSC - CSAT -1
(a) All X is/are Y.
The usage of ‘is/are’ is not important; whatever verb appears here will be independent of
interpretation. The simple translation of this statement is that all the elements of set X will
also be elements of set Y.
Another important thing to note is that, even if the statement does not have the prefix ‘all’,
(e.g. x is y) it will have the same interpretation.
Let us also try and understand this with Venn diagrams.

The second diagram shows a possibility that exists, in that the two sets X and Y are overlapping.

While solving questions, you should use the first diagram. And, as I have stated earlier, do not
get emotionally involved in trying to picturise the verb. The trick is- find out the verb, then
recognise the ‘doer’ of the verb (i.e. the ‘subject’ of the sentence) and put the subject in the
inner circle, while the object occupies the outer circle.

For example, “All men are blue”.

Here, the verb is ‘are’, and the subject ‘all men’. Hence the set of ‘men’ will be represented
by the inner circle, and the set of ‘blue’ by the outer circle.

Sometimes, if one becomes paranoid about being able to picturise stuff, things can get tricky.
For example, if the statement were “monkeys have brains”, one would be tempted to draw
the outer circle to represent the monkeys. Do NOT be tricked by the verb. Follow the same
rule that I have mentioned earlier. The verb here is “have”, the doer of which is “monkeys”.
Hence, make the inner circle to represent monkeys and the outer to represent brains.

¾¾ The presence or absence of the prefix “all” does not matter. The statement shall still be
treated as mentioned above. Hence “all X is Y”, is the same as “X is Y”.
¾¾ Put the subject of the sentence in the ‘inner’ circle.
¾¾ For solving a question, use the first diagram. The second diagram is a possibility to be
kept in mind for solving CR/ RC questions.

162 USPC - CSAT


UPSC - CSAT -1
(b) No X is/are Y.
This is a rather simpler statement to understand. It means that no elements of set X are
elements of set Y. Simply put, the elements of the two have nothing in common. These can
be easily represented as disjoint sets, i.e. two circles, not touching each other anywhere.


However, there are some other important things to learn here. Many people try to represent
the opposite of ‘all X is Y’ as ‘all X is NOT y’.
All engineers are not fools. (Implies that no engineer is a fool)
All engineers are not fools. (Implies that only some of them are. )
Since it is semantics at play here, such a negation is ambiguous.
Similarly, a negation of the nature “Not all X are Y” has comparable problems, and hence is
not deemed a valid negation of “All X are Y”.

¾¾ Can be represented by disjoint sets.


¾¾ “All X are not Y” / “Not all X are Y” are ambiguous premises.

(c) Some X is/are Y.


These premises have prefixes that look like- some, many, a lot of, most et al. Understand that
these words have little representative or absolute value, until pitted against their respective
‘whole’ numbers. Hence our comprehension of the same will have to be careful.
Let us then agree to interpret these two statements by concurring that

¾¾ If some X are Y, then some Y must definitely be X.


¾¾ The interpretation of the prefix “some, many etc.” will be “AT LEAST ONE”.
¾¾ If some X are Y, it does not imply that some X are then definitely NOT Y.

The first interpretation is fairly simple to understand. If some elements of set X are also
elements of set Y, those same elements are both X and Y. Hence some elements of Y
automatically become elements of set X.
The second point, when elaborated, means that in logic, the prefix ‘some’ in itself means

USPC - CSAT 163


UPSC - CSAT -1
nothing except “at least one”. Even if the prefix is ‘a lot, many, most, several’ etc. our
interpretation of the same shall remain ‘at least one’.
Now, for the third point, some logic books state that if the premise states ‘some X are y’, then
it definitely means that some X are NOT Y. This is bad reasoning. Just as we saw in inductive
reasoning erstwhile, if some elements of a set do show a certain trait, then we cannot for
certainty say EITHER that the rest will not show the same trait OR that they will. Hence, to
conclude from ‘Some X are Y’, as a necessity, that ‘Some X are not Y’, is simply not correct.
Time for a Venn interpretation.
The first diagram that I have presented below is what we shall use for solving questions. The rest
are just indicative of ‘possibilities’ that may exist, and with which we must familiarise ourselves,
for they will help us understand things better when we finally arrive at long CR questions.


In this diagram the shaded portion represents the area in which our ‘at least one X’ and ‘at
least one Y’ lie. This is the diagram we shall use for solving questions.

Here, the portion of X that coincides with the portion of Y is our area of concern. Also, please
understand that one line of argument may state that here ‘‘aren’t all Y’s, X’s too?’’ To this, a
logical response is that our premise concerns itself with some of the X’s being Y’s, not Y’s
being X’s. In the process, if all Y’s turn out to be X’s, it is just a possibility, and of course not
our primary concern. We had started with trying to prove that at least one X ought to be Y, and

164 USPC - CSAT


UPSC - CSAT -1
the diagram does justice to that. (Remember, we are dealing with all the possibilities here.)


Here again, one might point out that all of the X’s are Y’s. However, by now, you and I
understand that we had set out to prove that at least one X should be Y, and in the process if
all X’s DO happen to be Y’s, so be it. Our one X is still safely within Set Y, and our diagram,
yet again, does full justice to that.


This too is a possibility that exists. And again, our one X is still ensconced firmly within Set Y.

¾¾ If some X are Y, then some Y must definitely be X.


¾¾ The interpretation of the prefix “some, many etc.” will be “AT LEAST ONE”.
¾¾ If some X are Y, it does not imply that some X are then definitely NOT Y.
¾¾ For solving a question, we shall use the first diagram.

(d) Some X are/is not Y.


More often than not, it as a logically inconsistent premise. Although the statement “Some X
are not Y” CAN hold true as a conclusion, it is difficult to analyse as a premise. For instance,
let us try with
P1 – Some boys are not mature.

USPC - CSAT 165


UPSC - CSAT -1
Immediately with this premise you will have to go with three possible diagrams
simultaneously, i.e.


P2 – Some mature are fools/ All matures are fools/ No mature is a fool.
You understand that any of these three premises will have different impacts on the three
possible diagrams that we have made. With such a scenario we generally will not be able to
arrive at a sustainable conclusion at the third stage. In syllogisms, as you must have noticed
earlier, we do arrive at a conclusion at the third stage.
However, this statement DOES have validity as a conclusion.

For instance,
P1 – Some buckets are trees.

P2 – No tree is a fool

Now, in all of the three possible diagrams you can see that as an LN conclusion, we can safely
say that,
Some buckets are not fools. (i.e. the buckets that lie in intersection with trees.)

¾¾ Some X are/is not Y is generally an inconsistent premise


¾¾ Some X are/is not Y has an absolutely logical existence as a conclusion.

166 USPC - CSAT


UPSC - CSAT -1
Only X are/is Y.
If you encounter a statement such as “Only X are Y”, quickly convert it into “All Y are X”.
The diagram should be simple now- Y inside, X outside.


Let us see if the formula works or not !!
P1 – Only boys wear trousers.
If this be our premise, isn’t it easy to figure out that the moment I see someone wearing a
pair of trousers, without even looking further, I should be safely able to conclude that the
person is a ‘boy’? What I mean is that since the premise explicitly states that only boys can
wear trousers, then nobody else can wear them. Therefore if someone is wearing trousers, the
person OUGHT to be a boy, else our premise falls.
If, “only boys wear trousers”, then “all trousers are worn by boys”! Simple!

¾¾ Convert “only X are Y” to “All Y are X”, and then work with what you have learnt from
the “all” prefix statements, i.e. make Y the inner circle and X the outer circle.

Directions for questions I- VIII: Choose the set of three statements where the third statement can
be logically derived from the previous two.
I.
1. All Tricks are Loops. 2. No Puzzles are Loops.
3. All Puzzles are Tricks. 4. No Puzzles are Tricks.
5. All Puzzles are Loops. 6. All Loops are Tricks.
(A) 341 (B) 642 (C) 352 (D) 156

USPC - CSAT 167


UPSC - CSAT -1
(E) 631
II.
1. All Spaces are Bust. 2. No Curves are Spaces.
3. Some Curves are not Squares. 4. Some Squares are Bust.
5. All Spaces are Squares. 6. All Squares are Curves.
(A) 514 (B) 124 (C) 316 (D) 523
(E) 513
III.
1. All S are R. 2. All G are S.
3. All Y are R. 4. Some M are Y.
5. All Y are S. 6. No S are M.
(A) 654 (B) 153 (C) 361 (D) 145
(E) 564
IV.
1. Some T are S. 2. Some S are B.
3. No T are B. 4. All T are S.
5. All B are S. 6. No S are B.
(A) 165 (B) 463 (C) 245 (D) 541
(E) 425
V.
1. All cows are green. 2. Some Green are Red.
3. Some Green eat Grass. 4. All Green are Yellow.
5. Some Cows eat Grass. 6. Some Yellow are Brown.
(A) 123 (B) 153 (C) 364 (D) 423
(E) 621
VI.
1. Some footballers are cricketers. 2. All musicians are footballers.
3. Some guitarists are musicians. 4. All cricketers are footballers.
5. All cricketers are musicians. 6. Some guitarists are footballers.
(A) 643 (B) 264 (C) 542 (D) 21
168 USPC - CSAT
UPSC - CSAT -1
(E) 236
VII.
1. No man is a tree. 2. All fools are smokers.
3. Some men are smokers. 4. All trees are smokers.
5. All trees are fools. 6. All smokers have cancer.
(A) 345 (B) 154 (C) 354 (D) 524
(E) 126
Evaluate the following syllogisms (Q no. 8-15) and assess whether the argument is valid (i.e. LN)
or invalid (i.e. LP).
8. No physical actions are chance occurrences. All chance occurrences are random events. No
random events are physical actions.
9. All things describable by science are predictable events. No free decisions are things
describable by science. No free decisions are predictable events.
10. No things describable by science are uncaused happenings. All mental decisions are things
describable by science. No mental decisions are uncaused happenings.
11. All persons who are most free are person who decide most. All persons who decide least are
persons who are most free. All persons who decide least are persons who decide most.
12. Some snakes are dangerous creatures, for some snakes are poisonous animals and some
poisonous animals are dangerous creatures.
13. Some foreign cars are vehicles sold in America because all Triumphs are foreign cars and
some Triumphs are vehicles sold in America.
14. No loving persons are thoughtless persons. No loving persons are aggressive people. No
aggressive people are thoughtless persons.
15. If anything is metallic, then it is breakable. There are breakable ornaments. Therefore they
are metallic ornaments.
16. Find the valid argument(s) in the arguments given below.
1. Some cricketers snore. Some people who snore sleep. Some cricketers sleep.
2. All greats curse. Some greats cheat. Some people who curse cheat.
3. No fillers fall. Some who fall are peelers. No peeler is a filler.
4. All claws are clauses. No clause is a paw. No paws are claws.
(A) Only 1 (B) 2 and 4 (C) Only 4 (D) 1 and 3
(E) Only 2

USPC - CSAT 169


UPSC - CSAT -1
17. Find the valid argument(s) in the arguments given below.
1. Some cars are automatic. Some automatic are phones. Some cars are phones.
2. All preachers lie. Some liars are honest. Some preachers are honest.
3. All wolves are cunning. No cunning is a sly. No wolf is a sly.
4. Some bacon is fatty. All fatty are cancerous. Some bacon is cancerous.
(A) 1 and 3 (B) Only 4 (C) 3 and 4 (D) 2 and 4
(E) Only 2
The following questions appeared in CSAT 2011 :
Each of the following two items consists of four statements. Of these four statements, two cannot
both be true, but both can be false. Study the statements carefully and identify the two that satisfy the
above condition. Select the correct answer using the codes given below each set of statements :
1. Examine the following statements :
1. All animals are carnivorous. 2. Some animals are not carnivorous.
3. Animals are not carnivorous. 4. Some animals are carnivorous.
Codes :
(a) 1 and 3 (b) 1 and 2 (c) 2 and 3 (d) 3 and 4
2. Examine the following statements :
1. All trains are run by diesel engine. 2. Some trains are run by diesel engine.
3. No train is run by diesel engine. 4. Some trains are not run by diesel engine.
Codes :
(a) 1 and 2 (b) 2 and 3 (c) 1 and 3 (d) 1 and 4
3. Consider the following argument :
"In order to be a teacher one must graduate from college. All poets are poor. Some Mathema-
ticians are poets. No college graduate is poor."
Which one of the following is not a valid conclusion regarding the above argument?
(a) Some Mathematicians are not teachers.
(b) Some teachers are not Mathematicians.
(c) Teachers are not poor
(d) Poets are not teachers.


170 USPC - CSAT


UPSC - CSAT -1

Chapter

2 Syllogisms - 2

Convert these sentences into standard statements and represent in terms of venn diagrams
wherever possible.
1. At least one person is a flower
2. There exists a poem that is not a sonnet.
3. Not every animal that can fly is a bird
4. Only reptiles are lizards
5. Something is a painting but not a masterpiece
6. If anything is a sibling that is female, then it is a sister
7. Not every person who chooses not to fight is a coward.
Convert these sentences consisting of premise and conclusions into standard format and in-
dicate whether each argument is valid or not.
8. If anything is a capitalist, then it is not a hero. So, at least one thing is a capitalist but not a
hero.
9. Things are positrons only if they are smaller than atoms. Therefore, not all positrons are
smaller than an atom.
10. Nothing is an acid unless it is not a base. Therefore, each acid is a base.
11. Any person who is kept awake for over a week will go crazy. Thus, only persons who are
kept awake for over a week will go crazy.
12. No person who invented the airplane died flying an airplane. It follows that only persons
who died flying an airplane invented the airplane.
13. Assuming that “All idiots are fools” is true, state whether the statement is true, false or
cannot be determined.
(a) All non-fools are non-idiots. (b) No fools are non-idiots.
(c) Some idiots are not non-fools. (d) Some non-fools are non-idiots.
14. Assuming that”No psychiatrists are optimists” is true, state whether the statement is true,
false or cannot be determined.
(a) All psychiatrists are non-optimists.

USPC - CSAT 171


UPSC - CSAT -1
(b) No non-optimists are non-psychiatrists.
(c) Some non-optimists are not non-psychiatrists.
(d) No non-optimists are psychiatrists.
15. Assuming that” Some chemicals are poisons” is true, state whether the statement is true,
false or cannot be determined.
(a) Some poisons are chemicals. (b) Some non-chemicals are non-poisons.
(c) Some non-poisons are non-chemicals.(d) Some non-chemicals are poisons.
16. Assuming that” Some celebrities are not saints” is true, state whether the statement is true,
false or cannot be determined
(a) Some saints are not celebrities. (b) Some non-saints are celebrities.
(c) Some non-celebrities not non-saints. (d) Some celebrities are not non-saints.
Convert these syllogisms into standard format representation and indicate whether each
syllogism is valid or not.
17. Only greens are arrogant. At least one human is not arrogant. Therefore not all humans are
greens.
18. Every wicked person is self- deceived, for all liars are wicked, and every liar is self- de-
ceived.
19. No human is god. Something is both divine and human. So at least one divine being is not
god.
20. If anything is a mental event, then it is not a brain event. For only physical events are brain
events; and no mental events are physical events.
A combination of two statements
(a) Cannot both be true and cannot both be false.
(b) Cannot both be true but can both be false.
(c) Can both be true but cannot both be false
(d) Can both be true or can both be false
Using the code above, mark your answer
21. All roses are red flowers. No roses are red flowers.
22. No Apaches are pulsars. Some Apaches are pulsars.
23. Some radicals are miserable people. All radicals are miserable people.
24. Some leaders are followers. Some leaders are not followers.

172 USPC - CSAT


UPSC - CSAT -1
Following question appeared in CSAT 2012
1. Consider the following statements:
1. All machines consume energy.
2. Electricity provides energy.
3. Electrically operated machines are cheap to maintain.
4. Electrically operated machines do not cause pollution.
Which one of the following inferences can be drawn from the above statements?
(a) All machines are run by electric energy.
(b) There is no form of energy other than electricity.
(c) Most machines are operated on electric energy.
(d) Electrically operate machines are preferable to use.
2. Examine the following statements:
1. None but the rich can afford air-travel.
2. Some of those who travel by air become sick.
3. Some of those who become sick required treatment.
Which one of the following conclusions can be drawn from the above statements?
(a) All the rich persons travel by air.
(b) Those who travel by air become sick.
(c) All the rich persons become sick.
(d) All those who travel by air are rich.
3. Examine the following statements:
1. None but students are members of the club.
2. Some members of the club are married persons.
3. All married persons are invited for dance.
Which one of the following conclusions can be drawn from the above statements?
(a) All students are invited for dance.
(b) All married students of the club are invited for dance.
(c) All members of the club are married persons.
(d) None of the above conclusions can be drawn.

USPC - CSAT 173


UPSC - CSAT -1
4. Consider the following statements:
1. All X-brand cares parked here are white.
2. Some of them have radial tyres.
3. All X-brand cars manufactured after 1986 have radial tyres.
4. All cars are not X-brand.
Which one of the following conclusions can be drawn from the above statements?
(a) Only white cars are parked here.
(b) Some white X-brand cars with radial types are parked here.
(c) Cars other than X-brand cannot have radial tyres.
(d) Most of the X-brand cars are manufactured before 1986.
5. Consider the following statements:
1. All artists are whimsical.
2. Some artists are drug addicts.
3. Frustrated people are prone to become drug addicts.
From the above three statements it may be concluded that
(a) Artists are frustrated.
(b) Some drugs addicts are whimsical.
(c) All frustrated people are drug addicts.
(d) Whimsical people are generally frustrated.
6. Examine the following statements:
1. Only those who have a pair of binoculars can become the members of the birdwatcher’s
club.
2. Some members of the birdwatcher’s club have cameras.
3. Those members who have cameras can take part in photo-contests.
Which of the following conclusions can be drawn from the above statements?
(a) All those who have a pair of binoculars are members of the birdwatcher’s club.
(b) All members of the birdwatcher’s club have a pair of binoculars.
(c) All those who take part in photo-contests are members of the birdwatcher’s club.
(d) No conclusion can be drawn.

174 USPC - CSAT


UPSC - CSAT -1
7. Consider the following statement:
“Though quite expensive, television is not a luxury item, as one can learn many things
through television.”
Which one of the following is a valid inference from the above statement?
(a) All expensive things are regarded as luxury.
(b) All essential things for learning are not luxury.
(c) Television is essential for learning.
(d) Television is not a luxury item.
8. During the last summer vacation, Ankit went to a summer camp where he took part in hik-
ing, swimming and boating, This summer, he is looking forward to a music camp here he
hopes to sing, dance and learn to play the guitar.
Based on the above information, four conclusions, as given below, have been made. Which
one of these logically follows from the information given above?
(a) Ankit’s parents want him to play the guitar.
(b) Ankir prefers music to outdoor activities.
(c) Ankit goes to some type of camp every summer.
(d) Ankit likes to sing and dance.
9. Examine the following statements:
1. I watch TV only if I am bored.
2. I am never bored when I have my brother’s company.
3. Whenever I go to the theatre I take my brother along.
Which one of the following conclusions is valid in the context of the above statement?
(a) If I am bored, I watch TV.
(b) If I am bored, I seek my brother’s company.
(c) If I am not with my brother, then I watch TV.
(d) If I am not bored, I do not watch TV.


USPC - CSAT 175


UPSC - CSAT -1

Chapter

3 Syllogisms - 3

Identify Identify the missing step in each of the following arguments. Then put the argument into
standard form. Finally use a venn diagram to check for validity.
1. All Hindus are Indians. Hence, No Hindus are Asians.
2. If anything is a Judgement, then it is an Opinion. It follows that Judgements about thefts are
Opinions.
3. All Bengalis are Indians. Hence no Bengalis are socialists.
4. All Mormons are Cats. Therefore, all Purists are Cats.
5. Every politician who wants to win slings mud, and every politician wants to win.
6. No certainty should be rejected. So, no evidence should be rejected.
7. Atoms are indestructible because every simple substance is indestructible.
8. Only scientific statements are rational. It follows that judgements are never rational.
9. Every vice is harmful. Accordingly, every vice is a form of laziness.
10. Every event is caused by a deity. It follows that every sin is caused by a deity.

Practice Questions
Questions (1-10) In each of the following questions two statements are given and these state-
ments are followed by two conclusions numbered (1) and (2). Decide which of the given conclu-
sions logically follows from the two given statements.
Give answer:
(A) If only (1) conclusion follows (B) If only (2) conclusion follows
(C) If neither (1) nor (2) follows and (D) If both (1) and (2) follow.
1. Statements: Some actors are singers. All the singers are dancers.
Conclusions:
1. Some actors are dancers.
2. No singer is actor.
2. Statements: All the harmoniums are instruments. All the instruments are flutes.
Conclusions:
1. All the flutes are instruments.
2. All the harmoniums are flutes.

176 USPC - CSAT


UPSC - CSAT -1
3. Statements: Some mangoes are yellow. Some tixo are mangoes.
Conclusions:
1. Some mangoes are green.
2. Tixo is a yellow.
4. Statements: Some ants are parrots. All the parrots are apples.
Conclusions:
1. All the apples are parrots.
2. Some ants are apples.
5. Statements: Some papers are pens. All the pencils are pens.
Conclusions:
1. Some pens are pencils.
2. Some pens are papers.
6. Statements: All the actors are girls. All the girls are beautiful.
Conclusions:
1. All the actors are beautiful.
2. Some girls are actors.
7. Statements: All the windows are doors. No door is a wall.
Conclusions:
1. Some windows are walls.
2. No wall is a door.
8. Statements: All cups are books. All books are shirts.
Conclusions:
1. Some cups are not shirts.
2. Some shirts are cups.
9. Statements: Some cows are crows. Some crows are elephants.
Conclusions:
1. Some cows are elephants.
2. All crows are elephants.
10. Statements:All the pencils are pens. All the pens are inks.
Conclusions:
1. All the pencils are inks.
2. Some inks are pencils.
Questions (11-20): Convert the following statements into standard form (if not already) and
determine whether the syllogisms are valid.
11. Every cow loves horses. Not all farmers love horses. If follows that at least one farmer is not

USPC - CSAT 177


UPSC - CSAT -1
a cow.
12. No cowards ride bulls; therefore, some fools are not cowards since atleast one bull rider is a
fool.
13. Some works of art are books. All novels are books. So, some works of art are novels.
14. All sculptures are beautiful. Some beautiful things are paintings. So, some sculptures are
paintings.
15. All sadists are mean. All critics are mean. So, All critics are sadists.
16. Some chairs are not thrones. So, some thrones are not chairs.
17. Some married persons are persons who have disorders. Thus some persons who have disorders
are married persons.
18. No elephants are beetles. Consequently, no non-beetles are non-elephants.
19. Only persons who have conflicts are unhappy. Atleast one successful comedian is unhappy.
We may conclude that some successful comedians, are persons who have inner conflicts.
20. Some beliefs about aliens are not rational for beliefs are proportional to the evidence available.

178 USPC - CSAT


UPSC - CSAT -1

S e c t i o n - I I I

RC Questions from CSAT


2015-2017 Paper

USPC - CSAT 179


UPSC - CSAT -1

180 USPC - CSAT


UPSC - CSAT -1

Chapter
RC Questions on Inference
1
and Corollary
RC Questions on Inference
The Global Financial Stability Report finds that the share of portfolio investments from advanced
economies in the total debt and equity investments in emerging economies has doubled in the past
decade to 12 percent. The phenomenon has implications for Indian policy makers as foreign portfolio
investments in the debt and equity markets have been on the rise. The phenomenon is also flagged
as a threat that could compromise global financial stability in a chain reaction, in the event of United
States Federal Reserve's imminent reversal of its "Quantitative Easing" policy.
1. Which among the following is the most rational and critical inference that can be made
from the above passage ?
(a) Foreign portfolio investments are not good for emerging economies.
(b) Advanced economies undermine the global financial stability.
(c) India should desist from accepting foreign portfolio investments in the future.
(d) Emerging economies are at a risk of shock from advanced economies.
Open defecation is disastrous when practised in very densely populated areas, where it is impossible
to keep away human faeces from crops, wells, food and children's hands. Groundwater is also
contaminated by open defecation. Many ingested germs and worms spread diseases. They prevent the
body from absorbing calories and nutrients. Nearly one-half of India's children remain malnourished.
Lakhs of them die from preventable conditions. Diarrhoea leaves Indians' bodies smaller on average
than those of people in some poorer countries where people eat fewer calories. Underweight mothers
produce stunted babies prone to sickness who may fail to develop their full cognitive potential. The
germs released into environment harm rich and poor alike, even those who use latrines.
2. Which among the following is the most critical inference that can be made from the
above passage ?
(a) The Central and State governments in India do not have enough resources to afford a
latrine for each household.
(b) Open defecation is the most important public health problem of India.
(c) Open defecation reduces the human capital of India's workforce.
(d) Open defecation is a public health problem in all developing countries.

USPC - CSAT 181


UPSC - CSAT -1
India has suffered from persistent high inflation. Increase in administered prices, demand and
supply imbalances, imported inflation aggravated by rupee depreciation, and speculation — have
combined to keep high inflation going. If there is an element common to all of them, it is that many
of them are the outcomes of economic reforms. India's vulnerability to the effects of changes in
international prices has increased with trade liberalisation. The effort to reduce subsidies has resulted
in a continuous increase in the prices of commodities that are administered.
3. What is the most logical, rational and crucial message that is implied in the above
passage?
(a) Under the present circumstances, India should completely avoid all trade liberalisation
policies and all subsidies.
(b) Due to its peculiar socio-economic situation, India is not yet ready for trade liberalisation
process.
(c) There is no solution in sight for the problems of continuing poverty and inflation in
India in the near future.
(d) Economic reforms can often create a high inflation economy.
The ultimate aim of government is not to rule or control by fear, nor to demand obedience, but
conversely, to free every man from fear, that he may live in all possible security. In other words,
to strengthen his natural right to exist and work without injury to himself or others. The object of
government is not to change men from rational beings into beasts or puppets. It should enable them
to develop their minds and bodies in security, and to employ their reason unshackled.
4. Which among the following is the most logical and rational inference that can be made
from the above passage?
(a) The true aim of government is to secure the citizens their social and political freedom.
(b) The primary concern of government is to provide absolute social security to all its
citizens.
(c) The best government is the one that allows the citizens to enjoy absolute liberty in all
matters of life.
(d) The best government is the one that provides absolute physical security to the people of
the country.
India accounts for nearly a fifth of the world's child deaths. In terms of numbers, it is the highest in
the world — nearly 16 lakhs every year. Of these, more than half die in the first month of life. Officials
believe that the reason for this is the absence of steps to propagate basic health practices relating to
breast feeding and immunisation. Also the large reproductive population of 2-6 crore remains bereft
of care during the critical phases of pregnancy and post-delivery. Added to this is the prevalence of

182 USPC - CSAT


UPSC - CSAT -1
child marriages, anaemia among young women and lack of focus on adolescent sanitation, all of
which impact child death rates.
5. Which is the critical inference that can be made from the above passage ?
(a) A lot of Indians are illiterate and hence do not recognize the value of basic health
practices.
(b) India has a very huge population and the government alone cannot manage public health
services.
(c) Universalization and integration of maternal health and child health services can
effectively address the problem.
(d) The nutrition of women in child bearing age does not affect child mortality rate.
We are witnessing a dangerous dwindling of biodiversity in our food supply. The green revolution
is a mixed blessing. Over time farmers have come to rely heavily on broadly adapted, high yield
crops to the exclusion of varieties adapted to the local conditions. Monocropping vast fields with
the same genetically uniform seeds helps boost yield and meet immediate hunger needs. Yet high-
yield varieties are also genetically weaker crops that require expensive chemical fertilizers and toxic
pesticides. In our focus on increasing the amout of food we produce today. We have accidentally put
ourselves at risk for food shortages in future.
6. Which among the following is the most logical and critical inference that can be made
from the above passage?
(a) In our agricultural practices, we have become heavily dependent on expensive chemical
fertilizers and toxic pesticides only due to green revolution.
(b) Monocropping vast fields with high-yield varieties is possible due to green revolution
(c) Monocropping with high-yield varieties is the only way to ensure food security to
millions.
(d) Green revolution can pose a threat to biodiversity in food supply and food security in
the long run.
Climate adaptation may be rendered ineffective if policies are not designed in the context of other
development concerns. For instance, a comprehensive strategy that seeks to improve food security
in the context of climate change may include a set of corrdinated measures related to agricultural
extension, corp diversification, integrated water and pest management and agricultural information
services. Some of these measures may have to do with climate changes and others with economic
development.
7. What is the most logical and rational inference that can be made from the above passage?
(a) It is difficult to pursue climate adaptation in the developing countries.

USPC - CSAT 183


UPSC - CSAT -1
(b) Improving food security is a far more complex issue than climate adaptation.
(c) Every developmental activity is directly or indirectly linked to climate adaptation.
(d) Climate adaptation should be examined in tandem with other economic development
options.
Understanding of the role of biodiversity in the hydrological cycle enables better policymaking.
The term biodiversity refers to the variety of plants, animals, microorganisms, and the ecosystems in
which they occur. Water and biodiversity are interdependent. In reality, the hydrological cycle decides
how biodiversity functions. In turn vegetation and soil drive the movement of water. Every glass of
water we drink has, at least in part, passed through fish, trees, bacteria, soil and other organisms.
Passing through these ecosystems, it is cleansed and made. fit for consumption. The supply of water
is a critical service that the environment provides.
8. Which among the following is the most critical inference that can be made from the
above passage?
(a) Biodiversity sustains the ability of nature to recycle water.
(b) We cannot get potable water without the existence of living organisms.
(c) Plants, animals and microorganisms continuously interact among themselves
(d) Living organisms could not have come into existence without hydrological cycle.
In the last decade, the banking sector has been restructured with a high degree of automation and
products that mainly serve middle-class and upper middle-class society. Today there is a need for a
new agenda for the banking and non-banking financial services that does not exclude the common
man.
9. Which one of the following is the message that is essentially implied in the above passage?
(a) Need for more automation and more products of banks
(b) Need for a radical restructuring of our entire public finance system
(c) Need to integrate banking and non-banking institutions
(d) Need to promote financial inclusion.
To understand the nature and quantity of Government, proper for man, it is necessary to attend to
his character. As nature created him for social life, she fitted him for the station she intended. In all
cases she made his natural wants greater than his individual powers. No one man is capable, without
the aid of society, of supplying his own wants; and those wants, acting upon every individual, impel
the whole of them into society.
10. Which among the following is the most logical and rational inference that can be made
from the above passage?

184 USPC - CSAT


UPSC - CSAT -1
(a) Nature has created a great diversity in human society.
(b) Any given human society is always short of its wants.
(c) Social life is a specific characteristic of man
(d) Diverse natural wants forced man towards social system
About 15 per cent of global greenhouse gas emissions come from agricultural practices. This
includes nitrous oxide from fertilizers; methane from livestock, rice production, and manure
storage; and carbon dioxide (CO2) from burning biomass, but this excludes CO2 emissions from soil
management practices, savannah burning and deforestation. Forestry, land use, and land-use change
account for another 17 per cent of greenhouse gas emissions each year, three quarters of which come
from tropical deforestation. The remainder is largely from draining and burning tropical peatland.
About the same amount of carbon is stored in the world's peatlands as is stored in the Amazon
rainforest.
11. Which among the following is the most logical and rational inference that can be made
from the above passage?
(a) Organic farming should immediately replace mechanised and chemical dependant
agricultural practices all over the world.
(b) It is imperative for us to modify our land use practices in order to mitigate climate
change.
(c) There are no technological solutions to the problem of greenhouse gas emissions.
(d) Tropical areas are the chief sites of carbon sequestration.
"The conceptual difficulties in National Income comparisons between underdeveloped and
industrialized countries are particularly serious because a part of the national output in various
underdevelped countries is produced without passing through the commercial channels."
12. In the above statement, the author implies that :
(a) the entire national output produced and consumed in industrialized countries passes
through commercial channels.
(b) the existence of a non-commercialized sector in different underdeveloped countries
renders the national income comparisons over countries difficult.
(c) no part of national output should be produced and consumed without passing through
commercial channels.
(d) a part of the national output being produced and consumed without passing through
commercial channels is a sign of underdevelopment.
There has been a significant trend worldwide towards regionalism in government, resulting in

USPC - CSAT 185


UPSC - CSAT -1
a widespread transfer of powers downwards towards regions and communities since 1990s. This
process, which involves the creation of new political entities and bodies at a sub-national level and an
increase in their content and powers, is known as devolution. Devolution has been characterized as
being made up of three factors—political legitimacy, decentralization of authority and decentralization
of resources. Political legitimacy here means a mass demand from below for the decentralization
process, which is able to create a political force for it to take place. In many cases, decentralization
is initiated by the upper tier of government without sufficient political mobilization for it at the
grassroots level, and in such cases the decentralization process often does not fulfill its objectives.
13. Which among the following is the most logical, rational and critical inference that can
be made from the above passage?
(a) Emergence of powerful mass leaders is essential to create sub-national political entities
and thus ensure successful devolution and decentralization.
(b) The upper tier of government should impose devolution and decentralization on the
regional communities by law or otherwise.
(c) Devolution, to be successful, requires a democracy in which there is free expression of
the will of the people at lower level and their active participation at the grassroots level.
(d) For devolution to take place, a strong feeling of regionalism in the masses is essential.
The IMF has pointed out that the fast growing economies of Asia face the risk of falling into 'middle-
income trap'. It means that average incomes in these countries, which till now have been growing
rapidly, will stop growing beyond a point—a point that is well short of incomes in the developed
West. The IMF identifies a number of causes of middle-income trap—none of which is surprising—
from infrastructure to weak institutions, to less than favourable macroeconomic conditions. But the
broad, overall cause, says IMF, is a collapse in the growth of productivity.
14. Which among the following is the most logical, rational and critical inference that can
be made from the above passage?
(a) Once a country reaches middle-income stage, it runs the risk of falling productivity
which leads to stagnant incomes.
(b) Falling into middle-income trap is a general characteristic of fast growing economies.
(c) There is no hope at all for emerging Asian economies to sustain the growth momentum.
(d) As regards growth of productivity, the performance of Asian economies is not
satisfactory.
Climate change is likely to expose a large number of people to increasing environmental risks
forcing them to migrate. The international community is yet to recognize this new category of migrants.
There is no consensus on the definition and status of climate refugees owing to the distinct meaning

186 USPC - CSAT


UPSC - CSAT -1
the term refugees carry under international laws. There are still gaps in understanding how climate
change will work as the root cause of migration. Even if there is recognition of climate refugees,
who is going to provide protection? More emphasis has been given to international migration due to
climate change. But there is a need to recognize the migration of such people within the countries also
so that their problems can be addressed properly.
15. Which of the following is the most rational inference from the above passage?
(a) The world will not be able to cope with large scale migration of climate refugees.
(b) We must find the ways and means to stop further climate change.
(c) Climate change will be the most important reason for the migration of people in the
future.
(d) Relation between climate change and migration is not yet properly understood.
An air quality index (AQI) is a way to combine measurements of multiple air pollutants into
a single number or rating. This index is ideally kept constantly updated and available in different
places. The AQI is most useful when lots of pollution data are being gathered and when pollution
levels are normally, but not always, low. In such cases, if pollution levels spike for a few days, the
public can quickly take preventive action (like staying indoors) in response to an air quality warning.
Unfortunately, that is not urban India. Pollution levels in many large Indian cities are so high that
they remain well above any health or regulatory standard for large part of the year. If our index stays
in the Red/Dangerous' region day after day, there is not much any one can do, other than getting used
to ignoring it.
16. Which among the following is the most logical and rational inference that can be made
from the above passage?
(a) Our governments are not responsible enough to keep our cities pollution free.
(b) There is absolutely no need for air quality indices in our country.
(c) Air quality index is not helpful to the residents of many of our large cities.
(d) In every city, public awareness about pollution problems should increase.
Productive jobs are vital for growth and a good job is the best form of inclusion. More than half
of our population depends on agriculture, but the experience of other countries suggests that the
number of people dependent on agriculture will have to shrink if per capita incomes in agriculture are
to go up substantially. While industry is creating jobs, too many such jobs are low-productivity non-
contractual jobs in the unorganized sector, offering low incomes, little protection, and no benefits.
Service jobs are relatively of high productivity, but employment growth in services has been slow in
recent years.
17. Which among the following is the most logical and rational inference that can be made

USPC - CSAT 187


UPSC - CSAT -1
from the above passage?
(a) We must create conditions for the faster growth of highly productive service jobs to
ensure employment growth and inclusion.
(b) We must shift the farm workers to the highly productive manufacturing and service
sectors to ensure the economic growth and inclusion.
(c) We must create conditions for the faster growth of productive jobs outside of agriculture
even while improving the productivity of agriculture.
(d) We must emphasize the cultivation of high-yielding hybrid varieties and genetically
modified crops to increase the per capita income in agriculture.
A Landscape-scale approach to land use can encourage greater biodiversity outside protected
areas. During hurricane 'Mitch' in 1998, farms using ecoagricultural practices suffered 58 percent, 70
percent and 99 percent less damage in Honduras, Nicaragua and Guatemala, respectively, than farms
using conventional techniques. In Costa, vegetative windbreaks and fencerows boosted farmers'
income from pasture and coffee while also increasing bird diversity. Bee pollination is more effective
when agricultural fields are closer to natural or seminatural habitat, a finding that matters because 87
percent of the world's 107 leading crops depend on animal pollinators. In Costa Rica, Nicaragua and
Colombia silvopastoral systems that integrate trees with pastureland are improving the sustainability
of cattle production, and diversifying and increasing farmers' income.
18. Which among the following is the most logical and rational inference that can be made
from the above passage?
(a) Agricultural practices that enhance biodiversity can often increase farm output and
reduce the vulnerability to disasters.
(b) All the countries of the world should be encouraged to replace ecoagriculture with
conventional agriculture.
(c) Ecoagriculture should be permitted in protected areas without destroying the biodiversity
there.
(d) The yield of food crops will be very high if ecoagricultural practices are adopted to
cultivate them.
The medium term challenge for Indian manufacturing is to move from lower to higher tech sectors,
from lower to higher value-added sectors, and from lower to higher productivity sectors. Medium
tech industries are primarily capital intensive and resource processing; and high tech industries are
mainly capital and technology intensive. In order to push the share of manufacturing in overall GDP
to the projected 25 per cent, Indian manufacturing needs to capture the global market in sectors
showing a rising trend in demand. These sectors are largely high technology and capital intensive.

188 USPC - CSAT


UPSC - CSAT -1
19. Which among the following is the most logical and rational inference that can be made
from the above passage?
(a) India's GDP displays high value-added and high productivity levels in medium tech and
resource processing industries.
(b) Promotion of capital and technology intensive manufacturing is not possible in India.
(c) India should push up the public investments and encourage the private investments in
research and development, technology upgradation and skill development.
(d) India has already gained a great share in global markets in sectors showing a rising trend
in demand.
In a democratic State, where a high degree of Political maturity of the people obtains, the conflict
between the will of the sovereign law-making body and the organized will of the people seldom
occurs.
20. What does the above passage imply?
(a) In a democracy, force is the main phenomenon in the actual exercise of sovereignty.
(b) In a mature democracy, force to a great extent is the main phenomenon in the actual
exercise of sovereignty.
(c) In a mature democracy, use of force is irrelevant in the actual exercise of sovereignty.
(d) In a mature democracy, force is narrowed down to a marginal phenomenon in the actual
exercise of sovereignty.
What climate change will undeniably do is cause or amplify events that hasten the reduction of
resources. Competition over these diminishing resources would ensue in the form of political or
even violent conflict. Resource based conflicts have rarely been overt and are thus difficult to isolate.
Instead they take on veneers that appear more politically palatable. Conflicts over resources like water
are often cloaked in the guise of identity or ideology.
21. What does the above passage imply?
(a) Resource-based conflicts are always politically motivated.
(b) There are no political solutions to resolve environmental and resource based conflicts.
(c) Environmental issues contribute to resource stresses and political conflict.
(d) Political conflict based on identity or ideology cannot be resolved.
In the last two decades, the world's gross domestic product (GDP) has increased 50 percent,
whereas inclusive wealth has increased by a mere 6 percent. In recent decades, GDP-driven economic
performance has only harmed inclusive wealth like human capital; and natural capital like forests,
land and water. While the world's human capital which stands at 57 percent of total inclusive wealth
grew by only 8 percent, the natural which is 23 percent of total inclusive wealth declined by 30 per

USPC - CSAT 189


UPSC - CSAT -1
cent worldwide in the last two decades.
22. Which of the following is the most crucial inference from the above passage?
(a) More emphasis should be laid on the development of natural capital.
(b) The growth driven by GDP only is neither desirable nor sustainable.
(c) The economic performance of the countries of the world is not satisfactory.
(d) The world needs more human capital under the present circumstances
By 2020, when the global economy is expected to run short of 56 million young people, India, with
its youth surplus of 47 million, could fill the gap. It is in this context that labour reforms are often
cited as the way to unlock double-digit growth in India. In 2014, India's labour force was estimated
to be about 40 per cent of the population, but 93 per cent of this force was in unorganized sector.
Over the last decade, the compound annual growth rate (CAGR) of employment has slowed to 0.5
per cent, with about 14 million jobs created during last year when the labour force increased by about
15 million.
23. Which of the following is most rational inference from the above passage?
(a) India must control its population growth so as to reduce its unemployment rate.
(b) Labour reforms are required in India to make optimum use of its vast labour force
productively
(c) India is poised to achieve the double-digit growth very soon
(d) India is capable of supplying the skilled young people to other countries.
Vast numbers of Indian citizens without bank accounts live in rural areas, are financially and
functionally illiterate, and have little experience with technology. A research study was conducted in
a particular area in which electronic wage payments in Mahatma Gandhi National Rural Employment
Guarantee Scheme (MGNREGS) are meant to go directly to the poor. It was observed that recipients
often assume that the village leader needs to mediate the process, as was the case under the previous
paper-based system. Among households under this research study area who claimed to have at least
one bank account, over a third reported still receiving MGNREGS wages in cash directly from a
village leader.
24. What is the most logical, rational and crucial message that is implied in the above
passage?
(a) MGNREGS should be extended only to those who have a bank account.
(b) The paper-based system of payments is more efficient than electronic payment in the
present scenario.
(c) The goal of electronic wage payments was not to eliminate mediation by village leaders.
(d) It is essential to provide financial literacy to the rural poor.

190 USPC - CSAT


UPSC - CSAT -1
RC Questions on Corollary
Climate change is already making many people hungry all over the world, by disrupting crop
yields and pushing up prices. And it is not just food but nutrients that are becoming scarcer as the
climate changes. It is the poorest communities that will suffer the worst effects of climate change,
including increased hunger and malnutrition as crop production and livelihoods are threatened. On
the other hand, poverty is a driver of climate change, as desperate communities resort to unsustainable
use of resources to meet current needs.
1. Which among the following is the most logical corollary to the above passage ?
(a) Government should allocate more funds to poverty alleviation programmes and increase
food subsidies to the poor communities.
(b) Poverty and climate impacts reinforce each other and therefore we have to re-imagine
our food systems.
(c) All the countries of the world must unite in fighting poverty and malnutrition and treat
poverty as a global problem
(d) We must stop unsustainable agricultural practices immediately and control food prices.
Flamingos in large flocks in the wild are social and extremely loyal. They perform group mating
dances. Parents are very fond of their chicks, gathering them into creches for protection while both
males and females fly off to search for food.
2. Which among the following is the most logical corollary to the above passage ?
(a) Mass nesting in all species of birds is essential to ensure complete survival of their
offspring.
(b) Only birds have the capacity to develop social behaviour and thus can do mass nesting
to raise their chicks in safety.
(c) Social behaviour in some species of birds increases the odds of survival in an unsafe
world.
(d) All species of birds set up creches for their chicks to teach them social behaviour and
loyalty.
By killing transparency and competition, crony capitalism is harmful to free enterprise, opportunity
and economic growth. Crony capitalism, where rich and the influential are alleged to have received
land and natural resources and various licences in return for payoffs to venal politicians, is now a
major issue to be tackled. One of the greatest dangers to growth of developing economies like India is
the middle-income trap where crony capitalism creates oligarchies that slow down the growth.
3. Which among the following is the most logical corollary to the above passage?
(a) Launching more welfare schemes and allocating more finances for the current schemes

USPC - CSAT 191


UPSC - CSAT -1
are urgently needed.
(b) Efforts should be made to push up economic growth by other means and provide licences
to the poor.
(c) Greater transparency in the functioning of the government and promoting the financial
inclusion are needed at present.
(d) We should concentrate more on developing manufacturing sector than service sector.
An increase in human-made carbon dioxide in the atmosphere could initiate a chain reaction between
plants and microorganisms that would unsettle one of the largest carbon reservoirs on the planet - soil.
In a study, it was found that the soil, which contains twice the amount of carbon present in all plants
and Earth's atmosphere combined, could become increasingly volatile as people add more carbon
dioxide to the atmosphere. This is largely because of increased plant growth. Although a greenhouse
gas and a pollutant, carbon dioxide also supports plant growth. As trees and other vegetation flourish
in a carbon diloxide-rich future, their roots could stimulate microbial activity in soil that may in turn
accelerate the decomposition of soil carbon and its release into the atmosphere as carbon dioxide.
4. Which among the following is the most logical corollary to the above passage?
(a) Carbon dioxide is essential for the survival of microorganisms and plants.
(b) Humans are solely responsible for the release of carbon dioxide into the atmosphere
(c) Microorganisms and soil carbon are mainly responsible for the increased plant growth
(d) Increasing green cover could trigger the release of carbon trapped in soil.
Over the last decade, Indian agriculture has become more robust with record production of food
grains and oilseeds. Increased procurement, consequently, has added huge stocks of food grains in the
granaries. India is one of the world's top producers of rice, wheat, milk, fruits and vegetables. India
is still home to a quarter of all undernourished people in the world. On an average, almost half of the
total expenditure of nearly half of the households is on food.
5. Which among the following is the most logical corollary to the above passage?
(a) Increasing the efficiency of farm to-fork value chain is necessary to reduce the poverty
and malnutrition.
(b) Increasing the agricultural productivity will automatically eliminate the poverty and
malnutrition in India.
(c) India's agricultural productivity is already great and it is not necessary to increase it
further.
(d) Allocation of more funds for social welfare and poverty alleviation programmes will
ultimately eliminate the poverty and malnutrition in India.


192 USPC - CSAT


UPSC - CSAT -1

Chapter

2 RC Questions on
Assumptions
Governments may have to take steps which would otherwise be an infringement on the Fundamental
Rights of individuals, such as acquiring a person's land against his will, or refusing permission for
putting up a building, but the larger public interest for which these are done must be authorized by
the people (Parliament). Discretionary powers to the administration can be done away with. It is
becoming more and more difficult to keep this power within limits as the government has many
number of tasks to perform. Where discretion has to be used, there must be rules and safeguards to
prevent misuse of that power. Systems have to be devised which minimise, if not prevent, the abuse
of discretionary power. Government work must be conducted within a framework of recognised rules
and principles, and decisions should be similar and predictable.
1. Which among the following is the most logical assumption that can be made from the
above passage ?
(a) Government should always be given wide discretionary power in all matters of
administration.
(b) The supremacy of rules and safeguards should prevail as opposed to the influence of
exclusive discretion of authority.
(c) Parliamentary democracy is possible only if the Government has wider discretionary
power.
(d) None of the above statements is a logical assumption that can be made from this passage
The richer States have a responsibility to cut down carbon emissions and promote clean energy
investments. These are the States that got electricity, grew faster and now have high per capita income,
making them capable of sharing India's burden of becoming eco-friendly. Delhi, for example, can
help by generating its own clean electricity using solar rooftop panels or even help poor States finance
their clean energy projects. It is no secret that State Electricity Boards, which control 95% of the
distribution network, are neck-deep in losses. These losses further discourage State utilities from
adopting renewable energy as it is more expensive than fossil fuels.
2. Which among the following is the most logical and rational assumption that can be
made from the above passage ?
(a) The richer States must lead in the production and adoption of renewable energy.
(b) The poor States always have to depend on rich States for electricity.

USPC - CSAT 193


UPSC - CSAT -1
(c) The State Electricity Boards can improve their finances by undertaking clean energy
projects.
(d) The high economic disparity between the rich and poor States is the major cause of high
carbon emissions in India.
Our municipal corporations are understaffed. The issue of skills and competencies of the staff
poses an even greater challenge. Urban services delivery and infrastructure are complex to plan and
execute. They require a high degree of specialization and professionalism. The current framework
within which municipal employees, including senior management, are recruited does not adequately
factor in the technical and managerial competencies required. Cadre and recruitment rules only
specify the bare minimum in academic qualifications. There is no mention of managerial or technical
competencies, or of relevant work experience. This is the case with most municipal corporations.
They also suffer from weak organisation design and structure.
3. Which among the following is the most logical and rational assumption that can be
made from the above passage ?
(a) The task of providing urban services is a complex issue which requires the organisational
expansion of municipal bodies all over the country.
(b) Our cities can provide better quality of life if our local government bodies have adequate
staff with required skills and competencies.
(c) Lack of skilled staff is due to the absence of institutions which offer the requisite skills
in city management.
(d) Our country is not taking advantage of the demographic dividend to manage the
problems associated with rapid urbanization.
Many people in India feel that if we cut our defence expenditure on weapon-building, we can
create a climate of peace with our neighbours, subsequently reducing the conflict or creating a no-war
situation. People who proclaim such ideas are either the victims of war or the propagators of false
argument.
4. With reference to the above passage, which of the following is the most valid assumption?
(a) Building of weapons systems by us has instigated our neighbours to wage wars against
us.
(b) The greater spending on weapon-building by us would lessen the possibility of armed
conflict with our neighbours.
(c) It is necessary to have state of the art weapons systems for national security.
(d) Many people in India believe that we are wasting our resources on weapon-building.
All humans digest mother's milk as infants, but until cattle began, being domesticated 10,000

194 USPC - CSAT


UPSC - CSAT -1
years ago, children once weaned no longer needed to digest milk. As a result, they stopped making
the enzyme lactase, which breaks down the sugar lactose into simple sugars. After humans began
herding cattle, it became tremendously advantageous to digest milk, and lactose tolerance evolved
independently among cattle herders in Europe, the middle East and Africa. Groups not dependant on
cattle, such as the Chinese and Thai, remain lactose intolerant.
5. Which among the following is the most logical assumption that can be made from the
above passage?
(a) About 10,000 years ago, the domestication of animals took place in some parts of the
world.
(b) A permanent change in the food habits of a community can bring about a genetic change
in its members.
(c) Lactose tolerent people only are capable of getting simple sugars in their bodies.
(d) People who are not lactose tolerant cannot digest any dairy produt.
An innovative India will be inclusive as well as technologically advanced, improving the lives
of all Indians. Innovation and R&D can mitigate increases in social inequality and relieve the
pressures created by rapid urbanization. The growing divergence in productivity between agriculture
and knowledge-intensive manufacturing and services threatens to increase income inequality. By
encouraging India's R&D labs and universities to focus on the needs of poor people and by improving
the ability of informal firms to absorb knowledge, an innovation and research agenda can counter this
effect. Inclusive innovation can lower the costs of goods and services and create income - earning
opportunities for the poor people.
6. Which among the following is the most logical and rational assumption that can be
made from the above passage?
(a) Innovation and R&D is the only way to reduce rural to urban migration.
(b) Every rapidly growing country needs to minimize the divergence between productivity
in agriculture and other sectors.
(c) Inclusive innovation and R&D can help create an egalitarian society.
(d) Rapid urbanization takes place only when a country's economic growth is rapid.
The greatest blessing that technological progress has in store for mankind is not, of course, an
accumulation of material possessions. The amount of these that can be effectively enjoyed by one
individual in one lifetime is not great. But there is not the same narrow limit to the possibilities of
the enjoyment of leisure. The gift of leisure may be abused by people who have had no experience of
making use of it. Yet the creative use of leisure by a minority in societies has been the mainspring of
all human progress beyond the primitive level.

USPC - CSAT 195


UPSC - CSAT -1
7. With reference to the above passage, the following assumptions have been made :
1. People always see the leisure time as a gift and use it for acquiring more material
possessions.
2. Use of leisure by some people to produce new and original things has been the chief
source of human progress.
Which of these assumptions is/are valid?
(a) 1 only (b) 2 only (c) Both 1 and 2 (d) Neither 1 nor 2
There is more than a modicum of truth in the assertion that "a working knowledge of ancient
history is necessary to the intelligent interpretation of current events". But the sage who uttered these
words of wisdom might well have added something on the benefits of studying particularly the famous
battles of history for the lessons they contain for those of us who lead or aspire to leadership. Such
a study will reveal certain qualities and attributes which enabled the winners to win—and certain
deficiencies which caused the losers to lose and the student will see that the same pattern recurs
consistently, again and again, throughout the centuries.
8. With reference to the above passage, the following assumptions have been made:
1. A study of the famous battles of history would help us understand the modern warfare.
2. Studying the history is essential for anyone who aspires to be a leader.
Which of these assumptions is/are valid?
(a) 1 only (b) 2 only (c) Both 1 and 2 (d) Neither 1 nor 2


196 USPC - CSAT


UPSC - CSAT -1

Chapter

3 RC Questions on Main Idea

We generally talk about democracy but when it comes to any particular thing, we prefer a belonging
to our caste or community or religion. So long as we have this kind of temptation, our democracy will
remain a phoney kind of democracy. We must be in a position to respect a man as a man and to extend
opportunities for development to those who deserve them and not to those who happen to belong to
our community or race. This fact of favouritism has been responsible for much discontent and ill-will
in our country.
1. Which one of the following statements best sums up the above passage ?
(a) Our country has a lot of diversity with its many castes, communities and religions.
(b) True democracy could be established by providing equal opportunities to all.
(c) So far none of us have actually understood the meaning of democracy.
(d) It will never be possible for us to establish truly democratic governance in our country.
No Right is absolute, exclusive or inviolable. The Right of personal property, similarly, has to be
perceived in the larger context of its assumed legitimacy. The Right of personal property should unite
the principle of liberty with that of equality, and both with the principle of cooperation.
2. In the light of the argument in the passage, which one of the following statements is the
most convincing explanation ?
(a) The Right of personal property is a Natural Right duly supported by statutes and
scriptures.
(b) Personal property is a theft and an instrument of exploitation. The Right of personal
property is therefore violative of economic justice.
(c) The Right of personal property is violative of distributive justice and negates the
principle of cooperation.
(d) The comprehensive idea of economic justice demands that the Right of each person to
acquisition of property has to be reconciled with that of others.
The conflict between man and State is as old as State history. Although attempts have been made
for centuries to bring about a proper adjustment between the competing claims of State and the
individual, the solution seems to be still far off. This is primarily because of the dynamic nature of
human society where old values and ideas constantly yield place to new ones. It is obvious that if

USPC - CSAT 197


UPSC - CSAT -1
individuals are allowed to have absolute freedom of speech and action, the result would be chaos,
ruin and anarchy.
3. The author's viewpoint can be best summed up in which of the following statements ?
(a) The conflict between the claims of State and individual remains unresolved.
(b) Anarchy and chaos are the obvious results of democratic traditions.
(c) Old values, ideas and traditions persist despite the dynamic nature of human society.
(d) Constitutional guarantee of freedom of speech is not in the interest of society.
By 2050, the Earth's population will likely have swelled from seven to nine billion people. To fill
all those stomachs — while accounting for shifting consumption patterns, climate change, and a finite
amount of arable land and potable water — some experts say food production will have to double.
How can we make the numbers add up? Experts say higher yielding crop varieties and more efficient
farming methods will be crucial. So will waste reduction. Experts urge cities to reclaim nutrients and
water from waste streams and preserve farmland. Poor countries, they say, can improve crop storage
and packaging and rich nations could cut back on resource-intensive foods like meat.
4. Which one of the following statements best sums up the above passage ?
(a) The population of the world is growing very fast.
(b) Food security is a perennial problem only in developing countries.
(c) The world does not have enough resources to meet the impending food scarcity.
(d) Food security is increasingly a collective challenge.
As we look to 2050, when we will need to feed two billion more people, the question of which
diet is best has taken on new urgency. The foods we choose to eat in the coming decades will have
dramatic ramifications for the planet. Simply put, a diet that revolves around meat and dairy, a way
of eating that is on the rise throughout the developing world, will take a greater toll on the world's
resources than one that revolves around unrefined grains, nuts, fruits and vegetable.
5. What is the critical message conveyed by the above passage?
(a) Our increasing demand for foods sourced from animals puts a greater burden on our
natural resources.
(b) Diets based on grains, nuts, fruits and vegetables are best suited for health in developing
countries.
(c) Human beings change their food habits from time to time irrespective of the health
concerns.
(d) From a global perspective, we still do not know which type of diet is best for us.

198 USPC - CSAT


UPSC - CSAT -1
Disruption of traditional institutions, identifications and loyalties is likely to lead to ambivalent
situations. It is possible that some people may renew their identification with traditional groups
whereas others align themselves with new groups and symbols emergent from processes of political
development. In addition, political development tends to foster group awareness of a variety of class,
tribe, region, clan, language, religion, occupation and others.
6. Which one of the following is the best explanation of the above passage?
(a) Political development is not a unilinear process for it involves both growth and decay.
(b) Traditional societies succeed in resisting positive aspects of political development.
(c) It is impossible for traditional societies to break away from lingering loyalties.
(d) Sustenance of traditional loyalties is conducive to political development.
We live in digital times. The digital is not just something we use strategically and specifically to
do a few tasks. Our very perception of who we are, how we connect to the world around us, and the
ways in which we define our domains of life, labour and language are hugely structured by the digital
technologies. The digital is everywhere and; like air, invisible. We live within digital systems, we
live with intimate gadgets, we interact through digital media, and the very presence and imagination
of the digital has dramatically restructured our lives. The digital, far from being a tool, is a condition
and context that defines the shapes and boundaries of our understanding of the self, the society, and
the structure of governance.
7. Which among the following is the most logical and essential message conveyed by the
above passage?
(a) All problems of governance can be solved by using digital technologies.
(b) Speaking of digital technologies is speaking of our life and living.
(c) Our creativity and imagination cannot be expressed without digital media.
(d) Use of digital systems is imperative for the existence of mankind in future.
The States are like pearls and the Centre is the thread which turns them into a necklace; if the
thread snaps, the pearls are scattered.
8. Which one of the following views corroborates the above statement?
(a) A strong Centre and strong States make the federation strong.
(b) A strong Centre is a binding force for national integrity.
(c) A strong Centre is a hindrance to State autonomy.
(d) State autonomy is a prerequisite for a federation.
Really I think that the poorest he that is in England has a life to live, as the greatest he, and
therefore truly, I think it is clear that every man that is to live under a government ought first by his

USPC - CSAT 199


UPSC - CSAT -1
own consent to put himself under the government, and I do think that the poorest man in England is
not at all bound in a strict sense to that government that he has not had a voice to put himself under.
9. The above statement argues for
(a) distribution of wealth equally to all
(b) rule according to the consent of the governed
(c) rule of the poor
(d) expropriation of the rich
"The individual, according to Rousseau, puts his person and all his power in common under the
supreme direction of the General Will and in our corporate capacity we receive each member as an
indivisible part of the whole."
10. In the light of the above passage, the nature of General Will is best described as
(a) the sum total of the private wills of the individuals
(b) what is articulated by the elected representatives of the individuals
(c) the collective good as distinct from private wills of the individuals
(d) the material interests of the community
A successful democracy depends upon widespread interest and participation in politics, in which
voting is an essential part. To deliberately refrain from taking such an interest, and from voting, is a
kind of implied anarchy, it is to refuse one's political responsibility while enjoying the benefits of a
free political society.
11. This passage relates to
(a) duty to vote
(b) right to vote
(c) freedom to vote
(d) right to participate in politics
In a free country, the man who reaches the position of leader is usually one of outstanding character
and ability. Moreover, it is usually possible to foresee that he will reach such a position, since early
in life one can see his qualities of character. But this is not always true in the case of a dictator; often
he reaches his position of power through chance, very often through the unhappy state of his country.
12. The passage seems to suggest that
(a) a leader foresees his future position
(b) a leader is chosen only by a free country
(c) a leader must see that his country is free from despair
200 USPC - CSAT
UPSC - CSAT -1
(d) despair in a country sometimes leads to dictatorship
The man who is perpetually hesitating which of the two things he will do first, will do neither.
The man who resolves, but suffers his resolution to be changed by the first counter-suggestion of a
friend—who fluctuates from opinion to opinion and veers from plan to plan-can never accomplish
anything. He will at best be stationary and probably retrograde in all. It is only the man who first
consults wisely, then resolves firmly and then executes his purpose with inflexible perseverance,
undismayed by those petty difficulties which daunt a weaker spirit— that can advance to eminence
in any line.
13. The keynote that seems to be emerging from the passage is that
(a) we should first consult wisely and then resolve firmly
(b) we should reject suggestions of friends and remain unchanged
(c) we should always remain broad-minded
(d) we should be resolute and achievement-oriented
During the summer in the Arctic Ocean, sea ice has been melting earlier and faster, and the winter
freeze has been coming later. In the last three decades, the extent of summer ice has declined by about
30 per cent. The lengthening period of summer melt threatens to undermine the whole Arctic food
web, atop which stand polar bears.
14. Which among the following is the most crucial message conveyed by the above passage?
(a) Climate change has caused Arctic summer to be short but temperature to be high.
(b) Polar bears can be shifted to South Pole to ensure their survival.
(c) Without the presence of polar bears, the food chains in Arctic region will disappear.
(d) Climate change poses a threat to the survival of polar bears.
Why do people prefer open defecation and not want toilets or, if they have them, only use them
sometimes? Recent research has shown two critical elements: ideas of purity and pollutions, and
not wanting pits or septic tanks to fill because they have to be emptied. These are the issue that
nobody wants to talk about, but if we want to eradicate the practice of open defection, they have to be
confronted and dealt properly.
15. Which among the following is the most crucial message conveyed by the above passage?
(a) The ideas of purity and pollutions are so deep-rooted that they cannot be removed from
the minds of the people.
(b) People have to perceive toilet use and pit-emptying as clean and not polluting.
(c) People cannot change their old habits.
(d) People have neither civic sense nor sense of privacy.

USPC - CSAT 201


UPSC - CSAT -1
The very first lesson that should be taught to us when we are old enough to understand it, is that
complete freedom from the obligation to work is unnatural, and ought to be illegal, as we can escape
our share of the burden of work only by throwing it on someone else’s shoulders. Nature ordains that
the human race shall perish of famine if it stops working. We cannot escape from this tyranny. The
question we have to settle is how much leisure we can afford to allow ourselves.
16. The main idea of the passage is that
(a) it is essential for human beings to work
(b) there should be a balance between work and leisure
(c) working is a tyranny which we to face
(d) human's understanding of the nature of work is essential

202 USPC - CSAT


UPSC - CSAT -1

Chapter

4 Specific and Generic RC


Questions
The existence/establishment of formal financial institutions that offer safe, reliable, and alternative
financial instruments is fundamental in mobilising savings. To save, individuals need access to
safe and reliable financial institutions, such as banks, and to appropriate financial instruments and
reasonable financial incentives. Such access is not always available to all people in developing
countries like India and more so, in rural areas. Savings help poor households manage volatility in
cash flow, smoothen consumption, and build working capital. Poor households without access to a
formal savings mechanism encourage immediate spending temptations.
1. With reference to the above passage, consider the following statements :
1. Indian financial institutions do not offer any financial instruments to rural households to
mobilise their savings.
2. Poor households tend to spend their earnings/savings due to lack of access to appropriate
financial instruments.
Which of the statements given above is/are correct ?
(a) 1 only (b) 2 only (c) Both 1 and 2 (d) Neither 1 nor 2
2. What is the crucial message conveyed in the passage ?
(a) Establish more banks
(b) Increase the Gross Domestic Product (GDP) growth rate
(c) Increase the interest rate of bank deposits
(d) Promote financial inclusion
Climate change is a complex policy issue with major implications in terms of finance. All actions
to address climate change ultimately involve costs. Funding is vital for countries like India to design
and implement adaptation and mitigation plans and projects. Lack of funding is a large impediment
to implementing adaptation plans. The scale and magnitude of the financial support required by
developing countries to enhance their domestic mitigation and adaptation actions are a matter of
intense debate in the multilateral negotiations under the United Nations Framework Convention on
Climate Change (UNFCCC). The Convention squarely puts the responsibility for provision of financial
support on the developed countries, taking into account their contribution to the stock of greenhouse
gases (GHGs) in the atmosphere. Given the magnitude of the task and the funds required, domestic

USPC - CSAT 203


UPSC - CSAT -1
finances are likely to fall short of the current and projected needs of the developing countries. Global
funding through the multilateral mechanism of the Convention will enhance their domestic capacity
to finance the mitigation efforts.
3. According to the passage, which of the following is/are a matter of intense debate in the
multilateral negotiations under UNFCCC regarding the role of developing countries in
climate change ?
1. The scale and size of required financial support.
2. The crop loss due to climate change in the developing countries.
3. To enhance the mitigation and adaptation actions in the developing countries.
Select the correct answer using the code given below :
(a) 1 only (b) 2 and 3 only (c) 1 and 3 only (d) 1, 2 and 3
4. In this passage, the Convention puts the responsibility for the provision of financial
support on the developed countries because of
1. their higher level of per capita incomes.
2. their large quantum of GDP.
3. their large contribution to the stock of GHGs in the atmosphere.
Select the correct answer using the code given below :
(a) 1 only (b) 1 and 2 only (c) 3 only (d) 1,2 and 3
5. With regards to developing countries, it can be inferred from the passage that climate
change is likely to have implications on their
1. domestic finances.
2. capacity for multilateral trade.
Select the correct answer using the code given below :
(a) 1 only (b) 2 only (c) Both 1 and 2 (d) Neither 1 nor 2
6. Which one of the following is essentially discussed in the passage ?
(a) Conflict between developed and developing countries regarding support for mitigation.
(b) Occurrence of climate change due to excessive exploitation of natural resources by the
developed countries
(c) Lack of political will on the part of all the countries to implement adaptation plans
(d) Governance problems of developing countries as a result of climate change.
Set against a rural backdrop, 'Stench of kerosene' is the story of a couple, Guleri and Manak, who

204 USPC - CSAT


UPSC - CSAT -1
have been happily married for several years but do not have a child. Manak's mother is desperate to
have a grandchild to carry on the family name. Hence, she gets Manak remarried in Guleri's absence.
Manak, who acts as a reluctant but passive spectator, is meanwhile, informed by a friend that Guleri,
on hearing about her husband's second marriage, poured kerosene on her clothes and set fire to them.
Manak is heartbroken and begins to live as if he were a dead man. When his second wife delivers a
son, Manak stares at the child for a long time and blurts out, "Take him away ! He stinks of kerosene."
7. This is a sensitive issue-based story which tries to sensitise the readers about
(a) Male chauvinism and infidelity
(b) Love and betrayal
(c) Lack of legal safeguards for women
(d) Influence of patriarchal mindset
Individuals, groups and leaders who promote human development operate under strong
institutional, structural and political constraints that affect policy options. But experience suggests
broad principles for shaping an appropriate agenda for human development. One important finding
from several decades of human development experience is that focusing exclusively on economic
growth is problematic. While we have good knowledge about how to advance health and education, the
causes of growth are much less certain and growth is often elusive. Further, an unbalanced emphasis
on growth is often associated with negative environmental consequences and adverse distributional
effects. The experience of China, with its impressive growth record, reflects these broader concerns
and underlines the importance of balanced approaches that emphasize investments in the non-income
aspects of human development.
8. With reference to the above passage, consider the following statements :
1. In developing countries, a strong institutional framework is the only requirement for
human development and policy options.
2. Human development and economic growth are not always positively inter-related.
3. Focusing only on human development should be the goal of economic growth.
Which of the above statements is/are correct ?
(a) 1 only (b) 2 and 3 only (c) 2 only (d) 1,2 and 3
9. With reference to the above passage, the following assumptions have been made :
1. Higher economic growth is essential to ensure reduction in economic disparity.
2. Environmental degradation is sometimes a consequence of economic growth.
Which of the above is/are valid assumption/assumptions?
(a) 1 only (b) 2 only (c) Both 1 and 2 (d) Neither 1 nor 2

USPC - CSAT 205


UPSC - CSAT -1
Human history abounds in claims and theories confining the right of governing to a few select
citizens. Exclusion of the many is justified on the ground that human beings may be rightfully
segregated for the good of society and viability of the political process.
10. Which one of the following statements is least essential as a part of the argument in the
above passage?
(a) Man seeks control over external things affecting him.
(b) In society, there are 'super' and 'sub' human beings.
(c) Exceptions to universal citizen participation are conducive to systemic efficacy.
(d) Governing implies recognition of disparities in individual capacities.
Foods travel more than the people who eat them. Grocery stores and supermarkets are loaded
with preserved and processed foods. This, however, often leads to environmental threats, such as
pollution generated by long distance food transportation and wastage of food during processing
and transportation, destruction of rain forests, reduced nutritional content, increased demand for
preservation and packaging. Food insecurity also increases as the produce comes from regions that
are not feeding their own population properly.
11. With reference to the above passage, which of the following statements is/are true ?
1. Consuming regionally grown food and not depending on long travelled food is a part of
eco-friendly behaviour.
2. Food processing industry puts a burden on our natural resources.
Select the correct answer using the code given below:
(a) 1 only (b) 2 only (c) Both 1 and 2 (d) Neither 1 nor 2
I must say that, beyond occasionally exposing me to laughter, my constitutional shyness has been
of no disadvantage whatever. In fact I can see that, on the contrary, it has been all to my advantage.
My hesitancy in speech, which was once an annoyance, is now a pleasure. Its greatest benefit has
been that it has taught me the economy of words. I have naturally formed the habit of restraining my
thoughts. And I can now give myself the certificate that a thoughtless word hardly ever escapes my
tongue or pen. I do not recollect ever having had to regret anything in my speech or writing. I have
thus been spared many a mishap and waste of time. Experience has taught me that silence is part of
the spiritual discipline of a votary of truth. Proneness to exaggerate, to suppress or modify the truth,
wittingly or unwittingly, is a natural weakness of man, and silence is necessary in order to surmount
it. A man of few words will rarely be thoughtless in his speech; he will measure every word. We find
so many people impatient to talk. There is no chairman of a meeting who is not pestered with notes
for permission to speak. And whenever the permission is given the speaker generally exceeds the
time-limit, asks for more time, and keeps on talking without permission. All this talking can hardly be

206 USPC - CSAT


UPSC - CSAT -1
said to be of any benefit to the world. It is so much waste of time. My shyness has been in reality my
shield and buckler. It has allowed me to grow. It has helped me in my discernment of truth.
12. The author says that a thoughtless word hardly ever escapes his tongue or pen. Which
one of the following is not a valid reason for this?
(a) He has no intention to waste his time.
(b) He believes in the economy of words.
(c) He believes in restraining his thoughts.
(d) He has hesitancy in his speech.
13. The most appropriate reason for the author to be spared many a mishap is that
(a) he hardly utters or writes a thoughtless word.
(b) he is a man of immense patience.
(c) he believes that he is a spiritual person.
(d) he is a votary of truth.
14. For the author, silence is necessary in order to surmount
(a) constitutional shyness. (b) hesitancy in speech.
(c) suppression of thoughts. (d) tendency to overstate.
Accountability, or the lack of it, in governance generally, and civil services, in particular, is a major
factor underlying the deficiencies in governance and public administration. Designing an effective
framework for accountability has been a key element of the reform agenda. A fundamental issue is
whether civil services should be accountable to the political executive of the day or to society at large.
In other words, how should internal and external accountability be reconciled? Internal accountability
is sought to be achieved by internal performance monitoring, official supervision by bodies like the
central vigilance commission and comptroller and Auditor General, and judicial review of executive
decisions. Articles 311 and 312 of the Indian Constitution provide job security and safeguards to
the civil services especially the All India Services. The framers of the Constitution had envisaged
that provision of these safeguards would result in a civil service that is not totally subservient to the
political executive but will have the strength to function in larger public interest. The need to balance
internal and external accountability is thus built into the Constitution. The issue is where to draw the
line. Over the years, the emphasis seems to have tilted in favour of greater internal accountability of the
civil services to the political leaders of the day who in turn are expected to be externally accountable
to the society at large through the election process. This system for seeking accountability to society
has not worked out and has led to several adverse consequences for governance.
Some special measures can be considered for improving accountability in civil services. Provisions
of articles 311 and 312 should be reviewed and laws and regulations framed to ensure external

USPC - CSAT 207


UPSC - CSAT -1
accountability of civil services. The proposed civil Services Bill seeks to address some of these
requirements. The respective roles of professional civil services and the political executive should be
defined so that professional managerial functions and management of civil services are depoliticized.
For this purpose, effective statutory civil service boards should be created at the centre and in the
states. Decentralization and devolution of authority to bring government and decision making closer
to the people also helps to enhance accountability.
15. According to the passage, which of the following factor/factors led to the adverse
consequences for governance/public administration?
1. Inability of civil services to strike a balance between internal and external accountabilities.
2. Lack of sufficient professional training tot he officers of All India Services
3. Lack of proper service benefits in civil services.
4. Lack of Constitutional provisions to define the respective roles of professional civil
services vis-a-vis political executive in this context.
Select the correct answer using the code given below :
(a) 1 only (b) 2 and 3 only (c) 1 and 4 only (d) 2, 3 and 4
16. With reference to the passage, the following assumptions have been made :
1. Political executive is an obstacle to the accountability of the civil services to the society.
2. In the present framework of Indian polity, the political executive is no longer accountable
to the society.
Which of these assumptions is/are valid?
(a) 1 only (b) 2 only (c) Both 1 and 2 (d) Neither 1 nor 2
17. Which one of the following is the essential message implied by this passage?
(a) Civil services are not accountable to the society they are serving
(b) Educated and enlightened persons are not taking up political leadership.
(c) The framers of the Constitution did not envisage the problems being encountered by the
civil services.
(d) There is a need and scope for reforms to improve the accountability of civil services.
18. According to the passage, which one of the following is not a means of enhancing internal
accountability of civil services?
(a) Better job security and safeguards
(b) Supervision by Central Vigilance Commission
(c) Judicial review of executive decisions.

208 USPC - CSAT


UPSC - CSAT -1
(d) Seeking accountability through enhanced participation by people in decision making
process.
In general, religious traditions stress our duty to god, or the some universal ethical principle. Our
duties to one another derive from these. The religious concept of rights is primarily derived from our
relationship to this divinity or principle and the implication it has on our other relationships. This
correspondence between rights and duties is critical to any further understanding of justice. But for
justice to be practiced; virtue, rights and duties cannot remain formal abstractions. They must be
grounded in a community (common unity) bound together by a sense of common union (communion).
Even as a personal virtue, this solidarity is essential to the practice and understanding of justice.
19. With reference to the passage, the following assumptions have been made :
1. Human relationships are derived from their religious traditions.
2. Human beings can be duty bound only if they believe in god.
3. Religious traditions are essential to practice and understand justice.
Which of these assumption(s) is/are valid?
(a) 1 only (b) 2 and 3 only (c) 1 and 3 only (d) 1, 2 and 3
20. Which one of the following is the crux of this passage?
(a) Our duties to one another derive from our religious traditions.
(b) Having relationship to the divine principle is a great virtue.
(c) Balance between rights and duties is crucial to the delivery of justice in a society.
(d) Religious concept of rights is primarily derived from our relationship to god.
Biomass as fuel power, heat, and transport has the highest mitigation potential of all renewable
sources. It comes from agriculture and forest residues as well as from energy crops. The biggest challenge
in using biomass residues is a long-term reliable supply delivered to the power plant at reasonable costs;
the key problems are logistical constraints and the costs of fuel collection. Energy crops, if not managed
properly, compete with food production and may have undesirable impacts on food prices. Biomass
production is also sensitive to the physical impacts of a changing climate.
Projections of the future role of biomass are probably overestimated given the limits to the sustainable
biomass supply, unless breakthrough technologies substantially increase productivity. Climate-energ
models project that biomass use could increase nearly four-fold to around 150-200 exajoules, almost
a quarter of world primary energy in 2050. However the maximum sustainable technical potential of
biomass resources (both residues and energy crops) without disruption of food and forest resources
ranges from 80-170 exajoules a year by 2050, and only part of this is realistically and economically
feasible. In addition, some climate models rely on biomass-based carbon capture and storage, an
unproven technology, to achieve negative emissions and to buy some time during the first half of the

USPC - CSAT 209


UPSC - CSAT -1
century.
Some liquid biofuels such as corn-based ethanol, mainly for transport, may aggravate rather than
ameliorate carbon emissions on a life-cycle basis. Second genreation biofuels, based on ligno-cellulosic
feedstocks - such as straw, bagasse, grass and wood - hold the promise of sustainable production that is
high-yielding and emit low levels of greenhouse gases, but these are still in the R & D stage.
21. What is/are the present constraint/constraints in using biomass as fuel for power
generation?
1. Lack of sustainable supply of biomass
2. Biomass production competes with food production
3. Bio-energy may not always be low carbon on a life-cycle basis
Select the correct answer using the code given below :
(a) 1 and 2 only (b) 3 only
(c) 2 and 3 only (d) 1, 2 and 3
22. Which of the following can lead to food security problem?
1. Using agricultural and forest residues as feedstock for power generation.
2. Using biomass for carbon capture and storage
3. Promoting the cultivation of energy crops.
Select the correct answer using the code given below :
(a) 1 and 2 only (b) 3 only
(c) 2 and 3 only (d) 1, 2 and 3
23. In the context of using biomass, which of the following is/are the characteristic/
characteristics of the sustainable production of biofuel?
1. Biomass as a fuel for power generation could meet all the primary energy requirements
of the world by 2050
2. Biomass as a fuel for power generation does not necessarily disrupt food and forest
resources
3. Biomass as a fuel for power generation could help in achieving negative emissions,
given certain nascent technologies.
Select the correct answer using the code given below :
(a) 1 and 2 only (b) 3 only
(c) 2 and 3 only (d) 1, 2 and 3

210 USPC - CSAT


UPSC - CSAT -1
24. With reference to the passage, following assumptions have been made :
1. Some climate-energy models suggest that the use of biomass as a fuel for power
generation helps in mitigating greenhouse gas emissions.
2. It is not possible to use biomass as a fuel for power generation without disrupting food and
forest resources
Which of these assumptions is/are valid?
(a) 1 only (b) 2 only (c) Both 1 and 2 (d) Neither 1 nor 2
Safe and sustainable sanitation in slums has immeasurable benefits to women and girls in terms of
their health, safety, privacy and dignity. However, women do not feature in most of the schemes and
policies on urban sanitation. The fact that even now the manual scavenging exists, only goes to show
that not enough has been done to promote pour-flush toilets and discontinue the use of dry latrines. A
more sustained and rigorous campaign needs to be launched towards the right to sanitation on a very
large scale. This should primarily focus on the abolition of manual scavenging.
25. With reference to the above passage, consider the following statements :
1. Urban sanitation problems can be fully solved by the abolition of manual scavenging
only.
2. There is a need to promote greater awareness on safe sanitation practices in urban areas.
Which of the statements given above is/are correct?
(a) 1 only (b) 2 only (c) Both 1 and 2 (d) Neither 1 nor 2
The nature of the legal imperatives in any given state corresponds to the effective demands that
state encounters, and that these, in their turn, depend, in a general way, upon the manner in which
economic power is distributed in the society which the state controls.
26. The statement refers to :
(a) the antithesis of Politics and Economics.
(b) the interrelationship of Politics and Economics
(c) the predominance of Economics over Politics
(d) the predominance of Politics over Economics
Historically, the biggest challenge to world agriculture has been to achieve a balance between
demand for and supply of food. At the level of individual countries, the demand-supply balance
can be a critical policy issue for a closed economy, especially if it is a populous economy and
its domestic agriculture is not growing sufficiently enough to ensure food supplies, on an enduring
basis; it is not so much and not always, of a constraint for an open, and growing economy, which
has adequate exchange surpluses to buy food abroad. For the world as a whole, supply-demand

USPC - CSAT 211


UPSC - CSAT -1
balance is always an inescapable prerequisite for warding off hunger and starvation. However, global
availability of adequate supply does not necessarily mean that food would automatically move
from countries of surplus to countries of deficit if the latter lack in purchasing power. The uneven
distribution of hunger , starvation, under-or malnourishment, etc., at the world-level, thus owes itself
to the presence of empty-pocket hungry mouths, overwhelmingly confined to the underdeveloped
economies. Inasmuch as 'a two-square meal' is of elemental significance to basic human existence,
the issue of worldwide supply of food has been gaining significance, in recent times, both because
the quantum and the composition of demand has been undergoing big changes, and because, in recent
years, the capabilities of individual countries to generate uninterrupted chain of food supplies have
come under strain. Food production, marketing and prices, especially price-affordability by the poor
in the developing world, have become global issues that need global thinking and global solutions.
27. According to the above passage, which of the following are the fundamental solutions
for the world food security problem?
1. Setting up more agro-based industries
2. Improving the price affordability by the poor
3. Regulating the conditions of marketing
4. Providing food subsidy to one and all
Select the correct answer using the code given below :
(a) 1 and 2 only (b) 2 and 3 only
(c) 1, 3 and only (d) 1, 2, 3 and 4
28. According to the above passage, the biggest challenge to world agriculture is :
(a) to find sufficient land for agriculture and to expand food processing industries.
(b) to eradicate hunger in underdeveloped countries
(c) to achieve a balance between the production of food and non-food items.
(d) to achieve a balance between demand for and supply of food.
29. According to the above passage, which of the following helps/help in reducing hunger
and starvation in the developing economies?
1. Balancing demand and supply of food.
2. Increasing imports of food
3. Increasing purchasing power of the poor.
4. Changing the food consumption patterns and practices.
Select the correct answer using the code given below :

212 USPC - CSAT


UPSC - CSAT -1
(a) 1 only (b) 2, 3 and 4 only (c) 1 and 3 only (d) 1, 2, 3 and 4
30. The issue of worldwide supply of food has gained importance mainly because of :
1. overgrowth of the population worldwide.
2. sharp decline in the area of food production
3. limitation in the capabilities for sustained supply of food.
Select the correct answer using the code given below :
(a) 1 and 2 only (b) 3 only
(c) 2 and 3 only (d) 1, 2 and 3
Many farmers use synthetic pesticides to kill infesting insects. The consumption of pesticides in
some of the developed countries is touching 3000 grams/hectare. Unfortunately, there are reports that
these compounds possess inherent toxicities that endanger the health of the farm operators, consumers
and the environment. Synthetic pesticides are generally persistent in environment. Entering in food
chain they destroy the microbial diversity and cause ecological imbalance. Their indiscriminate use
has resulted in development of resistance among insects to insecticides, upsetting of balance in nature
and resurgence of treated populations. Natural pest control using the botanical pesticides is safer to
the user and the environment because they break down into harmless compounds within hours or
days in the presence of sunlight. Plants with pesticidal properties have been in nature for millions
of years without any ill or adverse effects on the ecosystem. They are easily decomposed by many
microbes common in most soil. They help in the maintenance of biological diversity, of predators
and the reduction of environmental contamination and human health hazards. Botanical pesticides
formulated from plants are biodegradable and their use in crop protection is a practical sustainable
alternative.
31. On the basis of the above passage, the following assumptions have been made:
1. Synthetic pesticides should never be used in modem agriculture.
2. One of the aims of sustainable agriculture is to ensure minimal ecological imbalance.
3. Botanical pesticides are more effective as compared to synthetic pesticides.
Which of the assumptions given above is/are correct?
(a) 1 and 2 only (b) 2 only
(c) 1 and 3 only (d) 1, 2 and 3
32. Which of the following statements is/are correct regarding bio-pesticides?
1. They are not hazardous to human health.
2. They are persistent in environment.

USPC - CSAT 213


UPSC - CSAT -1
3. They are essential to maintain the biodiversity of any ecosystem.
Select the correct answer using the code given below.
(a) 1 only (b) 1 and 2 only (c) 1 and 3 only (d) 1, 2 and 3
We have hard work ahead. There is no resting for any of us till we redeem our pledge in full, till
we make all the people of India what destiny intends them to be. We are citizens of a great country, on
the verge of bold advance, and we have to live up to that high standard. All of us, to whatever religion
we may belong, are equally the children of India with equal rights, privileges and obligations. We
cannot encourage communalism or narrow mindedness, for no nation can be great whose people are
narrow in thought or action.
33. The challenge the author of the above passage throws to the public is to achieve
(a) a high standard of living, progress and privileges
(b) equal privileges, fulfilment of destiny and political tolerance
(c) spirit of adventure and economic parity
(d) hard work, brotherhood and national unity
There is no harm in cultivating habits so long as they are not injurious. Indeed, most of us are little
more than bundle of habits. Take away our habits and the residuum would hardly be worth bothering
about. We could not get on without them. They simplify the mechanism of life. They enable us to do
a multitude of things automatically, which, if we had to give fresh and original thought to them each
time, would make existence an impossible confusion.
34. The author suggests that habits
(a) tend to make our lives difficult
(b) add precision to our lives
(c) make it easier for us to live
(d) tend to mechanize our lives


214 USPC - CSAT


UPSC - CSAT -1

Section - IV

Logical Reasoning and


Analytical Ability

USPC - CSAT 215


UPSC - CSAT -1

216 USPC - CSAT


UPSC - CSAT -1

Chapter

1 Simple Exercises

I. The government of an island nation is in the process of deciding how to spend its limited
income. It has $7 million left in its budget and eight programs to choose among. There is no
provision in the constitution to have a surplus, and each program has requested the minimum
amount they need; in other words, no program may be partially funded. The programs and
their funding requests are:
* Hurricane preparedness: $2.5 million
* Harbor improvements: $1 million
* School music program: $0.5 million
* Senate office building remodeling: $1.5 million
* Agricultural subsidy program: $2 million
* National radio: $0.5 million
* Small business loan program: $3 million
* International airport: $4 million
1. Senators from urban areas are very concerned about assuring that there will be funding
for a new international airport. Senators from rural areas refuse to fund anything until
money for agricultural subsidies is appropriated. If the legislature funds these two
programs, on which of the following could they spend the rest of the money?
A. the school music program and national radio
B. hurricane preparedness
C. harbor improvements and the school music program
D. small business loan program
E. national radio and senate office building remodeling
2. If the legislature decides to fund the agricultural subsidy program, national radio, and
the small business loan program, what two other programs could they fund?
A. harbor improvements and international airport
B. harbor improvements and school music program
C. hurricane preparedness and school music program
D. hurricane preparedness and international airport

USPC - CSAT 217


UPSC - CSAT -1
E. harbor improvements and hurricane preparedness
3. If the legislature decides to fund the agricultural subsidy program, national radio, and
the small business loan program, the only other single program that can be funded is
A. hurricane preparedness.
B. harbor improvements.
C. school music program.
D. senate office building remodeling.
E. international airport.
II. At a small company, parking spaces are reserved for the top executives: CEO, president, vice
president, secretary, and treasurer with the spaces lined up in that order. The parking lot guard
can tell at a glance if the cars are parked correctly by looking at the color of the cars. The cars
are yellow, green, purple, red, and blue, and the executives names are Alisha, Binoy, Maria,
Dev, and Eshwar.
¾¾ The car in the first space is red.
¾¾ A blue car is parked between the red car and the green car.
¾¾ The car in the last space is purple.
¾¾ The secretary drives a yellow car.
¾¾ Alisha's car is parked next to Dev's.
¾¾ Eshwar drives a green car.
¾¾ Binoy's car is parked between Maria's and Eshwar's.
¾¾ Dev's car is parked in the last space.
1. Who is the secretary?
A. Eshwar B. Dev C. Maria D. Binoy
E. Alisha
2. Who is the CEO ?
A. Alisha B. Binoy C. Maria D. Dev
E. Eshwar
3. What color is the vice president's car?
A. green B. yellow C. blue D. purple
E. red
III. Five cities all got more rain than usual this year. The five cities are: Lapataganj, Rampur,
Furasatganj, Chambal, and Jhumritalaiyya. The cities are located in five different areas of the
country: the mountains, the forest, the coast, the desert, and in a valley. The rainfall amounts
were: 12 inches, 27 inches, 32 inches, 44 inches, and 65 inches.

218 USPC - CSAT


UPSC - CSAT -1
¾¾ The city in the desert got the least rain; the city in the forest got the most rain.
¾¾ Furasatganj is in the mountains.
¾¾ Lapataganj got more rain than Chambal.
¾¾ Rampur got more rain than Jhumritalaiyya, but less rain than Furasatganj.
¾¾ Chambal got 44 inches of rain.
¾¾ The city in the mountains got 32 inches of rain; the city on the coast got 27 inches of rain.
1. Which city got the most rain?
A. Lapataganj B. Rampur C. Furasatganj D. Chambal
E. Jhumritalaiyya
2. How much rain did Rampur get?
A. 12 inches B. 27 inches C. 32 inches D. 44 inches
E. 65 inches
3. Which city is in the desert ?
A. Lapataganj B. Rampur C. Furasatganj D. Chambal
E. Jhumritalaiyya
4. Where is Chambal located?
A. the mountains B. the coast C. in a valley D. the desert
E. the forest
IV. Five roommates Raman, Salina, Tarun, Uma, and Salman each do one housekeeping task
mopping, sweeping, laundry, vacuuming, or dusting one day a week, Monday through Friday.
¾¾ Salman does not vacuum and does not do his task on Tuesday.
¾¾ Salina does the dusting, and does not do it on Monday or Friday.
¾¾ The mopping is done on Thursday.
¾¾ Tarun does his task, which is not vacuuming, on Wednesday.
¾¾ The laundry is done on Friday, and not by Uma.
¾¾ Raman does his task on Monday.
1. What task does Tarun do on Wednesday?
A. vacuuming B. dusting
C. mopping D. sweeping
E. laundry
2. What day does Uma do her task?
A. Monday B. Tuesday C. Wednesday D. Thursday
E. Friday

USPC - CSAT 219


UPSC - CSAT -1
3. What task does Salman do?
A. vacuuming B. dusting
C. mopping D. sweeping
E. laundry
4. What day is the vacuuming done?
A. Friday B. Monday C. Tuesday D. Wednesday
E. Thursday
5. When does Salina do the dusting?
A. Friday B. Monday C. Tuesday D. Wednesday
E. Thursday
Find the number that fits somewhere into the middle of the series. Some of the questions
involve both numbers and letters
1. F2, __, D8, C16, B32, ...?
A. A16 B. G4 C. E4 D. E3
2. 664, 332, 340, 170, ____, 89, ... ?
A. 85 B. 97 C. 109 D. 178
3. V, VIII, XI, XIV, __, XX, ... ?
A. IX B. XXIII C. XV D. XVII
4. 70, 71, 76, __, 81, 86, 70, 91, ... ?
A. 70 B. 71 C. 80 D. 96
5. 8, 43, 11, 41, __, 39, 17, ... ?
A. 8 B. 14 C. 43 D. 44
6. VI, 10, V, 11, __, 12, III, ... ?
A. II B. IV C. IX D. 14
7. (1/9), (1/3), 1, ____ , 9, ... ?
A. (2/3) B. 3 C. 6 D. 27
8. 83, 73, 93, 63, __, 93, 43, ... ?
A. 33 B. 53 C. 73 D. 93
9. 15, __, 27, 27, 39, 39, ... ?
A. 51 B. 39 C. 23 D. 15
10. 72, 76, 73, 77, 74, __, 75, ... ?
A. 70 B. 71 C. 75 D. 78

220 USPC - CSAT


UPSC - CSAT -1
11. J14, L16, __, P20, R22, ... ?
A. S24 B. N18 C. M18 D. T24
12. 4, 7, 25, 10, __, 20, 16, 19, ... ?
A. 13 B. 15 C. 20 D. 28
13. XXIV, XX, __, XII, VIII, ... ?
A. XXII B. XIII C. XVI D. IV
14. 0.15, 0.3, ____, 1.2, 2.4, ... ?
A. 4.8 B. 0.006 C. 0.6 D. 0.9
15. U32, V29, __, X23, Y20, ... ?
A. W26 B. W17 C. Z17 D. Z26

Look carefully for the pattern, and then choose which pair of numbers comes next.
1. 42 40 38 35 33 31 28
A. 25 22 B. 26 23 C. 26 24 D. 25 23
E. 26 22
2. 6 10 14 18 22 26 30
A. 36 40 B. 33 37 C. 38 42 D. 34 36
E. 34 38
3. 8 12 9 13 10 14 11
A. 14 11 B. 15 12 C. 8 15 D. 15 19
E. 8 5
4. 36 31 29 24 22 17 15
A. 13 11 B. 10 5 C. 13 8 D. 12 7
E. 10 8
5. 3 5 35 10 12 35 17
A. 22 35 B. 35 19 C. 19 35 D. 19 24
E. 22 24
6. 13 29 15 26 17 23 19
A. 21 23 B. 20 21 C. 20 17 D. 25 27
E. 22 20
7. 14 14 26 26 38 38 50
A. 60 72 B. 50 62 C. 50 72 D. 62 62

USPC - CSAT 221


UPSC - CSAT -1
E. 62 80
8. 44 41 38 35 32 29 26
A. 24 21 B. 22 19 C. 23 19 D. 29 32
E. 23 20
9. 34 30 26 22 18 14 10
A. 8 6 B. 6 4 C. 14 18 D. 6 2
E. 4 0
10. 32 31 32 29 32 27 32
A. 25 32 B. 31 32 C. 29 32 D. 25 30
E. 29 30
11. 7 9 66 12 14 66 17
A. 19 66 B. 66 19 C. 19 22 D. 20 66
E. 66 20
12. 3 8 10 15 17 22 24
A. 26 28 B. 29 34 C. 29 31 D. 26 31
E. 26 32
13. 4 7 26 10 13 20 16
A. 14 4 B. 14 17 C. 18 14 D. 19 13
E. 19 14
14. 32 29 26 23 20 17 14
A. 11 8 B. 12 8 C. 11 7 D. 32 29
E. 10 9
15. 16 26 56 36 46 68 56
A. 80 66 B. 64 82 C. 66 80 D. 78 68
E. 66 82
Translate from an imaginary language into English. Then, look for the word elements
that appear both on the list and in the answer choices.
1. gemolinea means fair warning
gerimitu means report card
gilageri means weather report
Which word could mean "fair weather"?
A. gemogila B. gerigeme C. gemomitu D. gerimita

222 USPC - CSAT


UPSC - CSAT -1
2. slar means jump
slary means jumping
slarend means jumped
Which word could mean "playing"?
A. clargslarend B. clergy
C. ellaclarg D. slarmont
3. jalkamofti means happy birthday
moftihoze means birthday party
mentogunn means goodness
Which word could mean "happiness"?
A. jalkagunn B. mentohoze
C. moftihoze D. hozemento
4. plekapaki means fruitcake
pakishillen means cakewalk
treftalan means buttercup
Which word could mean "cupcake"?
A. shillenalan B. treftpleka
C. pakitreft D. alanpaki
5. malgauper means peach cobbler
malgaport means peach juice
moggagrop means apple jelly
Which word could mean "apple juice"?
A. moggaport B. malgaauper
C. gropport D. moggagrop
6. peslligen means basketball court
ligenstrisi means courtroom
oltaganti means placement test
Which word could mean "guest room"?
A. peslstrisi B. vosefstrisi C. gantipesl D. oltastrisi
7. mallonpiml means blue light
mallontifl means blueberry
arpantifl means raspberry
Which word could mean "lighthouse"?

USPC - CSAT 223


UPSC - CSAT -1
A. tiflmallon B. pimlarpan C. mallonarpan D. pimldoken
8. briftamint means militant
uftonel means occupied
uftonalene means occupation
Which word could mean "occupant"?
A. elbrifta B. uftonamint C. elamint D. briftalene
9. morpirquat means birdhouse
beelmorpir means bluebird
beelclak means bluebell
Which word could mean "houseguest"?
A. morpirhunde B. beelmoki
C. quathunde D. clakquat
10. relftaga means carefree
otaga means careful
fertaga means careless
Which word could mean "aftercare"?
A. zentaga B. tagafer C. tagazen D. relffer
11. aptaose means first base
eptaose means second base
lartabuk means ballpark
Which word could mean "baseball"?
A. buklarta B. oseepta C. bukose D. oselarta
12. krekinblaf means workforce
dritakrekin means groundwork
krekinalti means workplace
Which word could mean "someplace"?
A. moropalti B. krekindrita C. altiblaf D. dritaalti


224 USPC - CSAT


UPSC - CSAT -1

Chapter

2 Data Arrangement

1. A, P, R, X, S and Z are sitting in a row. S and Z are in the centre. A and P are at the ends.
R is sitting to the left of A. Who is to the right of P?
A. A B. X C. S D. Z
2. A, B, C, D and E are sitting on a bench. A is sitting next to B, C is sitting next to D, D is
not sitting with E who is on the left end of the bench. C is on the second position from
the right. A is to the right of B and E. A and C are sitting together. In which position A
is sitting?
A. Between B and D B. Between B and C
C. Between E and D D. Between C and E

Directions for Questions 3-6:

P, Q, R, S, T, U, V and W are sitting round the circle and are facing the centre:
P is second to the right of T who is the neighbour of R and V.
S is not the neighbour of P.
V is the neighbour of U.
Q is not between S and W. W is not between U and S.
3. Which two of the following are not neighbours?
A. RV B. UV C. RP D. QW
4. Which one is immediate right to the V?
A. P B. U C. R D. T
5. Which of the following is correct?
A. P is to the immediate right of Q B. R is between U and V
C. Q is to the immediate left of W D. U is between W and S
6. What is the position of S?
A. Between U and V B. Second to the right of P
C. To the immediate right of W D. Data inadequate.

USPC - CSAT 225


UPSC - CSAT -1
Directions for Questions 7-10:
Five girls are sitting on a bench to be photographed. Seema is to the left of Rani and to the
right of Bindu. Mary is to the right of Rani. Reeta is between Rani and Mary.
7. Who is sitting immediate right to Reeta?
A. Bindu B. Rani C. Mary D. Seema
8. Who is in the middle of the photograph?
A. Bindu B. Rani C. Reeta D. Seema
9. Who is second from the right?
A. Mary B. Rani C. Reeta D. Bindu
10. Who is second from the left in photograph?
A. Reeta B. Mary C. Bindu D. Seema

Directions for Questions 11-14:


Six friends are sitting in a circle and are facing the centre of the circle. Deepa is between
Prakash and Pankaj. Priti is between Mukesh and Lalit. Prakash and Mukesh are opposite to
each other.
11. Who is sitting opposite to Prakash?
A. Mukesh B. Deepa C. Pankaj D. Lalit
12. Who is just right to Pankaj?
A. Deepa B. Lalit C. Prakash D. Priti
13. Who are the neighbours of Mukesh?
A. Prakash and Deepa B. Deepa and Priti
C. Priti and Pankaj D. Lalit and Priti
14. Who is sitting opposite to Priti?
A. Prakash B. Deepa C. Pankaj D. Lalit

Directions for Questions 15-18:


In an Exhibition seven cars of different companies - Cadillac, Ambassador, Fiat, Maruti,
Mercedes, Bedford and Fargo are standing facing to east in the following order:
Cadillac is next to right of Fargo.
Fargo is fourth to the right of Fiat.
Maruti is between Ambassador and Bedford.
Fiat which is third to the left of Ambassador, is at one end.

226 USPC - CSAT


UPSC - CSAT -1
15. Which pair of cars are on both the sides of Cadillac car?
A. Ambassador and Maruti B. Maruti and Fiat
C. Fargo and Mercedes D. Ambassador and Fargo
16. Which of the following statement is correct?
A. Maruti is next left of Ambassador. B. Bedford is next left of Fiat.
C. Bedford is at one end. D. Fiat is next second to the right of Maruti.
17. Which one of the following statements is correct?
A. Fargo car is in between Ambassador and Fiat.
B. Cadillac is next left to Mercedes car.
C. Fargo is next right of Cadillac.
D. Maruti is fourth right of Mercedes.
18. Which of the following groups of cars is to the right of Ambassador?
A. Cadillac, Fargo and Maruti B. Mercedes, Cadillac and Fargo
C. Maruti, Bedford and Fiat D. Bedford, Cadillac and Fargo

Directions for the following 5 (five) items:


Examine the information given in the following paragraph and answer the items that follow:
Guest lectures on five subjects viz., Economics, History, Statistics, English and Mathematics
have to be arranged in a week from Monday to Friday. Only one lecture can be arranged
on each day. Economics cannot be scheduled on Tuesday. Guest faculty for History is
available only on Tuesday. Mathematics lecture has to be scheduled immediately after the
day of Economics lecture. English lecture has to be scheduled immediately before the day of
Economics lecture.
19. Which lecture is scheduled on Monday?
A. History B. Economics C. Mathematics D. Statistics
20. Which lecture is scheduled between Statistics and English?
A. Economics B. History C. Mathematics D. No lecture
21. Which lecture is the last one in the week?
A. History B. English C. Mathematics D. Economics
22. Which lecture is scheduled on Wednesday?
A. Statistics B. Economics C. English D. History
23. Which lecture is scheduled before the Mathematics lecture?
A. Economics B. History C. Statistics D. English

USPC - CSAT 227


UPSC - CSAT -1
24. In five flats, one above the other, live five professionals. The professor has to go up to
meet his IAS officer friend. The doctor is equally friendly to all, and has to go up as
frequently as go down. The engineer has to go up to meet his MLA friend above whose
flat lives the professor’s friend. From the ground floor to the top floor, in what order do
the five professionals live?
A. Engineer, Professor, Doctor, IAS officer, MLA
B. Professor, Engineer, Doctor, IAS officer, MLA
C. IAS officer, Engineer, Doctor, Professor, MLA
D. Professor, Engineer, Doctor, MLA, IAS officer
25. Four political parties W, X, Y and Z decided to set up a joint candidate for the coming
parliamentary elections. The formula agreed by them was the acceptance of a candidate
by most of the parties. Four aspiring candidates, A, B, C and D approached the parties
for their tickets.
A was acceptable to W but not to Z.
B was acceptable to Y but not to X.
C was acceptable to W and Y.
D was acceptable to W and X.
When candidate B was preferred by W and Z, candidate C was preferred by X and Z,
and candidate A was acceptable to X but not to Y; who got the ticket?
A. A B. B C. C D. D
26. Ten new TV shows started in January — 5 sitcoms, 3 drama and 2 news magazines. By
April, only seven of the new shows were still on, five of them being sitcoms. Based on
the above information, four conclusions, as given below, have been made. Which one of
these logically follows from the information given above?
A. Only one news magazine show is still on.
B. Only one of the drama shows is still on.
C. At least one discontinued show was a drama.
D. Viewers prefer sitcoms over drama.
27. Read the passage given below and the two statements that follow (given on the basis of
the passage):
Four men are waiting at Delhi airport for a Mumbai flight. Two are doctors and the other two
are businessmen. Two speak Gujarati and two speak Tamil. No two of the same profession
speak the same language. Two are Muslims and two are Christians. No two of the same

228 USPC - CSAT


UPSC - CSAT -1
religion are of the same profession, nor do they speak the same language. The Tamil speaking
doctor is Christian.
1. The Christian-businessman speaks Gujarati.
2. The Gujarati-speaking doctor is a Muslim.
Which of the above statements is/are correct conclusion/conclusions?
A. 1 only B. 2 only C. Both 1 and 2 D. Neither 1 nor 2

Directions for questions 28-30:


A, B, C, D and E are members of the same family. There are two fathers, two sons, two wives,
three male and two females. The teacher was the wife of a lawyer who was the son of a doctor
E is not a male, neither also a wife of a professional. C is the youngest person in the family
and D is the eldest. B is a male.
28. How is D related to E?
A. Husband B. Son C. Father D. Wife
29. Who are the females in the group?
A. C and E B. C and D C. E and A D. D and E
30. Whose wife is the teacher?
A. C B. D C. A D. B

Directions for Questions 31 to 34:


Answer the questions after reading through the passage.
Six plays, P, Q, R, S, T and U are to be held during the week i.e., from Sunday to Saturday. In
the day, only one play can be shown, and the showing of the plays is subject to the following
conditions:
i. A two day gap should exist between the showing of plays T and S.
ii. The showing of U should be followed immediately by the showing of R.
iii. P cannot be shown on Thursday.
iv. Q should be shown on Tuesday and should not be followed by S.
v. There won’t be any play on a day. Friday or Sunday is not that day and just before this
day, S has to be shown.
31. No play is shown on:
A. Sunday B. Saturday C. Monday D. Tuesday

USPC - CSAT 229


UPSC - CSAT -1
32. On which day will the play R be shown?
A. Friday B. Saturday C. Thursday D. Monday
33. Which play is the last one to be shown?
A. S B. R C. P D. U
34. How many plays are shown between S and U?
A. One B. Two C. Three D. None of these

Directions for Question 35 to 37:


Refer to the data and answer the questions the follow:
(i) A, B, C, D, E and F are six members of a group. Out of these 3 are males and 3 are fe
males.
(ii) There are 2 electricians, 2 plumbers, one television star and one tailor in the group
(iii) B, E, C and A are two married couples, each one having a different profession.
(iv) E, a television star, wear a black gown, is married to a plumber in a brown dress. 2
people
wear black clothes, 2 wear brown clothes and the remaining people wear blue and
gray each.
(v) Both husbands and both wives wear the same coloured clothes respectively.
(vii) A is a male electrician and D is his twin sister who is also an electrician.
(viii) B is a tailor
35. Which of the following are the two married ladies?
A. E and C B. B and C C. B and E D. C and D
36. Who are the married couples?
A. AE, BC B. AB, EC C. AC, BE D. None of these
37. What colour dress does the unmarried lady wear?
A. Black B. Grey C. Blue D. Grey or Blue

Directions for Questions 38 to 42:


Refer to the data below and answer the questions that follow.
All the roads of city Z are either perpendicular or parallel to one another. The roads are all
straight. Road, A, B, C, D and E are parallel to one another. Roads G, H, I, H, J, K, L and M
are parallel to one another.

230 USPC - CSAT


UPSC - CSAT -1
i. Road A is 1 mile east of road B
ii. Road B is 1/2 mile west of C.
iii. Road D is 1 mile west of E.
iv. Road G is 1/2 mile south of H.
v. Road I is 1 mile north of J
vi. Road K is 1/2 mile north of L.
vii. Road K is 1 mile south of M
38. Which of the following statements is necessarily true?
A. I is 1 mile north of L B. D is 2 miles west of B
C. E and B intersect D. M is 1.5 miles north of L
39. If E is midway between B and C, then which of the following statement is false?
A. D is less than 1 mile from B.
B. C is less than 1.5 miles from D.
C. Distance from E to B added to distance of E to C is 1/2 mile.
D. D is 2 miles west of A
40. Which of the following possibilities would make two roads coincide?
A. L is 1/2 mile north of I B. D is 1/2 mile east of A
C. I is 1/2 mile north of K D. C is 1 mile west of D
41. If X is parallel to I & X is 1/2 mile south of J & 1 mile north of G, then which road would
be ½ mile apart?
A. I and X B. X and H C. J and G D. J and H
42. If road E is midway between B and C, then the distance between A and D is
A. 1/2 mile B. 1 mile C. 1.75 miles D. 2.5 miles

Directions for Question 43 to 44:


Answer the questions on the basis of the information given below.
Age Consultants have three consultants Gyani, Medha and Budhi. The sum of the number of
projects handled by Gyani, Medha and Budhi individually is equal to the number of projects
in which Medha is involved. All three consultants are involved together in 6 projects. Gyani
works with Medha in 14 projects. Budhi has 2 projects with Medha but without Gyani,
and 3 projects with Gyani but without Medha. The total number of projects for New Age
Consultants is one less than twice the number of projects in which more than one consultant

USPC - CSAT 231


UPSC - CSAT -1
is involved.
43. What is the number of projects in which Medha alone is involved?
A. Uniquely equal to zero. B. Uniquely equal to 2.
C. Uniquely equal to 4. D. Can’t be determined uniquely.
44. What is the number of projects in which Gyani alone is involved?
A. Uniquely equal to zero. B. Uniquely equal to 1.
C. Uniquely equal to 4. D. Can’t be determined uniquely.

Directions for Q. 45 to 48:


Consider the following statements where every person gets exactly one different dish:
i. Ria will not get soup unless Janet gets hot coffee.
ii. Gia will not get gums unless Veena gets soup.
iii. Veena will not get tea unless Gia gets soup.
iv. Janet will not get gums unless Ria gets hot coffee.
v. Janet will not get hot coffee unless Veena gets gums.
vi. Gia will not get hot coffee unless Ria gets tea.
vii. Gia will not get tea unless Ria gets hot coffee.
viii. Ria will not get hot coffee unless Gia gets soup.
ix. Veena will not get gums unless Ria gets the hot coffee.
x. Janet will not get tea unless Ria gets gums.
xi. Gia will not get soup unless Ria gets gums.
45. Who gets gums?
A. Ria B. Gia C. Janet D. Veena
46. Who gets soup?
A. Janet B. Veena C. Gia D. Ria
47. Who gets hot coffee?
A. Gia B. Veena C. Ria D. Janet
48. Who does Janet get?
A. Hot coffee B. Soup C. Gums D. Tea

232 USPC - CSAT


UPSC - CSAT -1
Directions for Q. 49 – 53:
Refer to following passage:
Six participants in the National Billiards Championship, who played in the super six stage
of the championship all belonged to different states. The six states are Gujarat, Orissa,
Karnataka, Maharasthra, MP and UP. The six participants are aged 18, 26, 32, 34, 38, 44
years (not necessarily in order).
1. Pravin is the oldest while Laxman is the youngest player.
2. Player from MP is aged 32
3. Minal comes from Orissa but Laxman is not from Gujarat
4. Pankaj and Kunal belong to Karnataka and UP resp. They are not aged 38 or 18
5. Asim, 32 is not from Maharashtra or Gujarat
6. Minal, Laxman and Pankaj are neither the oldest nor in their twenties.
49. Which of the following statements must be true?
A. Pravin 44, is from Orissa B. Kunal, 26, is from MP
C. Pankaj, 26, is from Karnakata D. Laxman, 18, is from Maharashtra
50. Which of the following statements must not be true?
A. Pravin 44, belongs to Gujarat B. Pankaj, 34, belongs to Karnataka
C. Asim, 32, belongs to Orissa D. Laxman, 18, belongs to Maharasthra
51. Pravin belongs to the state of
A. Gujarat B. Orissa C. Maharashtra D. None of these
52. Which player is aged 34?
A. Kunal B. Pankaj C. Pravin D. Kunal or Pankaj
53. Which player is in his 20s?
A. Minal B. Pankaj C. Kunal D. Pravin
54. Every month Chess Federation of India publishes ranking of Indian Chess Players.
They actually complement the FIDE lists which are brought out at longer intervals. It
was seen, observing monthly lists for last year that top six players in the list remained
same throughout the year but there was considerable mutual change of rankings among
these six. Thus ranking for January 2003 as follows:
January 2003 was as follows:
1. P. Harikrishna 2. D. Barua
3. K. Humpy 4. S. Chanda
USPC - CSAT 233
UPSC - CSAT -1
5. K. Ramesh 6. S. S Ganguli
The list of rankings for Feb 2003 had an entirely different look with each of the six ranked in
a position from the previous one. The following facts are known:
1. No one else had his/her ranking changed by as many places as D. Barua, whose change
in ranking was the greatest of the six.
2. The product of Chanda’s ranking for the two months was the same as product of Gan
guli’s ranking for the two months.
Who was ranked 5th in the list for February 2003?
A. P. Harikrishna B. D. Barua
C. K. Humpy D. None of these


234 USPC - CSAT


UPSC - CSAT -1

Chapter

3 Blood Relations + Que.


from Previous Year
1. Pointing to a photograph of a boy Dharmendra said, "He is the son of the only son of
my mother." How is Dharmendra related to that boy?
A. Brother B. Uncle C. Cousin D. Father
2. If A + B means A is the mother of B; A - B means A is the brother B; A % B means A is
the father of B and A x B means A is the sister of B, which of the following shows that P
is the maternal uncle of Q?
A. (Q - N + M x P) B. (P + S x N – Q)
C. (P - M + N x Q) D. (Q - S % P)
3. If A is the brother of B; B is the sister of C; and C is the father of D, how D is related to
A?
A. Brother B. Sister
C. Nephew D. Cannot be determined
4. If A + B means A is the brother of B; A - B means A is the sister of B and A x B means A
is the father of B. Which of the following means that C is the son of M?
A. (M - N x C + F) B. (F - C + N x M)
C. (N + M - F x C) D. (M x N - C + F)
5. Introducing Ranbir, Kareena said, "He is the son of the daughter of the father of my
uncle." How is the boy related to the girl?
A. Brother B. Nephew C. Uncle D. Son-in-law
6. Pointing to a photograph Rekha says, "He is the son of the only son of my grandfather."
How is the man in the photograph related to Rekha?
A. Brother B. Uncle C. Cousin D. Data is inadequate
7. If A + B means A is the brother of B; A x B means A is the son of B; and A % B means B
is the daughter of A then which of the following means M is the maternal uncle of N?
A. M + O x N B. M % O x N + P C. M + O % N D. None of these
8. If D is the brother of B, how B is related to C? To answer this question which of the
statements is/are necessary?

USPC - CSAT 235


UPSC - CSAT -1
1. The son of D is the grandson of C. 2. B is the sister of D.
A. Only 1 B. Only 2
C. Either 1 or 2 D. 1 and 2 both are required
9. If A + B means A is the father of B; A - B means A is the brother B; A % B means A is the
wife of B and A x B means A is the mother of B, which of the following shows that M is
the maternal grandmother of T?
A. M x N % S + T B. M x N - S % T
C. M x S - N % T D. M x N x S % T
10. Pointing to a photograph. jeetendra said, "He is the son of the only daughter of the
father of my brother." How jeetendra is related to the man in the photograph?
A. Nephew B. Brother C. Father D. Maternal Uncle
11. Imran said to Aamir, "That boy playing with the football is the younger of the two
brothers of the daughter of my father's wife." How is the boy playing football related to
Imran?
A. Son B. Brother C. Cousin D. Brother-in-law
12. Pointing a photograph X said to his friend Y, "She is the only daughter of the father of
my mother." How X is related to the person of photograph?
A. Daughter B. Son C. Nephew D. Cannot be decided
13. Amrita who is the sister-in-law of Arbaaz, is the daughter-in-law of Helen. Salim is the
father of Salman who is the only brother of Arbaaz. How is Helen related to Arbaaz?
A. Mother-in-law B. Aunt
C. Wife D. None of these
14. If A + B means A is the sister of B; A x B means A is the wife of B, A % B means A is the
father of B and A - B means A is the brother of B. Which of the following means T is the
daughter of P?
A. (P x Q%R+S–T) B. (P x Q%R-T+S)
C. (P x Q%R+T–S) D. (P x Q%R+S+T)
15. Pointing to a woman, Tushar said, "Her granddaughter is the only daughter of my
brother." How is the woman related to Tushar?
A. Sister B. Grandmother C. Mother-in-law D. Mother
16. Sanjay said - "This girl is the wife of the grandson of my mother". How is Sanjay
related to the girl?
A. Father B. Grandfather C. Husband D. Father-in-law
236 USPC - CSAT
UPSC - CSAT -1
17. A and B are children of D. Who is the father of A? To answer this question which of the
statements (1) and (2) is necessary?
1. C is the brother of A and the son of E. 2. F is the mother B.
A. Only (1) B. Only (2) C. Either (1) or (2) D. (1) and (2) both
18. Pointing towards Dilip, Saira said, "His mother is the only daughter of my mother."
How is Saira related to Dilip?
A. Mother B. Grandmother C. Sister D. Daughter
19. If P $ Q means P is the brother of Q; P # Q means P is the mother of Q; P * Q means P
is the daughter of Q in A # B $ C * D, who is the father?
A. D B. B C. C D. Data is inadequate
20. Introducing Sunny, Abhay says, "She is the wife of only nephew of only brother of my
mother." How Sunny is related to Abhay?
A. Wife B. Sister C. Sister-in-law D. Data is inadequate
21. If A + B means A is the brother of B; A % B means A is the father of B and A x B means
A is the sister of B. Which of the following means M is the uncle of P?
A. M % N x P B. N x P % M C. M + S % R % P D. M + K % T x P
22. Pointing to Kishore, Ashok said, "I am the only son of one of the sons of his father."
How is Kishore related to Ashok?
A. Nephew B. Uncle C. Father or Uncle D. Father
23. Introducing Hema, Abhay said, "She is the mother of the only daughter of my son."
How is Hema is related to Abhay?
A. Daughter B. Sister-in-law C. Wife D. Daughter-in-law
24. If A + B means B is the brother of A; A x B means B is the husband of A; A - B means A
is the mother of B and A % B means A is the father of B, which of the following relations
shows that Q is the grandmother of T?
A. Q - P + R % T B. P x Q % R – T
C. P x Q % R + T D. P + Q % R - T
25. A3P means A is the mother of P;A4P means A is the brother of P
A9P means A is the husband of P;A5P means A is the daughter of P
Which of the following means that K is the mother-in-law of M?
A. M9N3K4J B. M9N5K3J C. K5J9M3N D. K3J9N4M

USPC - CSAT 237


UPSC - CSAT -1
26. Pointing to a photograph Kajol said, "He is the son of the only son of my grandfather."
How is the man in the photograph related to Kajol?
A. Brother B. Uncle C. Son D. Data is inadequate
27. Pointing to a person, Gabbar said, "His only brother is the father of my daughter's
father". How is the person related to Gabbar?
A. Father B. Grandfather C. Uncle D. Brother-in-law
28. P is the mother of K; K is the sister of D; D is the father of J. How is P related to J?
A. Mother B. Grandmother C. Aunt D. Data inadequate
29. If P $ Q means P is the father of Q; P # Q means P is the mother of Q and P * Q means
P is the sister of Q, then N # L $ P * Q shows which of the relation of Q to N?
A. Grandson B. Granddaughter C. Nephew D. Data is inadequate
30. If A $ B means A is the brother of B; A @ B means A is the wife of B; A # B means A is
the daughter of B and A * B means A is the father of B, which of the following indicates
that U is the father-in-law of P?
A. P@Q$T#U*W B. P@W$Q*T#U
C. P@Q$W*T#U D. P@Q$T#W*U

Questions asked in UPSC


1. Consider the following relationships among members of a family of six persons A, B, C,
D, E and F:
1. The number of males equals that of females.
2. A and E are sons of F.
3. D is the mother of two, one boy and one girl.
4. B is the son of A.
5. There is only one married couple in the family at present.
Which one of the following inferences can be drawn from the above?
(a) A, B and C are all females. (b) A is the husband of D.
(c) E and F are children of D. (d) D is the daughter of F.
2. Consider the following:
A+ B means A is the son of B. A - B means A is the wife of B.
What does the expression P + R - Q mean?
(a) Q is the son of P. (b) Q is the wife of P.
238 USPC - CSAT
UPSC - CSAT -1
(c) Q is the father of P. (d) None of the above
3. Four persons A, B, C and D consisting of two married couples are in a group. Both the
women are shorter than their respective husbands. A is the tallest among the four. C is
taller than B. D is B's brother. In this context, which one of the following statements is
not correct ?
(a) All four have family ties. (b) B is the shortest among the four.
(c) C is taller than D. (d) A is B's husband.
4. Consider the following:
A, B, C, D, E, F, G and H are standing in a row facing North.
B is not neighbour of G.
F is to the immediate right of G and neighbour of E.
G is not at the extreme end.
A is sixth to the left of E.
H is sixth to the right of C.
Which one of the following is correct in respect of the above?
(a) C is to the immediate left of A. (b) D is immediate neighbour of B and F.
(c) G is to the immediate right of D. (d) A and E are at the extreme ends.
5. Five people A, B, C, D and E are seated about a round table. Every chair is spaced
equidistant from adjacent chairs.
(i) C is seated next to A
(ii) A is seated two seats from D
(iii) B is not seated next to A
On the basis of above information, which of the following must be true?
1. D is seated next to B
2. E is seated next to A
3. D and C are separated by two seats
Select the correct answer using the code given below :
(a) 1 only (b) 1 and 2 only
(c) 3 only (d) Neither 1 nor 2 nor 3

USPC - CSAT 239


UPSC - CSAT -1
6. In a certain code, '256' means 'red colour chalk', '589' means 'green colour flower' and
'254' means 'white colour chalk'. The digit in the code that indicates `white' is
(a) 2 (b) 4 (c) 5 (d) 8
7. A military code writes SYSTEM as SYSMET and NEARER as AENRER. Using the
same code, FRACTION can be written as :
(a) CARFTION (b) FRACNOIT
(c) NOITCARF (d) CARFNOIT
8. Consider the following statements followed by two conclusions :
Statements : Some men are great.
Some men are wise.
Conclusion I: Men are either great or wise.
Conclusion II: Some men are neither great nor wise.
Which one of the following is correct?
(a) Only conclusion I is valid (b) Only conclusion II is valid
(c) Both the conclusions are valid (d) Neither of the conclusions is valid
9. All good athletes want to win and all athletes who want to win eat a well-balanced diet;
therefore all athletes who do not eat a well-balanced diet are bad athletes.
The best conclusion from this statement is that
(a) no bad athlete wants to win.
(b) no athlete who does not eat a well-balanced diet is a good athlete.
(c) every athlete who eats a well-balanced diet is a good athlete.
(d) all athletes who want to win are good athletes.
10. There are some nectar-filled flowers on a tree and some bees are hovering on it. If one
bee lands on each flower, one bee will be left out. If two bees land on each flower, one
flower will be left out. The number of flowers and bees respectively are :
(a) 2 and 4 (b) 3 and 2 (c) 3 and 4 (d) 4 and 3
11. A person walks 12 km due north, then 15 km due east, after that 19 km due west and
then 15 km due south. How far is he from the starting point?
(a) 5 km (b) 9 km (c) 37 km (d) 61 km
12. What is the missing number “X” of the series 7, X, 21, 31, 43?
(a) 11 (b) 12 (c) 13 (d) 14

240 USPC - CSAT


UPSC - CSAT -1
13. Consider the following matrix
3 8 10 2 ? 1
6 56 90 2 20 0

What is the missing number at '?' in the matrix?


(a) 5 (b) 0 (c) 7 (d) 3
14. There are 4 horizontal and 4 vertical lines, parallel and equidistant to one another on a
board. What is the maximum number of rectangles and squares that can be formed?
(a) 16 (b) 24 (c) 36 (d) 42
15. In a plane, line X is perpendicular to line Y and parallel to line Z; line U is perpendicular
to both lines V and W; line X is perpendicular to line V. Which one of the following
statements is correct ?
(a) Z, U and W are parallel.
(b) X, V and Y are parallel.
(c) Z, V and U are all perpendicular to W.
(d) Y, V and W are parallel.


USPC - CSAT 241


UPSC - CSAT -1

Chapter

4 Data Sufficiency

Mark option A if Statement (1) ALONE is sufficient, but statement (2) is not sufficient.
¾¾ Mark option B if Statement (2) ALONE is sufficient, but statement (1) is not sufficient.
¾¾ Mark option C if BOTH statements TOGETHER are sufficient, but NEITHER statement
ALONE is sufficient.
¾¾ Mark option D if EACH statement ALONE is sufficient.
¾¾ Mark option E if Statements (1) and (2) TOGETHER are NOT sufficient.
1. What is the value of x?
(1) x is negative. (2) 2x = -4
2. How old is Raja?
(1) Eight years ago Raja was half as old as he is now.
(2) Four years from now Raja will be twice as old as he was six years ago.
3. If a bottle is to be selected at random from a certain collection of bottles, what is the
probability that the bottle will be defective?
(1) The ratio of the number of bottles in the collection that are defective to the number that
are not defective is 3:500.
(2) The collection contains 3,521 bottles
4. Radha and Geeta are in a line to purchase tickets. How many people are in the line?
(1) There are 20 people behind Radha and 20 people in front of Geeta.
(2) There are 5 people between Radha and Geeta.
5. Was 70 the average (arithmetic mean) marks on a class test?
(1) On the test, half of the class had marks below 70 and half of the class had marks
above 70.
(2) The lowest marks on the test was 45 and the highest marks on the test was 95.
6. What was Ratan’s average driving speed in miles per hour during a 15-minute interval?
(1) He drove 10 miles during this interval.
(2) His maximum speed was 50 miles per hour and his minimum speed was 35 miles per
hour during this interval.

242 USPC - CSAT


UPSC - CSAT -1
7. If the Playground is rectangular, what is its width?
(1) The ratio of its length to its width is 7 to 2.
(2) The perimeter of the playground is 396 meters.
8. A coal company can choose to transport coal to one of its customers by railroad or by
truck. If the railroad charges by the mile and the trucking company charges by the ton,
which means of transporting the coal would cost less than the other?
(1) The railroad charges Rs. 5,000 plus Rs. 0.01 per mile per railroad car used, and the
trucking company charges Rs. 3,000 plus Rs. 85 per ton.
(2) The customer to whom the coal is to be sent is 195 miles away from the coal company.
9. If x and y are consecutive odd integers, what is the sum of x and y?
(1) The product of x and y is negative.
(2) One of the integers is equal to –1.
10. Who types at a faster rate, Bunty or Babli?
(1) The difference between their typing rates is 10 words per minute.
(2) Babli types at a constant rate of 80 words per minute.
11. In a certain packinghouse, grapes are packed in bags and the bags are packed in cases.
How many grapes are in each case that is packed?
(1) The grapes are always packed 5 to a bag and the bags are always packed 8 to a case.
(2) Each case is always 80 percent full.
12. A rectangular floor that is 4 meters wide is to be completely covered with non-overlap-
ping square tiles, each with side of length 0.25 meter, with no portion of any tile remain-
ing. What is the least number of such tiles that will be required?
(1) The length of the floor is three times the width.
(2) The area of the floor is 48 square meters.
13. A certain company paid bonuses of Rs. 125 to each of its executive employees and Rs. 75
to each of its nonexecutive employees. If 100 of the employees were nonexecutives, how
many were executives?
(1) The company has a total of 120 employees.
(2) The total amount that the company paid in bonuses to its employees was Rs10.000.
14. What fraction of his salary did Mr. Singh put into savings last week?
(1) Last week Mr. Singh put Rs. 1700 into savings.
(2) Last week Mr. Singh put 5% of his salary into savings.
15. If the price of a magazine is to be doubled, by what percent will the number of maga-

USPC - CSAT 243


UPSC - CSAT -1
zines sold decrease?
(1) The current price of the magazine is Rs 1.00.
(2) For every Rs 0.25 of increase in price, the number of magazines sold will decrease by
10 percent of the number sold at the current price.
16. If a rope is cut into three pieces of unequal length, what is the length of the shortest of
these pieces of rope?
(1) The combined length of the longer two pieces of rope is 12 meters.
(2) The combined length of the shorter two pieces of rope is 11 meters.
17. At a certain university, if 50 percent of the people who inquire about admission policies
actually submit applications for admission, what percent of those who submit applica-
tions for admission enroll in classes at the university?
(1) Fifteen percent of those who submit applications for admission are accepted at the
university.
(2) Eighty percent of those who are accepted send a deposit to the university.
18. Is Vasant taller than Janaki?
(1) Vasant is taller than Anupama.
(2) Anupama is not as tall as Janaki.
19. On a certain day it took Birju three times as long to drive from home to work as it took
Seeta to drive from home to work. How many kilometers did Birju drive from home to
work?
(1) Seeta drove 10 kilometers from home to work, and the ratio of distance driven from
home to work time to drive from home to work was the same for Birju and Seeta that
day.
(2) The ratio of distance driven from home to work time to drive from home to work for
Seeta that day was 64 kilometers per hour.
20. All trainees in a certain aviator training program must take both a written test and a
flight test. If 70 percent of the trainees passed the written test, and 80 percent of the
trainees passed the flight test, what percent of the trainees passed both tests?
(1) 10 percent of the trainees did not pass either test.
(2) 20 percent of the trainees passed only the flight test.
21. At a certain picnic, each of the guests was served either a single scoop or a double scoop
of ice cream. How many of the guests were served a double scoop of ice cream?
(1) At the picnic, 60 percent of the guests were served a double scoop of ice cream.
(2) A total of 120 scoops of ice cream were served to all the guests at the picnic.

244 USPC - CSAT


UPSC - CSAT -1
22. If l and w represent the length and width, respectively, of a rectangle, what is the perim-
eter?
(1) 2l + w = 40 (2) l + w = 25
23. How much did a certain telephone call cost?
(1) The call lasted 53 minutes
(2) The cost for the first 3 minutes was 5 times the cost for each additional minute.
24. What is the total number of employees in the personnel and data processing divisions of
Company S?
(1) The number of employees in the data processing division is 3 more than twice the
number of employees in the personnel division.
(2) The number of employees in the data processing division is 15.
25. Is 1 < x < 2?
(1) 0 < x (2) x < 3

Assignment
1. Ketki's raise increased his salary by what percent?
(1) Ketki's raise was Rs. 1,200.
(2) Ketki's raise increased his taxes to Rs. 1,700.
2. Mouse population X doubles every week. How many weeks from now will population X
first exceed 1,000,000 ?
(1) The mouse population is now 65,536.
(2) Fifteen weeks ago the mouse population was 2.
3. If no student took test T more than once, how many students took test T?
(1) The average (arithmetic mean) of the students' scores on test T was 72.
(2) The sum of he students' scores on test T was 2,232.
4. If x + 2y = 6, what is the value of x?
(1) 2x + y = 9 (2) 3x + 2y = 14
5. Last year 4/5 of the applicants for a job on a police force passed the physical examina-
tion. If 3/4 of the applicants who passed the physical examination also passed the writ-
ten examination, how many of the applicants passed both examinations?
(1) The number of applicants who did not pass either examination was equal to the number
who passed the written examination only.
(2) There was a total of 100 applicants.

USPC - CSAT 245


UPSC - CSAT -1
6. Is the integer n even?
(1) n2 -1 is odd. (2) is an integer.
7. If today is Chetna's birthday, how old is Carol?
(1) 6 years ago she was half her present age.
(2) 3 years from now she will be 3 times as old as she was 7 years ago.
8. If x, y, and z are positive, what is the value of x ?
(1) x + y = z + y (2) z–y=4–y
9. If y > 0, is y greater than x?
(1) 3x = 2y (2) x + y = 5
10. Did the population of Country S increase by less than 20 percent from 1965 to 1975?
(1) The population of Country S in 1965 was 180 million.
(2) The population of Country S in 1975 was 1.17 times what it was in 1965.
11. If the measures of' the three interior angles of a triangle are y°, 15x°, and 18x°, what is
the value of y
(1) x =5 (2) 15x + y = 90
12. What is the average (arithmetic mean) of x and y?
(1) x/2 + y/2 = 10 (2) x = 2y
13. How many bags of grass seed were used for rectangular lawn X?
(1) Lawn X has a perimeter of 720 feet.
(2) One bag of grass seed was used for each 5,000 square feet of lawn X
14. If n is a positive integer, is n divisible by at least six positive integers?
(1) n is the product of three different prime numbers.
(2) n = 30
15. If b is the product of three consecutive positive integers c, c + 1, and c + 2, is b a multiple
of 24?
(1) b is a multiple of 3, (2) c is odd.
16. If x and y are integers, is x + y divisible by 6?
(1) x is divisible by 6. (2) y is divisible by 6.
17. In a given class, what is the average (arithmetic mean) height per pupil?
(1) The average (arithmetic mean) height of the girls in the class is 61 inches.
(2) The average (arithmetic mean) height of the boys in the class is 64 inches.


246 USPC - CSAT


UPSC - CSAT -1

Chapter

5 Questions from Previous


Papers - 1
Directions for the following 2 (two) items :
Read the following passage and, answer the 2 (two) items that follow :
A, B, C, D, E and F are cousins. No two cousins are of the same age, but all have birthdays
on the same day of the same month. The youngest is 17 years old and the oldest E is 22 years
old. F is somewhere between B and D in age. A is older than B. C is older than D. A is one
year older than C.
1. Which one of the following is possible?
(a) D is 20 years old (b) F is 18 years old
(c) F is 19 years old (d) F is 20 years old
2. What is the number of logically possible orders of all six cousins in terms of increasing
age?
(a) 1 (b) 2 (c) 3 (d) 4
3. Two men, Anil and David, and two women, Shabnam and Rekha are in a sales group.
Only two speak Tamil. The other two speak Marathi. Only one man and one woman can
drive a car. Shabnam speaks Marathi. Anil speaks Tamil. Both Rekha and David can
drive.
Which of the following statements is true ?
(a) Both the Tamil speakers can drive a car.
(b) Both the Marathi speakers can drive a car.
(c) Both of those who can drive a car speak Marathi.
(d) One of those who can drive a car speaks Tamil.
4. A society consists of only two types of people— fighters and cowards. Two cowards are
always friends. A fighter and a coward are always enemies. Fighters are indifferent to
one another. If A and B are enemies, C and D are friends, E and F are indifferent to each
other, A and E are not enemies, while B and F are enemies.
Which of the following statements is correct?
(a) B, C and F are cowards. (b) A, E and F are fighters.

USPC - CSAT 247


UPSC - CSAT -1
(c) B and E are in the same category. (d) A and F are in different categories
5. A ate grapes and pineapple; B ate grapes and oranges; C ate oranges, pineapple and
apple; D ate grapes, apple and pineapple. After taking fruits, B and C fell sick. In the
light of the above facts, it can be said that the cause of sickness was:
(a) Apple (b) Pineapple (c) Grapes (d) Oranges

Directions for the following 5 (five) items :


Read the following passage and, answer the 5 (five) items that follow:
There are five persons in a group – P, Q, R, S and T. The group has one doctor, one lawyer and
one artist. P and S are unmarried students. T is a man married to one of the group members.
Q is the brother of P and is neither doctor nor artist. R is not doctor.
6. Who is the doctor?
(a) T (b) P (c) Q (d) R
7. Who is the artist?
(a) P (b) Q (c) R (d) T
8. Who is the spouse of R?
(a) P (b) T (c) Q (d) S
9. Who is the lawyer?
(a) P (b) Q (c) R (d) S
10. Who of the following is definitely a man?
(a) P (b) S (c) Q (d) None of the above.

Directions for the following 3 (Three) items:


Read the following passage and, answer the 3 (Three) items that follow:
Six boxes A, B, C, D, E and F have been painted with six different colours viz., violet, indigo,
blue, green, yellow and orange and arranged from left to right (not necessarily either kept or
painted with the colours in the same order). Each box contains a ball of any one of the fol-
lowing six games: cricket, hockey, tennis, golf, football and volleyball (not necessarily in the
same order). The golf ball is in violet box and is not in the box D. The box A which contains
tennis ball is orange in colour and is at the extreme right. The hockey ball is neither in box D
nor in box E. The box C having cricket ball is painted green. The hockey ball is neither in the
box painted blue nor in the box painted yellow. The box C is fifth from right and next to box
B. The box B contains volleyball. The box containing the hockey ball is between the boxes
containing golf ball and volleyball.
11. Which one of the following boxes contains the golf ball?

248 USPC - CSAT


UPSC - CSAT -1
(a) F (b) E (c) D (d) None of the above
12. Which of the following statements is/are correct?
(a) D is painted yellow (b) F is painted indigo
(c) B is painted blue (d) All of the above
13. The football is in the box of which colour?
(a) Yellow
(b) Indigo
(c) Cannot be determined as data are inadequate
(d) Blue

Direction for the following 3 (three) items :


Consider the given information and answer the three items that follow.
When three friends A, B and C met, it was found that each of them wore an outer garment of
a different colour. In random order, the garments are: jacket, sweater and tie; and the colours
are: blue, white and black. Their surnames in random order are: Ribeiro, Kumar and Singh.
Further, we know that:
1. neither B nor Ribeiro wore a white sweater
2. C wore a tie
3. Singh’s garment was not white
4. Kumar does not wear a jacket
5. Ribeiro does not like to wear the black colour
6. Each of the friends wore only one outer garment of only one colour
14. What is C’s surname?
(a) Riberio (b) Kumar
(c) Singh (d) Cannot be determined
15. What is the colour of the tie?
(a) Black (b) Blue
(c) White (d) Cannot be determined
16. Who wore the sweater?
(a) A (b) B
(c) C (d) Cannot be determined

USPC - CSAT 249


UPSC - CSAT -1
17. Usha runs faster than Kamala, Priti runs slower than Swati, Swati runs slower than
Kamala. Who is the slowest runner?
(a) Kamala (b) Priti (c) Swati (d) Usha
18. If A runs less faster than B, and B runs as fast but not faster than C; then, as compared
to A, C runs
(a) slower than A (b) faster than A
(c) with same speed as A (d) Given data is not sufficient to determine
19. Examine the following statements :
1. Lady's finger is tastier than cabbage.
2. Cauliflower is tastier than lady's finger.
3. Cabbage is not tastier than peas.
The conclusion that can be drawn from these statements is that
(a) peas are as tasty as lady's finger.
(b) peas are as tasty as cauliflower and lady's finger.
(c) cabbage is the least tasty of the four vegetables.
(d) cauliflower is tastier than cabbage.
20. Consider the following statements:
1. Either A and B are of the same age or A is older than B
2. Either C and D are of the same age or D is older than C
3. B is older than C
Which of the following conclusions can be drawn from the above statements?
(a) A is older than B (b) B and D are of the same age
(c) D is older than C (d) A is older than C

Directions for the following 3 (three) items :


Consider the given information and answer the three items that follow.
A, B, C, D, E, F and G are Lecturers from different cities—Hyderabad, Delhi, Shillong, Kan-
pur, Chennai, Mumbai and Srinagar (not necessarily in the same order) who participated in
a conference. Each one of them is specialized in a different subject, viz., Economics, Com-
merce, History, Sociology, Geography, Mathematics and Statistics (not necessarily in the
same order).
Further

250 USPC - CSAT


UPSC - CSAT -1
1. Lecturer from Kanpur is specialized in Geography
2. Lecturer D is from Shillong
3. Lecturer C from Delhi is specialized in Sociology
4. Lecturer B is specialized in neither History nor Mathematics
5. Lecturer A who is specialized in Economics does not belong to Hyderabad
6. Lecturer F who is specialized in Commerce belongs to Srinagar
7. Lecturer G who is specialized in Statistics belongs to Chennai
21. Who is specialized in Geography?
(a) B (b) D
(c) E (d) Cannot be determined as data are inadequate
22. To which city does the Lecturer specialized in Economics belong?
(a) Hyderabad (b) Mumbai
(c) Neither Hyderabad nor Mumbai (d) Cannot be determined as data are inadequate
23. Who of the following belongs to Hyderabad?
(a) B (b) E
(c) Neither B nor E (d) Cannot be determined as data are inadequate
24. In a school, there are five teachers A, B, C, D and E. A and B teach Hindi and English.
C and B teach English and Geography. D and A teach Mathematics and Hindi. E and B
teach History and French. Who teaches maximum number, of subjects?
(a) A (b) B (c) D (d) E

Directions for the following 3 (three) items :


Consider the given information and answer the three items that follow.
Eight railway stations A, B, C, D, E, F, G and H are connected either by two-way passages or
oneway passages. One-way passages are from C to A, E to G, B to F, D to H, G to C, E to C
and H to G. Two-way passages are between A and E, G and B, F and D, and E and D.
25. While travelling from C to H, which one of the following stations must be passed
through?
(a) G (b) E (c) B (d) F
26. In how many different ways can a train navel from F to A without passing through any
station more than once?
(a) 1 (b) 2 (c) 3 (d) 4

USPC - CSAT 251


UPSC - CSAT -1
27. If the route between G and C is closed, which one of the following stations need not be
passed through while travelling from H to C?
(a) E (b) D (c) A (d) B
28. In a group of six women, there are four tennis players, four postgraduates in Sociology,
one postgraduate in Commerce and three bank employees. Vimala and Kamla are the
bank employees while Amala and Komala are unemployed. Komala and Nirmala are
among the tennis players. Amala, Kamla, Komala and Nirmala are postgraduates in
Sociology of whom two are bank employees. If Shyamala is a postgraduate in Commerce,
who among the following is both a tennis player and a bank employee?
(a) Amala (b) Komala (c) Nirmala (d) Shyamala
29. Six boys A, B, C, D, E and F play a game of cards. Each has a pack of 10 cards. F
borrows 2 cards from A and gives away 5 to C who in turn gives 3 to B while B gives 6
to D who passes 1 to E. Then the number of cards possessed by D and E is equal to the
number of cards possessed by
(a) A, B and C (b) B, C and F
(c) A, B and F (d) A, C and F
30. In a test, Randhir obtained more marks than the total marks obtained by Kunal and
Debu. The total marks obtained by Kunal and Shankar are more than those of Randhir.
Sonal obtained more marks than Shankar. Neha obtained more marks than Randhir.
Who amongst them obtained highest marks?
(a) Randhir (b) Neha
(c) Sonal (d) Data are inadequate
31. Shahid and Rohit start from the same point in opposite directions. After each 1 km,
Shahid always turns left and Rohit always turns right. Which of the following statements
is correct?
(a) After both have travelled 2 km, the distance between them is 4 km.
(b) They meet after each has travelled 3km.
(c) They meet for the first time after each has travelled 4 km.
(d) They go on without ever meeting again.
32. A person X was driving in a place where all roads ran either north-south or east-
west, forming a grid. Roads are at a distance of 1 km from each other in a parallel. He
started at the intersection of two roads, drove 3 km north, 3 km west and 4 km south.
Which further route could bring him back to his starting point, if the same route is not
repeated?

252 USPC - CSAT


UPSC - CSAT -1
(a) 3 km east, then 2 km south (b) 3 km east, then 1 km north
(c) 1 km north, then 2 km west (d) 3 km south, then 1 km north
33. A person climbs a hill in a straight path from point ‘O’ on the ground in the direction
of north-east and reaches a point ‘A’ after travelling a distance of 5 km. Then, from the
point ‘A’ he moves to point ‘B’ in the direction of north-west. Let the distance AB be 12
km. Now, how far is the person away from the starting point ‘O’?
(a) 7 km (b) 13 km (c) 17 km (d) 11 km
34. In a town, 45% population read magazine A, 55% read magazine B, 40% read magazine
C, 30% read magazines A and B, 15% read magazines B and C, 25% read magazines
A and C; and 10% read all the three magazines. What percentage do not read any
magazine?
(a) 10% (b) 15% (c) 20% (d) 25%
35. Out of 130 students appearing in an examination, 62 failed in English, 52 failed in
Mathematics, whereas 24 failed in both English and Mathematics. The number of
students who passed finally is
(a) 40 (b) 50 (c) 55 (d) 60
36. In a group of persons travelling in a bus, 6 persons can speak Tamil, 15 can speak
Hindi and 6 can speak Gujarati. In that group none can speak any other language. If
2 persons in the group can speak two languages only and one person can speak all the
three languages, then how many persons are there in the group?
(a) 21 (b) 22 (c) 23 (d) 24
37. Consider the following statements :
1. A man had a wife, two sons and two daughters in his family.
2. The daughters were invited to a feast and the male members of the family went out to
take part in a picnic.
3. The man's father did not return from his work.
Which of the following statements is true?
(a) Only the man's wife was left at home.
(b) It is likely that the man's wife was left at home.
(c) None was left at home.
(d) More than one person was left at home.
38. Geeta : Naresh has become a better boxer since he started meditation.
Radha : Impossible. A boxer's most important asset is his aggressiveness.
USPC - CSAT 253
UPSC - CSAT -1
Radha's statement reflects her belief that.
(a) meditation tends to make a person less aggressive.
(b) meditation has little or no effect on the person who practises it.
(c) Naresh was a poor boxer earlier because he was not aggressive enough.
(d) Naresh would not have taken to meditation as he was a boxer.

Directions for the following 2 (two) items:


Consider the given information and answer the two items that follow.
No supporters of 'party X', who knew Z and supported his campaign strategy, agreed for the
alliance with 'party Y’; but some of them had friends in 'party Y'.
39. With reference to the above information, which one among the following statements
must be true?
(a) Some supporters of 'party Y' did not agree for the alliance with the `party X'.
(b) There is at least one supporter of `party Y' who knew some supporters of 'party X' as a
friend.
(c) No supporters of 'party X' supported Z's campaign strategy.
(d) No supporters of 'party X' knew Z.
40. With reference to the above information, consider the following statement
1. Some supporters of 'party X' knew Z
2. Some supporters of 'party X', who opposed Z's campaign strategy, knew Z.
3. No supporters of 'party X' supported Z's campaign strategy.
Which of the statements given above, is/are not correct?
(a) 1 only (b) 2 and 3 only (c) 3 only (d) 1, 2 and 3


254 USPC - CSAT


UPSC - CSAT -1

Chapter

6 Questions from Previous


Papers - 2
Directions for the following 4 (four) items:
Read the following statements and answer the four items that follow:
Five cities P, Q, R, S and T are connected by different modes of transport as follows:
P and Q are connected by boat as well as rail.
S and R are connected by bus and boat.
Q and T are connected by air only.
P and R are connected by boat only.
T and R are connected by rail and bus.
1. Which mode of transport would help one to reach R starting from Q, but without
changing the mode of transport?
(a) Boat (b) Rail (c) Bus (d) Air
2. If a person visits each of the places starting from P and gets back to P, which of the
following places must he visit twice?
(a) Q (b) R (c) S (d) T
3. Which one of the following pairs of cities is connected by any of the routes directly
without going to any other city?
(a) P and T (b) T and S (c) Q and R (d) None of these
4. Between which two cities among the pairs of cities given below are there maximum
travel options available?
(a) Q and S (b) P and R (c) P and T (d) Q and R

Directions for the following 3 (three) items:


Read the following passage and answer the three items that follow:
A tennis coach is trying to put together a team of four players for the forthcoming tournament.
For this 7 players are available: males A, Band C; and females W, X, Y and Z. All players
have equal capability and at least 2 males will be there in the team. For a team of four, all
players must be able to play with each other. But, B cannot play with W, C cannot play with

USPC - CSAT 255


UPSC - CSAT -1
Z and W cannot play with Y.
5. If Y is selected and B is rejected, the team will consist of which one of the following
groups?
(a) A, C, Wand Y (b) A, C, X and Y
(c) A, C, Y and Z (d) A, W, Y and Z
6. If B is selected and Y is rejected, the team will consist of which one of the following
groups?
(a) A, B, C and W (b) A, B, C and Z
(c) A, B, C and X (d) A, W, Y and Z
7. If all the three males' are selected, then how many combinations of four member teams
are possible?
(a) 1 (b) 2 (c) 3 (d) 4
8. The music director of a film wants to select four persons to work on "different aspects
of the composition of a piece of music. Seven persons are available for this work; they
are Rohit, Tanya, Shobha, Kaushal, Kunal, Mukesh and J aswant. Rohit and Tanya will
not work together. Kunal and Shobha will not work together. Mukesh and Kunal want
to work together.
Which of the following is the most acceptable group .of people that can be selected by the
music director?
(a) Rohit, Shobha, Kunal and Kaushal (b) Tanya, Kaushal, Shobha and Rohit
(c) Tanya, Mukesh, Kunal and Jaswant (d) Shobha, Tanya, Rohit and Mukesh

Directions for the following 3 (three) items:


Examine carefully the following statements and answer the three items that follow:
Out of four friends A, B, C and D,
A and B play football and cricket,
B and C play cricket and hockey,
A and D play basketball and football,
C and D play hockey and basketball.
9. Who does not play hockey?
(a) D (b) C (c) B (d) A
10. Who plays football, basketball and hockey?
(a) D (b) C (c) B (d) A

256 USPC - CSAT


UPSC - CSAT -1
11. Which game do B, C and D play?
(a) Basketball (b) Hockey (c) Cricket (d) Football
12. Geeta is older than her cousin Meena, Meena's brother Bipin is older than Geeta. When
Meena and Bipin visit Geeta, they like to play chess. Meena wins the game more often
than Geeta. Based on the above information, four conclusions, as given below, have
been made. Which one of these logically follows from the information given above?
(a) While playing chess with Geeta and Meena, Bipin often loses.
(b) Geeta is the oldest among the three.
(c) Geeta hates to lose the game.
(d) Meena is the youngest of the three.
13. A, B, C, D and E belong to five different cities P, Q, R, Sand T (not necessarily in that
order). Each one of them comes from a different city. Further it is given that:
1. B and C do not belong to Q.
2. B and E do not belong to P and R.
3. A and C do not belong to R, S and T.
4. D and E do not belong to Q and T.
Which one of the following statements is not correct?
(a) C belongs to P (b) D belongs to R
(c) A belongs to Q (d) B belongs to S
14. Seven men, A, B, C, D, E, F and G are standing in a queue in that order. Each one is
wearing a cap of a different colour like violet, indigo, blue, green, yellow, orange and
red. D is able to see in front of him green and blue, but not violet. E can see violet and
yellow, but not red. G can see caps of all colours other than orange. If E is wearing an
indigo coloured cap, then the colour of the cap worn by F is
(a) Blue (b) Violet (c) Red (d) Orange
15. Consider the following statements:
(i) A primary group is relatively smaller in size.
(ii) Intimacy is an essential characteristic of a primary group.
(iii) A family may be an example of a primary group.
In the light of the above statements, which one of the following is true?
(a) All families are primary groups.

USPC - CSAT 257


UPSC - CSAT -1
(b) All primary groups are families.
(c) A group of smaller size is always a primary group.
(d) Members of a primary group know each other intimately.
16. “Price is not the same thing as value. Suppose that on a day the price of everything
viz., coal, bread, postage, stamps, a day’s labour, the rent of houses, etc. were to double.
Prices then would certainly rise, but values of all things except one would not.”
The writer wants to say that if prices of all things were doubled
(a) the values of all things would remain constant.
(b) the values of the things sold would be doubled.
(c) the values of the things bought would be halved.
(d) the value of money only would be halved.
17. “Liberty, therefore, is never real unless the Government can be called to account when
it invades rights.”
Which one of the following is the best justification of the above statement?
(a) In the realization that the government can be brought to book in a court of law.
(b) In identifying a man as a political unit in a way which distinguishes him from other citizens
(c) In a decentralized society wherein the basic needs of men can find satisfaction
(d) In the understanding that liberty and restraints are complementary.
18. The number of deaths among the army personnel is 8 in 1000, but among the civilian
population it is 20 per 1000.
Which one of the following inferences can be drawn from this statement?
(a) It is better to join the army.
(b) The relationship is fortuitous.
(c) Quality of Life Index is very high within the armed forces.
(d) The groups cannot be compared due to their heterogeneity.
19. Given the statement : “Buses are the cause of more accidents than cars, and trucks
cause fewer accidents than buses”, which of the following conclusions can we draw?
(a) There are more buses on the road than trucks.
(b) Car drivers are more careful than bus drivers.
(c) Truck drivers are more skilled than either car or bus drivers.
(d) None of the above

258 USPC - CSAT


UPSC - CSAT -1
20. “If political leadership fails to emerge, there is likelihood of military taking over power
in developing countries. Radical student groups or labour may try to raise revolution
but they are not likely to compete with the military. Military intervention, rule, and
withdrawal from politics is closely related to a society’s level of political development”.
In the context of political development, the assumption in the above passage is that
(a) political leadership is not an effective instrument.
(b) military fills in political vacuum.
(c) military intervention is inevitable for development.
(d) None of the above
21. Four friends, A, B, C and D distribute some money among themselves in such a manner
that A gets one less than B, C gets 5 more than D, D gets 3 more than B. Who gets the
smallest amount?
(a) A (b) B (c) C (d) D
22. There are some balls of red, green and yellow colour lying on a table. There are as many
red balls as there are yellow balls. There are twice as many yellow balls as there are
green ones. The number of red balls
(a) is equal to the sum of yellow and green balls.
(b) is double the number of green balls.
(c) is equal to yellow balls minus green balls.
(d) cannot be ascertained.
23. There are seven persons up on a ladder, A, B, C, D, E, F and G (not in that order). A
is further up than E but is lower than C. B is in the middle. G is between A and B. E is
between B and F. If F is between E and D, the person on the bottom step of the ladder
will be
(a) B (b) F (c) D (d) E
24. Consider that:
1. A is taller than B.
2. C is taller than A.
3. D is taller than C.
4. E is the tallest of all.
If they are made to sit in the above order of their height, who will occupy the mid position?
(a) A (b) B (c) C (d) D

USPC - CSAT 259


UPSC - CSAT -1
25. Four persons, Alok, Bhupesh, Chander and Dinesh have a total of Rs. 100 among
themselves. Alok and Bbupesh between them have as much money as Chander and
Dinesh between them, but Alok has more money than Bhupesh; and Chander has only
half the money that Dinesh has. Alok has in fact Rs. 5 more than Dinesh has. Who has
the maximum amount of money?
(a) Alok (b) Bhupesh (c) Chander (d) Dinesh
26. Six books are labeled A, B, C, D, E and F and are placed side by side. Books B, C, E and
F have green covers while others have yellow covers. Books A, B and D are new while
the rest are old volumes. Books A, B and C are law reports while the rest are medical
extracts. Which two books are old medical extracts and have green covers?
(a) B and C (b) E and F (c) C and E (d) C and F
27. A question paper must have a question on one of the eight poets: A, B, C, D, E, F, G or H.
The first four belong to the medieval period while the rest are considered modern poets.
Generally, modern poets figure in the question paper in alternate years. Generally those
who like H like G also; and those who like F like E also. The paper-setter does not like to
ask about F as he has written a book on F, but he likes F. Last year, the paper contained
a question on A. On the basis of the information given, this year’s paper is most likely
to contain a question on
(a) C (b) E (c) F (d) H
28. In a group of six women there are four dancers, four vocal musicians, one actress and
three violinists. Girija and Vanaja are among the violinists while Jalaja and Shailaja do
not know how to play on the violin. Shailaja and Tanuja are among the dancers. Jalaja,
Vanaja, Shailaja and Tanuja are all vocal musicians and two of them are also violinists.
If Pooja is an actress, who among the following is certainly a dancer and a violinist?
(a) Jalaja (b) Pooja (c) Shailaja (d) Tanuja
29. The letters L, M, N, O, P, Q, R, S and T in their order are substituted by nine integers
1 to 9 but not in that order. 4 is assigned to P. The difference between P and T is 5. The
difference between N and T is 3. What is the integer assigned to N?
(a) 7 (b) 5 (c) 4 (d) 6
30. Examine the following statements:
George attends Music Classes on Monday.
He attends Mathematics classes on Wednesday.
His Literature classes are not on Friday.
He attends History classes on the day following the day of his Mathematics classes.

260 USPC - CSAT


UPSC - CSAT -1
On Tuesday, he attends his Sports classes.
If he attends just one subject in a day and his Sunday is free, then he is also free on
(a) Monday (b) Thursday (c) Saturday (d) Friday

Directions for the following 3 (three) items :


Read the passage given below and answer the items that follow.
A, B, C, D, E, F are members of a family. They are engineer, stenographer, doctor, draughtsman,
lawyer and judge (not in order). A, the engineer is married to the lady stenographer. The judge
is married to the lawyer. F, the draughtsman is the son of B and brother of E. C, the lawyer is
the daughter-in-law of D. E is the unmarried doctor. D is the grandmother of F. There are two
married couples in the family.
31. What is the profession of B?
(a) Judge (b) Lawyer
(c) Draughtsman (d) Cannot be determined
32. Which of the following is/are a couple/couples?
(a) AD only (b) BC only
(c) Both AD and BC (d) Both AC and BD
33. What is the profession of D?
(a) Judge (b) Stenographer
(c) Doctor (d) Cannot be determined
34. Three persons A,B and C wore shirts of black, blue and orange colours (not necessarily
in that order) and pants of green, yellow and orange (not necessarily in that order). No
person wore shirt and pant of the same colour. Further it is given that
1. A did not wear shirt of black colour
2. B did not wear shirt of blue colour
3. C did not wear shirt of orange colour
4. A did not wear pants of green colour
5. B wore pants of orange colour
What were the colours of pants and shirt worn by C, respectively?
(a) Orange and black (b) Green and blue
(c) Yellow and blue (d) Yellow and black
35. Only six roads A,B,C,P,Q and R connect a military camp to the rest of the country. Only

USPC - CSAT 261


UPSC - CSAT -1
one out of A,P and R is open during storms. P is closed during floods. In this context,
which one of the following statements is correct?
(a) Under normal conditions only three roads are open
(b) During storms, at least one road is open
(c) During floods, only three roads are open
(d) During calamities all roads are closed
36. Six squares are coloured front and back, red(R), blue(B), yellow(Y), green(G), white(W)
and orange(O) and are hinged together as shown in the figure given below. If they are
folded to form a cube, what would be the face opposite to white face?


(a) R (b) G (c) B (d) O
37. The outer surface of a 4 cm x 4 cm x 4 cm cube is painted completely in red. It is sliced
parallel to the faces to yield sixty four 1 cm x 1 cm x 1 cm small cubes. How many small
cubes do not have painted faces?
(a) 8 (b) 16 (c) 24 (d) 36
38. Consider the following matrix with one empty block in the lower extreme corner:

Which of the following figures could fit in the empty block and thus complete the matrix?

(a) (b)


(c) (d)

262 USPC - CSAT


UPSC - CSAT -1
39. With reference to the figure given below, the number of different routes from S to T
without retracing from U and / or V, is


(a) 3 (b) 6 (c) 9 (d) 18
40. A cube has six numbers marked 1, 2, 3, 4, 5 and 6 on its faces. Three views of the cube
are shown below:


What possible numbers can exist on the two faces marked A and B, respectively on the cube?

(a) 2 and 3 (b) 6 and 1 (c) 1 and 4 (d) 3 and 1




USPC - CSAT 263


UPSC - CSAT -1

264 USPC - CSAT


UPSC - CSAT -1

Section - V

Data Interpretation

USPC - CSAT 265


UPSC - CSAT -1

266 USPC - CSAT


UPSC - CSAT -1

Chapter

1 Model Exercises

Study the given tables and answer the questions based on it.
I. Expenditures of a Company (in Lakhs) per Annum Over the given Years.
Item of Expenditure
Year
Salary Fuel and Transport Bonus Interest on Loans Taxes
1998 288 98 3.00 23.4 83
1999 342 112 2.52 32.5 108
2000 324 101 3.84 41.6 74
2001 336 133 3.68 36.4 88
2002 420 142 3.96 49.4 98
1. What is the average amount of interest per year which the company had to pay during
this period?
(A) INR 32.43 lakhs (B) INR 33.72 lakhs
(C) INR 34.18 lakhs (D) INR 36.66 lakhs
2. The total amount of bonus paid by the company during the given period is approxi-
mately what percent of the total amount of salary paid during this period?
A. 0.1% B. 0.5% C. 1% D. 1.25%
3. Total expenditure on all these items in 1998 was approximately what percent of the total
expenditure in 2002?
A. 62% B. 66% C. 69% D. 71%
4. The total expenditure of the company over these items during the year 2000 is?
A. INR 544.44 lakhs B. INR 501.11 lakhs
C. INR 446.46 lakhs D. INR 478.87 lakhs
5. The ratio between the total expenditure on Taxes for all the years and the total expendi-
ture on Fuel and Transport for all the years respectively is approximately?
A. 4:7 B. 10:13 C. 15:18 D. 5:8

USPC - CSAT 267


UPSC - CSAT -1
II. Number of Candidates Appeared and Qualified in a Competitive Examination from
Different States Over the Years.
State Year
1997 1998 1999 2000 2001
App. Qual. App. Qual. App. Qual. App. Qual. App. Qual.
M 5200 720 8500 980 7400 850 6800 775 9500 1125
N 7500 840 9200 1050 8450 920 9200 980 8800 1020
P 6400 780 8800 1020 7800 890 8750 1010 9750 1250
Q 8100 950 9500 1240 8700 980 9700 1200 8950 995
R 7800 870 7600 940 9800 1350 7600 945 7990 885

6. Total number of candidates qualified from all the states together in 1997 is approxi-
mately what percentage of the total number of candidates qualified from all the states
together in 1998?
A. 72% B. 77% C. 80% D. 83%
7. What is the average candidates who appeared from State Q during the given years?
A. 8700 B. 8760 C. 8990 D. 8920
8. In which of the given years the number of candidates appeared from State P has maxi-
mum percentage of qualified candidates?
A. 1997 B. 1998 C. 1999 D. 2001
9. What is the percentage of candidates qualified from State N for all the years together,
over the candidates appeared from State N during all the years together?
A. 12.36% B. 12.16% C. 11.47% D. 11.15%
10. The percentage of total number of qualified candidates to the total number of appeared
candidates among all the five states in 1999 is?
A. 11.49% B. 11.84% C. 12.21% D. 12.57%
The following table gives
III. The percentage of marks obtained by seven students in six different subjects in an ex-
amination. The Numbers in the Brackets give the Maximum Marks in Each Subject.
Subject (Max. Marks)
Student Maths Chemistry Physics Geography History Computer Science
(150) (130) (120) (100) (60) (40)
Ayush 90 50 90 60 70 80
Aman 100 80 80 40 80 70
Sajal 90 60 70 70 90 70
Rohit 80 65 80 80 60 60
Muskan 80 65 85 95 50 90
Tanvi 70 75 65 85 40 60
Tarun 65 35 50 77 80 80

268 USPC - CSAT


UPSC - CSAT -1
11. What are the average marks obtained by all the seven students in Physics? (rounded off
to two digit after decimal)
A. 77.26 B. 89.14 C. 91.37 D. 96.11
12. The number of students who obtained 60% and above marks in all subjects is?
A. 1 B. 2 C. 3 D. None
13. What was the aggregate of marks obtained by Sajal in all the six subjects?
A. 409 B. 419 C. 429 D. 449
14. In which subject is the overall percentage the best?
A. Maths B. Chemistry C. Physics D. History
15. What is the overall percentage of Tarun?
A. 52.5% B. 55% C. 60% D. 63%
IV. Sales of Books (in thousand numbers) from Six Branches - B1, B2, B3, B4, B5 and B6 of
a publishing Company in 2000 and 2001.


16. What is the ratio of the total sales of branch B2 for both years to the total sales of
branch B4 for both years?
A. 2:3 B. 3:5 C. 4:5 D. 7:9
17. Total sales of branch B6 for both the years is what percent of the total sales of branches
B3 for both the years?
A. 68.54% B. 71.11% C. 73.17% D. 75.55%
18. What percent of the average sales of branches B1, B2 and B3 in 2001 is the average sales
of branches B1, B3 and B6 in 2000?
A. 75% B. 77.5% C. 82.5% D. 87.5%

USPC - CSAT 269


UPSC - CSAT -1
19. What is the average sales of all the branches (in thousand numbers) for the year 2000?
A. 73 B. 80 C. 83 D. 88
20. Total sales of branches B1, B3 and B5 together for both the years (in thousand numbers)
is?
A. 250 B. 310 C. 435 D. 560
V. Foreign Exchange Reserves Of a Country. (in million US $)


21. The ratio of the number of years, in which the foreign exchange reserves are above the
average reserves, to those in which the reserves are below the average reserves is?
A. 2:6 B. 3:4 C. 3:5 D. 4:4
22. The foreign exchange reserves in 1997-98 was how many times that in 1994-95?
A. 0.7 B. 1.2 C. 1.4 D. 1.5
23. For which year, the percent increase of foreign exchange reserves over the previous
year, is the highest?
A. 1992-93 B. 1993-94 C. 1994-95 D. 1996-97
24. The foreign exchange reserves in 1996-97 were approximately what percent of the aver-
age foreign exchange reserves over the period under review?
A. 95% B. 110% C. 115% D. 125%
25. What was the percentage increase in the foreign exchange reserves in 1997-98 over
1993-94?
A. 100 B. 150 C. 200 D. 620

270 USPC - CSAT


UPSC - CSAT -1
VI. The bar graph given below shows the data of the production of paper (in lakh
tonnes) by three different companies X, Y and Z over the years.
Production of Paper (in lakh tonnes) by Three Companies X, Y and Z over the Years.


26. For which of the following years, the percentage rise/fall in production from the previ-
ous year is the maximum for Company Y?
A. 1997 B. 1998 C. 1999 D. 2000
27. What is the ratio of the average production of Company X in the period 1998-2000 to
the average production of Company Y in the same period?
A. 1:1 B. 15:17 C. 23:25 D. 27:29
28. The average production for five years was maximum for which company?
A. X B. Y C. Z D. X and Z both
29. In which year was the percentage of production of Company Z to the production of
Company Y the maximum?
A. 1996 B. 1997 C. 1998 D. 1999
30. What is the percentage increase in the production of Company Y from 1996 to 1999?
A. 30% B. 45% C. 50% D. 60%
31. What is the difference between the production of Company Z in 1998 and Company Y
in 1996?
A. 2,00,000 tons B. 20,00,000 tons
C. 20,000 tons D. 2,00,00,000 tons

USPC - CSAT 271


UPSC - CSAT -1
VII. Various Expenditures (in percentage) Incurred in Publishing a Book


32. If for a certain quantity of books, the publisher has to pay Rs. 30,600 as printing cost,
then what will be amount of royalty to be paid for these books?
A. Rs. 19,450 B. Rs. 21,200 C. Rs. 22,950 D. Rs. 26,150
33. What is the central angle of the sector corresponding to the expenditure incurred on
Royalty?
A. 15º B. 24º C. 54º D. 48º
34. The price of the book is marked 20% above the C.P. If the marked price of the book is
Rs. 180, then what is the cost of the paper used in a single copy of the book?
A. Rs. 36 B. Rs. 37.50 C. Rs. 42 D. Rs. 44.25
35. If 5500 copies are published and the transportation cost on them amounts to Rs. 82500,
then what should be the selling price of the book so that the publisher can earn a profit
of 25%?
A. Rs. 187.50 B. Rs. 191.50 C. Rs. 175 D. Rs. 180
36. Royalty on the book is less than the printing cost by:
A. 5% B. 33 1/5 % C. 20% D. 25%
The following pie charts exhibit the distribution of the overseas tourist traffic from India.
The two charts shows the tourist distribution by country and the age profiles of the tourists
respectively.

272 USPC - CSAT


UPSC - CSAT -1
VIII. Distribution of Overseas Tourist Traffic from India.


37. What percentage of Indian tourist went to either USA or UK ?
A. 40 % B. 50 % C. 60 % D. 70 %
38. The ratio of the number of Indian tourists that went to USA to the number of Indian
tourists who were below 30 years of age is ?
A. 2:1 B. 8:3
C. 3:8 D. Cannot be determined
39. If amongst other countries, Switzerland accounted for 25% of the Indian tourist traffic,
and it is known from official Swiss records that a total of 25 lakh Indian tourists had
gone to Switzerland during the year, then find the number of 30-39 year old Indian tour-
ists who went abroad in that year ?
A. 18.75 lakh B. 25 lakh C. 50 lakh D. 75 lakh
The following pie chart give the information about the distribution of weight in the human
body according to different kinds of components. Study the pie charts and answer the question.

USPC - CSAT 273


UPSC - CSAT -1
IX. Distribution of Weight in Human Body


40. What percentage of proteins of the human body is equivalent to the weight of its skin ?
A. 41.66 % B. 43.33 %
C. 44.44 % D. Cannot be determined
41. How much of the human body is neither made of bones or skin ?
A. 40 % B. 50 % C. 60 % D. 70 %
42. What is the ratio of the distribution of proteins in the muscles to that of the distribution
of proteins in the bones ?
A. 2:1 B. 2:3
C. 3:2 D. Cannot be determined
X. Exports from Three Companies Over the Years (in Rs. crore)

274 USPC - CSAT


UPSC - CSAT -1
43. For which of the following pairs of years the total exports from the three Companies
together are equal?
A. 1995 and 1998 B. 1996 and 1998
C. 1997 and 1998 D. 1995 and 1996
44. Average annual exports during the given period for Company Y is approximately what
percent of the average annual exports for Company Z?
A. 87.12% B. 89.64% C. 91.21% D. 93.33%
45. In which year was the difference between the exports from Companies X and Y the
minimum?
A. 1994 B. 1995 C. 1996 D. 1997
46. What was the difference between the average exports of the three Companies in 1993
and the average exports in 1998?
A. Rs. 15.33 crores B. Rs. 18.67 crores
C. Rs. 20 crores D. Rs. 22.17 crores
47. In how many of the given years, were the exports from Company Z more than the aver-
age annual exports over the given years?
A. 2 B. 3 C. 4 D. 5
The following line graph gives the ratio of the amounts of imports by a company to the
amount of exports from that company over the period from 1995 to 2001.
XI. Ratio of Value of Imports to Exports by a Company Over the Years.

USPC - CSAT 275


UPSC - CSAT -1
48. If the imports in 1998 was Rs. 250 crores and the total exports in the years 1998 and
1999 together was Rs. 500 crores, then the imports in 1999 was ?
A. Rs. 250 crores B. Rs. 300 crores
C. Rs. 357 crores D. Rs. 420 crores
49. The imports were minimum proportionate to the exports of the company in the year ?
A. 1995 B. 1996 C. 1997 D. 2000
50. What was the percentage increase in imports from 1997 to 1998 ?
A. 72 B. 56
C. 28 D. Data inadequate
51. If the imports of the company in 1996 was Rs. 272 crores, the exports from the company
in 1996 was
A. Rs. 370 crores B. Rs. 320 crores
C. Rs. 280 crores D. Rs. 275 crores
52. In how many of the given years were the exports more than the imports ?
A. 1 B. 2 C. 3 D. 4
Study the following line graph and answer the questions based on it.
XII. Number of Vehicles Manufactured by Two companies ove the Years (Number in Thou-
sands)

276 USPC - CSAT


UPSC - CSAT -1
53. What is the difference between the number of vehicles manufactured by Company Y in
2000 and 2001 ?
A. 50000 B. 42000
C. 33000 D. 21000
54. What is the difference between the total productions of the two Companies in the given
years ?
A. 19000 B. 22000 C. 26000 D. 28000
55. What is the average numbers of vehicles manufactured by Company X over the given
period ? (rounded off to nearest integer)
A. 119333 B. 113666 C. 112778 D. 111223
56. In which of the following years, the difference between the productions of Companies X
and Y was the maximum among the given years ?
A. 1997 B. 1998 C. 1999 D. 2000
57. The production of Company Y in 2000 was approximately what percent of the produc-
tion of Company X in the same year ?
A. 173 B. 164 C. 132 D. 97


USPC - CSAT 277


UPSC - CSAT -1

Chapter

2 D. I. Questions in CSAT So Far

CSAT - 2011
1. Consider the four age pyramids given below namely A, B, C and D representing four
different countries.

Which one of them indicates the declining population?


(a) A (b) B (c) C (d) D
2. The followings figures has four curves namely A, B, C and D, Study the figure and an-
swer the item that follows.

Which curve indicates the exponential growth ?


(a) A (b) B (c) C (d) D

278 USPC - CSAT


UPSC - CSAT -1
Directions for the following 2 (two) items:
The following pie charts show the break-up of disease categories recorded in the patients
from two towns, Town A and Town B. Pie charts plot the disease Categories as percentage of
the total number of patients. Based on these, answer the two items that follow the charts.

3. Which of the two towns has a higher number of persons with Diabetes?
(a) Town A (b) Town B
(c) Same in Town A and Town B (d) No inference can be drawn
4. What can we say about persons with more than one disease from these graphs ?
(a) There are likely to be persons with more than one disease in Town A.
(b) There are likely to be persons with more than one disease in Town B.

USPC - CSAT 279


UPSC - CSAT -1
(c) There'are likely to be persons with more than one disease in both Towns A and B.
(d) No inference can be drawn.
5. Consider the followmg Velocity-Time graph. It shows two trains starting simultane-
ously on parallel tracks.

With reference to the above graph, which one of the following statements is not correct ?
(a) Train B has an initial acceleration greater than that of Train A.
(b) Train B is faster than Train A at all times.
(c) Both trains have the same velocity at time to'
(d) Both trains travel the same distance in time to units.
6. Consider the following distance - time graph. The graph shows three athletes A, Band C
running side by side for a 30 km race.

280 USPC - CSAT


UPSC - CSAT -1
With reference to the above graph consider the following statements :
1. the race was won by A.
2. B was ahead of A up to 25 km 26 mark.
3. C ran very slowly from the begining.
Which of the statements given above is/are correct ?
(a) 1 only (b) 1 and 2 only
(c) 2 and 3 only (d) 1, 2 and 3

Directions for the following 3 (Three) items:


Read the passage given below, study the graph that follows and answer the three items given
belwo the figure.
During a party, a person was exposed to cotaminated water. A few days later, he developed
fever and loose motions. He suffered for some days before going to a doctor for treatment.
On starting the treatment, he soon became better and recovered completely a few days later.
The following graph shows different phases of the person's disease condition as regions A,
B, C, D and E of the curve.

7. Which region/regions of the curve correspond/corresponds to incubation phase of the


infection ?
(a) A only
(b) B only
(c) Band C
(d) No part of the curve indicates the incubation phase
8. Which region of the curve indicates that the person began showing the symptoms of
infection ?
(a) A (b) B (c) C (d) D

USPC - CSAT 281


UPSC - CSAT -1
9. Which region of the curve indicates that the treatment yielded effective relief?
(a) C (b) D
(c) E (d) The curve does not indicate the treatment
10. There are four routes to travel from city A to city B and six routes from city B to city C.
How many routes are possible to travel from the city A to city C ?
(a) 24 (b) 12 (c) 10 (d) 8

CSAT – 2013
11. Consider the following diagrams:
x men, working at constant speed, do a certain job in y days. Which one of these diagrams
shows the relation between x and y?


(a) diagram I (b) diagram II
(c) diagram III (d) diagram IV

282 USPC - CSAT


UPSC - CSAT -1
12. Four cars are hired at the rate of Rs. 6 per km plus the cost of diesel at Rs. 40 a litre. In
this context, consider the details given in the following table:
Mileage Total
Car Hours
(km/) Payment <
A 8 20 2120
B 10 25 1950
C 9 24 2064
D 11 22 1812
Which car maintained the maximum average speed?
(a) Car A (b) Car B
(c) Car C (d) Car D
Direction for the following 5 (five) items:
Study the two figures given below and answer the five items that follow:

Figure 1: Number of professors in selected disciplines in a University by sex

USPC - CSAT 283


UPSC - CSAT -1

Figure 2: Age of Physics Professors


13. How many Physics professors belong to the age group 35 - 44?
(a) 18 (b) 16 (c) 14 (d) 12
14. Which one of the following diciplines has the highest ratio of males to females?
(a) Physics (b) Mathematics
(c) Chemistry (d) Economics
15. What percentage of all Psychology professors are females?
(a) 40% (b) 50% (c) 60% (d) 70%
16. If the number of female Physics professors in the age group 25 - 34 equals 25% of all the
Physics professors in that age group, then what is the number of male Physics profes-
sors in the age group 25 - 34?
(a) 9 (b) 6 (c) 3 (d) 2
17. If the Psychology professors in the University constitute 2% of all the professors in the
University, then what is the number of professors in the University?
(a) 400 (b) 500 (c) 600 (d) 700

CSAT – 2014
Directions for the following 4 (four) items:
The following graph shows the average profit of two fruit-sellers A and B in thousands (Rs.)
per year from the year 1995 to 2000.
Consider the graph and answer the 4 (four) items that follow:

284 USPC - CSAT


UPSC - CSAT -1


18. In which year is the average profit of A and B same?
(a) 1995 (b) 1996 (c) 1997 (d) 1998
19. What is the difference between the average profit of B and A in the year 1998?
(a) - Rs. 100 (b) - Rs. 1,000 (c) + Rs. 600 (d) - Rs. 300
20. How much more average profit did A make in the year 2000 than in the year 1999?
(a) Rs. 200 (b) Rs. 1,000 (c) Rs. 1,500 (d) Rs. 2,000
21. What is the trend of the average profit of B from the year 1997 to the year 2000?
(a) Non-increasing (b) Non -decreasing
(c) Steady (d) Fluctuating
22. The following table shows the marks obtained by two students in different subjects:

Students Maximum Students Maximum
Subjects
A Marks B Marks
English 60 100 80 150
Psychology 70 100 70 100
History 30 100 60 100
Sanskrit 50 50 15 25
The difference in the mean aggregate percentage marks of the students is
(a) 2.5% (b) 13.75% (c) 1.25% (d) Zero

USPC - CSAT 285


UPSC - CSAT -1
23. Which one of the following figures has the above figure embedded in it?

Year 1992 1993 1994 1995
Population in lakhs 20 21 22 23
Income in crores (Rs.) 1010 1111 1225 1345
Which one of the following statements correct in respect of the above data ?
(a) Population increased by 5% or more every year.
(b) Income increased by 10% or more every year.
(c) Per capita income was always above 5,000.
(d) Per capita income was highest in 1994.

CSAT – 2015
24. The graph below depicts the earnings of A and B over the period 2000 to 2010:


From the graph, which one of the following can be concluded?
(a) On the average A earned more than B during this period.
(b) On the average B earned more than A during this period.
(c) The earnings of A and B were equal during this period.
(d) The earnings of A were less as compared to B during this period.

286 USPC - CSAT


UPSC - CSAT -1
25. Year-wise variation of the price of a certain commodity is shown in the following graph:


The price of the commodity in the year 1990
(a) must have been Rs. 10/-
(b) must have been Rs. 12/-
(c) must have been anywhere between Rs. 10/- and Rs. 20/-
(d) is higher than that in the year 1991
26. The proportion of expenditure on various items by two families A and B are represented
in the following Bar Charts:

USPC - CSAT 287


UPSC - CSAT -1
From these charts, we can conclude that
(a) Family A spent more money on food than Family B.
(b) Family B spent more money on food than Family A.
(c) Family A and Family B spent the same amount on food.
(d) The expenditure on food by Family A and Family B cannot be compared.

CSAT – 2018
27. The figure drawn below gives the velocity graphs of two vehicles A and B. The straight line
OKP represents the velocity of vehicle A at any instant, whereas the horizontal straight
line CKD represents the velocity of vehicle B at any instant. In the figure, D is the point
where perpendicular from P meets the horizontal line CKD such that PD= 1/2 LD :


What is the ratio between the distances covered by vehicles A and B in the time interval OL?
(a) 1 : 2 (b) 2 : 3 (c) 3 : 4 (d) 1 : 1
28. Consider the following graph :

288 USPC - CSAT


UPSC - CSAT -1
Which one of the following statements is not correct with reference to the graph given above?
(a) On 1st June, the actual progress of work was less than expected.
(b) The actual rate of progress of work was the greatest during the month of August.
(c) The work was actually completed before the expected time.
(d) During the period from 1st April to 1st September, at no time was the actual progress
more than the expected progress.
Consider the following graph in which the birthrate and death rate of a country are given, and
answer the two items that follow.

29. Looking at the graph, it can be inferred that from 1990 to 2010
(a) population growth rate has increased
(b) population growth rate has decreased
(c) growth rate of population has remained stable
(d) population growth rate shows no trend
30. With reference to the above graph, consider the following statements considering 1970
as base year:
1. Population has stabilized after 35 years.
2. Population growth rate has stabilized after 35 years.
3. Death rate has fallen by 10% in the first 10 years.
4. Birthrate has stabilized after 35 years.

USPC - CSAT 289


UPSC - CSAT -1
Which of the above are the most logical and rational statements that can be made from the
above graph?
(a) 1 and 2 only (b) 1, 2 and 3
(c) 3 and 4 (d) 2 and 4
31. Average hourly earnings per year (E) of the workers in a firm are represented in figures
A and B as follows :


(a) values of E are different
(b) ranges (i.e., the difference between the maximum and the minimum) of E are different
(c) slopes of the graphs are same
(d) rates of increase of E are different

290 USPC - CSAT


UPSC - CSAT -1
32. Consider the following figures A and B :

The manufacturing cost and projected sales for a product are shown in the above figures A
and B respectively. What is the minimum number of pieces that should be manufactured to
avoid a loss?
(a) 2000 (b) 2500 (c) 3000 (d) 3500
33. Consider the following graphs. The curves in the graphs indicate different age groups
in the populations of two countries A and B over a period of few decades:

USPC - CSAT 291


UPSC - CSAT -1


With reference to the above graphs, which of the following are the most logical and rational
inferences that can be made?
1. Over the last two and a half decades, the dependency ratio for country B has decreased.
2. By the end of next two and a half decades, the dependency ratio of country A will be
much less than that of country B.
3. In the next two decades, the work-force relative to its total population will increase in
country B as compared to country A.
Select the correct answer using the code given below.
(a) 1 and 2 only (b) 2 and 3 only
(c) 1 and 3 only (d) 1, 2 and 3
34. The graph given below indicates the changes in key policy rates made by the Central
Bank several times in a year:

292 USPC - CSAT


UPSC - CSAT -1
Which one of the following can be the most likely reason for the Central Bank for such an
action?
(a) Encouraging foreign investment
(b) Increasing the liquidity
(c) Encouraging both public and private savings
(d) Anti-inflationary stance
Directions for the following 2 (two) items :
The following table gives the GDP growth rate and Tele-density data of different States of a
country in a particular year. Study the table and answer th.e two items that follow.
Per Capita GDP Growth
States Tele- density
Income ($) Rate (%)
States 1 704 9.52 70.27
States 2 419 5.31 35.88
States 3 254 10.83 50.07
States 4 545 9.78 5.94
States 5 891 10.8 76.12
States 6 1077 11.69 77.5
States 7 900 8.88 14.86
States 8 395 5.92 6
States 9 720 7.76 82.25
States 10 893 9.55 96.7
States 11 363 4.7 57.7
States 12 966 7.85 63.8
States 13 495 9.37 52.3
States 14 864 5.46 97.9
States 15 497 7.48 62.3
States 16 777 7.03 93.8
States 17 335 5.8 49.9
States 18 599 7.49 47.84
35. With reference to the above table, which of the following is/are the most logical and
rational inference! Inferences that can be made?
1. Higher per capita income is generally associated with higher Tele-density.
2. Higher GDP growth rate always ensures higher per capita income.

USPC - CSAT 293


UPSC - CSAT -1
3. Higher GDP growth rate does not necessarily ensure higher Tele density.
Select the correct answer using the code given below.
(a) 1 only (b) 2 and 3 (c) 1 and 3 (d) 3 only
36. With reference to the above table, the following assumptions have been made:
1. Nowdays, prosperity of an already high performing State cannot be sustained without
making further large investments in its telecom infrastructure.
2. Nowadays, a very high Tele-density the most essential condition for promoting the
business and economic growth in a State.
Which of the above assumptions is/are valid?
(a) 1 only (b) 2 only (c) Both 1 and 2 (d) Neither 1 nor 2
37. The following graph indicates the composition of our tax revenue for a period of two
decades :

With reference to the above graph, which of the following is/are the most logical and rational
inference/ inferences that can be made?
1. During the given period, the revenue from Direct Taxes as percentage of gross tax rev-
enue has increased while that of Indirect Taxes decreased.
2. The trend in the revenue from Excise Duty demonstrates that the growth of manufactur-
ing sector has been negative during the given period.
Select the correct answer using the code given below.
(a) 1 only (b) 2 only (c) Both 1 and 2 (d) Neither 1 nor 2

294 USPC - CSAT


UPSC - CSAT -1
Directions for the following 3 (three) items:
The following three items are based on the graph given below which shows imports of three
different types of steel over a period of six months of a year. Study the graph and answer the
three items that follow.

38. By how much (measured in thousands of tons) did the import of sheet steel exceed the
import of coil steel in the first three months of the year?
(a) 11 (b) 15 (c) 19 (d) 23
39. What was the approximate total value (in $) of sheet steel imported over the six months
period?
(a) 45,555 (b) 50,555 (c) 55,550 (d) 65,750
40. What was the approximate ratio of sheet steel and scrap steel imports in the first three
months of the year?
(a) 1 : 1 (b) 1.2 : 1 (c) 1.4 : 1 (d) 1.6 : 1


USPC - CSAT 295


UPSC - CSAT -1

296 USPC - CSAT


UPSC - CSAT -1

S e c t i o n - V I

Sectional Tests

USPC - CSAT 297


UPSC - CSAT -1

298 USPC - CSAT


UPSC - CSAT -1

Chapter

1 Reading Comprehension

Read the following passages and answer the questions that follow.

Passage I

Literature and history are twin sisters, inseparable. In the days of our own grandfathers, and for
many generations before them, the basis of education was the Greek and Roman classics for the
educated, and the Bible for all. In the classical authors and in the Bible, history and literature were
closely intervolved, and it is that circumstance which made the old form of education so stimulating
to the thought and imagination of our ancestors. To read the classical authors and to read the Bible
was to read at once the history and the literature of the three greatest races of the ancient world. No
doubt the classics and the Bible were read in a manner we now consider uncritical but they were read
according to the best tenets of the time and formed a great humanistic education. Today the study
both of the classics and of the Bible has dwindled to small proportions. What has taken their place?
To some extent the vacuum has been filled by a more correct knowledge of history and a wider range
of literature. But I fear that the greater part of it has been filled up with rubbish.
1. Which of the following statements best reflects the underlying tone of the passage?
1. Literature and history are mutually exclusive
2. Literature and history are complementary to each other
3. The study of literature is meaningless without any knowledge of history.
4. Literature and history are inseparably linked together in the classics and the Bible
(a) 1 and 4 (b) 1 and 3 (c) 3 only (d) 4 only
2. The author of the above passage says that in the past the basis of education for all
people, irrespective of their intellectual calibre, was
(a) Greek and Roman classics (b) The Bible
(c) A correct knowledge of history (d) A wider range of literature
3. The author of the above passage says that the classics and the Bible were read by his
ancestors
1. methodically and with discretion
2. in a manner that broadened their view of life.

USPC - CSAT 299


UPSC - CSAT -1
3. with great emphasis on their literary values
4. without critical discrimination but in the light of their humanistic culture
(a) 1 and 2 only (b) 3 only
(c) 2 and 4 only (d) 4 only
4. According to the author of the above passage, the old form of education, based on the
study of the classics and of the Bible, has
(a) succeeded in creating interest in history
(b) laid the basis of human civilization
(c) had a gradual decline in our time
(d) been rejuvenated in the context of modern education
5. The author of the above passage fears that the greater part of the vacuum created by
lack of interest in the classics and the Bible has been filled up by
(a) a richer sense of history (b) a wider range of literature
(c) worthless ideas (d) a new philosophy of life

Passage II

Constitutional democracy acts through a prescribed division of functions between legislature,


executive and judiciary. Populist democracy regards such division of functions as cumbersome and
arbitrary impediments that act overtly or covertly against the will of the people. Populism sets great
store by achieving political objectives swiftly and directly through mass mobilization in the form of
rallies, demonstrations and other spectacular displays of mass support. Constitutionalism, on the other
hand, seeks to achieve its objectives methodically through the established institutions of governance.’
In older Western democracies, the principal focus of decision-making has been the legislature and
the executive. Strikes and street protests were, and are, rare. On the other hand, populist methods
and techniques have had a strong appeal in countries like India, ‘countries that were latecomers to
development and without the strong foundations for the rule of law required for the success of a
constitutional democracy.’
Populist movements, draw on ‘the Gandhian tradition of civil disobedience used with great effect
during the nationalist movement’. It will be hard to deny that agitations, demonstrations and rallies
undertaken in the name of civil disobedience have increasingly become coercive not only in their
consequences but even in their intentions.’ By showing contempt for elected leaders, populism
called into question all forms of public authority. Thus, ‘populism has not only become a part of our
democracy, but from time to time it puts forward its demands in a very imperious form. When that
happens, many naturally feel that the Constitution itself is under threat. However, there are plenty of

300 USPC - CSAT


UPSC - CSAT -1
deficiencies in the constitutional democracy, existing today in India. ‘In a parliamentary democracy’,
the obligations of constitutional morality are expected to be equally binding on the government and
the opposition. In India, the same political party treats these obligations very differently when it is in
office and when it is out of it. This has contributed greatly to the popular perception of our political
system as being amoral. ’Owing to the hypocrisy and arrogance of politicians in power, the people of
India have gradually learnt that their own elected leaders can be as deaf to their pleas as the ones who
came from outside. On the other side, the corruption and corrosion of our political class has led to an
increasing disenchantment with formal processes of decision-making. The continuing co-existence
within India of these two, somewhat opposed, understandings of democracy is no surprise.
6. What can be a suitable title for the passage?
(a) The death of democracy in India.
(b) The problems of democracy in India
(c) Competing versions of democracies in India
(d) Alternatives to democracy
7. With which of the following statements is the author likely to agree?
1. India would not have seen so many strikes had it become a democratic republic much
earlier as others.
2. The politicians cannot be blamed for the way constitution is being brought to question
by public
3. Decision making tends to be less chaotic in the western democracies
4. Gandhian traditions are being misused in India.
(a) All of the above (b) 1, 2 and 3
(c) 1, 3 and 4 (d) 2, 3 and 4
8. Which of the following are true about the views that ‘Populist Democracy’ holds?
1. It values the mandate that members of parliament enjoy
2. It considers decision making a tedious process.
3. They conduct themselves in the true spirit of gandhism
(a) 1 only (b) 2 only (c) 3 only (d) None of these
9. Which of the following features should constitutional democracy imbibe to become
more favourable to countries like India?
(a) Maintaining fair-play in actions whether enjoying power or not
(b) Mass mobilization and rallies to attract people

USPC - CSAT 301


UPSC - CSAT -1
(c) Periodically bring about changes in constitution
(d) Condemn the executive class as it is seeped in corruption
10. Which of the following would pose as a contradiction to the author’s arguments?
1. Occupy movements’ success in US and UK
2. Jan Lok Pal agitation creating uproar in India
(a) 1 only (b) 2 only (c) Both 1 and 2 (d) Neither 1 nor 2

Passage III

The hypothesis of an expanding Earth has never attracted notable support, and if it were not for
the historical example of continental drift, such indifference might be a legitimate response to an
apparently improbable concept. It should be remembered, however, that drift too was once regarded as
illusory, but the idea was kept alive until evidence from physicists compelled geologists to reinterpret
their data.
Of course, it would be as dangerous to overreact to history by concluding that the majority must
now be wrong about expansion as it would be to reenact the response that greeted the suggestion that
the continents had drifted. The cases are not precisely analogous. There were serious problems with
the pre-drift world view that a drift theory could help to resolve, whereas Earth expansion appears
to offer no comparable advantages. If, however, physicists could show that the Earth’s gravitational
force has decreased with time, expansion would have to be reconsidered and accommodated.
11. The passage indicates that one reason why the expansion hypothesis has attracted little
support is that it will not
1. overcome deficiencies in current geologic hypotheses
2. clarify theories concerning the Earth’s gravitational forces
3. accommodate relevant theories from the field of physics
(a) 1 and 2 only (b) 2 only
(c) 3 only (d) 1 and 3 only
12. The final acceptance of a drift theory could best be used to support the argument that
(a) physicists are reluctant to communicate with other scientists
(b) improbable hypotheses usually turn out to be valid
(c) there should be cooperation between different fields of science
(d) there is a need for governmental control of scientific research

302 USPC - CSAT


UPSC - CSAT -1
13. In developing his argument, the author warns against
(a) relying on incomplete measurements
(b) introducing irrelevant information
(c) rejecting corroborative evidence
(d) making unwarranted comparisons
14. It can be deduced from the passage that the gravitational force at a point on the Earth’s
surface is
1. representative of the geologic age of the Earth
2. analogous to the movement of land masses
3. proportional to the size of the Earth
(a) 1 only (b) 2 only (c) 3 only (d) None of the above

Passage IV

Punctually at midday he opened his bag and spread out his professional equipment, which consisted
of a dozen cowrie shells, a square piece of cloth with obscure mystic charts on it, a note book, and
a bundle of Palmyra writing. His forehead was resplendent with sacred ash and vermilion, and his
eyes sparkled with a sharp abnormal gleam which was really an outcome of a continual searching
look for customers, but which his simple clients took to be a prophetic light and felt comforted.
The power of his eyes was considerably enhanced by their position placed as they were between
the painted forehead and the dark whiskers which streamed down his cheeks: even a half-wit’s eyes
would sparkle in such a setting. To crown the effect he wound a saffron-coloured turban around his
head. This colour scheme never failed. People were attracted to him as bees are attracted to cosmos
or dahlia stalks.
15. From the description of this passage one can make out the person to be a:
(a) Snake Charmer (b) Footpath vendor
(c) Astrologer (d) Priest
16. The eyes of the person described, sparkled because:
(a) He was sitting under midday sun
(b) He was always looking for possible clients
(c) His forehead was bright with ash and vermilion
(d) He was full of joy

USPC - CSAT 303


UPSC - CSAT -1
17. The person opened his bag
(a) to search for something he needed
(b) to indicate the end of his work
(c) to keep his professional equipment
(d) to take out things for display
18. The tone of the description is
(a) sad (b) neutral (c) ironic (d) sympathetic

Passage V

During adolescence, the development of political ideology becomes apparent in the individual.
As such, political ideology is dim or absent at the beginning of adolescence. Its acquisition by the
adolescent, in even the most modest sense, requires the acquisition of relatively sophisticated cognitive
skills: the ability to manage abstractness, to synthesize and generalize, to imagine the future. These
are accompanied by a steady advance in the ability to understand principles.
The child’s rapid acquisition of political knowledge also promotes the growth of political ideology
during adolescence. By knowledge I mean more than the dreary “facts,” such as the composition of
state government that the child is exposed to in the conventional ninth-standard civics course. Nor
do I mean only information on current political realities. These are facets of knowledge, but they are
less critical than the adolescent’s absorption, often unwitting, of a feeling for those many unspoken
assumptions about the political system that comprise the common ground of understanding—for
example, what the state can appropriately demand of its citizens, and vice versa, or the proper
relationship of government to subsidiary social institutions, such as the schools and churches. Thus
political knowledge is the awareness of social assumptions and relationships as well as of objective
facts. Much of the naiveté that characterizes the younger adolescent’s grasp of politics stems not from
an ignorance of “facts” but from conventions of the system, of what is and is not customarily done,
and of how and why it is or is not done.
Yet I do not want to overemphasize the significance of increased political knowledge in forming
adolescent ideology. Over the years I have become progressively disenchanted about the centrality of
such knowledge and have come to believe that much current work in political socialization, by relying
too heavily on its apparent acquisition, has been misled about the tempo of political understanding
in adolescence. Just as young children can count numbers in series without grasping the principle
of ordination, young adolescents may have in their heads many random bits of political information
without a secure understanding of those concepts that would give order and meaning to the information.
Children’s minds pick up bits and pieces of data. But until the adolescent has grasped the functions
that concepts and principles provide, the data remain fragmented, random, disordered.

304 USPC - CSAT


UPSC - CSAT -1
19. The author’s primary purpose in the passage is to
(a) clarify the kinds of understanding an adolescent must have in order to develop a political
ideology
(b) dispute the theory that a political ideology can be acquired during adolescence
(c) explain why adolescents are generally uninterested in political arguments
(d) suggest various means of encouraging adolescents to develop personal political
ideologies
20. According to the author, which of the following contributes to the development of
political ideology during adolescence?
1. Conscious recognition by the adolescent of his or her own naiveté
2. Intuitive understanding of relationships among various components of society
3. Evaluation by the adolescent of the general principles encompassing his or her specific
political ideas
(a) 1 only (b) 1 and 2 only (c) 2 only (d) 2 and 3 only
21. The author uses the term “common ground of understanding” to refer to
1. familiar legislation regarding political activity
2. the experiences that all adolescents share
3. a society’s general sense of its own political activity
4. a society’s willingness to resolve political tensions
(a) 4 only (b) 3 only (c) 1 and 3 (d) 2 and 3
22. It can be inferred from the passage that the author would be most likely to agree with
which of the following statements about schools?
(a) They should present political information according to carefully planned, schematic
arrangements.
(b) They themselves constitute part of a general sociopolitical system that adolescents are
learning to understand.
(c) If they were to introduce political subject matter in the primary grades, students would
understand current political realities at an earlier age.
(d) They are ineffectual to the degree that they disregard adolescents’ political naiveté.

USPC - CSAT 305


UPSC - CSAT -1
23. Which of the following best summarizes the author’s evaluation of the accumulation of
political knowledge by adolescents?
(a) It is unquestionably necessary, but its significance can easily be overestimated.
(b) It is important, but not as important as is the ability to appear knowledgeable.
(c) It delays the necessity of considering underlying principles.
(d) It is primarily relevant to an understanding of limited, local concerns, such as county
politics.
24. Which of the following statements best describes the organization of the author’s
discussion of the role of political knowledge in the formation of political ideology during
adolescence?
(a) He acknowledges its importance, but then modifies his initial assertion of that importance.
(b) He consistently resists the idea that it is important, using a series of examples to support
his stand.
(c) He wavers in evaluating it and finally uses analogies to explain why he is indecisive.
(d) He begins by questioning conventional ideas about its importance, but finally concedes
that they are correct.

Passage VI

The life cycle of a butterfly is very dramatic and action-packed, because so much happens in so
short a time. The most interesting is the caterpillar stage. They are very vulnerable to attacks and use
camouflage to protect themselves. Generally their colours help them blend with the surroundings.
Also, their bodies have thin lines much like the veins of a leaf. Some caterpillars use the leaf to make
themselves invisible. Some of them are even beautiful, especially the ones that are grass-yellow. This
shade of green is fabulous but strangely the adult is not so beautiful.
Butterflies have many enemies but most stay away from them because they are very bitter to the
taste! This saves them from many aggressors. The swallowtail caterpillar emits a strong odour to
keep its enemies at bay. Some caterpillars simply drop to the ground and climb back up the tree only
after the enemy has gone.
25. The life cycle of butterfly is said to be dramatic because
1. the changes in its life cycle take place fast
2. they exhibit different colours and are beautiful
(a) 1 only (b) 2 only (c) 1 and 2 only (d) None of the above

306 USPC - CSAT


UPSC - CSAT -1
26. Which of the following methods are not used by caterpillar to save itself from the
enemies?
1. Making their appearance akin to the leaves
2. Changing the shape of their bodies very fast
3. Giving off smells that repulse the enemies
Select the correct answer using the code given below:
(a) 2 only (b) 2 and 3 only (c) 1 and 3 only (d) 1, 2 and 3
27. The expression ‘Very vulnerable to attacks’ in the passage means
(a) prone to attack others (b) prone to being overcome by enemies
(c) they provoke others to attack (d) they are immune to attacks from others

Passage VII

If the growing trends towards political opportunism and partisan action by Constitutional authorities
spread further, there is no guarantee over the long run that the democratic values at the national level
will continue to be safe. In case political alliances at the centre also become unstable and short-
lived, political opportunism of the type witnessed in some states may also occur at the federal level
without any safety valves in place. At present, the Centre has the power to impose President’s rule in
a state in the event of prolonged instability. At the federal level, no such recourse is available as the
Union Cabinet is supreme and the President can act only on its advice. Some countervailing action
is, therefore, necessary to curb the worst kind of political opportunism, which has the potential of
undermining the very foundations of a stable, prosperous, and democratic India.
As a counter to the above view, it may be argued that concerns about the survival of India’s
democracy are as old as the birth of the republic. It will be recalled that at the time of Independence,
there was worldwide skepticism about the further of India as a united democratic republic. India
had then an untried government combined with widespread communal violence and social disorder.
In view of its immense regional, linguistic, and religious diversity, it was apprehended that India
would soon break up, or at least go back to an authoritarian regime of some kind. And yet, because
of its inherent strengths and pre-Independence history of inclusiveness of various conflicting forces,
democracy in India has not only survived but has also become ‘deep and strong’. India has also
emerged as one of the fastest- growing developing countries, with strong social cultural and economic
bonds, across its regions.
28. What is the main idea of the passage?
(a) Indian democracy has proved every critic wrong and is moving from strength to strength
(b) Democratic values are constantly under threat in India, but India has a history of having

USPC - CSAT 307


UPSC - CSAT -1
dealt with it successfully
(c) Indian democratic values are set for a collapse and if political opportunism continues,
it will lead to social unrest
(d) India is growing fast due to its foundations of democracy
29. The author’s forecast about future of democratic values in India is
(a) Guarded optimism (b) Concerned fear
(c) Desperate and calling for reforms (d) Unbridled prosperity
30. Which of the following is not a threat to democracy?
1. Political opportunism
2. Frequent elections
3. Political alliances devoid of common ideologies
4. Growth of regional parties
(a) 4 only (b) 4 and 2 (c) 2, 3 and 4 (d) 4 and 3
31. What might be categorized as a ‘safety valve’ that has been referred to in the passage?
1. A Coalition coming into power when no single party gets majority
2. Centre stepping in when continuous political instability continues in state
3. President accepting the cabinet’s recommendation on imposing emergency at the centre
(a) 1 only (b) 2 only (c) 3 only (d) 2 and 3

Passage VIII

Colonialism had powerful political impacts on the colonies too. Modern science, technology,
medicine and even political ideas inevitably diffused into the colonies through the colonizers. The
technical and administrative ability of the colonial powers enabled them to politically unify and rule
vast areas of colonies. The British formed one political unit out of India, from Baluchistan to Assam,
from Kashmir to Kanyakumari, a feat no Indian king had been capable of throughout the long history
of India. The native people educated in the schools and colleges established by the colonial power not
only imbibed technical knowledge but also the literature and political philosophy of the colonisers.
The hypocrisy of the rules then became clear to them. The British preached and partially practiced
democracy at home but not in India! They prescribed the writings of John Locke and Edmund Burke
to Indian students, but as to implementing their ideas in India – forget it! Their conviction of racial
superiority had no basis in the biological sciences that they taught. Last but not least, the concept of
nationalism, which undoubtedly originated in Europe, also percolated to the colonies.

308 USPC - CSAT


UPSC - CSAT -1
Nationalism is a difficult concept to definite. Uniformity of religion, language or ethnicity within
a region does not necessarily lead to a nation. The Arabs of the Middle East share all these features
but are split into many nations. Switzerland on the other hand speaks three different languages but
is a unified nation for centuries. The best way of defining it is to say that a nation is a political unit
held together by the belief of its citizens, however diverse they are that they share a common destiny.
Being under a common colonial yoke for centuries also promoted the idea that the victims were all
together! This way colonialism made an undoubted contribution to Asian and African nationalism.
32. What is the primary purpose of the author?
(a) To show a few selected but far-reaching effects of colonialism on the colonies
(b) To show how colonies helped build nationalism
(c) To show how Indians struggled under British rule
(d) To show the double standards of the British.
33. With which of the following statements is the author most likely to agree?
1. The British were the first to give the identity of a country to India,
2. The British were champions of democracy
3. The British also contributed in enlargement of the literary and philosophical horizons
(a) 1 and 3 (b) 2 and 3 (c) 1 and 2 (d) 1, 2 and 3
34. Nationality as a concept is
1. Complex to understand
2. brought about by common hardships faced
3. difficult to achieve in the face of vast differences.
(a) 1, 2 and 3 (b) 1 and 2 (c) 2 and 3 (d) 1 and 3
35. What can be inferred about John Locke and Edmund Burke?
1. They preached freedom and democracy
2. They were the greatest British philosophers of their time
3. Their writings might have helped Indians understand western concepts of democracy
4. They opposed Indian nationalism
(a) 1 and 2 only (b) 1 and 3 only
(c) 2 and 4 only (d) 2 and 3 only

USPC - CSAT 309


UPSC - CSAT -1
Passage IX

Liberalism places the freedom of individuals, their presumptive equality and claim to be treated
with dignity at the centre of attention. India has made considerable progress in creating space for
the de jure recognition of individual rights. But our political culture far too often immobilises the
claims of individual freedom in the face of community identity or group coercion, putting at risk
assorted values, from freedom of expression to gender equality. The idea that India is a federation
of communities and the task of politics is to keep a balance between them can have deeply illiberal
consequences. It traps individuals in the tyranny of compulsory identities. It readily mobilises state
power against individuals in the name of community sentiment. Diversity should be an outcome of
individuals freely exercising choices.
Second, liberalism has a presumptive faith in citizens. The Indian state has acquired inordinate
powers over citizens by setting itself up as the vanguard of society. The state is often needed to secure
justice and reform society. But, cutting across party lines, there is a more insidious idolisation of the
state that is legitimised by a pervasive distrust of citizens. The state knows better than the citizens;
citizens cannot be trusted to make choices. And perhaps more damagingly, this distrust of citizens is
licence to micromanage them. The state is all virtue, society all vice, so society needs superintendence.
This construction of the citizen as incapable and untrustworthy is deeply entrenched in administrative
practice. No liberal society can flourish on the basis of a pervasive distrust of citizens.
Third, liberalism distrusts concentrations of power, wherever they are found. Nothing has damaged
Indian liberalism more than the idea that Indian liberalism simply replaces the power of the state with
the power of the large Corporations. But temperamentally, a genuine liberalism has been as suspicious
of the monopolies and inordinate influence of private actors as it is of state power. It also believes in
‘the Art of Separation’: the considerations and norms appropriate to one sphere of activity should not
contaminate another. Politics has to be shielded from economic power, considerations appropriate to
culture have to be shielded from politics and so on.
Fourth, liberals are not radical democrats. But they recognise that participation is necessary to
secure rights, foster a sense of citizenship, prevent power from becoming remote, and for producing
decisions that are legitimate. For this reason they are committed to forms of self-government where
possible. For all the talk of decentralisation, none of the political parties thinks of local governments
as genuine sites of self-government. They think of them as, at best, instrumental conduits for plans
hatched at higher levels of government.
36. What can be a suitable title of the passage?
(a) ‘The art of Separation’ (b) Liberalism- the highest virtue
(c) Liberalism- What it is made of? (d) Indian Liberalism- Where it is headed?

310 USPC - CSAT


UPSC - CSAT -1
37. Which of the following can be inferred from the passage?
1. Individualism is at the core of Liberalism
2. Assertion of various identities by citizens is helpful for building liberal values
3. The state has every power to interfere in what it thinks is best for society
(a) 1 only (b) 1 and 2 only (c) 2 and 3 only (d) 1 and 3 only
38. With which of the following statements is the author most likely to agree?
1. Pursuit of Freedom should come before diversity.
2. Local self- governments are genuine forms of governments
3. Private firms will exploit opportunities created by liberalism.
4. The state needs to be actively involved in keeping different castes and communities that
exist in India, together.
(a) 1, 2, 3 and 4 (b) 1, 2 and 3 only
(c) 1 and 2 only (d) None of the above
39. With respect to the passage which of the following statements can be inferred?
1. The citizens cannot be entrusted to make critical decisions
2. Liberalism is more of a sensibility than a set of principles
3. India lags behind most countries in upholding individual rights legally.
(a) 1 and 2 only (b) 2 only
(c) 1 only (d) 2 and 3 only

Passage X

While primary education provides the three ‘R’s, it rarely imparts the skills and knowledge
necessary for employment in a job that ensures decent wages and living conditions. So, elementary
education is not a terminal level of education. Most of the literacy and primary/elementary education
programmes in the country do not impart literacy that is sustainable, which means that they do not
guarantee that children do not relapse into illiteracy. Further, even if elementary education imparts
some valuable attributes in terms of attitudes and skills and is able to take people from below the
poverty line to above it, the level of ascent is often not very high. The danger of their falling below
the poverty line at any time remains high.
On the other hand, it is higher education that consolidates the gains received from elementary and
secondary education and provides skills that could be useful in the labour market, while helping in
innovating technology and sustaining growth. It is higher education that keeps people above the poverty

USPC - CSAT 311


UPSC - CSAT -1
line without the danger of their falling back into the poverty trap, whether it is educational poverty
or income poverty. Higher education takes people much beyond the poverty line by improving the
social, occupational and economic mobility of households. Robust research evidence exists to show
that higher education contributes to development and has a poverty-alleviating effect. It enhances the
earnings of individuals and thereby contributes to economic development. It thus makes a significant
contribution to reducing absolute as well as relative poverty. It also contributes to improvement in
human development indicators such as infant mortality and life expectancy. In all, higher education is
a very important “human capability” and a “human freedom, a freedom that helps in attaining other
freedoms.
40. Which of the following is true about elementary education?
1. It does not impart useful skills
2. It does not even teach children basic concepts of reading and writing
3. It does not ensure that students remain literate
(a) 1 and 2 alone (b) 2 and 3 alone
(c) 1 and 3 alone (d) 1, 2 and 3
41. What is the main idea of the passage?
(a) Higher education, more than elementary education can usher in development and
upward mobility of the down-trodden.
(b) Elementary education needs to be given a boost along with higher education.
(c) Higher education can eliminate poverty.
(d) Employment opportunities are less for people who have had only elementary education
42. The passage can be a good material on the discussion of which of the following topics?
(a) Employment opportunities for the rural poor
(b) How to increase India’s human development indicators
(c) The basics of sustainable growth
(d) Which should get priority funding from government- Elementary or Higher Education?

Passage XI

Many philosophers disagree over the definition of morality, but most disputants fall into one of
two categories: egocentrics, who define morality as the pursuit of self-fulfillment, and socio-centrics,
who define morality as an individual’s obligations to society. Where does the truth lie? Fortunately,
the stem of the word “morality” provides some clues. The word “mores” originally referred to the
customs of pre-literate cultures. Mores, which embodied each culture’s ideal principles for governing

312 USPC - CSAT


UPSC - CSAT -1
every citizen, were developed in the belief that the foundation of a community lies in the cultivation
of individual powers to be placed in service to the community. These mores were concerned with
such skills as food-gathering and warfare as well as an individual’s relationships with others. Thus,
I submit, “morality” must be concerned with what is honored by the community at large. However,
self-fulfillment is important to morality because unfulfilled citizens, no matter how virtuous, cannot
perform the duties morality assigns them.
43. The primary purpose of this passage is to
(a) summarize an argument (b) resolve a dispute
(c) trace a word’s origin (d) prove a hypothesis
44. According to the passage, mores in preliterate cultures concerned such skills as warfare
and food-gathering because these skills were
1. characteristic of an individual’s self-fulfillment
2. examples of a culture’s traditions
3. demonstrations of an individual’s contributions to the community
(a) 1 and 2 only (b) 2 only
(c) 3 only (d) 2 and 3 only
45. It can be inferred from the passage that the author would be most likely to agree with
which of the following statements regarding sociocentrics and egocentrics?
1. The position of the sociocentrics is stronger than that of the egocentrics.
2. The positions of the egocentrics and sociocentrics are of equal merit.
(a) 1 only (b) 2 only (c) 1 and 2 (d) Neither 1 nor 2
46. With which of the following statements regarding the relationship between the individual
and morality would the author be most likely to agree?
(a) Failure in social obligations is the price of success in individual endeavors.
(b) The unfulfilled citizen cannot fulfill his moral obligations to the community.
(c) Morality is unconcerned with conflicts among citizens.
(d) The unfulfilled citizen is without virtue.

Passage XII

Although the development of new infrastructure (such public facilities as power plants, schools,
and bridges) is usually determined by governmental planning, sometimes this development can be
planned more flexibly and realistically by private investors who anticipate profit from the collection
of user fees. Such profits can contribute to the financing of more infrastructure if demand proves
USPC - CSAT 313
UPSC - CSAT -1
great enough, whereas reluctance of developers to invest in such projects can signal that additional
infrastructure is not needed. During the economic boom of the 1990’s, for example, the state of
Andhra Pradesh authorized private developers to build a 300 crore toll road. These developers
obtained the needed right-of-way from property owners, but by 1993 they still had not raised the
necessary financing. The unwillingness of investors to finance this project does not negate the viability
of privately financed roads; rather, it illustrates a virtue of private financing. If a road appears unlikely
to attract enough future traffic to pay for the road, then it should not be built.
47. The primary purpose of the passage is to
(a) build a case for increasing the development of new infrastructure
(b) advocate an alternative to government financing of infrastructure
(c) explain the failure of a privately financed venture
(d) suggest the types of infrastructure most appropriate for private financing
48. The passage implies that the “governmental planning” mentioned may lead to which of
the following problems?
1. Improper use of profits derived from user fees
2. Unrealistic decisions about developing new infrastructure
(a) 1 only (b) 2 only (c) 1 and 2 (d) Neither 1 nor 2
49. According to the passage, which of the following is false of the toll road mentioned?
1. After it was built, it attracted too little traffic to pay for its construction.
2. It was partially financed by the state of Andhra Pradesh.
3. Its development was authorized during an economic boom.
(a) 1 and 2 only (b) 2 and 3 only
(c) 1 and 3 only (d) 1, 2 and 3
50. The passage suggests that which of the following would occur if a privately financed
bridge that proved to be profitable failed after a number of years to meet the demands
of traffic?
(a) Private developers who financed the bridge would rely on governmental authorities to
develop new infrastructure.
(b) User fees would be increased so that usage would become more costly.
(c) Governmental authorities would be reluctant to rely on private contractors to develop a
new bridge.
(d) Profits generated by user fees would be used to help finance the construction of new
infrastructure to alleviate the traffic problem.

314 USPC - CSAT


UPSC - CSAT -1
Passage XIII

No doubt, the ‘green revolution’ has led to self-sufficiency in food production but it has also brought
with it the formidable problem of poisoning of food grains and other eatables. This is caused by
excessive use of chemicals on crops and pesticide residues. It has also created havoc by exterminating
the species of useful parasites and viruses which keep pests under control. Scientists are now worried
about the resurgence of such formidable pests in menacing proportions which seem to undermine all
that they have achieved in agricultural production.
51. From the reading of the passage, which one of these statements do you think is correct?
1. The ‘green revolution’ has solved all problems in agriculture
2. Application of chemicals has resulted in everlasting preservation of grains
3. The ‘green revolution’ is a mixed blessing
(a) 1 only (b) 1 and 3 only (c) 3 only (d) 2 and 3 only
52. The statement “the green revolution has also … species of useful parasites and viruses”
means
(a) all parasites and viruses keep pests under control
(b) pesticides and chemicals kill parasites and viruses, which control pests
(c) the pests are controlled by parasites
(d) application of chemicals to grains has created havoc
53. Which one of the following statements best reflects the underlying implication of the
passage?
(a) Man’s effort to control nature to his advantage has always created unseen dangers side
by side
(b) Research in one area leads to a challenge for further research in the same field
(c) At present, research in preservation of agricultural production is at the cross-roads
(d) The excessive use of chemicals and pesticides is dangerous
54. Which one of these phrases best helps to bring out the precise meaning of ‘menacing
proportions?
(a) To an extent which becomes threatening
(b) Assuming dimensions that cause concern
(c) Unimagined, dangerous proportion
(d) Harmful size

USPC - CSAT 315


UPSC - CSAT -1
Passage XIV

Democracy is not merely a guarantee of adult franchise; it also creates conditions for participation
in the political process. Has India been successful in this regard?
It is difficult to arrive at a conclusive answer because of the apparent paradox one confronts when
conceptualizing Indian democracy: on the one hand, popular zeal, which reaches the level of hysteria
at times during the elections, almost evaporates once the politicians take over political authority and
thus hardly functions as the custodian of both the democratic process and its value system. What is
probably more alarming is the gradual erosion of the institutions that are critical for democracy in
its classical liberal sense. The perversion of the electoral system that fails to neutralize the forces
challenging its very existence highlights a major lacuna in the political arrangement forced on India,
drawing on feudal instincts and primordial loyalties. So, democracy, a western concept, has failed to
evolve in India, in its true form. Or, it seems possible, given its short history that democracy in India
is passing through a transition and will triumph eventually. Or, since the form of democracy is linked
largely to the socio-economic compulsions of the day, India is likely to redefine its nature and contour
since its socioeconomic environment is entirely dissimilar to that of the west.
Nonetheless, India is perhaps the only example showing that it is possible to maintain, sustain
and strengthen a functioning democracy in a very poor country despite enormous diversity in terms
of language, religion, culture and ethnicity. It is most striking because according to the classical
liberal discourse democracy cannot strike roots in multi-ethnic societies. Democracy in India is
thus ‘a phenomenon’ that, ‘by most accounts, should not have existed, flourished or, indeed long
endured’. The growing consolidation of democratic processes can be attributed to the emergence of
complementary social and political institutions, nurtured and sustained by an alert people despite the
rising tide of communalism and other divisive tendencies.
The evolving nature of democracy in India hardly corresponds to any copybook description. One
can thus safely argue that India is a creative democracy for it is being not only constantly reinvented,
but also redesigned to capture the new experiments in a non-western socio-political context. The
Indian democratic experiment is innovative not only in terms of articulation, but also in substance.
Political institutions that hold the spirit of democracy are constantly restructured in view of the
constantly changing socio-economic milieu, giving it distinctive localized characteristics within the
larger universal paradigm of liberal democracy.
55. What is the passage thematically centered on?
(a) Evaluating democracy in India
(b) The failure of western democracy in India
(c) The problems facing Indian democracy
(d) India : A creative democracy

316 USPC - CSAT


UPSC - CSAT -1
56. Which of the following factors is the author concerned about as weakening Indian
democracy
1. The hyper-enthusiasm that people display during elections.
2. Inability to break barriers of caste and creed and other long-standing but obsolete
bindings.
3. The opinion of many that Indian democracy should have collapsed long ago.
(a) 1 only (b) 1 and 2 only (c) 1, 2 and 3 (d) 2 and 3 only
57. According to the passage, which of the following is valid?
1. The meaning of democracy is to give the right to vote to everybody.
2. ‘Pillars’ of democracy in India are not able to protect themselves from their ‘enemies’
(a) 1 only (b) 2 only (c) 1 and 2 (d) Neither 1 nor 2
58. With which of the following is the author most likely to agree?
1. In order to succeed India must follow the principles of western democracy.
2. Indian democracy is sure to emerge stronger in the years to come
(a) 1 only (b) 2 only (c) 1 and 2 (d) Neither 1 nor 2
59. Which of the following can be said about Indian democracy?
(a) Most people believed that it cannot survive for long.
(b) Indian democracy has not changed in any form since its beginning.
(c) Based on established theories it is possible to predict the major events in Indian
democracy
(d) Ethnic boundaries are becoming blunt day-by-day.

Passage XV

Men and women should be treated primarily as people, and not primarily as members of opposite
sexes. Their shared humanity and common attributes should be stressed not their gender difference.
Neither sex should be stereotyped or arbitrarily assigned to a leading or secondary role. Women
and men should be treated with the same respect, dignity and seriousness. Women should not be
described by physical attributes when men are being described by mental attributes or professional
position. Instead, both sexes should be dealt with in the same terms. References to a man’s or woman’s
appearance, charm or intuition should be avoided when irrelevant.
60. Men and women should be treated first
(a) as human beings (b) as belonging to opposite sexes

USPC - CSAT 317


UPSC - CSAT -1
(c) in terms of their physical attributes (d) as stereotypes
61. Dealing with both sexes “in the same terms” means
(a) stereotyping both men and women
(b) describing them by physical attributes
(c) treating them equally
(d) describing women by mental attributes
62. All the suggestions in the passage are meant
(a) to make women more powerful
(b) to reduce the power of men
(c) to minimise the possibility of conflict between men and women
(d) to remove gender discrimination

Passage XVI

When vegetation sprouts in the desert, it is a good sign but when the ice in the Arctic and Antarctic
begins to turn green, there is something terribly wrong. Reports say that an iceberg, approximately
the size of New York city, has broken off from the icy continent. An Argentine team discovered huge
cracks in the polar ice caps. These developments can have serious implications. If polar ice fields
melt, our coastal cities might be submerged, and sea levels across the world could rise between 3.65
and 6.09 metre in different parts of the earth. This is probably due to global warming.
63. What can be considered as a “good sign” in the desert?
(a) when trees grow there
(b) when it snows in the desert
(c) when ice caps melt and there is water
(d) when there is vegetation found in the desert
64. The melting of ice fields in the Arctic and Antarctic regions is dangerous because:
(a) the ice will turn green and poisonous
(b) it will cause huge floods which will destroy coastal regions
(c) it will create global warming and will badly affect our climate
(d) huge cracks will develop all over the world
65. Polar ice caps develop huge cracks because:
(a) of the movement of the earth

318 USPC - CSAT


UPSC - CSAT -1
(b) of the breaking off of icebergs
(c) of the crowding of cities like New York
(d) of rising temperatures
66. What do you think is the intention of the author?
(a) to describe strange phenomena in nature
(b) to report findings of research teams working in the polar regions
(c) to make us aware of the dangers of global warming
(d) to compare developments in deserts and Arctic regions caused by global warming

Passage XVII

History with its flickering lamp stumbles along the trail of the past and kindles with pale gleams the
passions of the former days. What is the worth of all this? The only guide to a man is his conscience;
the only shield to his memory is the rectitude and sincerity of his actions. It is very imprudent to walk
through life without this shield, because we are so often mocked at by the failure of our hopes and
upsetting of our calculations; but with this shield, however the fates may play, we march always in
the ranks of honour.
67. In the given context, the best meaning of the word ‘conscience’ is
(a) consciousness (b) conformity to what is right
(c) the virtue of being right (d) the sense of right and wrong
68. The word ‘however’ in the expression “however the fates may play’ may be replaced by
(a) yet (b) by whatever means (c) no matter how (d) nevertheless
69. The author’s suggestion is that rectitude and sincerity of our actions help us ultimately
(a) to walk through life prudently
(b) to meet the challenge of fates successfully
(c) to fulfill our hopes and calculations
(d) to be remembered as a virtuous and honourable man
70. The extract is taken from the speech of a very eminent person. The language and
particularly the metaphors used seem to suggest that the writer is
(a) a religious preacher attracting his disciples to the paths of virtue
(b) an honest businessman declaring his business policy
(c) a teacher of history addressing his students about the value of history
(d) a great politician inspiring the nation with patriotism

USPC - CSAT 319


UPSC - CSAT -1
Passage XVIII

The importance of early detection of tuberculosis (TB), regular treatment and nutritious food
are just not known widely enough. Often TB victims discontinue the treatment when the symptoms
disappear, without waiting for a complete cure; the next attack is more virulent from bacteria which
have thus become drug-resistant.
Anti-TB drugs are produced in India. The capability to meet the country’s requirements of anti-TB
drugs in full already exists. Yet millions of Indians suffer from TB and thousands of them die every
year.
Voluntary organizations and government agencies are doing commendable work. But we have so
far tackled only the fringe of the problem. What is now needed is a nation-wide determination to fight
TB. India eradicated smallpox with a national campaign. We can eradicate TB too.
71. Treatment is discontinued- by TB victims, when
(a) they think that the disease is completely cured
(b) the apparent signs of TB are no longer visible to them
(c) they run out of resources like money or medicine
(d) they are attacked by drug-resistant bacteria
72. Millions of Indians suffer from TB, because
(a) people discontinue the treatment too soon or do not start the treatment early enough
(b) India does not produce anti-TB drugs of the required quality
(c) anti-TB drugs are not available at a reasonable price
(d) people do not have nutritious food
73. When the treatment of TB is discontinued too early
(a) the old symptoms reappear
(b) the patient gets better, although slowly
(c) the disease appears in a new, more dangerous form
(d) the patient must get good, nutritious food
74. ‘The fringe of the problem’ means
(a) the basic cause of the problem
(b) the root of the problem
(c) the side effects of the drugs
(d) the edge of the problem, not the main point
75. Who or what become ‘drug-resistant’, according to the passage?
(a) TB patients who are treated for a long time
(b) People who do not want to take medicine for their illness

320 USPC - CSAT


UPSC - CSAT -1
(c) TB bacteria that have not been fully eradicated
(d) Patients who have discontinued the treatment

Passage XIX

It was a very cold evening and so few people were seen out on the streets. I did not go out
myself although it was my habit not to keep indoors after sunset. So I closed all the doors and
windows of my room, took the book which had been lying opened on the table, and tried to read it.
The cold was getting so severe that I started shivering, so I wrapped myself up with a bigger blanket.
But I could not continue reading the book because I was nearly rendered incapable of turning the
pages.
76. The author’s habit was to:
(a) read in the evening (b) sleep in the evening
(c) go out in the evening (d) play in the evening
77. If it was not so cold the author would have :
(a) liked to work (b) liked to sit and look out to the streets
(c) liked to read (d) liked to have a stroll
78. The author could not continue reading the book because:
(a) he did not like the book (b) he was feeling tired
(c) he was feeling very cold (d) the lights had gone off
79. There were not many people outside because:
(a) it was a rainy evening (b) it was a cold evening
(c) it was a dark evening (d) there was heavy snow outside
80. Before wrapping himself as mentioned in the passage, the author:
(a) did not feel cold (b) was wrapped but not sufficiently
(c) was not wrapped (d) had not wrapped because


USPC - CSAT 321


UPSC - CSAT -1

Chapter

2 Syllogisms

Evaluate the following arguments (1-10) and decide whether they are valid are not.

1. Alif likes Laila. Laila likes Hanif. Therefore Alif likes Hanif.
2. Some rocks are not gems. All emeralds are gems. Therefore, some rocks are not emeralds.
3. All birds are animals. No tree is a bird. Therefore, No tree is an animal.
4. No capitalists are charitable. All charitable people are helpful. So, No capitalists are helpful.
5. All murderers are criminals. Therefore all non-murderers are non-criminals.
6. Some fat people are not chain smokers. All Japanese are fat people. So, some chain smokers
are not Japanese.
7. Either we use nuclear power or we reduce our consumption of energy. If we use nuclear
power, then we place our lives at great risk. If we reduce our consumption of energy, then
we place ourselves under extensive governmental control. So, either we place our lives at
great risk or we place ourselves under extensive governmental control.
8. Some intelligent people are highly immoral. All highly immoral people are unhappy.
Therefore some unhappy people are not intelligent.
9. All the really hot new thinkers are using principles from sociobiology. It’s the new wave in
ethics. So, you should accept the principles of sociobiology.
10. All beautiful paintings are colourful objects. No Charcoal drawings are beautiful paintings.
Therefore, no charcoal drawings are colourful objects.
11. Examine the following statements and conclusions :
Statement I : All men are married.
Statement II: Some men are educated.
Conclusion I : Some married are educated.
Conclusion II : Some educated are married.
Which of the following is correct?
a) Only conclusion I follows
b) Only conclusion II follows

322 USPC - CSAT


UPSC - CSAT -1
c) Either conclusion I or conclusion II follows
d) Both conclusion I and conclusion II follow
The next 4 items are based on the given information which is followed by two possible
inferences. Use the following options to answer to each of the items?
a) Only inference I follows
b) Only inference II follows
c) Both inferences I and II follow
d) Neither inference I nor II follows
12. All apples are bananas. Some apples are oranges.
I) Some oranges are bananas
II) Some bananas are orange
13. All flowers are rivers. All rivers are stars.
I) All flowers are stars
II) All stars are flowers
14. All officers are trained. All trained are efficient.
I) All officers are efficient
II) All efficient people are officers
15. No ice cream is sweet. Some ice creams are flavoured.
I) No flavoured thing is sweet
II) No sweet thing is flavoured
16. Statement:
There are many Indians who are honest. Mohan is an Indian.
Assumptions :
I. Mohan is honest.
II. Mohan is not honest.
(1) Only I is implicit. (2) Only II is implicit
(3) Both I and II are implicit (4) Neither I nor II is implicit
17. Statements:
I. All birds are dogs. II. Some dogs are cats.

USPC - CSAT 323


UPSC - CSAT -1
Assumptions :
I. Some cats are not dogs.
II. All dogs are not birds.
(1) Only I is implicit. (2) Only I is implicit
(3) Both I and II are implicit. (4) Neither I nor II is implicit
In each of the following Questions from 19-21 , two statements are given followed by four
conclusions numbered I, II, III, IV. You have to take the given statement as true and decide
which of the given conclusions logically follows from the given statements disregarding
commonly known facts.
18. Statements : All branches are flowers. All flowers are leaves
Conclusion : (I) All branches are leaves (II) All leaves are branches
(III) All flowers are branches (IV) Some leaves are branches
(1) None follows (2) Only I and IV follows
(3) only II and III follows (4) All follows
19. Statements : All politicians are honest. All honest are fair
Conclusion :
(I) Some honest are politicians (II) No honest are politicians.
(III) Some fair are politician (IV) All fair are politicians
(1) None follows (2) Only I follows
(3) Only I and II follows (4) Only I and III follows
20. Statements : All aeroplanes are trains. Some trains are chairs.
Conclusion :
(I) Some aeroplanes are chairs (II) Some chairs are aeroplanes
(III) Some chairs are trains (IV) Some trains are aeroplanes
(1) None follows (2) Only I and II follows
(3) Only II and III follows (4) Only III and IV follows

Directions for Questions Nos 21- 23.


Each question contains six statements followed by four sets of combinations of three. Choose
the set in which the statements are logically related.

21. A. All apples are fruits.


324 USPC - CSAT
UPSC - CSAT -1
B. All fruits are sweet.
C. All apples are sweet.
D. All apples are priced.
E. All apples are red.
F. All fruits are available.
(1) ABC (2) BCD (3) ADE (4) DEF
22.
A. All snakes are reptiles. B. All reptiles are not snakes.
C. All reptiles are cold-blooded. D. All snakes lay eggs.
E. All reptiles lay eggs. F. Snakes are cold-blooded.
(1) ABE (2) BED (3) ABD (4) ACF
23.
A. Some men are bald.
B. Bald men are intelligent.
C. Raman is a man.
D. Raman is bald.
E. Raman is intelligent.
F. All men are intelligent.
(1) ABF (2) BDE (3) CDB (4) EBF
24. If it is false that ‘no person can operate an industrial plant for the purpose of any scheduled
industry in an air pollution control area without the prior consent of the State Board’, then,
which of the following may be validly inferred ?
a) Some persons obtained the consent of the State Board but did not run a plant as industry
in an air pollution control area
b) Someone did not obtain the consent of the State Board but ran a plant commercially in
an air pollution control urea
c) The State Board is the only authority to give authorization for opening industry in
restricted air pollution control area
d) Some areas are under the jurisdiction of the State Board
25. If it is true that ‘all pollutants are harmful’, identify which of the following is invalid to

USPC - CSAT 325


UPSC - CSAT -1
infer from it?
a) Pollutants constitute a subset, of harmful things
b) No pollutants are non-harmful
c) If anything is harmful, it is a pollutant
d) Some pollutants are harmful
26. In the following question three statements are followed by three conclusions numbered I,
II and III. Assume the three statements are true even if they are at variance with commonly
known facts. Then pick the correct answer from the choices given below.
Statements:
All dogs like to run. Some dogs like to swim. Some dogs look like their masters.
Conclusions:
I. All dogs who like to swim look like their masters.
II. Dogs who like to swim also like to run.
III. Dogs who like to run do not look like their masters.
a. I only
b. II only
c. II and III only
d. None of the given conclusion follows.
27. Examine the following statements :
1. All colours are pleasant 2. Some colours are pleasant
3. No colour is pleasant 4. Some colours are not pleasant
Given that statement 4 is true, what can be definitely concluded ?
a) 1 and 2 are true b) 1 is false
c) 2 is false d) 3 is true
28. Examine the following statement’s and conclusion, and then choose-the logically
conclusive answer using the code given below ;
Statement I : A triangle has three angles.
Statement II :A square has four angles.
Conclusion : A polygon has more than three angles.
Code :

326 USPC - CSAT


UPSC - CSAT -1
a) The conclusion drawn is definitely true
b) The conclusion drawn is definitely false
c) The conclusion drawn is either probably true or probably false
d) The conclusion drawn is irrelevant.
29. In the following question two statements are followed by two conclusions numbered I and
II. Assume the two statements are true even if they are at variance with commonly known
facts. Then pick the correct answer from the choices given below.
A. Only conclusion I follows
B. Only conclusion II follows
C. Both conclusions I & II follow
D. Neither conclusion I nor conclusion II follows
Statements:
Some chairs are not tables.
Table is a book.
Conclusions:
I. Table is a chair.
II. Some chairs are not book.
30. The following question consists of five statements followed by options consisting of three
statements put together in a specific order. Choose the option which indicates a valid
argument, that is, where the third statement is a conclusion drawn from the preceding two
statements.
A. All business schools have a placement department.
B. TMS is a business school.
C. TMS has a placement department.
D. Some business schools have a placement department.
E. TMS has only placement department.
a. ACB b. DEB c. DBC d. BAC


USPC - CSAT 327


UPSC - CSAT -1

Chapter

3 Logical Reasoning and


Analytical Ability
Directions for Questions 1- 3:
1. Five girls- Seema, Reema, Neeta, Mona and Veena have total five tickets of movie the-
atres- Priya, Chanakya, M2K, PVR Saket, Satyam where movies - Gangster, Khiladi, Hero,
SaalaamNamaste and Iqbal are currently playing. Each girl has one movie ticket of one of
the five theatres.
2. Movie Gangster is running in Priya theatre whose ticket is not with Veena and Seema.
3. Mona has ticket of Iqbal movie.
4. Neeta has ticket for the M2K theatre. Veena has the ticket of Satyam theatre where Khiladi
is not running.
5. In PVR Saket theatre Saalaam Namaste is running.
1. Which is the correct combination of the Theatre - Girl - Movie ?
(a) M2K-Neeta-Hero (b) Priya — Mona — Gangster
(c) Satyam — Veena — Iqbal (d) PVR Saket - Seema — Saalaam Namaste
2. Which movie is running at Chanakya?
(a) Gangster (b) Iqbal (c) Hero (d) Data inadequate
3. Who is having the ticket of the movie Hero?
(a) Reema (b) Veena (c) Seema (d) Mona

Directions for questions 4 - 5:


Atul has been assigned the task of allotting offices to six faculty members. The offices are
numbered 1-6 and arranged in a row. Only a six- foot high divider separates them. So voices,
sounds and cigarette smoke flow easily from one office to another:
a. Sandhya needs to use the telephone quite often through the day
b. Vikas and Kunal need adjacent offices as they need to consult each other often while
working.
c. Aditi is a senior employee and has to be allotted the office number 5, having the biggest
window.

328 USPC - CSAT


UPSC - CSAT -1
d. Mahipal requires silence in the offices next to his
e. Tejash, Vikas, and Mahipal are all smokers.
f. Aditi finds tobacco smoke allergic and the offices next to hers have to be occupied by
non-smokers.
Unless specifically stated, all the employees maintain an atmosphere of silence during office
hours.
4. The ideal candidate to occupy the office number adjacent to Sandhya would be
A. Aditi B. Vikas C. Tejash D. Mahipal
5. The three employees who are smokers should be seated in the offices
A. 1, 2 and 4 B. 2, 3 and 6 C. 1, 2 and 3 D. 1, 2 and 6
6. Find the number of triangles in the given figure.


A. 8 B. 10 C. 12 D. 14
7. In the adjoining figure, if the centres of all the circles are joined by horizontal and verti-
cal lines, then find the number of squares that can be formed.


A. 6 B. 7 C. 8 D. 1
8.

A. 1 B. 2 C. 3 D. 4

USPC - CSAT 329


UPSC - CSAT -1
9.

A. 1 B. 2 C. 3 D. 4
10.

A. 1 B. 2 C. 3 D. 4
11.

A. 1 B. 2 C. 3 D. 4
12. Here are some words translated from an artificial language.
moolokarn means blue sky
wilkospadi means bicycle race
moolowilko means blue bicycle
Which word could mean "racecar"?
A. wilkozwet B. spadiwilko C. moolobreil D. spadivolo
13. Statements:
I. Some drivers are technicians
II. All technicians are engineers
III. Some engineering are lectures
Conclusions
1. Some technicians are lectures
2. Some lectures are drivers
3. All engineers are technician
4. Some engineers are drivers
A. only 3 follows B. only 4 follows

330 USPC - CSAT


UPSC - CSAT -1
C. Only 3 and 4 follows D. None of the above
14. Mr Raghav went in his car to meet his friends John. He Drove 30 kms towards north
and then 40 kms towards west. He then turned to south and covered 8 kms. Further he
turned to east and moved 26 kms. Finally he turned right and drove 10 kms and then
turned left to travel 19 kms. How far and in which direction is he from the starting
point?
A. East of starting point, 5 kms B. East of starting point, 13 kms
C. North East of starting point, 13 kms D. North East of starting point, 5 kms

Directions for questions 15 - 16:


For these questions, what is the missing element in the sequence represented by the question
mark?
15. 1, 1, 2, 6, 24,?, 720
A. 100 B. 104 C. 108 D. 120
16. 2, 12, 30, 56, ?, 132, 182
A. 116 B. 76 C. 90 D. 86

Directions for questions 17 -18:


two statements are given, followed by two inferences A and B. Assume the statements to be
true, mark your answer as:
A. If only inference A follows, B. If only inference B follows,
C. If both A and B follow, D. If neither A nor B follows
17. Statements:
Some doctors are fools.
Some fools are rich.
Inferences:
A: Some doctors are rich.
B: Some rich are doctors.
18. Statements:
All pedestrians are poor.
All poor are honest.
Inferences:
A: All honest are pedestrians.

USPC - CSAT 331


UPSC - CSAT -1
B: All pedestrians are honest
19. A sheet has been folded in the manner as shown in X and Y and punched as in Z above.
Choose from the following how will it look when unfolded?

A. B.

C. D.
20. What is the minimum number of colours required to fill the spaces in the given diagram
without any two adjacent spaces having the same colour?


A. 6 B. 5 C. 4 D. 3
21. A cuboid has six sides of different colours. The red side is opposite to black. The blue
side is adjacent to white. The brown side is adjacent to blue. The red side is face down.
Which one of the following would be the opposite to brown?
A. Red B. Black C. White D. Blue
22. In the figure, number in any cell is obtained by adding two numbers in the cells directly
below it. For example, 9 in the second row is obtained by adding the two numbers 4 and
5 directly below it. The value of (X – Y) is ?

332 USPC - CSAT


UPSC - CSAT -1
A. 2 B. 3 C. 4 D. 5

Directions questions 23 to 25:


A bus has exactly six stops numbered 1 to 6 on its route. The bus first stops at stop number 1
and then at stop number 2, 3, 4, 5 and 6 respectively. After the bus leaves stop 6, the bus turns
and returns to stop number 1 and repeats the cycle. The stops are at six buildings that are, in
alphabetical order: A, B, C, D, E, and F. Further information is given below:
1. E is the third stop
2. B is the sixth stop
3. The stop D is the stop immediately before F.
4. C is the stop immediately before A.
23. If the stop C comes after D, which among the following must be the stop immediately
before C?
A. D B. F C. E D. A
24. If A comes at an even numbered stop, which among the following must be the stop im-
mediately before B?
A. C B. A C. E D. F
25. In case a passenger boards the bus at F, rides past three stops, and gets off at B, which
of the following must be true?
A. C is stop one B. F is stop 5
C. D is stop one D. E is stop 4

Directions for Questions 26 to 27:


If all the three statements, marked (i), (ii) and (iii) are true, then which one of the following
deductions, marked A, B, C and D can be MOST LOGICALLY deduced:
26. (i) Whenever milk is kept in front of a child, he/she starts crying.
(ii) Children cry if they are hungry.
(iii) Unhappy children are hungry.
A. When hungry, a child likes milk. B. A child crying means he/she is unhappy.
C. A happy child does not cry. D. An unhappy child usually cries.
27. (i) Whenever there is a fire, the fire alarm goes off
(ii) If the sprinklers do not start, the fire alarm does not go off.
(iii) If the sprinklers start, an automatic alarm is set off at the fire department.

USPC - CSAT 333


UPSC - CSAT -1
A. If an automatic alarm is set off at the fire department, that means there must be a fire.
B. If the sprinklers do not start, the automatic alarm at the fire department is not set off.
C. Whenever there is a fire, an automatic alarm is set off in the fire department.
D. If there is no fire, no automatic alarm is set off in the fire department.
28. Five friends – Pari, Panna, Palli, Priya and Prit went for shopping and each of them
bought one of the items from among a mobile phone, sandals, purse, watch and sun-
glasses. Following information is also known.
1. If Pari purchased sandals, then Priya will purchase a watch.
2. If Panna purchased a mobile phone, then Pari will purchase a watch.
3. If Priya purchased sunglasses, then Panna will purchase a mobile phone.
4. If Prit purchased a purse, then Palli will purchase sunglasses.
Which of the following is a valid combination of items bought by each of the five friends?
A. Pari - mobile phone, Panna - sandals, Palli - purse, Priya - sunglasses and Prit - watch
B. Pari - sandals, Panna - sunglasses, Palli - purse, Priya - mobile phone and Prit - watch
C. Pari - watch, Panna - mobile phone, Palli - sandals, Priya - sunglasses, Prit - purse
D. None of these
29. For a problem in an entrance examination, there are four questions 1, 2, 3 and 4. For
each of these questions there are four separate alternatives marked A, B, C and D,
amongst which only one alternative is correct. Four different students' answers to the
four questions 1, 2, 3 and 4 are BCBA, DBBA, BCDA and DCCB respectively. All of
these four students got exactly two correct answers. If now another student's answers to
questions 1, 2, 3 and 4 are BCAA, then the number of correct answers obtained by this
student is:
A. 1 B. 2 C. 3 D. 4

Directions for questions 30 to 31:


are based on the following information: There are three different cable channels namely
Ahead, Luck and Bang. In a survey it was found that 85% of viewers respond to Bang, 20%
to Luck, and 30% to Ahead. 20% of viewers respond to exactly two channels and 5% to none.
30. What percentage of the viewers responded to all three?
A. 10 B. 12 C. 14 D. None of these
31. Assuming 20% respond to Ahead and Bang, and 16% respond to Bang and Luck, what
is the percentage of viewers who watch only Luck?

334 USPC - CSAT


UPSC - CSAT -1
A. 20 B. 10 C. 16 D. None of these

Directions for questions 32-33 :


two statements are given, followed by two inferences (i) and (ii). Assume the statements to
be true, mark your answer as:
A. If only inference (i) follows, B. If only inference (ii) follows,
C. If both (i) and (ii) follow, D. If neither (i) nor (ii) follows
32. Statements:
All mothers are aunts.
All aunts are ladies.
Inferences:
(i): All mothers are ladies. (ii): All aunts are mothers.
33. Statements:
All goats are cows.
Some goats are lambs.
Inferences
(i): All goats are lambs. (ii): Some lambs are cows.

Directions for questions 34-36:


two premises are given (i) and (ii). Assume the statements to be true, and mark logically valid
conclusions given in the options.
34. i. All persons who worry are people who live in the future.
ii. All persons who plan ahead are people who live in the future.
A. Al l Persons who worry are people who plan ahead.
B. All people who plan ahead are people who worry.
C. Both A and B
D. None of the above
35. i. No passive individuals are folks who live well.
ii. All self-actualized persons are folks who live well.
A. No self-actualized persons are passive individuals
B. No passive individuals are self-actualized persons
C. Both A and B

USPC - CSAT 335


UPSC - CSAT -1
D. None of the above
36. i. All people cherishing living are persons living excellent lives.
ii. All folks who live well are people cherishing living.
A. All folks who live well are persons living excellent lives.
B. All persons living excellent lives are folks who live well
C. Both A and B
D. None of the above
37. Find the next term in the numerical series: 8 16 24 36 48 64 ?
A. 80 B. 56 C. 100 D. 91
38. In a certain code, EASE is written as GUCG. How is CUT written in that code?
A. UWE B. VWE C. EWU D. CWF
39. If BRIDGE is written as EULGJH in a certain code, how will FRUIT be written in that
code?
A. IUXLW B. IVLXW C. IUWXL D. IUXVT

Directions for Questions 40 - 43:


In each question below is given a statement followed by two courses of action numbered I
and II. You have to assume everything in the statement to be true and on the basis of the in-
formation given in the statement, decide which of the suggested courses of action logically
follow(s) for pursuing. Give answer
(a) Only I follows (b) Only II follows
(c) Neither I nor II follows (d) Both I and II follow
40. Statement: There has been an unprecedented increase in the number of requests for
berths in most of the long distance trains during the current holiday season.
Courses of Action:
I. The railway authority should immediately increase the capacity in each of these trains
by attaching additional coaches.
II. The people seeking accommodation should be advised to make their travel plan after
the holiday.
41. Statement: Every year large number of devotees die due to severe cold on their way to
the shrine located at the top of the mountain range.
Courses of Action:

336 USPC - CSAT


UPSC - CSAT -1
I. The devotees should be discouraged to visit the shrine without having proper warm
clothes and other amenities.
II. The government should provide warm clothes and shelter to all the devotees visiting the
shrine.

Directions for questions 42 – 45:


Answer the questions based on the following information.
Mr. Mansingh has five sons – Arun, Mahi, Rohit, Nilesh and Saurav, and three daughters
– Tamanna, Kuntala and Janaki. Three sons of Mr. Mansingh were born first followed by
two daughters. Saurav is the eldest child and Janki is the youngest. Three of the children are
studying at Trinity School and three are studying at St. Stefan. Tamanna and Rohit study at
St. Stefan school. Kuntala, the eldest daughter, plays chess. Mansorover school offers cricket
only, while Trinity school offers chess. Beside, these schools offer no other games. The chil-
dren who are at Mansorover school have been born in succession. Mahi and Nilesh are crick-
eters while Arun plays football. Rohit who was born just before Janki, plays hockey.
42. Arun is the _________ child of Mr. Mansingh.
A. 2nd B. 3rd C. 6th D. 5th
43. Saurav is a student of which school?
A. Trinity B. St. Stefan
C. Mansorover D. Cannot be determined
44. What game does Tamanna play?
A. Cricket B. Hockey
C. Football D. Cannot be determined
45. Which of the following pairs was not born in succession (ignore the order)?
A. Mahi and Nilesh B. Kuntala and Arun
C. Rohit and Janki D. Arun and Rohit
46. If it is true that ‘some engineers are graduates’ which of the following is definitely false?
A. Some engineers are not graduates B. All engineers are graduates
C. No Engineers are graduates D. None of the above

USPC - CSAT 337


UPSC - CSAT -1
47. Which one of the following diagrams illustrates relationship among Ants, Insects, and
living beings?

(a) (b)

(c) (d)
48. Find the next term in the alphanumerical series. A1Z, C2W, E6T, G21Q, ?
(a) I 88 N (b) H 66 O (c) I 88 O (d) H 66 N

Directions for questions 49-51:


Professor Marathe works only on Mondays, Tuesdays, Wednesdays, Fridays, and Saturdays.
She performs four different activities – Lecturing, Conducting quizzes, correcting papers
and working on consultancy projects. Each working day she performs exactly one activity in
the morning and exactly one activity in the afternoon. During each week her work schedule
MUST satisfy the following restrictions:
• She conducts quizzes on exactly three mornings.
• If she conducts quizzes on Monday, she does not conduct a quiz on Tuesday.
• She lectures in the afternoon on exactly two consecutive calendar days.
• She corrects papers on exactly one morning and three afternoons.
• She works on consultancy project on exactly one morning.
• On Saturday, she neither lectures nor conducts quizzes.
49. On Wednesdays, the professor could be scheduled to?
(a) Work on a consultancy project in the morning and conduct a quiz in the afternoon
(b) Lecture in the morning and correct papers in the afternoon.
(c) Conduct a quiz in the morning and lecture in the afternoon
(d) Conduct a quiz in the morning and work on consultancy project in the afternoon.
50. Which of the following statements must be true?
(a) There is one day on which she corrects papers both in the morning and in the afternoon.
(b) She works on the consultancy project on one of the days on which she lectures.

338 USPC - CSAT


UPSC - CSAT -1
(c) She works on consultancy project on one of the days on which she corrects papers.
(d) She lectures on one of the days on which she conducts quiz.
51. If the Professor conducts a quiz on Tuesday, then her schedule for correcting papers
could be?
(a) Monday morning, Monday afternoon, Friday morning, Friday afternoon.
(b) Monday morning, Friday afternoon, Saturday morning, Saturday afternoon.
(c) Monday afternoon, Wednesday morning, Wednesday afternoon, Saturday afternoon.
(d) Wednesday afternoon, Friday afternoon, Saturday morning, Saturday afternoon.

Directions for questions 52 - 53:


Read the information below and answer the questions given below:
It is possible to arrange eight of the nine numbers 2, 3, 4, 5, 7, 10, 11, 12, 13 in the vacant
squares of the 3 by 4 array shown below so that the arithmetic average of the numbers in each
row and column is the same integer.
1 15
9
14
52. The arithmetic average is:
(a) 6 (b) 7 (c) 8 (d) 9
53. Which one of the nine numbers must be left out when completing the array?
(a) 4 (b) 5 (c) 7 (d) 10
54. Below are two statements are followed by two possible inferences. Choose the correct
option.
Statements:
1. All students stay in the hostel.
2. Some of the students are intelligent.
Inferences
A. Every student is intelligent.
B. All the intelligent students stay in the hostel.
(a) Only inference A follows (b) Only inference B follows
(c) Both inference A and B follow (d) Neither inference A nor B follows

USPC - CSAT 339


UPSC - CSAT -1
55. The below Venn diagram shows a city population which read three popular daily News-
papers Hindustan Times (HT), The Times of India (TI) and Navbharat Times (NT):


If a person is randomly selected from the city population and it is found that he reads at least
one of the three newspapers then the person belongs to which part of the population?
(a) g (b) a + b + c (c) P - h (d) P-g
56. Six football teams play a tournament. Each team plays every other team exactly once.
The winner of a game gets 3 points, the loser gets 0 points. If a game ends in a draw
each team gets 1 point. Altogether the teams got 40 points. How many games did end in
a draw?
(a) 2 (b) 3 (c) 4 (d) 5
57. Six friends A, B, C, D, E and F are seated in a circle facing centre. If F is between A and
D, C, is between E and B, E is not between D and C, and D is 2nd to the left of C; which
one of the following is the position of D?
(a) 2nd to the right of A (b) Next to the right of B
(c) 3rd to the left of B (d) 4th to the right of A
58. From a novel, which starts from page number 1, some of the leaves have been torn off.
The sum of these page numbers is 342. How many leaves have been torn off?
(a) 6 (b) 9
(c) 12 (d) cannot be determined
59. In the word HEIRARCHICAL, If the first and second, third and fourth, fourth and
fifth, fifth and sixth letters are interchanged up to the last letter, which are the two po-
sition from the left on which R would appear and on which positions would C appear
twice?
(a) R – 3 and 5; C – 8 and 9 (b) R – 9 and 10; C – 4 and 5
(c) R – 4 and 5; C – 7 and 8 (d) 4 and 5; C – 7 and 8

340 USPC - CSAT


UPSC - CSAT -1
60. In the following series, what numbers should replace the question marks?
–1, 0, 1, 0, 2, 4, 1, 6, 9, 2, 12, 16, ? ? ?
(a) 11, 18, 27 (b) –1, 0, 3
(c) 3, 20, 25 (d) Cannot be ascertained
61. Here are some words translated from an artificial language.
dionot means oak tree
blyonot means oak leaf
blycrin means maple leaf
Which word could mean “maple syrup”
(a) blymuth (b) hupponot (c) patricrin (d) crinweel
62. Gita is older than her cousin Mita. Mita’s brother, Bhanu is older than Gita. When Mita
and Bhanu are visiting Gita, all three like to play a game of Monopoly. Mita wins more
often than Gita does. Which of the following can be concluded from the above?
(a) When he plays Monopoly with Mita and Gita. Bhanu often loses.
(b) Of the three, Gita is the oldest
(c) Gita hates to lose at Monopoly
(d) Of the three, Mita is the youngest.
63. Priya is taller than Tiya and shorter than Siya. Riya is shorter than Siya and taller than
Priya. Riya is taller than Diya, who is shorter than Tiya.
Arrange them in order of asending heights.
(a) Priya – Siya – Riya – Tiya - Diya (b) Riya – Siya – Priya – Diya - Tiya
(c) Siya – Riya – Priya – Tiya - Diya (d) Siya – Priya – Riya – Diya - Tiya
64. Statement 1: All chickens are birds.
Statement 2: Some chickens are hens.
Statement 3: Female birds lay eggs.
If the above statement are facts, then which of the following must also be a fact?
I. All birds lay eggs.
II. Hens are birds.
III. Some chickens are not hens.
(a) II only (b) II and III only

USPC - CSAT 341


UPSC - CSAT -1
(c) I, II and III (d) None of the statement is a known fact
65. Statement 1: Pictures can tell a story.
Statement 2: All storybooks have pictures.
Statement 3: Some storybooks have words.
If the above statement are facts, then which of the following must also be a fact?
I. Pictures can tell a story better than words can.
II. The stories in storybook are very simple
III. Some storybooks have both words and pictures.
(a) I only (b) II only
(c) III only (d) None of the statement is a known fact
66. If IQS : LNV, then JRM : ?
(a) OKS (b) MOP (c) NIP (d) MOQ

Directions for Questions 67 – 68:


Some information is provided in the paragraph below. Answer the questions based on this
information
A weekly television show routinely stars six actors, J, K, L, M, N and O. Since the show has
been on the air for a long time, some of the actors are good friends and some do not get along
at all. In an effort to keep peace, the director sees to it that friends work together and enemies
do not. Also, as the actors have become more popular, some of them need time off to do other
projects. To keep the schedule working, the director has a few things she must be aware of:
• J will only work on episodes on which M is working
• N will not work with K under any circumstances.
• M can only work every other week, in order to be free to film a movie.
• At least three of the actors must appear in every weekly episode.
67. In a show about L getting a job at the same company J already works for and K used
to work for, all three actors will appear. Which of the following is true about the other
actors who may appear?
(a) M, N and O must all appear. (b) M may appear and N must appear.
(c) M must appear and O may appear (d) O may appear and N may appear

342 USPC - CSAT


UPSC - CSAT -1
68. Next week, the show involves N’s new car and O’s new refrigerator. Which of the follow-
ing is true about the actors who may appear?
(a) M, J, L and K all may appear. (b) J, L, and K must appear
(c) L and K must appear. (d) Only L may appear.
69. Immediately after leaving his house, Ritvik turned right and walked for 40m. Then he
turned left and walked for 20mts. Then he again took a left turn and walked for 30mts.
There he met a friend and turned right to go to the coffee shop 20 mts away. After hav-
ing coffee, he walked back straight for 40mts in the direction he had come from. How
far is he from his house?
(a) 20 m (b) 0 m (c) 10 m (d) 40 m
70. Find the missing alphabet.
H C ?
B F E
P R T
(a) Y (b) O (c) D (d) G
71. In a four-day period – Monday through Thursday – each of the following temporary
office workers worked only one day, each a different day. Jai was scheduled to work on
Monday, but he traded with Raj, who was originally scheduled to work on Wednesday.
Farid traded with Kajal, who was originally scheduled to work on Thursday. Finally,
Jai traded with Kajal. After all the switching was done, who worked on Tuesday?
(a) Jai (b) Farid (c) Raj (d) Kajal
72. If a clock is kept on the table in such a way that at 3: 10 pm, the hour hand points south,
after how much time will the minute hand point east?
(a) 20 minutes (b) 35 minutes
(c) 50 minutes (d) 90 minutes
73. How many triangles are there in the following figure?


(a) 8 (b) 10 (c) 12 (d) 16

USPC - CSAT 343


UPSC - CSAT -1
Directions for Questions No. 74 - 78 :
Consider the following information.
A cube is painted red on two adjacent faces, black on the faces opposite to the red faces and
green on the remaining faces. It is then cut into 64 smaller cubes of equal size.
74. How many cubes are there which have no face painted?
(a) 0 (b) 4 (c) 8 (d) 12
75. How many cubes have only one face painted?
(a) 16 (b) 24 (c) 36 (d) 48
76. How many cubes have two faces painted?
(a) 8 (b) 16 (c) 24 (d) 30
77. How many cubes are there with three faces painted?
(a) 8 (b) 10 (c) 12 (d) 14
78. How many cubes have one face green and one of the adjacent faces black or red?
(a) 8 (b) 16 (c) 24 (d) 28
79. Some boys are sitting in a row. P is sitting fourteenth from the left and Q is seventh from
the right. If there are four boys between P and Q, how many boys are there in the row,
considering that Q is to the right of P?
(a) 25 (b) 21 (c) 20 (d) 18
80. A bus for Delhi leaves every thirty minutes from a bus stand. An enquiry clerk told a
passenger that the bus had already left ten minutes back and the next bus will leave at
9•35 a.m. At what time did the enquiry clerk give this information to the passenger?
(a) 8.55 a.m. (b) 9.08 a.m. (c) 9.10 a.m. (d) 9.15 a.m.


344 USPC - CSAT


UPSC - CSAT -1

Chapter

4 LRAA + D.I.

Directions: Questions 1 – 5:
The pie charts above present the relative break-up of the sources of income and the heads
of expenditure for a family for the year 2007. The total annual income of the family is Rs.
8,00,000. Due to the revised pay scales, the incomes of the father and the mother have in-
creased by 40% and 30% respectively in 2008. Education cost doubled in 2008 whereas, all
other expenses (in Rs.) remain same in 2008.
1. Assuming that the relative contribution of different sources of income remains the same
across all heads of expenditure, how much of the father’s income is spent on medical
expenses for the year 2007?
A. Rs. 32, 000 B. Rs. 25, 000
C. Rs. 54, 000 D. Rs. 80, 000
2. What percentage of the total income is spent on education in 2008?
A. 20% B. 30% C. 50% D. 40%

USPC - CSAT 345


UPSC - CSAT -1
3. In 2008, what is the amount saved by the family?
A. Rs. 50, 000 B. Rs. 40, 000
C. Rs. 10, 000 D. Rs. 15, 000
4. What is the total amount spent on clothing and medical expenses during 2007 and 2008?
A. Rs. 3, 20, 000 B. Rs. 2, 40, 000
C. Rs. 4, 40, 000 D. Rs. 5, 20, 000
5. In 2008, what percentage of income is spent on bills and miscellaneous expenses?
A. 5% B. 12% C. 16% D. 25%

Directions for Questions 6 to 8:


For admission to various affiliated colleges, a university conducts a written test with four
different sections, each with a maximum of 50 marks. The table above gives the aggregate as
well as the sectional cut-off marks fixed by six different colleges affiliated to the university. A
student will get admission only if he/she gets marks greater than or equal to the cut-off marks
in each of the sections and his/her aggregate marks are at least equal to the aggregate cut-off
marks as specified by the college.
Sectional Cut-offs Aggregate
Section A Section B Section C Section D Cut-off marks
College 1 42 42 42 176
College 2 45 45 175
College 3 46 171
College 4 43 45 178
College 5 45 43 180
College 6 41 44 176

6. Bharna got calls from all colleges. What could be the minimum aggregate marks obtained
by her1?
A. 180 B. 181 C. 196 D. 176
7. Aditya did not get a call from even a single college. What could be the maximum aggre-
gate marks obtained by him?
A. 181 B. 176 C. 184 D. 196
8. Charlie got calls from two colleges. What could be the minimum marks obtained by him
in a section’?
A. 0 B. 21 C. 25 D. 35

346 USPC - CSAT


UPSC - CSAT -1
Directions for Questions 9 - 11:
The table shows the number of people who responded to a survey about their favourite style
of music.
Age 15-20 21-30 31+
Classical 6 4 17
Pop 7 5 5
Rock 6 12 14
lazz 1 4 11
Blues 2 3 15
Hip-Hop 9 3 4
Ambient 2 2 2
33 33 68

9. What percentage of respondents under 31 indicated that Blues is their favourite style of
music?
A. 7.1 B. 7.6 C. 8.3 D. 14.1
10. What percentage of respondents aged 21-30 indicated a favourite style other than Rock
music?
A. 64% B. 60% C. 75% D. 36%
11. What percentage of the total sample indicated that Jazz is their favourite style of mu-
sic?
A. 6% B. 8% C. 22% D. 12%

Directions for Question Nos. 12 – 13:


These questions are based on the graph which shows the demand and production statistics of
5 T.V. companies

USPC - CSAT 347


UPSC - CSAT -1
12. What is the difference between the average demand and the average production of the
five companies taken together?
A. 1400 B. 400 C. 280 D. 138
13. The production of the company D is how many times of the production of the company
A?
A. 1.8 B. 1.5 C. 2.5 D. 1.11

Directions for Question Nos. 14 – 15:


The following questions are based on the pie-chart given which gives the expenditure in-
curred in printing a magazine:


14. What is the angle for the sector representing paper cost?
A. 10 degrees B. 36 degrees
C. 23.5 degrees D. 45 degrees
15. For a given issue of the magazine, the 2% of the cost is Rs. 2,000 and the print-run is
12,500 copies. What should be the sale price if the publisher desires a profit of 5 per-
cent?
A. Rs.5 B. Rs.7.5 C. Rs.8 D. Rs.8.40

Directions for (Q. Nos. 16 – 19) :


These questions consist of a question and two statements numbered I and II given below it.
You have to decide whether the data provided in the statements are sufficient to answer the
question.
Read both the statements and give your answer as follows:
A. If the data in statement I alone is sufficient to answer the question

348 USPC - CSAT


UPSC - CSAT -1
B. If the data in statement II alone is sufficient to answer the question
C. If the data in statement I and II together are not sufficient to answer the question
D. If the data in statement I and II together are necessary to answer the question
16. What is the height of Ashok?
I. Nisheeth is 5'-8" tall and is 3" taller than Ashok.
II. Kapil is 4" taller than Ashok
17. Who is C’s partner in a game of cards involving four players?
I. D is sitting opposite to A.
II. B is sitting right of A and left of C
18. What is the average weight of the 3 new team members who are recently included into
the team?
I. The average weight of the team increases by 20 kg.
II. The 3 new men substitute 3 earlier members whose weighs are 64 kg, 75 kg, and 66 kg.
19. Goats are put into three sheds of large, medium and small sizes. Large shed contains
more goats than medium shed and medium in turn contains more goats than small.
How many goats are there in the small shed?
I. The large shed and the medium sheds contain 13 goats in total.
II. The medium and small sheds contain 11 goats in total.

Directions for Q 20 to 21:


Mark (a): if the question can be answered with the help of statement I alone,
Mark (b): if the question can be answered with the help of any one statement independently,
Mark (c): if the question can be answered with the help of both statements together but no by
any statement alone.
Mark (d): if the questions cannot be answered even with the help of both statements together.
20. Is the number p, a prime number?
I. p is not a composite number
II. p is an even number
21. Three friends, P, Q and R are wearing hats, either black or white. Each person can see
the hats of the other two persons. What is the colour of P‘s hat?
I. P says that he can see one black hat and one white hat.
II. Q says that he can see one white hat and one black.

USPC - CSAT 349


UPSC - CSAT -1
22. Who among Siddhartha, Nikunj, Vipul and Mukul is the youngest?
I. Vipul is younger than Mukul but older than Siddhartha and Nikunj.
II. Mukul is the oldest.
III. Siddhartha is older than Nikunj.
A. Only I B. Only I and II C. Only II and III D. Only I and III
23. Five persons - A, B, C, D and E are sitting in a row. Who is sitting in the middle?
I. B is between E and C.
II. B is to the right of E.
III. D is between A and E.
A. Only I and II B. Only II and III
C. Only I and III D. All I, II and III
24. In a certain code, 'XYZ' means 'We are friends'. Which letter stands for 'We' ?
I. 'PYN' means 'They are classmates'.
II. 'ZMS' means 'We love them'.
III. 'PX' means 'Hello friends',
A. Only II B. Only I and III C. All I, II and III D. None of these
Directions for Questions 25-29:
Study the following information carefully to answer these questions.
Percentage of employees in various departments of an organization (represented in a pie
chart) and the male-female ratio (shown in table).
Total No. of Employees in the organization = 2500

350 USPC - CSAT


UPSC - CSAT -1
Department Male : Female
Admin 7:5
Accounts 2:3
HR 5:3
Marketing 7:8
IT 3:4
Operations 5:4
Logistics 6:5
Printing 2:1

25. What is the ratio of male employees in Administration to those in printing Department?
A. 7:4 B. 4:7 C. 3:4 D. 7:3
26. What is the difference between the total number of employees in IT and that in opera-
tions department?
A. 75 B. 150 C. 100 D. 50
27. What is the ratio of the total number of males in HR and marketing to the total number
of females in these two departments?
A. 13 : 15 B. 15 : 13 C. 13 : 17 D. 17 : 14
28. How many female employees are there in the HR Department?
A. 250 B. 120 C. 125 D. 150
29. What is the difference between the numbers of male and female employees in Logistics
Department?
A. 50 B. 25 C. 75 D. 100

For Questions 30-33,


each question is followed by two statements, (1) and (2). Mark your answer as:
a. If statement (1) alone is sufficient but statement (2) alone is not sufficient to answer the
question asked.
b. If statement (2) alone is sufficient but statement (1) alone is not sufficient to answer the
question asked.
c. If both statements (1) and (2) together are sufficient to answer the question or either
statement is sufficient alone.
d. If statements (1) and (2) are not sufficient to answer the question asked and additional
data is needed to answer the statements.

USPC - CSAT 351


UPSC - CSAT -1
30. A rectangular box (shown below) is 48 centimeters long, 32 centimeters wide, and 15
centimeters high. The box is filled to capacity with k identical cylindrical cans that
stand upright in rows and columns, as indicated in the figure above. If the cans are 15
centimeters high, what is the value of k?


(1) Each of the cans has a radius of 4 centimeters.
(2) 6 of the cans fit exactly along the length of the carton.
31. The table gives the number of calories and grams of protein per kilogram of foods S &
T. If a total of 7 kilograms of S and T are combined to make a certain food mixture, how
many kilograms of food S are in the mixture?
Food No. of Cal/Kg No.of Gms. of Protein/Kg
S 2,000 150
T 1,500 90

(1) Mixture has a total of 12,000 calories.


(2) Mixture has a total of 810 grams of protein
32. The sum of the ages of Amar, Akbar and Anthony is 53 years. If Anthony, whose ages
are a perfect square, is the oldest, then who is the youngest?
I. Sum of ages of Amar and Akbar is a perfect square.
II. Ages (in years) of all of them are distinct odd natural numbers.
33. If Argentina beats Brazil by 3 goals in a soccer match, how many goals were scored by
Argentina?
I. Both terms were tied at the end of first half.
II. The number of goals scored by both the teams in the first half as well as the second
half was a perfect square.

Directions for Questions 34 - 38:


Data of different states regarding population of states in the year 1998. Total population of
the given States = 3276000.

352 USPC - CSAT


UPSC - CSAT -1


Sex and Literacy wise Population Ratio
States Sex Literacy
M F Literate Illiterate
A.P 5 3 2 7
M.P 3 1 1 4
Delhi 2 3 2 1
Goa 3 5 3 2
Bihar 3 4 4 1
U.P. 3 2 7 2
T.N. 3 4 9 4

34. What will be the percentage of total number of males in U.P., M.P. and Goa together to
the total population of all the given states?
A. 25% B. 27.5% C. 28.5% D. 31.5%
35. What was the total number of illiterate people in A.P. and M.P. in 1998?
A. 876040 B. 932170 C. 981550 D. 1161160
36. What is the ratio of the number of females in T.N. to the number of females in Delhi?
A. 7 : 5 B. 9 : 7 C. 13 : 11 D. 15 : 14
37. What was the number of males in U.P. in the year 1998?
A. 254650 B. 294840 C. 321470 D. 341200
38. If in the year 1998, there was an increase of 10% in the population of U.P. and 12% in
the population of M.P. compared to the previous year, then what was the ratio of popu-
lations of U.P. and M.P. in 1997?
A. 42 : 55 B. 48 : 55 C. 7 : 11 D. 4 : 5

Directions for Questions 39 – 43:


Analyse the following chart showing the exports and imports of Lampat Ltd. and answer the

USPC - CSAT 353


UPSC - CSAT -1
questions based on this chart. Total exports and imports for the given period is 1970 and 2170
respectively.


39. Approximately by what percentage are the total Exports greater/ smaller than the total
imports for the given period?
(a) Greater by 9 percent (b) Smaller by 10 percent
(c) Smaller by 9 percent (d) Greater by 10 percent
40. If the absolute difference between imports and exports for each year are ranked in as-
cending order, which year gets 1st rank?
(a) 2010 (b) 2008 (c) 2009 (d) None of the above
41. In which year was the largest annual percentage increase in exports recorded?
(a) 2007 (b) 2005 (c) 2009 (d) 2008
42. Which year saw the second largest annual percentage increase in imports?
(a) 2010 (b) 2005 (c) 2006 (d) None of the above
43. What is the approximate percentage point difference in the maximum annual percent-
age increase in export and the annual percentage decrease in Imports in 2003?
(a) 28 (b) 48 (c) 64 (d) 12

Directions for Questions 44 – 45:


Study the following pie charts regarding sales of 5 models of cars for the years 2010 and
2011, and answer the questions.

354 USPC - CSAT


UPSC - CSAT -1

44. If the 2010 sales for all car models is 80,000 and these have grown by 25% in 2011, then
what is the approximate increase in the number of Figo cars sold in 2011 over 2010?
(a) 5,000 (b) 12,200 (c) 4,500 (d) 2,200
45. If the 2010 sales for all car models is 80,000 and these have grown by 25% in 2011, then
how many models have grown more than the average growth rate for all the models
taken together?
(a) 2 (b) 3 (c) 4 (d) None of the above
46. A large volume of water is gushing through a pipe which narrows at the outlet. At which
point, will the water flow fastest?

(a) A (b) B (c) C (d) D

USPC - CSAT 355


UPSC - CSAT -1
47. The following table shows the percentage of male and female coffee drinkers and non-
coffee drinkers in two Towns A and B. If the total population of the Towns A and B be
10,000 and 20,000 respectively, then what is the total number of female coffee drinkers
in both towns?

Town A Town B
Attributes
Male Female Male Female
Coffee drinkers 40% 5% 25% 15%
Non-coffee drinkers 20% 35% 30% 30%
(a) 8,000 (b) 6,000 (c) 3,500 (d) 2,500
48. From the graph below, who out of the four persons A, B, C and D saves the least per-
centage of his monthly income?


(a) A (b) B (c) C (d) D
49. The position - time (x - t) graph for motion of a body is given below :


The number representing acceleration of the body shown in the graph, would be =?
(a) Some number > 0 (b) Some negative number
(c) 0 (d) Cannot be determined

356 USPC - CSAT


UPSC - CSAT -1
Directions for questions 50 – 52:
A team of scientists observed the growth of a plant and tabulated the data in the following
table. The time count began from the time, when the plant length is 15 cm.
Length at
Time
given
(Day's)
time (cm)
0 15
15 31
45 63
60 79
90 111
150 175
180 207

50. What was the length of the plant 2 months after the beginning of the time period?
(a) 15 cm (b) 31 cm (c) 63 cm (d) 79 cm
51. During the five month period after the beginning of the time period, the length of the
plant increased by
(a) 500% (b) 833.33% (c) 1066.66% (d) 1280%
52. At time t, measured in days after the beginning of the time period, which of the follow-
ing gives the length of the plant in accordance with the table?

16t 16t
15 + 15 +
(a) 10 t (b) 15 (c) 15 (d) t + 25
53. What is the number of roots of A to B? (Assuming the path is allowed in the direction of
arrow only.)


(a) 8 (b) 11 (c) 13 (d) None of the above

USPC - CSAT 357


UPSC - CSAT -1
54. All even integers greater than -1000 are arranged in six columns in the table below. In
which column does 2004 appear?
A B C D E F
-998 -996 -994 -992 -990 -988
-986 -984 -982 -980 -978 -976
-974 -972 … …. … …

(a) A (b) B (c) C (d) D

Directions for (55-60) :


The table shows the results and points scored from a league of six teams. Using this table
answer the following questions:
Team Won Drawn Lost Points
A 3 1 0 10
B 3 0 1 9
C 2 0 2 6
D 2 0 2 6
E 0 1 2 1
F 0 0 3 0

55. Which of the following combinations describe the points for a win, draw and loss re-
spectively that a team earns?
(a) 4,-2, 0 (b) 3, 2, -1 (c) 3, 1 ,0 (d) 4, 0, -1
56. How many games have been played?
(a) 8 (b) 9 (c) 10 (d) 11
57. If each team plays every other team once during the season, how many games remain to
be played?
(a) 4 (b) 3 (c) 5 (d) 6
58. The team with the most points at the end of the season wins the league. How many
teams could achieve this?
(a) 1 (b) 2
(c) 3 (d) Cannot be determined
59. The team with the least points at the end of the season wins the league. How many teams
can finish bottom of the table?
(a) 1 (b) 2

358 USPC - CSAT


UPSC - CSAT -1
(c) 3 (d) Cannot be determined
60. What is the highest possible value for the points of all the teams combined at the end of
the season?
(a) 34 (b) 38 (c) 42 (d) 44

Directions for (61 - 65) :


Interpret the following graph showing rainfall throughout a year in cm and answer the
questions:


61. Which month had the highest absolute increase in rainfall compared to the previous
month?
(a) November (b) December (c) March (d) August
62. Which month had the highest decrease in rainfall compared to the previous month?
(a) December (b) February (c) April (d) September
63. Which month had the lowest increase in rainfall compared to the previous month?
(a) March (b) August (c) October (d) May
64. What is the total rainfall for the year?
(a) 4.4 (b) 14.4 (c) 24.4 (d) 34.4
65. Which month was closest to the average monthly rainfall for the year?
(a) February (b) August (c) March (d) January

USPC - CSAT 359


UPSC - CSAT -1
Directions for Qns. (66 - 67):
One –bedroom flats were most popular new homes in Pune last year, both for living and as
investments. The good news is that more of these flats are being built; the bad news is that
they are getting smaller and more expensive.
66. Based on the above passage, we can conclude that It has become popular to buy flats as
investments in Pune. One-bedroom flats are becoming more expensive.
(a) Only I follows (b) Only II follows
(c) Both I and II follow (d) Neither I and II follow
67. Based on the above passage, we can conclude that
I. Buyers who look to invest often rent out their flats.
II. Pune is a very expensive city to live.
(a) Only I follows (b) Only II follows
(c) Both I and II follow (d) Neither I and II follow

Directions for questions. (68-69):


The number of IT jobs in India has more than doubled in the last five years. But while women
represent over half of the potential workforce in India, they form only 22 per cent of the IT
workforce, compared with 29 per cent as recently as 2004, according to government figures.
68. Based on the above passage, we can conclude that
I. More than 50% of the potential workforce in India is comprised of women.
II. The IT workforce is dominated by women.
(a) Only I follows (b) Only II follows
(c) Both I and II follow (d) Neither I and II follow
69. Based on the above passage, we can conclude that
I. 7 per cent of women working in IT have moved to work elsewhere.
II. There are fewer women working in IT today than in 1997.
(a) Only I follows (b) Only II follows
(c) Both I and II follow (d) Neither I and II follow
70. A family consisting of two adults and three children has booked a holiday to Kashmir.
The children are one, seven and ten years old. With children’s discount, the whole trip
costs Rs.18,000. How much does the trip cost for the children?
I. The family pays Rs. 1200 for the one-year-old.
II. The seven- and the ten-year-old each pay 40 per cent of the price of an adult.
There is sufficient information in order to solve the question:
360 USPC - CSAT
UPSC - CSAT -1
(a) In (I) but not in (II) (b) In (II) but not in (I)
(c) In (I) and (II) together (d) In (I) and (II) separately

Directions for Qns. (71 - 72):


Each of the questions given below consists of a statement and/or a question and two state-
ments numbered I and II given below it. Read both statements and
• Give answer (a) if the data in Statement I alone is sufficient to answer the question.
• Give answer (b) if the data in Statement II alone are sufficient to answer the question.
• Give answer (c) if the data either in Statement I or in Statement II alone are sufficient to
answer the question.
• Give answer (d) if the data in both Statements I and II together are necessary.
71. The towns A, B and C are on a straight line. Town C is between A and B. The distance
from A to B is 100 km. How far is A from C?
I. The distance from A to B is 25% more than the distance from C to B.
II. The distance from A to C is one-fourth of the distance C to B.
72. Average age of employees working in a department is 30 years. In the next year, ten
workers will retire. What will be the average age in the next year?
I. Retirement age is 60 years.
II. There are 50 employees in the department.

Directions for Qns. (73 - 77):


Each of the questions below consists of a question and two statements numbered I and II
given below. Read both the statements and answer-
(a) if the data in statement I alone is sufficient to answer the question,
(b) if the data in statement II alone is sufficient to answer the question.
(c) if the data even in both statements I and II together are not sufficient
(d) if the data in both statements I and II together are necessary to answer the question.
73. Who among Meera, Jyoti, Pinki and Sadhana is the tallest?
I. Meera is not as tall as Pinki or Sadhana
II. Jyoti is taller than Sadhana
74. How is N related to M?
I. O is the husband of N.
II. Z is N’s daughter and has a brother M.
75. All men are boys. Are some boys thrifty?

USPC - CSAT 361


UPSC - CSAT -1
I. Some men are thrifty.
II. Some boys are not thrifty.
76. How is X related to W?
I. V is the husband of X.
II. W is the brother of Z and Z's mother is X.
77. Is this college co-educational?
I. There are more female teachers than male teachers in this college.
II. A girl known to me was the topper from this college.
78. Who is the uncle of L?
I. P, brother of M, is father of L. M is father of S.
II. R is father of L’s cousin.
(a) I alone is sufficient (b) II alone is sufficient
(c) Either I alone or II alone is sufficient (d) Both I and II together are
not sufficient
79. Who is to the North-East of R?
I. S is to the South-East of N, who is to the South-West of P, who is to the North of Q.
II. T is to the North-West of Q who is to the South of P.
III. R, who is to the North of S, is midway between N and Q, N being to the West of R.
(a) All I, II, III together are required
(b) Only I and III together are sufficient
(c) Only II and III together are sufficient
(d) Either I and III together or II and III together are sufficient
80. What is the rank of Suresh in the class of 17 students?
I. Nitin, having 13th rank from the bottom, is six ranks ahead of Bhupesh, who is two
ranks behind Suresh.
II. Bhupesh is four ranks ahead of Kamlesh
III. Bhupesh is two ranks behind Suresh and Kamlesh’s rank is 15.
(a) Only I alone is sufficient
(b) Either I alone or II and III together are sufficient
(c) Only II and III together are sufficient
(d) Only I and III together are sufficient

362 USPC - CSAT
UPSC - CSAT -1
Notes

USPC - CSAT 363


UPSC - CSAT -1
Notes

364 USPC - CSAT


UPSC - CSAT -1
Notes

USPC - CSAT 365


UPSC - CSAT -1
Notes

366 USPC - CSAT


UPSC - CSAT -1
Notes

USPC - CSAT 367


UPSC - CSAT -1
Notes

368 USPC - CSAT

You might also like